Top 45 đề thi HSG Toán 9 có lời giải chi tiết

Giới thiệu đến quý thầy, cô giáo và các em học sinh lớp 9 đề thi chọn học sinh giỏi môn Toán các trường THPT và cơ sở Giáo dục – Đào tạo trên toàn quốc.  giúp bạn ôn tập kiến thức, chuẩn bị tốt kì thi sắp tới. Mời bạn đọc đón xem.

SỞ GIÁO DỤC VÀ ĐÀO TO
THÀNH PHỐ CẦN THƠ
Đề chính thức
THI CHỌN HỌC SINH GIỎI LỚP 9 THCS
CẤP THÀNH PHỐ-NĂM HỌC 2012-2013
Khóa ngày 11/04/2013
MÔN THI: TOÁN
Thời gian làm bài: 150 phút, không kể thời gian phát đề.
Câu 1 (5,0 điểm)
1. Cho biểu thức P =
2m +
16m + 6
m + 2
m 3
+
m 2
m 1
+
3
m + 3
2
a) Rút gọn P .
b) Tìm giá trị tự nhiên của m để P số tự nhiên.
2. Tính giá trị (a
3
+ 15a 25)
2013
với a =
3
p
13 7
6 +
3
p
13 + 7
6.
Câu 2 (5,0 điểm)
1. Giải phương trình:
x + 5 +
3 x 2
15 2x x
2
+ 1
= 0.
2. Tìm giá trị của m để hệ phương trình sau nghiệm:
2x
2
+ mx 1 = 0
mx
2
x + 2 = 0
Câu 3 (5,0 điểm)
1. Tìm tất cả các số nguyên dương x, y, z thỏa
1
x
+
1
y
+
1
z
= 2.
2. Cho hai số x, y thỏa mãn:
x + y 2
x
2
+ y
2
+ xy = 3
Tìm giá trị lớn nhất, giá trị nhỏ nhất của biểu thức T = x
2
+ y
2
xy.
Câu 4 (2,0 điểm)
Cho đường tròn (O; R) và hai điểm A, B nằm ngoài đường tròn sao cho OA = 2R. Tìm điểm M
trên đường tròn để M A + 2MB đạt giá trị nhỏ nhất.
Câu 5 (3,0 điểm)
Cho tam giác ABC ba c nhọn nội tiếp đường tròn (O; R). Gọi P một điểm di động trên
cung BC không chứa A.
1. Gọi M, N lần lượt hình chiếu vuông góc hạ từ A xuống P B, P C. Chứng minh rằng đường
thẳng M N luôn đi qua một điểm cố định.
2. Gọi I, D, E chân các đường cao lần lượt hạ từ A, B, C xuống các cạnh BC, CA, AB.
Chứng minh rằng chu vi tam giác IDE không đổi khi A, B, C thay đổi trên đường tròn
(O; R) sao cho diện tích của tam giác ABC luôn bằng a
2
.
—–HẾT—–
Ghi chú: Giám thị coi thi không giải thích thêm.
SỞ GIÁO DỤC VÀ ĐÀO TO
THÀNH PHỐ CẦN THƠ
Đề chính thức
THI CHỌN HỌC SINH GIỎI LỚP 9 THCS
CẤP THÀNH PHỐ-NĂM HỌC 2012-2013
Khóa ngày 11/04/2013
MÔN THI: TOÁN
Thời gian làm bài: 150 phút, không kể thời gian phát đề.
HƯỚNG DẪN CHẤM
(Hướng dẫn chấm này có 03 trang.)
U NỘI DUNG ĐIỂM
1(5,0đ)
1. (3,5 điểm)
a) Điều kiện: m 0, m 6= 1 0,5đ
P =
m + 1
m 1
2,0đ
b) P = 1 +
2
m 1
0,5đ
Để P N = m {4; 9} 0,5đ
2.(1,5 điểm)
a =
3
p
13 7
6 +
3
p
13 + 7
6 = a
3
= 26 15a 1,0đ
a
3
+ 15a 25 = 1 = (a
3
+ 15a 25)
2013
= 1 0,5đ
2(5,0đ)
1. (2,5 điểm)
Điều kiện: 5 x 3
0,5đ
Đặt t =
x + 5 +
3 x, t
2
= 8 + 2
15 2x x
2
= t 2
2
Phương trình đã cho dạng: t
2
t 6 = 0
t = 3
t = 2 (loại)
1,0đ
t = 3
x + 5 +
3 x = 3
4x
2
+ 8x 59 = 0
x =
2 + 3
7
2
x =
2 3
7
2
1,0đ
2. (2,5 điểm)
Đặt x
2
= y 0. Hệ trở thành:
mx + 2y = 1
x + my = 2
0,5đ
Hệ luôn nghiệm:
x =
m + 4
m
2
+ 2
y =
1 2m
m
2
+ 2
0 (m
1
2
)
0,5đ
Ta có: x
2
= y
m + 4
m
2
+ 2
2
=
1 2m
m
2
+ 2
0,5đ
(m + 1) (m
2
m + 7) = 0 m = 1 1,0đ
3(5,0đ) 1. (3,0 điểm)
Tiếp
U NỘI DUNG ĐIỂM
Không mất tính tổng quát giả sử: 1 x y z
= 2 =
1
x
+
1
y
+
1
z
3
x
= x = 1
1,0đ
=
1
y
+
1
z
= 1
2
y
=
y = 1 ( )
y = 2 = z = 2
1,0đ
Vậy (1; 2; 2) và các hoán vị của chúng nghiệm của phương trình đã cho 1,0đ
2. (2,0 điểm)
Hệ
(
x + y 2
x
2
+ y
2
+ xy = 3
(
x + y = 2 a (a 0)
x
2
+ y
2
+ xy = 3
0,5đ
Do đó:
(
x + y = 2 a
xy = (2 a)
2
3
, = S
2
4P 0 = 0 a 4 0,5đ
T = x
2
+ y
2
+ xy 2xy = 9 2(2 a)
2
0,5đ
min T = 1 khi x = 1, y = 1 hoặc x = 1, y = 1
max T = 9 khi x =
3, y =
3 hoặc x =
3, y =
3
0,5đ
4(2,0đ)
O
A
B
C
M
M
0
Gọi C điểm trên đoạn thẳng OA sao cho OC =
R
2
, ta điểm C cố định 0,5đ
Dễ thấy OCM đồng dạng OMA = M A = 2MC 0,5đ
Ta M A + MB BC (không đổi)
MA + 2M B = 2(M B + M C) 2BC
0,5đ
Dấu “=” xảy ra khi và chỉ khi M nằm giữa B và C
Vậy khi điểm M giao điểm của đoạn BC và đường tròn (O) thì MA+2MB
đạt giá trị nhỏ nhất
0,5đ
5(3,0đ) 1. (2,0 điểm)
Tiếp
U NỘI DUNG ĐIỂM
O
A
B
C
P
N
D
I
E
M
A
0
Kẻ AI BC, I BC cố định. Ta
\
BMA =
[
BIA = 90
nên tứ giác
AMBI nội tiếp hay
[
AIM =
\
ABM
Ta lại tứ giác ABP C nội tiếp nên
\
ABM =
[
ACP
Do đó
[
AIM =
[
ACP (1)
1,0đ
Mặt khác
[
AIC =
\
ANC = 90
nên tứ giác AINC nội tiếp, suy ra
[
ACP +
[
AIN = 180
(2)
0,5đ
Từ (1) và (2) suy ra
[
AIM +
[
AIN = 180
0,5đ
Vậy đường thẳng MN luôn đi qua điểm cố định I
2. (1,0 điểm)
Tứ giác BCDE nội tiếp suy ra
\
AED =
[
ACB
Kéo dài AO cắt (O; R) tại điểm A
0
. Ta có:
[
EAO +
\
AED =
\
BAA
0
+
[
ACB = 90
= AO DE = S
AEOD
=
1
2
AO.DE =
1
2
R.DE
0,5đ
Tương tự ta cũng có: S
BEOI
=
1
2
R.EI, S
CDOI
=
1
2
R.ID
Vậy: S
ABC
= S
AEOD
+ S
BIOE
+ S
CDOI
=
1
2
R.(DE + EI + ID )
= DE + EI + ID =
2S
ABC
R
=
2a
2
R
(không đổi)
0,5đ
—–HẾT—–
Ghi chú:
Mọi cách giải đúng khác đáp án đều cho điểm tối đa.
SGO DC VÀ ĐÀO TO
THÀNH PH ĐÀ NẴNG
ĐỀ CHÍNH THỨC
K THI CHỌN HỌC SINH GIỎI LP 9
M HC 2010-2011
n thi: TN
Thời gian: 150 phút (khôngnh thi gian giao đề)
i 1. (2,0 điểm)
Cho biu thức:
2
a 1 a a 1 a a a a 1
M
a a a a a a

với a > 0, a 1.
a) Chứng minh rng
M 4.
b) Với nhng giá tr nào của a thì biểu thức
6
N
M
nhn g tr nguyên?
i 2. (2,0 điểm)
a) Cho các hàm s bậc nhất:
y 0,5x 3
,
và
y mx
có đ th lần
lượt các đường thẳng (d
1
), (d
2
) và (
m
). Với những g tr nào của tham s m thì
đường thẳng (
m
) ct hai đường thng (d
1
) và (d
2
) ln lượt ti hai điểm A và B sao
cho đim A có hoành độ âm còn đim B có hoành độ dương?
b) Trên mặt phng tọa đ Oxy, cho M và N là hai đim pn bit, di đng ln
lượt trên trục hoành trên trc tung sao cho đường thẳng MN luôn đi qua điểm c
đnh
I(1; 2)
. Tìm hệ thức liên h giữa hoành đ ca M và tung đ ca N; từ đó, suy
ra giá tr nh nhất của biu thức
22
11
.
Q
OM ON

i 3. (2,0 điểm)
a) Gii hệ phương trình:
17 2 2011
2 3 .


x y xy
x y xy
b) Tìm tất cc giá tr ca x, y, z sao cho:
1
x y z z x (y 3).
2
i 4. (3,0 điểm)
Cho đường tròn (C) với tâm O và đường kính AB c đnh. Gi M là đim di
đng trên (C) sao cho M không trùng với các đim A và B. Ly C là đim đi xứng
ca O qua A. Đường thng vuông góc vi AB tại C cắt đường thng AM ti N.
Đường thng BN cắt đường tròn (C ) ti điểm thhai E. Các đường thng BM
CN ct nhau tại F.
a) Chứng minh rằng các đim A, E, F thẳng hàng.
b) Chứng minh rằng tích AMAN không đi.
c) Chứng minh rng A là trngm ca tam giác BNF khi và ch khi NF ngắn
nhất.
i 5. (1,0 điểm)
Tìm ba chữ số tận cùng ca tích của mười hai s nguyên dương đầu tiên.
---HẾT---
H và tên thí sinh: ................................................. S báo danh: ........................
Chữ ký ca giám th 1: ............................. Chữ ký của giám th 2: ...........................
SGO DC VÀ ĐÀO TO
THÀNH PH ĐÀ NẴNG
KÌ THI CHỌN SINH HỌC SINH GIỎI LP 9
M HC 2010-2011
n thi: TN
HƯỚNG DN CHM N TN LỚP 9
Dưi đây là sơ lưc biểu điểm của đề thi Học sinh giỏi lớp 9. Các Giám khảo thảo luận
thống nhất tm chi tiết lời giải cũng như thang điểm của biểu điểm đã tnh bày. Tchấm có thể
phân chia nhỏ thang điểm đến 0,25 điểm cho từng ý của đề thi. Tuy nhiên, điểm từng bài, tng u
không đưc thay đổi. Nội dung thảo luận và đã thống nhất khi chấm đưc ghi vào biên bản cụ thể để
việc chấm phúc khảo sau này đưc thống nhất và chính xác.
Học sinh có li giải khác đúng, chính xác nhưng phải nằm trong chương tnh đưc học t
bài làm đúng đến ý nào giám khảo cho điểm ý đó.
Việc làm tròn số điểm bài kiểm tra được thực hiện theo quy định của BGiáo dục và Đào
tạo tại Quyết định số 40/2006/BGD-ĐT.
BÀI-Ý
Đ -ĐÁP ÁN
ĐIỂM
Bài 1
Cho biu thức:
2
a 1 a a 1 a a a a 1
M
a a a a a a

vi a > 0, a 1.
a) Chng minh rằng
M 4.
b) Vi những giá tr nào ca a thì biu thc
6
N
M
nhận giá tr nguyên.
2,00
1.a
(1,25đ)
Do a > 0, a 1 nên:
a a 1 ( a 1)(a a 1) a a 1
a a a( a 1) a


0,25
2
a a a a 1 (a 1)(a 1) a(a 1) (a 1)(a a 1) a a 1
a a a a(1 a) a(1 a) a
0,25
a1
M2
a

0,25
Do
a 0; a 1
nên:
2
( a 1) 0 a 1 2 a
0,25
2a
M 2 4
a
0,25
1.b
(0,75đ)
Ta có
63
0N
M2
do đó N chỉthnhn được một giá tr nguyên 1
0,25
Mà N = 1
6a
1
a 1 2 a

a 4 a 1 0
2
( a 2) 3
a 2 3 hay a 2 3
(phù hợp)
0,25
Vy, N nguyên
2
a (2 3)
0,25
Bài 2
a) Cho các hàm số bậc nht:
y 0,5x 3
,
y 6 x
và
y mx
đồ thị lần lưt
là các đường thng (d
1
), (d
2
) và (
m
). Với những giá tr nào ca tham số m t đưng
thẳng (
m
) cắt hai đường thng (d
1
) và (d
2
) ln lưt ti hai đim A B sao cho đim
Ahnh độ âmn điểm Bhnh độ dương?
b) Tn mt phng tọa độ Oxy, cho M và N là hai đim phân bit, di động ln lượt
tn trc hoành và tn trục tung sao cho đưng thng MN luôn đi qua đim cố định
I(1; 2)
. Tìm hthức liên hệ gia hnh độ ca M tung độ của N; từ đó, suy ra giá
2,00
tr nhnht ca biểu thức
22
11
.
Q
OM ON

2.a
(0,75đ)
Điu kiện để (
m
) là đồ thị hàm số bậc nhất là
m0
0,25
Phương trình hnh độ giao điểm ca (d
1
) và (
m
) là:
0,5x 3 mx
(m 0,5)x 3
Điu kn để phương trình nàynghiệm âm là
m 0,5 0 hay m 0,5
0,25
Phương trình hnh độ giao điểm ca (d
2
) và (
m
) là:
6 x mx
(m 1)x 6
Điu kn để phương trình nàynghimơng là
m 1 0 hay m 1
Vy điu kiện cần tìm là:
1 m 0,5; m 0
0,25
2.b
(1,25đ)
Đặt m = x
M
và n = y
N
mn 0 và m 1 (*)
Nên đường thng qua ba đim M, I, N có dạng: y = ax+b
0,25
0 am b
2 a b
nb


hệ thức liên hgia m và n là
2m n mn
0,25
Chia hai vế cho mn 0 ta được:
12
1
mn

(**)
22
2 2 2 2
1 2 1 4 4 1 1 2 1
15
m n m n mn m n m n
0,25
22
1 1 1
Q;
m n 5
dấu = xy ra khi
21
;
mn
kết hợp (**): m = 5, n = 2,5 (thỏa (*))
0,25
Vy giá tr nhnhất ca Q là
1
5
0,25
Bài 3
a) Giải hệ phương trình:
17 2 2011
2 3 .


x y xy
x y xy
(1)
b) Tìm tất cả các giá tr của x, y, z sao cho:
1
x y z z x (y 3)
2
(2)
2,0 đ
3.a
(1,25đ)
Nếu
0xy
t
17 2
1 1007
9
2011
9
490
(1)
1 2 9
1 490
3
1007
9
x
yx
y
y
yx
x


(phù hp)
0,50
Nếu
0xy
t
17 2
1 1004
2011
9
(1) 0
12
1 1031
3
18
yx
y
xy
yx
x






(loại)
0,25
Nếu
0xy
t (1)
0xy
(nhn).
0,25
KL: Hệ có đúng 2 nghim là
(0;0)
và
99
;
490 1007



0,25
3.b
(0,75đ)
Điu kiện x ≥ 0; y z 0; z x 0 yzx 0
0,25
(2)
2 x 2 y z 2 z x x y z z x 3
2 2 2
( x 1) ( y z 1) ( z x 1) 0
0,25
x1
y z 1
z x 1


x1
y3
z2
(thỏa điều kin)
0,25
Bài 4
Cho đường tròn (C ) với tâm O và đưng kính
AB cố định. Gi M đim di động tn (C )
sao cho M không trùng với các đim A và B.
Ly C là đim đối xng ca O qua A. Đưng
thẳng vuông góc với AB ti C cắt đưng thng
AM tại N. Đưng thng BN cắt đường tn (C
) tại đim thhai là E. Các đường thng BM và
CN cắt nhau ti F.
a) Chng minh rằng các điểm A, E, F thng
hàng.
b) Chng minh rng ch AMAN không
đổi.
c) Chng minh rằng A là trọng m ca tam
giác BNF khi và chỉ khi NF ngn nht.
C
( )
F
E
N
C
O
A
B
M
3,0 đ
4.a
(1,00đ)
MN BF
và
BC NF
0,25
A là trc tâm ca tam giác BNF
0,25
FA NB
Li
AE NB
0,25
Nên A, E, F thng ng
0,25
4.b
(0,75đ)
CAN MAB
, nên hai tam giác ACN và AMB đồng dạng.
0,25
Suy ra:
AN AC
AB AM
0,25
Hay
2
AM AN AB AC 2R
không đổi (với R n kính đưng tròn (C ))
0,25
4.c
(1,25đ)
Ta có
2
BA BC
3
nên A là trong tâm tam giác BNF C là trung đim NF (3)
0,25
Mặt khác:
CAN CFM
, nên hai tam giác CNA và CBF đồng dạng
2
CN AC
CN CF BC AC 3R
BC CF
0,25
Áp dụng bất đẳng thức Cô-si, ta:
NF CN CF 2 CN CF 2R 3
không đổi
0,25
Nên: NF ngn nhất CN =CF C là trung đim NF (4)
0,25
(3) và (4) cho ta: A là trong tâm tam giác BNF NF ngắn nhất
0,25
Bài 5
Tìm ba chữ số tn cùng ca tích ca mưi hai số nguyên dương đầu tiên.
0,75
(1,00đ)
Đặt: S = 123456789101112
100
S
3467891112 (1) là một snguyên
hai ch số tn cùng ca S là 00
0,50
Mặt khác, trong sut quá trình nhân liên tiếp các tha số vế phi ca (1), nếu chỉ để ý
đến chsố tận cùng, ta thấy
100
S
chsố tận cùng là 6 ( 34=12; 26=12; 27=14;
48=32; 29=18; 811=88; 812=96)
0,25
Vy ba chữ số tn cùng ca S là 600
0,25
--- Hết ---
3.b
(0,75đ)
Điu kiện x ≥ 0; y z 0; z x 0 y ≥ zx 0
0,25
Theo BĐT Cauchy:
x 1 y z 1 z x 1
x ; y z ; z x
2 2 2
1
VP x y z z x (y 3) VT
2
0,25
Do đó
x1
y z 1
z x 1


x1
y3
z2
thỏa điu kiện
0,25
PHÒNG GD-ĐT CẨM THỦY KỲ THI CHN HỌC SINH GII TOÁN 9 (Đ S 3)
năm học : 2011 - 2012
Môn : TOÁN
(Thời gianm bài: 150 phút: Vòng 2)
i 1 ( 3,0 đim)
Cho các s dương: a; b và x =
1
2
2
b
ab
. Xét biu thức P =
b
xaxa
xaxa
3
1
1. Chứng minh Pc đnh. Rút gọn P.
2. Khi a b thay đi, hãym giá tr nh nhất của P.
i 2 (3,0 đim)
m x; y; z tho mãn hệ sau:
xzz
zyy
yxx
3623
2423
223
3
3
3
i 3 ( 3,0 đim)
Vi mi s nguyên dương n 2008, đặt S
n
= a
n
+b
n
, với a =
2
53
; b
=
2
53
.
1. Chứng minh rng vi n ≥ 1, ta có S
n + 2
= (a + b)( a
n + 1
+ b
n + 1
) ab(a
n
+ b
n
)
2. Chứng minh rng vi mi n tho mãn điều kin đề bài, S
n
là số nguyên.
3. Chứng minh S
n
2 =
2
2
15
2
15
nn
. Tìm tất c các số n đ S
n
2 là s chính pơng.
i 4 (5,0 đim)
Cho đoạn thng AB và đim E nm giữa điểm A và điểm B sao cho AE < BE.
V đường tròn (O
1
) đường kính AE và đường tròn (O
2
) đường kính BE. V tiếp
tuyến chung ngoài MN của hai đường tròn trên, vi M là tiếp điểm thuc (O
1
) và N
là tiếp đim thuc (O
2
).
1. Gọi F là giao điểm ca các đường thẳng AM và BN. Chứng minh rng đường
thng EF vuông góc với đường thẳng AB.
2. Vi AB = 18 cm AE = 6 cm, v đường tròn (O) đường kính AB. Đường
thng MN cắt đường tròn (O) ở C và D, sao cho điểm C thuộc cung nh AD. Tính
đ dài đoạn thẳng CD.
Bài 5: (4đ): Cho ABC đường thng d ct AB và AC trung tuyến AM theo thtự . Là
E , F , N .
a) Chng minh :
AN
AM
AF
AC
AE
AB 2
b) Gi sử đường thng d // BC. Trên tia đối ca tia FB ly đim K, đường thng KN ct
AB ti P đường thng KM ct AC tại Q.
Chng minh PQ//BC.
Bài 6: (2 đim)
Cho 0 < a, b,c <1 .Chứng minh rằng :
accbbacba
222333
3222
------------- HT-------------
ỚNG DẪN CHẤM: ĐỀ SỐ 3
Câu 1. (3,0 đim)
Tóm tắt lời gii
Đim
1. (2.0 đim)
Ta: a; b; x > 0
a + x > 0 (1)
Xét a x =
0
1
)1(
2
2
b
ba
(2)
Ta a + x > a x 0
0 xaxa
(3)
T(1); (2); (3)
P xác định
Rút gọn:
Ta: a + x =
1
)1(
1
2
2
2
2
b
ba
b
ab
a
1
)1(
2
b
a
bxa
a - x =
1
)1(
1
2
2
2
2
b
ba
b
ab
a
1
1
2
b
a
bxa
P =
bbb
bb
b
b
a
b
b
a
b
b
a
b
b
a
b
3
1
11
11
3
1
11
1
)1(
1
1
1
)1(
22
22
Nếu 0 < b < 1
P =
bbb 3
4
3
1
2
2
Nếu b
1
P =
b
b
b
b
3
13
3
1
2
2. (1.0 đim)
Xét 2 trường hp:
Nếu 0 < b < 1, a dương tu ý thì P =
b3
4
P
4
3
Nếu b
1
, a dương tuý thì P =
3
2
3
1
33
1 b
b
b
b
b
0,25
0,25
0,25
0,25
0,25
0,25
0,25
0,25
Ta:
3
2
3
1
3
b
b
, du bng xy ra khi và ch khi b = 1
Mặt khác:
3
2
3
2
b
, du bng xy ra khi và ch khi b = 1
Vậy P
3
4
3
2
3
2
, dấu bng xy ra khi và ch khi b = 1
KL: Giá trị nh nht ca P =
3
4
0,25
0,25
0,25
0,25
Câu 2 (3,0 đim)
Tóm tắt lời gii
Đim
Biến đổi tương đương h ta
)2(3)1)(2(
)2(2)1)(2(
2)1)(2(
2
2
2
xzz
zyy
yxx
Nhân các vế ca 3 phương trình với nhau ta được:
(x - 2)(y - 2) (z - 2)(x+1)
2
(y+1)
2
(z+1)
2
= - 6(x - 2)(y - 2) (z - 2)
(x - 2)(y - 2) (z - 2)
6)1()1()1(
222
zyx
= 0
(x - 2)(y - 2) (z - 2) = 0
x = 2 hoặc y = 2 hoc z = 2
Với x = 2 hoặc y = 2 hoc z = 2 thay vào h ta đu x = y = z = 2
Vậy với x = y = z = 2 tho mãn h đã cho
1,00
0,50
0,25
0,25
0,25
0,50
0,25
Câu 3 (3,0 đim)
Tóm tắt lời gii
Đim
1. (1,0 đim)
Với n 1 thì S
n + 2
= a
n+2
+ b
n+2
(1)
Mặt khác: (a + b)( a
n + 1
+b
n + 1
) ab(a
n
+b
n
) = a
n+2
+ b
n+2
(2)
T(1); (2) ta có điu phi chng minh
2. (1.0 đim)
Ta: S
1
= 3; S
2
= 7
Do a + b =3; ab =1 nên theo 1 ta: với n 1 thì S
n+2
= 3S
n+1
- S
n
Do S
1
, S
2
Z nên S
3
Z; do S
2
, S
3
Z nên S
4
Z
Tiếp tc quá trình trên ta được S
5
; S
6
;...; S
2008
Z
3. (1.0 đim)
Ta S
n
2 =
2
2
1
2
5
2
1
2
5
22
nn
=
n
nn
2
15
2
15
2
2
15
2
15
22
=
2
2
15
2
15
nn
đpcm
0,25
0,50
0,25
0,25
0,25
0,25
0,25
0,25
0,25
Đặt a
1
=
2
15
; b
1
=
2
15
a
1
+ b
1
=
5
; a
1
b
1
= 1
Xét U
n
=
11
nn
ab
Với n 1 thì U
n+2
= (a
1
+ b
1
)(a
1
n+1
- b
1
n + 1
) a
1
b
1
(a
1
n
- b
1
n
)
U
n+2
=
5
U
n+1
U
n
Ta U
1
= 1
Z; U
2
=
5
Z; U
3
= 4
Z; U
4
= 3
5
Z;...
Tiếp tc quá trình trên ta được U
n
nguyên
n lẻ
Vậy S
n
2 số chính phương
n = 2k+1 với k
Z và 0
k
1003
0,25
0,25
Câu 4 (5,0 đim)
Tóm tắt lời gii
Đim
1. (2,5 đim) O
1
M; O
2
N
MN
O
1
M/ / O
2
N
Do O
1
; E; O
2
thẳng hàng nên
MO
1
E =
NO
2
B
Các tam giác O
1
ME; O
2
NB lần lượt cân ti O
1
và O
2
nên ta có:
MEO
1
=
NBO
2
(1)
Mặt khác ta có:
AME = 90
0
MAE +
MEO
1
= 90
0
(2)
MAE +
NBO
2
= 90
0
AFB = 90
0
T giác FMEN 3 góc vuông
Tgiác FMEN hình chữ nht
NME =
FEM
(3)
Do MN
MO
1
MNE +
EMO
1
= 90
0
(4)
Do tam giác O
1
MEn ti O
1
MEO
1
=
EMO
1
(5)
T(3); (4); (5) ta có:
FEM +
MEO
1
= 90
0
hay
FEO
1
= 90
0
pcm)
2. (2,5 đim)
Ta EB = 12 cm
O
1
M = 3 cm < O
2
N = 6 cm
MN cắt AB tại S với A nm gia S và B.
Gọi I là trung đim CD
CD
OI
OI// O
1
M //O
2
N
2
1
2
1
SO
SO
NO
MO
SO
2
= 2SO
1
SO
1
+O
1
O
2
= 2SO
1
SO
1
= O
1
O
2
0,25
0.25
0,25
0,25
0,50
0,25
0,25
0,25
0,25
0,25
0,25
0,5
0,25
0,25
0,5
0,25
0,25
F
O
1
O
2
O
E
A
B
C
M
I
N
D
S
Do O
1
O
2
= 3 + 6 = 9 cm
SO
1
= O
1
O
2
= 9 cm
SO =SO
1
+ O
1
O = 15cm
Mặt khác:
11
SO
SO
MO
OI
OI = 5 cm
Xét tam giác COI vng tại I ta có: CI
2
+ OI
2
= CO
2
CI
2
+ 25 = CO
2
Ta: CO = 9 cm
CI
2
+ 25 = 81
CI =
56
CD = 4
14
cm
Câu 5 (2,0 đim)
Đim
a)
K
EFCSBI //,
),( AMSI
Ta:
AN
AS
AF
AC
AN
AI
AE
AB
,
)(
AN
AS
AN
AI
AF
AC
AE
AB
Ta:
CSMBIM
(cgc)
MSIM
Vậy:
AMMSIMAIAIASAI 2
Thay o (*) ta đượcpcm)
1,0
0,5
Khi
NBCEFBCd ////
trung đim ca EF
+TF k đường thng song song với AB cắt KP tại L
Ta:
LFEPcgcNFLNFP )(
Do đó :
)1(
KB
KF
PB
LF
PB
EP
+TB kẻ đường thng song song với AC ct
KM ti H
Ta
)(cgcCMQBMH
QCBH
0,5
0,5
0,5
0,5
E
E
I
S
M
N
C
B
A
K
P
Q
F
L
E
N
M
C
B
A
Do đó:
)2(
KB
KF
BH
FQ
QC
FQ
T
(1) (2) //
FP FQ
va PQ BC
PB QC
pcm)
0,5
Bài 6: 2 đim)
Do a <1
2
a
<1 và b <1
Nên
2 2 2
1 . 1 0 1 0a b a b a b
Hay
baba
22
1
(1)
Mặt khác 0 <a,b <1
32
aa
;
3
bb
332
baab
Vậy
baba
233
1
ơng tự ta có
acca
cbcb
233
233
1
1
accbbacba
222333
3222
0,5
0,5
0,25
0,25
0,5
UBND HUYỆN
PHÒNG GIÁO DỤC - ĐÀO TẠO
ĐỀ THI CHỌN HỌC SINH GIỎI HUYỆN
NĂM HỌC 2013-2014
MÔN: TOÁN LỚP 9
Thời gianm bài 150 phút không k thời gian giao đ
Bài 1: (4 đim) Cho biu thc:
x y x y
x y 2xy
P : 1
1 xy
1 xy 1 xy










.
a) Rút gn biu thc P.
b) Tính giá tr ca P với
2
x
23
.
Bài 2: (4 đim) Trong mt phng tọa độ Oxy, gi (D) và (L) ln lưt là đồ thị ca hai hàm
số:
13
yx
22
và
yx
.
a) Vẽ đồ thị (D) và (L).
b) (D) và (L) cắt nhau ti M và N. Chng minh OMN là tam giác vuông.
Bài 3: (4 điểm) Giải phương trình:
4 3 2
6x 5x 38x 5x 6 0
.
Bài 4: (2 điểm) Qua đnh A của hình vuông ABCD cạnh là a, vmột đưng thng cắt cnh
BC M và cắt đường thẳng DC ở I.
Chng minh rằng:
2 2 2
1 1 1
AM AI a

.
Bài 5: (6 điểm)
Cho hai đường tròn ( O ) và ( O
/
) ngi nhau. Đường ni tâm OO
/
cắt đường tn ( O )
và ( O
/
) tại các đim A, B, C, D theo thứ t trên đưng thng. Kẻ tiếp tuyến chung ngi EF, E
(
Đ CHÍNH THC
O ) và F
( O
/
). Gọi M là giao đim ca AE và DF; N là giao đim ca EB và FC. Chứng minh
rằng:
a) T giác MENF là hình chữ nhật.
b) MN
AD.
c) ME.MA = MF.MD.
---------- Hết ----------
UBND HUYỆN
PHÒNG GIÁO DỤC - ĐÀO TẠO
ĐÁP ÁN HƯỚNG DẪN CHẤM THI
K THI CHỌN HỌC SINH GIỎI HUYỆN
NĂM HỌC 2013-2014-MÔN: TOÁN LỚP 9
Bài
Đáp án
Đim
1
ĐKXĐ:
x 0;y 0;xy 1
.
0,5 đ
a)
Mẫu thức chung là 1 – xy
( x y)(1 xy) ( x y)(1 xy)
1 xy x y 2xy
P:
1 xy 1 xy

x x y y y x x x y y y x
1 xy
.
1 xy 1 x y xy
2( x y x)
2 x(1 y) 2 x
(1 x)(1 y) (1 x)(1 y) 1 x
0,5 đ
0,5 đ
0,5 đ
b)
2
2 2(2 3)
x 3 2 3 1 ( 3 1)
43
23
2
x ( 3 1) 3 1 3 1
2
2( 3 1) 2 3 2
P
1 ( 3 1) 1 3 2 3 1
2( 3 1) 6 3 2
P
13
5 2 3



0,5 đ
0,5 đ
0,5 đ
0,5 đ
2
a)
Đồ thị
13
yx
22
:
3
x 0 y
2
y 0 x 3
Đồ thị
x khi x 0
yx
x khi x 0


Đồ thị như nh v:
0,5 đ
0,5 đ
1 đ
b)
Đồ thị (D) và (L) cắt nhau ti hai đim có tọa độ M(1; 1) và N( - 3; 3)
Ta có: OM =
22
1 1 2
OM
2
= 2
ON =
22
3 ( 3) 3 2
ON
2
= 18
0,5 đ
0,5 đ
MN =
22
(1 3) (1 3) 20
MN
2
= 20
Vì: OM
2
+ ON
2
= MN
2
Vy: tam giác OMN vuông tại O
0,5 đ
0,5 đ
3
Ta thy x = 0 kng phi nghim ca phương trình
Chia cả 2 vế ca phương trình cho x
2
ta được:
2
2
56
6x 5x 38 0
xx
2
2
11
6(x ) 5(x ) 38 0
xx
Đặt
1
yx
x

thì:
22
2
1
x y 2
x
Ta được pt: 6y
2
5y 50 = 0 <=> (3y 10)(2y + 5) = 0
Do đó:
10 5
y và y
32
* Vi
10
y
3
thì:
2
1 10
x 3x 10x 3 0
x3
<=> (3x 1)(x 3) = 0 <=>
1
2
1
x
3
x3
* Vi
5
y
2

thì:
2
15
x 2x 5x 2 0
x2
<=> (2x + 1)(x + 3) = 0 <=>
3
4
1
x
2
x2


1 đ
1 đ
1 đ
1 đ
4
J
M
C
D
I
B
A
Vẽ Ax
AI cắt đường thng CD ti J.
Ta có
AIJ vuông ti A, có AD là đưng cao thuộc cạnh huyn IJ, nên:
2 2 2
1 1 1
AD AJ AI

(1)
Xét hai tam giác vuông ADJ và ABM, ta:
AB = AD = a;
DAJ BAM
(góc có cạnh tươngng vuông góc)
ADJ = ABM
. Suy ra: AJ = AM
Thay vào (1) ta đưc:
2 2 2 2
1 1 1 1
AD AM AI a
(đpcm)
0,5 đ
0,5 đ
0,5 đ
0,5 đ
5
H
D
E
M
F
O
I
N
O
/
B
C
A
a)
Ta có
0
AEB CFD 90
(góc ni tiếp chn nữa đưng tròn)
Vì EF là tiếp tuyến chung ca hai đường tn (O) và (O
/
), nên:
OE
EF và OF
EF => OE // O
/
F
=>
/
EOB FO D
(góc đồng vị) =>
/
EAO FCO
Do đó MA // FN, mà EB
MA => EB
FN
Hay
0
ENF 90
.
T giác MENF
O
E N F 90
, nên MENF là hình chnht
0,5 đ
0,5 đ
0,5 đ
0,5 đ
b)
Gi I giao điểm ca MN và EF; H giao đim ca MN và AD
Vì MENF là nh ch nht, nên
IFN INF
Mặt khác, trong đưng tn (O
/
):
1
IFN FDC sđ FC
2

=>
FDC HNC
Suy ra
FDC
đồng dạng
HNC
(g g)
=>
O
NHC DFC 90
hay MN
AD
0,5 đ
0,5 đ
0,5 đ
0,5 đ
c)
Do MENF là hình ch nhật, n
MFE FEN
Trong đưng tròn (O) có:
1
FEN EAB sđ EB
2

=>
MFE EAB
Suy ra
MEF
đồng dạng
MDA
(g g)
=>
ME MF
MD MA
, hay ME.MA = MF.MD
0,5 đ
0,5 đ
0,5 đ
0,5 đ
Lưu ý: Nếu học sinh giải theo cách khác, nếu đúng và phù hợp vi kiến thức trong chương
tnh đã học thì hai Giám khảo chấm thi thống nhất việc phân bố điểm của cách giải đó, sao cho
không làm thay đổi tổng điểm của bài (hoặc ý) đã nêu trong hưng dẫn này./.
Đ THI HỌC SINH GIỎI TOÁN 9
Thi gian: 150 pt( không kể thời gian giao đề)
Câu1: ( 5đ)
Cho biÓu thøc M =
x
x
x
x
xx
x
2
3
3
12
65
92
a. T×m ®iÒu kiÖn cña x ®Ó M cã nghÜa rót gän M
b. T×m x ®Ó M = 5
c. T×m x
Z ®Ó M
Z.
Câu: 2(2đ). Cho 4a
2
+b
2
=5ab vi 2a>b>0.
Tính giá tr ca biu thức:
22
4 ba
ab
P
Câu 3(4đ)
a. Tìm giá tr nhnhất ca biểu thc
12
683
2
2
xx
xx
A
b. Chng minh rằng vi mi số thực a,b,c ta có
cabcabcba
222
Câu: 4 (4đ)
a. Phân tích đa thc sau thành nhân tử: x
3
+y
3
+z
3
-3xyz
b. Gii phương trình : x
4
+2x
3
-4x
2
-5x-6=0
Câu: 5 (5đ) Cho nh bình hành ABCD có đường chéo AC ln n đường chéo BD. Gi E, F
ln lưt là nh chiếu ca B và D xung đưng thẳng AC.
1) Tứ giác BEDF là hình sao?
2) Gọi CH và CK ln lưt là đường cao ca tam giác ACB và tam giác ACD.Chng minh
rằng.
a. Tam giác CHK và tam giác ABC đồng dạng .
b. AB.AH+AD.AK=AC
2
ĐÁP ÁN
Câu: 1(5đ)
a) ĐK
9;4;0 xxx
0,5đ
Rút gn M =
32
2123392
xx
xxxxx
0,5đ
Biến đổi ta có kết qu: =
32
2
xx
xx
0,5đ
=
3
1
23
21
x
x
xx
xx
b)
)(164
5
3
1
5 M
TMxx
x
x
c) M =
3
4
1
3
43
3
1
xx
x
x
x
0,5đ
Do M
z
nên
3x
là ước ca 4
3x
nhn các giá tr: -4;-2;-1;1;2;4 0,5đ
49;25;16;4;1 x
do
4x
49;25;16;1x
0,5đ
Câu: 2 (2đ)
Phân tích được 4a
2
+b
2
=5ab thành (a-b)(4a-b)=0 0,5đ
<=> a=b hoặc 4a=b 0,5đ
Lập lun chỉ ra a=b (nhn) 4a=b (loại) 0,5đ
Tính đưc
3
1
34
2
2
22
a
a
ba
ab
P
0,5đ
Câu: 3 (4đ)
a. Viết đưc
2
)1(
)2(
2
12
44242
2
2
2
22
x
x
xx
xxxx
A
1,5đ
Lập lun min A = 2 khi x-2= 0 => x= 2 0,5đ
b. biến đổi
cabcabcba
222
<=> 2a
2
+2b
2
+2c
2
≥2ab+2bc+2ca 0,5đ
<=> a
2
-2ab+b
2
+b
2
-2bc +c
2
+c
2
-2ca+a
2
≥0 0,5đ
<=> (a-b)
2
+(b-c)
2
+(c-a)
2
≥ 0 0,5đ
Lập lun => khng định 0,5đ
Câu: 4 (4đ)
a. x
3
+y
3
+z
3
-3xyz
= x
3
+3x
2
y+3xy
2
+y
3
+z
3
-3x
2
y-3xy
2
-3xyz 0,5đ
= (x+y)
3
+z
3
3xyz(x+y+z) 0,5đ
= (x+y+z)(x
2
+2xy+y
2
+z
2
-xz-yz)-3xy(x+y+z) 0,5đ
=(x+y+z)(x
2
+y
2
+z
2
-xy-yz-zx) 0,5đ
b. Gii phương trình : x
4
+2x
3
-4x
2
-5x-6=0
<=> x
4
-2x
3
+4x
3
-8x
2
+4x
2
-8x + 3x-6=0 0,5đ
<=> x
3
(x-2)+4x
2
(x-2)+4x(x-2)+3(x-2)=0 0,5đ
<=> (x-2)(x
3
+4x
2
+4x+3)=0 0,25đ
<=> (x-2)(x
3
+3x
2
+x
2
+3x+x+3) =0 0,25đ
<=> (x-2)[x
2
(x+3)+x(x+3)+(x+3)]=0 0,25đ
<=> (x-2)(x+3)(x
2
+x+1) =0 0,25đ
Câu: 5 (5đ)
1. Chỉ ra Tam giác ABE = Tam giác CDF 0,5đ
=>BE=DF . BE//DF cùng vuông góc với AC 0,2
=> BEDF là hình bình nh 0,25đ
2.a. Chỉ ra góc CBH = góc CDK 0,5đ
=> tam giác CHB đồng dạng vi Tam giác CDK (g,g) 0,25đ
CD
CK
CB
CH
0,25đ
Chỉ ra CB//AD,CK vuông góc CB=> CK vuông góc CB 0,25đ
Chỉ ra góc ABC = góc HCK ( cùng bù vi BAD) 0,25đ
Chỉ ra
CD
CK
CB
CH
hay
AB
CK
CB
CH
AB=CD 0,25đ
Chỉ ra tam giác CHK đồng dạng tam giác BCA (c-g-c) 0,25đ
b. chỉ ra tam giác AFD = tam giác CEB => AF=CE 0,5đ
chỉ ra tam giác AFD đồng dạng vi tam giác AKC 0,25đ
=> AD.AK=AF.AC => AD.AK=CE.AC (1) 0,5đ
Chỉ ra tam giác ABE đồng dạng với tam giác ACH 0,25đ
=> AB.AH=AE.AC (2) 0,25đ
Công theo vế (1) và (2) ta đưc
B
A
F
E
D
K
C
H
AD.AK+ AB.AH =CE.AC+ AE.AC =(CE+AE)AC=AC
2
0,25đ
Lưu ý: Học sinh làmch khác đúng vẫn cho điểm tối đa
PHÒNG GIÁO DC ĐÀO
TẠO HUYN KIM THÀNH
ĐỀ THI CHỌN HC SINH GIỎI HUYN
M HC 2012 2013
n: Toán 9
Thời gian làm bài: 120 phút
Đề gm 01 trang
i 1: (4,0 điểm)
a) Rút gn biểu thức A =
2 9 3 2 1
5 6 2 3
x x x
x x x x

b) Cho x, y, z thoả mãn: xy + yz + xz = 1.
Hãynh giá tr biểu thức: A =
2 2 2 2 2 2
2 2 2
(1 )(1 ) (1 )(1 ) (1 )(1 )
(1 ) (1 ) (1 )
y z z x x y
x y z
x y z


i 2: (3,0 điểm)
a) Cho hàm số : f(x) = (x
3
+ 12x 31)
2012
Tính f(a) tại a =
33
16 8 5 16 8 5
b) Tìm s tự nhiên n sao cho n
2
+ 17 là s chính phương?
i 3: (4,0 điểm)
Giải các pơng trình sau:
a)
1 4 3xx
b)
2
4 5 2 2 3x x x
i 4: (3,0 điểm)
a) m x; y tha mãn:
2 4 4x y y x xy
b) Cho a; b; c là các s thuộc đon
1;2
tha mãn: a
2
+ b
2
+ c
2
= 6 hãy chứng minh
rng:
a + b + c
0
i 5: (6,0 điểm)
Cho tam giác ABC nhn; các đường cao AK; BD; CE cắt nhau tại H.
a) Chứng minh:
2 2 2
2 2 2
KC AC CB BA
KB CB BA AC


b) Gi sử: HK =
1
3
AK. Chứng minh rằng: tanB.tanC = 3
c) Gi sS
ABC
= 120 cm
2
BÂC = 60
0
. Hãynh dinch tam giác ADE?
TRƯNG THCS TỢNG VŨ
Tổ KHTN
NG DN GII ĐỀ THI HSG HUYỆN KIM THÀNH
NĂM HỌC 2012 2013
n: Toán 9
Thời gian: 120
u 1: (4 điểm)
a/ Rút gn biu thức A =
2 9 3 2 1
5 6 2 3
x x x
x x x x

ĐKXĐ: x
4; x
9
A =
2 9 3 2 1 2 9 9 2 3 2 2
23
2 3 2 3 2 3
x x x x x x x x x
xx
x x x x x x

=
12
1
3
23
xx
x
x
xx


b/ Cho x, y, z tho mãn: xy + yz + xz = 1.
Hãynh: A =
2 2 2 2 2 2
2 2 2
(1 )(1 ) (1 )(1 ) (1 )(1 )
(1 ) (1 ) (1 )
y z z x x y
x y z
x y z


Gợi ý: xy + yz + xz = 1
1 + x
2
= xy + yz + xz + x
2
= y(x + z) + x(x + z) = (x +
z)(x + y)
Tương tự: 1 + y
2
= …; 1 + z
2
= ….
u 2: (3 điểm)
a/ Cho hàm số : f(x) = (x
3
+ 12x 31)
2012
Tính f(a) tại a =
33
16 8 5 16 8 5
b/ Tìm s tự nhiên n sao cho n
2
+ 17 là s chính phương?
Giải
a/Từ a=
33
16 8 5 16 8 5
3
33
3
32 3 16 8 5 16 8 5 16 8 5 16 8 5 32 12aa



n a
3
+ 12a = 32
Vy f(a) = 1
b/ Gi sử: n
2
+ 17 = k
2
(k
) và k > n
(k n)(k + n) = 17
1
8
17
kn
n
kn



Vy với n = 8 tha mãn yêu cu bài toán.
u 3: (4 điểm)
Giải các pơng trình sau:
a/
1 4 3xx
b/
2
4 5 2 2 3x x x
Giải
a/ ĐK:
41x
Bình phương 2 vế:
1 4 2 (1 )(4 ) 9 (1 )(4 ) 2x x x x x x
2
0
4 3 4 ( 3) 0
3
x
x x x x
x

(tha mãn)
Vy phương trình có 2 nghiệm: x = 0; x = -3
b/
2
4 5 2 2 3x x x
ĐKXĐ: x
3
2
2
2 1 2 3 2 2 3 1 0x x x x
2
2
10
1 2 3 1 0 1
2 3 1
x
x x x
x


vậy phương tnh có nghim duy
nhất x = -1
u 4: (3 điểm)
a/ Tìm x; y tha mãn:
2 4 4x y y x xy
b/ Cho a; b; c là các số thuc đoạn
1;2
tha mãn: a
2
+ b
2
+ c
2
= 6 hãy chứng minh
rng: a + b + c
0
Giải
a/
2 4 4 .2. 4 .2. 4x y y x xy x y y x xy
Xét VP =
.2. 4 .2. 4x y y x
theo BĐT cosi:
4 4 4 4
2 4 ;2 4
2 2 2 2
y y x x
yx
vậy VP
xy = VT
Du = xảy ra khi:
42
8
42
x
xy
y


b/ Do a; b; c thuc đon
1;2
n a + 1
0; a 2
0 nên (a + 1)(a 2)
0
Hay: a
2
a 2
0
a
2
a + 2
Tương tự: b
2
b + 2; c
2
c + 2
Ta có: a
2
+ b
2
+ c
2
a + b + c + 6 theo đu bài: a
2
+ b
2
+ c
2
= 6 n: a + b + c
0
u 5: (6 điểm)
Cho tam giác ABC nhn; các đường cao AK; BD; CE cắt nhau tại H.
a/ Chứng minh:
2 2 2
2 2 2
KC AC CB BA
KB CB BA AC


b/ Gi sử: HK =
1
3
AK. Chứng minh rằng: tanB.tanC = 3
c/ Gi sS
ABC
= 120 cm
2
C = 60
0
. Hãynh diện ch tam giác ADE?
Giải
a/ S dng đnh pytago:
2 2 2 2 2 2 2
2 2 2 2 2 2
()
( ) ( )
AC CB BA AK KC BK CK AB
CB BA AC BK CK BA AK KC
=
2
2
2 2 . 2 ( )
2 2 . 2 ( )
CK BK CK CK CK BK CK
BK BK CK BK BK CK BK



b/ Ta có: tanB =
AK
BK
; tanC =
AK
CK
Nên: tanBtanC =
2
.
AK
BK CK
(1)
H
E
D
K
C
B
A
Mt khác ta có:
B HKC
mà: tanHKC =
KC
KH
Nên tanB =
KC
KH
ơng tự tanC =
KB
KH
2
.
tan .tan
KB KC
BC
KH

(2)
T (1)(2)
2
2
tan .tan
AK
BC
KH




Theo gt: HK =
1
3
AK
tan .tan 3BC
c/ Ta chứng minh được:
ABC
ADE
đồng dạng vậy:
2
ABC
ADE
S
AB
S AD



(3)
Mà BÂC = 60
0
n
0
30ABD 
AB = 2AD(4)
T (3)(4) ta có:
2
4 30( )
ABC
ADE
ADE
S
S cm
S
SỞ GO DỤC VÀ ĐÀO TẠO K THI CHỌN HC SINH GII TNH
THANH A NĂM HC 2011 - 2012
MÔN: TOÁN
Lớp 9 thcs
Thời gian làm bài 150 phút không kể thời gian phát đề
Ngày thi: 23 tháng 3 năm 2012
Câu I (4đ)
Cho biểu thức P =
1 8 3 1 1 1
:
10
3 1 3 1 1 1
x x x
x
x x x x
æ ö æ ö
- + - +
÷÷
çç
÷÷
+-
çç
÷÷
çç
÷÷
÷÷
çç
-
+ - - - - -
è ø è ø
1) Rút gọn P
2) Tính giá trị của P khi x =
44
223
223
223
223
Câu II (4đ)
Trong cùng một hệ toạ độ, cho đưng thẳng d: y = x – 2 và parabol (P): y = - x
2
. Gọi A và B là
giao điểm của d và (P).
1) Tính độ dài AB.
2) Tìm m đđưng thẳng d’: y =- x = m cắt (P) tại hai điểm C và D sao cho
CD = AB.
Câu III (4đ)
§Ò CHÝNH THøC
1) Gii hệ phương tnh
.
2
1
2
2
2
y
x
y
x
y
x
2) Tìm nghiệm nguyên của phương tnh 2x
6
+ y
2
2 x
3
y = 320
Câu IV (6đ)
Cho tam giác nhọn ABC có AB > AC. Gọi M là trung điểm của BC; H là trực tâm; AD, BE, CF
là các đưng cao của tam giác ABC. Kí hiệu (C
1
) và (C
2
) lần lưt là đường tròn ngoại tiếp tam
giác AEFDKE, vi K là giao điểm của EF và BC. Chứng minh rằng:
1) ME là tiếp tuyến chung của (C
1
) và (C
2
).
2) KH
AM.
Câu V (2đ)
Với
1;;0 zyx
. Tìm tt cả các nghiệm của phương tnh:
zyxyzx
z
xyz
y
zxy
x
3
111
S GIÁO DỤC ĐÀO TẠO
THANH A
KỲ THI CHỌN HỌC SINH GII CẤP TNH LỚP 9
NĂM HỌC 2011-2012
Môn : TOÁN
Ngày thi :18/02/2012
Câu 1:ĐK
1)
3 1 9 1 2 1 4
:.
10
1 1 3
xx
P
x
xx
éù
- + - +
êú
=
êú
-
- - -
êú
ëû
( )
1. 1 3
3( 1 3)
.
10
2 1 4
xx
x
P
x
x
- - -
-+
=
-
-+
3 1( 10)( 1 2) 3( 2)
2(10 )( 1 4) 2( 5)
x x x x
P
x x x
- - - - -
= = -
- - - -
b)
22
44
44
3 2 2 3 2 2
(3 2 2) (3 2 2) 3 2 2 3 2 2
3 2 2 3 2 2
x
+-
= - = + - - = + - -
-+
=> x=
1 2 ( 2 1) 2+ - - =
x>1
Vậy P=0
Câu II:
1) Hoành độ giao điểm là nghiệm phương trình
x
2
+x-2=0
=> x=1 hoặc x=2
Vậy A(1,-1) và B(-2;-4) hoặc A(-2;-4) vàB(1;-1)
2)Để (d’) cắt (P) tại 2 điểm phân biệt thì pơng tnh x
2
-x+m=0 (1)
có hai nghiệm phân bit <=>
0D>
<=>
1
4
m <
Ta có khoảng cách AB
2
=18
để CD = AB <=> (x
1
-x
2
)
2
+(y
1
-y
2
)
2
=18
<=>(x
1
-x
2
)
2
=9
<=>(x
1
+x
2
)
2
-4x
1
x
2
=9
<=>1-4m-9=0=> m=-2(TM)
Vậy C(-1,-3) và D(2;0) hoặc D(-1;-3) hoặc C(2;0
Câu III
1,ĐK x
¹
0, y
¹
0
Đặt x=ky ( k
¹
0)
.
2
1
2
2
2
y
x
y
x
y
x
<=>
2
( ) 2
11
( 1)
2
k k y
y
k
ì
ï
+=
ï
ï
ï
í
ï
+=
ï
ï
ï
î
(1)
Nếu k=-1 thì hệ phương tnh (1) vô nghiệm nên hệ phương trình đã cho vô nghiệm
Nếu k
¹
-1
t (1) =>
2
()
4
1
k k k
k
+
=
+
=> k=2 hoặc k = -2
Nếu k=2 =>
21
( , ) ( ; )
33
xy=
Nếu k = -2 => (x;y)=(-2;1)
2, Từ 2x
6
+ y
2
x
3
y = 320 <=>(x
3
-y)
2
+(x
3
)
2
=320
=> (x
3
)
2
£
320
mà x nguyên nên
2x £
Nếu x=1 hoặc x=-1 thì y không nguyên (loại)
Nếu x=2=> y=-2 hoặc y=6
Nếu x=-2 => y=-6 hoặc y=2
Vậy phương trình đã cho có 4 cặp nghiệm (x;y) là(2;-2);(2;6);(-2;-6);(-2;2)
Câu IV: 1) Ta có
µ
µ
0
90EF==
nên t giác AEHF nội tiếp một đường tròn tâm cnh là
(C
1
) là trung điểm AH
·
¼
1
2
EAH sdEH=
(1)
mà
·
·
EAH CBE=
(2) ( cùng phụ với góc ACD)
·
·
MEB CBE=
(3)( do đương trung tuyến ứng vi cạng huyền)
Từ (1), (2) và (3) ta có
·
¼
1
2
MEH sdEH=
=> ME là tiếp tuyến đưng tròn tâm (C
1
)
B
F
E
K
C
C
D
M
N
A
2, gọi giao điểm AM với KH là N trước tiên chứng minh 5 điểm A,E,H,N,F cùng thuộc một
đưng tròn
Ta thấy
·
·
·
·
·
·
AF ;ANE ACB E AFE ANE ACB= = = > =
=> nghĩa là C,M,N, F cùng thuộc một đưng tròn
chứng minh A,E,N, B nội tiếp
do đó
·
0
90KNM =
KH
AM
Câu V:: do vai trò x,y,z n nhau nên
01x y z£ £ £ £
Nếu x= 0 =>
2
3
11
1 1 1
( ) ( )
11
( 1)( 1 ) 1 1
(1 )( ) (1 )( )
yz
z zy y z
yz
z y z zy y z y z
y y z z
z y z yz y z y z
+=
+ + +
= > - + - =
+ + + + +
- + + -
= > + =
+ + + + +
Ta có VT
³
0 mà VP < 0 nên trong trường hợp này không nghim
Nếu x khác 0
01x y z£ £ £ £
011 xz
<=>
zxzx 1
>0
01
01
zzxx
zxzx
đúng với mi
1;0 zx
.
Dấu=” xảy ra khi: x=z=1.
+ Ta có:
zxzx 1
zyxzxy 1
zyx
x
zxy
x
1
+ Tương tự:
zyx
y
xyz
y
1
zyx
z
yzx
z
1
1
111
zyx
zyx
yzx
z
xyz
y
zxy
x
VT
. (1)
+ Mặt khác, vì:
31;;0 zyxzyx
1
3
33
zyx
VP
Dấu=” xảy ra khi : x=y=z=1. (2)
+ T (1) và (2)
VPVT
chỉ đúng khi:
1VPVT
.
Khí đó x=y=z=1.
* Vy pơng trình có nghim duy nhất:
1;1;1;; zyx
.
S GIÁO DC VÀ ĐÀO TO K THI CHN HC SINH GII VÒNG TNH LP 9 THCS
TNH KIÊN GIANG NĂM HC 2012-2013
HƯỚNG DN CHM MÔN TOÁN 9
Câu Đá
p
án Đim
Câu 1a
(1,25đ)
- Hàm s y = (m
2
– 2m)x + m
2
– 1 nghch biến
m
2
– 2m < 0
m(m – 2) < 0
>
<
<
>
>
<
<
>
)(
2
0
2
0
02
0
02
0
loai
m
m
m
m
m
m
m
m
0 < m < 2 (1)
- Ct trc tung :
m
2
– 1 = 3
m =
2
±
(2)
T (1) và (2)
m
0,25
0,25
0,25
0,5
Câu 1b
(1,5đ)
Tìm giá tr nh nht ca :
M = 5x
2
+ y
2
+ z
2
- z – 4x – 2xy – 1
M = x
2
- 2xy + y
2
+ 4x
2
– 4x + 1 + z
2
- z +
4
9
4
1
= (x – y)
2
+ (2x – 1)
2
+
2
2
1
z
4
9
-
4
9
Giá tr nh nht ca M =
4
9
2
1
0
2
1
012
0
===
=
=
=
zyx
z
x
yx
0,25
0,5
0,25
0,5
Câu 1c
(1,25đ)
Cho x + y = - 5 và x
2
+ y
2
= 11. Tính x
3
+ y
3
Ta có : x
3
+ y
3
= (x+y)(x
2
+ y
2
– xy) = -5(11 – xy) (1)
Mà x + y = -5
x
2
+ y
2
+2xy = 25
11 + 2xy = 25
xy = 7 (2)
T (1) và (2)
x
3
+ y
3
= -5(11- 7) = -20
0,25
0,5
0,5
Câu 2a
(2,0đ)
Rút gn : A =
()
x
x
xxxx
xxxx
+
++
+++
3
2
1.2:
923
965
22
22
ĐK : -3 < x < 3
A =
()()
()()
x
x
x
x
xxxxx
xxxxx
+
+++
++++
3
2
3
3
2:
3.323
3.323
=
()
[
]
()
[]
x
x
xxxxx
xxxxx
+
+++
++++
3
3
2:
.3233
3323
=
x
x
x
x
+
+
3
3
2:
3
3
=
2
1
0,25
0,5
0,5
0,5
0,25
Câu 2b
(2,0đ)
Cho a, b c tha mãn :
cbacba ++
=++
1111
Tính giá tr biu thc Q = (a
27
+ b
27
)(b
41
+ c
41
)(c
2013
+
a
2013
)
Ta có :
cbacba
+
+
=++
1111
ccbaba
1111
+
+
=+
()
()
cbac
ba
ab
ba
++
+
=
+
(a+b)c(a+b+c) = -ab(a+b)
(a+b)[c(a+b+c) +ab] = 0
(a+b)[c(a+c)+bc +ab] = 0
(a+b)[c(a+c) +b(a+c)] = 0
(a+b)(a+c)(b+c) = 0
=
=
=
=+
=+
=+
ac
cb
ba
ac
cb
ba
0
0
0
- Thế vào tính được Q = 0
0,25
0,25
0,5
0,25
0,75
Câu 3a
(2,0đ)
Gii phương trình :
31710
33
=++ xx
()
3
3
33
31710 =++ xx
x + 10 + 17 – x + 3.
3
)17)(10( xx +
.3 = 27
(x+10)(17 – x) = 0
x = -10 , x = 17
0,5
0,5
0,5
0,5
Câu 3b
2,0đ
Gii h phương trình :
=+
=
+
+
+
1923
2
32
5
5
32
yx
x
y
y
x
(vi
5,
2
3
>> yx )
Đặt
0
5
32
>=
+
m
y
x
m +
2
1
=
m
(
)
101012
2
2
===+ mmmm
(nhn)
825321
5
32
=+==
+
yxyx
y
x
Gii h
=
=
=+
=
=+
=
2
5
1923
1624
1923
82
y
x
yx
yx
yx
yx
0,5
0,5
0,5
0,5
Câu 4
(4,0đ)
Câu a
(1,0đ)
(1,25đ)
Câu b
(1,25đ)
a) Chng minh : KD = CI và EF//AB.
– Cminh ABID, ABCK là hình bình hành
DI = CK (cùng bng AB)
DI + IK = CK + IK
DK = CI
- C/m : ΔAEB đồng dng Δ KED (g.g)
KD
AB
EK
AE
=
Δ AFB đồng dng Δ CFI (g.g)
A
FAB
FC CI
=
Mà KD = CI (cmtrên)
AE AF
EF / /KC
EK FC
=⇒
(Đlí Talet đảo trong Δ AKC)
b) Chng minh AB
2
= CD. EF.
Ta có : Δ KED đồng dng ΔAEB (cmtrên)
EB
DE
AB
DK
=
EB
EBDE
A
B
ABDK
+
=
+
EB
DB
AB
KCDK
=
+
EB
DB
AB
DC
=
(1)
Do EF//DI (theo CMT: EF//KC, I
KC)
Hình 0,5đ
0,5
0,25
0,25
0,5
0,25
0,5
0,5
0,25
DB DI DB AB
EB EF EB EF
=⇒ =
(2) (Vì DI = AB)
T (1) và (2)
EFDCAB
EF
AB
AB
DC
.
2
==
0,5
Câu 5
4,0đ
Câu a
(1,75đ)
Câu b
(075đ)
Câu c
(1,0đ)
a) Chng minh MC + MB = MA ?
- Trên MA ly D sao cho MD = MB
ΔMBD cân ti M
góc BMD = góc BCA = 60
0
(cùng chn cung AB)
ΔMBD đều
- Xét ΔMBC và ΔDBA
Ta có :
MB = BD (vì ΔMBD đều)
BC = AB (vì ΔABC đều)
Góc MBC = góc DBA (cùng cng góc DBC bng 60
0)
ΔMBC = ΔDBA (c-g-c)
MC = DA
Mà MB = MD (gt)
MC + MB = MA
b) Xác định v trí ca đim M để tng MA + MB +
MC đạt giá tr ln nht.
Ta có : MA là dây cung ca (O;R)
MA
2R
MA + MB + MC
4R (không đổi)
Du “ = “ xy ra
MA là đường kính
M là đim chính gia ca cung BC
c) CMR : MH + MK + MQ =
(
)
23 2'
3
SS
R
+
Ta có
MACMBCMAB
SSS
ACMQBCMKABMH
++=++
2
.
2
.
2
.
AB.(MH + MK + MQ ) = 2 (S + 2S’)
Tính hoc nói AB là cnh tam giác đều ni tiếp (O;R)
Hình 0,5đ
0,25
0,5
0,5
0,5
0,25
0,25
0,25
0,25
0,25
xO
A
D
C
B
M
H
Q
K
Lưu ý : Hc sinh gii cách khác đúng cho trn s đim
AB = R
3
MH + MK + MQ =
(
)
R
SS
3
'232 +
0,25
0,25
Họ tên TS:........................................... SốBD:...................... Chữ ký GT1:..................
SỞ GIÁO DỤC VÀ ĐÀO TẠO
NINH THUẬN
ề thi chính thức)
KỲ THI CHỌN HỌC SINH GIỎI CẤP TỈNH
NĂM HỌC 2012 2013
Khóa ngày: 18 / 11 / 2012
Môn thi: TOÁN - Cấp: THCS
Thời gian làm bài: 150 phút
(Không kể thời gian phát đề)
ĐỀ:
ề thi có 01 trang)
Bài 1 (5,0 điểm):
Tim tt cả c cp s thc x y tha n bất đng thc sau:
2 2
2 2 2 0 x y x y
Bài 2 (4,0 điểm):
m tt cả c nghiệm nguyên dương của phương trình:
1 1 1
7
x y
Bài 3 (5,0 điểm):
Cho nh thang ABCD (AD // BC). Hai đường phân giác trong của c A
c B ct nhau tại đim E, hai đường phân giác trong của c C c D ct
nhau tại đim F.
a) Chng minh rng: EF // AD.
b) nh độ dài đoạn EF thông qua c cạnh của nh thang ABCD.
Bài 4 (3,0 điểm):
Cho s thc A = 2 + 2
2
28 1n
, vi n nguyên. Chng minh rng nếu A
s nguyên thì A mt s chính phương (bng nh phương của mt s
nguyên).
Bài 5 (3,0 điểm):
Trong hình vuông độ i cạnh bng 1 cho 151 đim bt k. Chng minh
rng ít nht 7 điểm đã cho nm trong mt nh tròn có bán kính bng
1
7
.
------- HT -------
SỞ GIÁO DỤC VÀ ĐÀO TẠO
QUẢNG NINH
ĐỀ THI CHÍNH THỨC
KỲ THI CHỌN HỌC SINH GIỎI CẤP TỈNH
LỚP 9 NĂM HỌC 2012 – 2013
MÔN: TOÁN
(Bảng A)
Ngày thi: 20/3/2013
Thời gian làm bài: 150 phút
(không kể thời gian giao đề)
Họ và tên, chữ ký
của giám thị số 1:
...............................
...............................
(Đề thi này có 01 trang)
Bài 1. (4,5 điểm)
a) Chứng minh đẳng thức:
3
3
3 3 3
1 2 4
2 1
9 9 9
= +
.
b) Giải hệ phương trình :
2
2
(2013 2012) 1
( 2012) 2013
x y
x y
=
+ =
.
Bài 2. (3,5 điểm)
Cho hàm số bậc nhất y = mx + m - 1
(*)
(với m là tham số).
a) Tìm các giá trị của m để đồ thị của hàm số
(*)
tạo với các trục tọa độ Oxy một
tam giác có diện tích bằng 2.
b) Chứng minh rằng đồ thị của hàm số
(*)
luôn đi qua một điểm cố định với mọi
giá trị của m.
Bài 3. (4,0 điểm)
Cho x, y, z là ba số thực dương thoả mãn xyz = 1.
Tìm giá trị lớn nhất của biểu thức
1xz
1
1zy
1
1yx
1
A
333333
++
+
++
+
++
=
.
Bài 4. (6,0 điểm)
Cho tam giác ABC ba góc nhọn nội tiếp đường tròn m O. Gọi I một điểm
trên cung nhỏ AB (I không trùng với A và B). Gọi M, N, P theo thứ thình chiếu của
điểm I trên các đường thẳng BC, AC, AB.
a) Chứng minh rằng ba điểm M, N, P thẳng hàng.
b) Xác định vị trí của điểm I để đoạn thẳng MN có độ dài lớn nhất.
Bài 5. (2,0 điểm)
Giải phương trình sau: (x+3)
28xx)x)(12(4 =++
.
.......................Hết.....................
Họ và tên thí sinh:.............................................................Số báo danh:...............
SỞ GIÁO DỤC VÀ ĐÀO TẠO
QUẢNG NINH
ĐỀ THI CHÍNH THỨC
KỲ THI CHỌN HỌC SINH GIỎI CẤP TỈNH
LỚP 9 NĂM HỌC 2012 – 2013
MÔN: TOÁN
(Bảng B)
Ngày thi: 20/3/2013
Thời gian làm bài: 150 phút
(không kể thời gian giao đề)
Họ và tên, chữ ký
của giám thị số 1:
..............................
...............................
(Đề thi này có 01 trang)
Câu 1. (4,0 điểm)
Cho biểu thức P
=
(
)
2 2 1
1 1
:
1
x x
x x x x
x
x x x x
+
+
+
với
>0; 1.
x x
a) Rút gọn biểu thức P.
b) Tìm x nguyên để P nhận giá trị nguyên.
Câu 2. (4,0 điểm)
Cho ba số thực a, b, c thỏa mãn đồng thời:
2 2 2
a + b + c = 6
a + b + c = 12
.
Tính giá trị của biểu thức P =
2013 2013 2013
(a - 3) + (b - 3) + (c - 3)
.
Câu 3. (4,0 điểm)
Giải phương trình:
2 2
2( 4 ) 4 5 13 0
x x x x
+ =
.
Câu 4
. (6,0 điểm)
Cho đường tròn (O) BC một y cung không đi qua m O. Điểm A bất
nằm trên cung lớn BC của đường tròn (O) sao cho điểm O luôn nằm trong tam giác
ABC (A
B; C). Các đường cao AD, BE, CF cắt nhau tại H.
a) Chứng minh tứ giác BFEC nội tiếp.
b) Đường cao AD cắt đường tròn (O) tại I. Chứng minh I đối xứng với H qua BC.
c) Gọi M là trung điểm của BC. Chứng minh AH = 2OM.
Câu 5.
(2,0 điểm)
Cho ba số thực dương x, y, z thỏa mãn
1 1 1
2.
1 1 1x y z
+ +
+ + +
m giá trị lớn nhất của biểu thức P = xyz.
-----------------Hết----------------
Họ và tên thí sinh :……………………………………………..Số báo danh :………..
.
1
`SỞ GD&ĐT QUẢNG NINH
ĐỀ THI CHÍNH THỨC
HƯỚNG DẪN CHẤM THI CHỌN HỌC SINH GIỎI CẤP TỈNH
LỚP 9 NĂM HỌC 2012 – 2013
Môn: TOÁN (BẢNG A)
(Hướng dẫn chấm này có 04 trang)
Bài Sơ lược bài giải Điểm
Bài 1
4,5đ
Câu a
2,5
điểm
Đặt
3
3
2 2
a a
= =
.
Đẳng thức cần chứng minh tương đương với:
3
2
3
9
1
1
aa
a
+
=
0,5
).1(9)1(1)1(9
322
3
=++= aaaaaa
0,5
Biến đổi vế trái:
2 3 2 2 2
2 2 2
3
( 1) ( 1) ( 1)
3( 1)( 1) 3( 1)( 1)( 1)
3( 1)( 1) 3( 1)(2 1) 9( 1)
a a a a a a
a a a a a a a
a a a a
+ = + +
= + = + +
= + = + =
Vậy đẳng thức được chứng minh.
1,5
Câu b
2,0
điểm
2. ta thấy
0
x
=
không là nghiệm. hệ phương trình tương đương
với:
2
2
1
2013 2012
2013
2012
y
x
y
x
=
+ =
(*)
0,5
Đặt:
1
t
x
=
, hệ (*)
2
2 2
2
2013 2012 0
2013 2013
2013 2012 0
t y
t y y t
y t
+ =
=
+ =
( )( 2013) 0
2013
y t
t y t y
y t
=
+ + =
=
0,5
* Trường hợp
2
2013 2012 0,
y t t t
= + =
Giải PT được :
1 2
1; 2012
t t
= =
0,5
* Trường hợp
2 2
2013 2013 2013 2012 0
y t t t
= + + + =
, PT vô
nghiệm
Vậy hệ có nghiêm (
1 1 2 2
1
( 1; 1);( ; 2012)
2012
x y x y= = = =
0,5
Bài 2
3,5đ
Câu a
2,0
điểm
Vì (*) là hàm số bậc nhất nên m
0
. (1)
Điều kiện để đồ thị của (*) tạo với các trục tọa độ Oxy một tam
giác là m
1. (2)
0,25
0,25
Gọi A là giao điểm của đường thẳng (*) với trục tung
A(0; m-1) nên độ dài OA = | m - 1|.
Gọi B là giao điểm của đường thẳng (*) với trục hoành
B(
m
m1
; 0) nên độ dài OB = |
m
m1
|.
0,25
0,25
2
Câu b
1,5
Điểm
S
ABC
= 2
2
1
OA.OB = 2
OA.OB = 4.
(m - 1)
2
= 4|m|
0,25
*Với m > 0 thì m
2
- 2m + 1 = 4m
m
2
- 6m + 1 = 0
m
1
= 3 – 2
2
; m
2
= 3 + 2
2
.
*Với m < 0 thì m
2
- 2m + 1 = - 4m
m
2
+ 2m +1 = 0
m = -1
Vậy m
{ -1; 3 - 2
2
; 3 + 2
2
} thỏa mãn điều kiện (1) và (2).
0,25
0,25
0,25
Gọi M(x
0
; y
0
) là điểm cố định thuộc đồ thị (*) khi và chỉ khi:
y
0
= mx
0
+ m – 1
m
R
(x
0
+ 1)m – (y
0
+ 1) = 0
m
R
0 0
0 0
1 0 1
1 0 1
x x
y y
+ = =
+ = =
Vậy đồ thị của (*) luôn đi qua một điểm
cố định M(-1; -1)
m
R
0,75
0,75
Bài 3
4
điểm
Ta có (x - y)
2
0
với
x, y
R
x
2
- xy + y
2
xy.
0,5
Mà x; y > 0 nên x + y > 0.
0,5
Mà x
3
+ y
3
= (x + y)(x
2
- xy + y
2
)
(x + y)xy.
0,5
x
3
+ y
3
+1 = x
3
+ y
3
+ xyz
(x + y)xy + xyz.
x
3
+ y
3
+1
xy(x + y + z) > 0.
0,5
Tương tự chứng minh được:y
3
+ z
3
+1
yz(x + y + z) > 0.
z
3
+ x
3
+1
zx(x + y + z) > 0.
0,5
A
z)yxz(x
1
z)yyz(x
1
z)yxy(x
1
++
+
++
+
++
0,5
A
z)yxyz(x
zyx
++
+
+
xyz
1
=
A
1
.
0,5
Vậy giá trị lớn nhất của A là 1 khi x = y = z = 1.
0,5
Bài 4
Câu a
3 điểm
Từ giả thiết ta có:
IPA +
INA = 180
0
tứ giác IPAN nội tiếp
IPN = IAN ( cùng chắn cung IN) (1)
0,75
Lại có
IPB =
IMB = 90
0
tứ giác IPMB là tứ giác nội tiếp
MPI + IBM = 180
0
(2)
0,75
Vì I
(O)
CAI +
IBM = 180
0
(3) 0,5
Từ (2) và (3)
MPI =
CAI (4) 0,5
Từ (4) và (1)
MPI +
IPN =
CAI +
IAN = 180
0
Suy ra M, P, N thẳng hàng.
0,5
Câu b
3 điểm
Tứ giác IPMB là tứ giác nội tiếp nên
IBA =
IMN
( cùng chắn cung IP) (5)
0,5
3
Tứ giác INAP là tứ giác nội tiếp nên
INM =
IAB
( cùng chắn cung IP) (6)
0,5
Từ (5) và (6)
tam giác IMN đồng dạng với tam giác IBA
0,5
ABMN1
IA
IN
IB
IM
BA
MN
==
0,5
Dấu “ =’’xy ra
AN
BM
IAC = IBC = 90
0
CI là đường kính của (O).
0,5
Vậy MN lớn nhất bằng AB
I đối xứng với C qua O.
0,5
Bài 5
2 điểm
(x+3).
28xx)x)(12(4 =++
(*)
Điều kiện xác định: - 12
x
4
0,25
Đặt x + 3 = u;
vx)x)(12(4
=+
0,25
u
2
+ v
2
= x
2
+ 6x + 9 + 48 - 8x – x
2
= 57 - 2x
u
2
+ v
2
- 1 = 2(28 - x) (1)
0,25
Theo đề bài ta có uv = 28 - x (2)
0,25
Từ (1) và (2) ta có u
2
+ v
2
- 1 = 2uv
(u - v)
2
= 1
=
=
1vu
1vu
=
+=
1vu
1vu
0,5
i) Với u = v +1
2xx)x)(12(4 +=+
(điều kiện: x
2
)
Giải phương trình được x = - 3 +
31
( thỏa mãn).
0,25
ii) Với u = v - 1
4xx)x)(12(4 +=+
(điều kiện: x
4
)
Giải phương trình được x = - 4 + 4
2
( thỏa mãn)
=> S = {-4 +4
2
; -3 +
31
}.
0,25
4
p
n
m
o
B
A
C
i
Hình vẽ bài 4
Các lưu ý khi chấm:
1. Hướng dẫn chấm này chỉ trình bày lược một cách giải. Bài làm của học sinh
phải chi tiết, lập luận chặt chẽ, tính toán chính xác mới được điểm tối đa.
2. Các cách giải khác nếu đúng vẫn cho điểm. Tổ chấm thống nhất cho điểm
thành phần của câu nhưng không vượt quá số điểm của câu hoặc phần đó.
3. Bài 4 không vẽ hình không cho điểm cả bài. Bài 4 câu b tìm được vị trí điểm
I không chứng minh không cho điểm.
4. Mọi vấn đề phát sinh trong quá trình chấm phải được trao đổi trong tổ chấm
và chỉ cho điểm theo sự thống nhất của cả tổ.
5. Điểm toàn bài là tổng số điểm đã chấm. Không làm tròn.
.......................Hết.....................
1
`SỞ GD&ĐT QUẢNG NINH
ĐỀ THI CHÍNH THỨC
HƯỚNG DẪN CHẤM THI CHỌN HỌC SINH GIỎI CẤP TỈNH
LỚP 9 NĂM HỌC 2012 – 2013
Môn: TOÁN (BẢNG B)
(Hướng dẫn chấm này có 03 trang)
Câu Tóm tắt lời giải
Cho
điểm
Câu 1
(4điểm )
a, P =
+
+
++
++
)1)(1(
)1(2
:
)1(
)1)(1(
)1(
)1)(1(
2
xx
x
xx
xxx
xx
xxx
0,5
=
+
+
++
1
)1(2
:
11
x
x
x
xx
x
xx
0,5
=
)1(2
1
.
11
++++
x
x
x
xxxx
0,5
=
)1(2
1
.
2
+
x
x
x
x
1
1
+
=
x
x
.
0,5
b, P =
1
2
1
1
1
+=
+
xx
x
0,5
Để P nhận giá trị nguyên thì
1x
Ư(2).
0,25
*
4211 === xxx
*
9321
===
xxx
0,5
*
0011
===
xxx
(loại).
*
121
==
xx
(loại).
0,5
Vậy x nhận các giá trị nguyên 4 ; 9 thì P nhận các giá trị nguyên lần
lượt là 3; 2.
0,25
Câu 2
( 4điểm)
2 2 2
6
12
a b c
a b c
+ + =
+ + =
2 2 2
4 4 4 24
12
a b c
a b c
+ + =
+ + =
.
1,0
Từ hai phương trình ta suy ra:
2 2 2
4 4 4 12 0
a b c a b c
+ + + =
. 0,75
2 2 2
( 2) ( 2) ( 2) 0
a b c
+ + =
2
( 2) 0
a
;
2
( 2) 0
b
;
2
( 2) 0
c
với
mọi số thực a, b, c.
1,0
2
2
2
( 2) 0
( 2) 0
( 2) 0
a
b
c
=
=
=
( 2) 0
( 2) 0
( 2) 0
a
b
c
=
=
=
2
2
2
a
b
c
=
=
=
.
0,75
Vậy P =
2013 2013 2013
( 3) ( 3) ( 3)
a b c + +
=
2013 2013 2013
( 1) ( 1) ( 1) 3
+ + =
.
0,5
Câu 3
(4điểm)
2 2
2 2
2( 4 ) 4 5 13 0
2( 4 5) 4 5 3 0
x x x x
x x x x
+ =
+ =
1,0
2
Điều kiện
2
4 5 0
x x
1
x
hoặc
5
x
(*)
0,25
Đặt :
2
4 5
t x x
=
; ( t
0)
2 2
4 5
t x x
=
0,75
Phương trình đã cho trở thành:
2
2 3 0 ( 1)(2 3) 0
t t t t
+ = + =
t = 1 hoặc t = -
2
3
(loại).
1,0
Với t = 1 ta có :
2
4 5 1
x x
=
2
4 6 0
2 10
x x
x
=
= ±
0,75
Vậy phương trình có nghiệm là
2 10
x = ± ( thỏa mãn điều kiện (*)).
0,25
Câu 4
(6 điểm)
a
, Có
BFC=90
0
(vì CF là đường cao của tam giác ABC)
BEC = 90
0
(vì BE là đường cao của tam giác ABC)
1,0
Như vậy từ hai đỉnh F và E cùng nhìn cạnh BC dưới một góc vuông
Suy ra hai điểm E và F cùng nằm trên đường tròn đường kính BC
0,75
Vậy tứ giác BFEC nội tiếp.
0,25
b
, Tứ giác ABDE nội tiếp do có
BDA =
BEA = 90
0
DBE =
DAE (hai góc nội tiếp cùng chắn cung DE) hay
CBE=
IAC (1)
IBC=
IAC ( góc nội tiếp chắn cung IC) (2)
Từ (1) và (2)
CBE =
IBC
BC là tia phân giác góc IBH .
1,0
Ta lại có BC
HD nên tam giác IBH cân tại B
Suy ra BC cũng là trung trực của HI
Vậy I và H đối xứng nhau qua BC
1,0
c,
Kẻ đường kính AK suy ra : KB // CH ( cùng vuông góc với AB )
KC//BH ( cùng vuông góc với AC )
tứ giác BHCK là hình bình hành và M là giao điểm hai đường chéo.
1,0
M là trung điểm của HK 0,5
OM là đường trung bình của tam giác AHK
AH = 2OM ( đpcm) 0,5
Câu 5
(2 điểm)
1 1 1
(1 ) (1 ) 2
1 1 1 ) 1 1 (1 )(1 )
y z yz
x y z y z y z
+ = +
+ + + + + + +
(1)
0,75
Tương tự :
1
2
1 (1 )(1 )
zx
y x z
+ + +
(2)
1
2
1 (1 )(1 )
xy
z x y
+ + +
(3)
0,5
3
Nhân ba bất đẳng thức cùng chiều (1), (2), (3) với nhau ta được
xyz
1
8
0,5
Suy ra giá trị lớn nhất của P =
1
8
khi x = y = z =
1
2
.
0,25
H
O
M
K
I
D
F
E
C
B
A
Hình vẽ bài 4
Các chú ý khi chấm
1.
Hướng dẫn chấm này chỉ trình bày sơ lược một cách cách giải. Bài làm của học
sinh phải chi tiết, lập luận chặt chẽ, tính toán chính xác mới được điểm tối đa.
2.
Các cách giải khác nếu đúng vẫn cho điểm. Tổ chấm trao đổi và thống nhất điểm
chi tiết nhưng không vượt quá số điểm dành cho câu hoặc phần đó.
3.
Với bài 4 không cho điểm nếu không có hình vẽ.Có thể chia nhỏ điểm thành
phần nhưng không dưới 0,25 điểm và phải thống nhất trong tổ chấm.
………………… Hết ……………….
S GIÁO DC VÀ ĐÀO TO
THANH HOÁ
Đề chính thc
S báo danh
K THI CHN HC SINH GII TNH
Năm hc 2010- 2011
Môn thi: Toán
Lp: 9 THCS
Thi gian: 150 phút (không k thi gian giao đề)
Ngày thi: 24/03/2011
(Đề thi có 01 trang, gm 05 câu).
Câu I. (5,0 đim).
1) Cho phương trình:
2
2210.xmxm−+=
Chng minh phương trình luôn có hai nghim
12
,
x
x vi mi m. Tìm giá tr ln nht ca biu thc
12
22
12 12
23
2(1 )
xx
P
x
xxx
+
=
++ +
khi m thay đổi.
2) (a). Cho ba s hu t a, b, c tho mãn
111
.
abc
+
=
Chng minh rng
222
A
abc=++
là s hu t.
(b). Cho ba s hu t
,,
x
yz
đôi mt phân bit. Chng minh rng:
222
111
()()()
B
x
yyzzx
=++
−−
là s hu t.
Câu II. (5,0 đim).1) Gii phương trình:
22
10
.
119
xx
xx
⎛⎞⎛⎞
+=
⎜⎟⎜⎟
−+
⎝⎠⎝⎠
2) Gii h phương trình:
2
2
3
23
11
14
1
4.
xx
yy
xx
x
yyy
⎛⎞
+
++=
⎜⎟
⎝⎠
+
++=
Câu III. (2,0 đim). Cho tam giác đều ABC, các đim D, E ln lượt thuc các cnh AC, AB,
sao cho BD, CE ct nhau ti P và din tích t giác ADPE bng din tích tam giác BPC.
Tính
.BPE
Câu IV. (4,0 đim). Cho đường tròn tâm O và dây cung AB c định (OAB
). P là đim di động
trên đon thng AB (
,PAB và P khác trung đim AB). Đường tròn tâm C đi qua đim
P tiếp xúc vi đường tròn (O) ti A. Đường tròn tâm D đi qua đim P tiếp xúc vi đường
tròn (O) ti B. Hai đường tròn (C) và (D) ct nhau ti N (
NP
).
1) Chng minh rng
ANP BNP= bn đim O, D, C, N cùng nm trên mt đường tròn.
2) Chng minh rng đường trung trc ca đon ON luôn đi qua đim c định khi P di động.
Câu V. (4,0 đim).
1)
Cho
12 45
, ,....,aa a 45 s t nhiên dương thon
12 45
.... 130.aa a
<
<< Đặt
1
, ( 1,2,...,44).
jj j
da a j
+
=− = Chng minh rng ít nht mt trong 44 hiu
j
d xut hin ít
nht 10 ln.
2)
Cho ba s dương ,,abctho mãn:
22 22 22
2011.ab bc ca++ ++ +=
Chng minh rng:
222
1 2011
.
22
abc
bc ca ab
++
++ +
............................................................. HT ........................................................
Thí sinh không được s dng tài liu.
Cán b coi thi không gii thích gì thêm
.
S GD & ĐT THANH HOÁ
HƯỚNG DN CHM
ĐỀ CHÍNH THC
(Gm có 3 trang)
K THI CHN HC SINH GII TNH
NĂM HC 2010 - 2011
MÔN THI: TOÁN
LP: 9 THCS
Ngày thi: 24 - 3 - 2011
Câu Ý
Hướng dn chm
Đim
Ta có
2
'( 1) 0,mmΔ=
nên phương trình có hai nghim vi mi m.
0,5
Theo định lí viet, ta có
12 12
2, 2 1xx mxx m
+
==
, suy ra
2
41
42
m
P
m
+
=
+
1,0
1)
2,5đ
2
2
(2 1)
11.1,
42
m
Max P
m
=− =
+
khi
1
.
2
m
=
1,0
T gi thiết suy ra 2220ab bc ca
−=
0,5
2a)
1,5đ
Suy ra
2
()Aabcabc=+=+
là s hu t
1,0
Đặt
111
,,abc
x
yyzxz
===
−−
suy ra
111
.
abc
+
=
0,5
Câu I
6 đ
2b)
1,0đ
Áp dng câu 2a) suy ra
222
111
()()()
B
x
yyzzx
=++
−−
là s hu t.
0,5
Đk: 1.
x
≠± Phương trình tương đương vi
2
2
222
222
10 2 2 10
20.
11 19 1 19
xx x x x
xx x x x
⎛⎞
⎛⎞
+− = =
⎜⎟
⎜⎟
+−
⎝⎠
⎝⎠
1,0
Đặt
2
2
2
,
1
x
t
x
=
ta được phương trình
2
10 5
0
93
tt t
−== hoc
2
3
t
=
0,5
Vi
5
,
3
t = ta được
2
2
25
13
x
x
=
(vô nghim)
0,5
1)
2,5đ
Vi
2
,
3
t =−
ta được
2
2
22
13
x
x
=
suy ra
1
.
2
x
=
±
0,5
Đk:
0.y
H tương đương vi
2
2
3
3
11
4
11
4.
xx
yy
x
xx
yy y
+++=
⎛⎞
+
++=
⎜⎟
⎝⎠
0,5
Câu II
6 đ
2)
2,5đ
Đặt
1
,
ux
y
x
v
y
=+
=
ta được h
22
32
24 440 2
1.
24 42
uuv u u u
v
uuv uu v
⎧⎧
+
−= −+= =
⎪⎪
⇔⇔
⎨⎨
=
−= +=
⎪⎪
⎩⎩
1,0
Vi
2
1,
u
v
=
=
ta được
1
2
1
1.
1
x
x
y
xy
y
+=
=
⎨⎨
=
=
(tho mãn điu kin)
1,0
K EF AC ti F,
D
GBC
ti G.
Theo gi thiết
()()
A
DPE BPC
SS
=
() ()
.
ACE BCD
SS⇒=
0,5
A
CBC EFDG=⇒=
AC
=
Suy ra
.
A
EF CDG AE CGΔ=Δ =
0,5
Do đó
()AEC CDB c g c DBC ECAΔ=Δ =
0,5
Câu
III
2đ
0
60BPE PBC PCB PCD PCB⇒=+=+=
0,5
1,0
Gi Q là giao đim ca các tiếp tuyến
chung ca (O) vi (C), (D) ti A, B
tương ng.
Suy ra

.ANP QAP QBP BNP===
Ta có


ANB ANP BNP QAP QBP=+=+
0
180 AQB=− , suy ra NAQB ni tiếp (1).
D thy t giác OAQB ni tiếp (2)
T (1) và (2) suy ra 5 đim O, N, A, Q, B
cùng nm trên mt đường tròn.
0,5
0,5
Suy ra các đim O, N, A, B cùng nm tn
mt đường tròn.
0,5
1)
3,0đ
Ta có

22OCN OAN OBN ODN===
,
suy ra bn đim O, D, C, N cùng nm
trên mt đường tròn.
0,5
Câu
IV
4,0đ
2)
1,0đ
Gi E là trung đim OQ, suy ra E c định và E là tâm đường tròn đi qua
các đim N, O, D, C. Suy ra đường trung trc ca ON luôn đi qua đim E c
định.
1,0
1 2 44 2 1 3 2 45 44 45 1
... ( ) ( ) ... ( ) 130 1 129.dd d aa aa a a a a+++ = + ++ = = (1)
0,5
1)
2,0
đ
Nếu mi hiu
( 1,2,....,44)
j
dj
=
xut hin không quá 10 ln thì
12 44
... 9(1 2 3 4) 8.5 130dd d+++ ++++ = u thun vi (1).
Vy phi có ít nht mt h ( 1,...,44)
j
dj
=
xut hin không ít hơn 10 ln
1,5
Câu V
2đ
2)
2,0đ
Ta có
22 2
2( ) ( )ab ab+≥+.
0,5
A
O
N
C
D
B
P
Q
E
H
GHI CHÚ: Nếu hc sinh gii cách khác mà đúng thì vn cho đim ti đa.
Suy ra
()()()
222 2 2 2
22 2 2 2 2
222
abc a b c
bc ca ab
bc ca ca
++ + +
++ +
+++
Đặt
22 22 22
,,,
x
bcy caz ab=+ =+ =+
suy ra
222 22 2 2 22
22 22 22
yzx zxy xyz
VT
x
yz
+− + +
≥++
22 2
1( ) () ( )
22 2
22
yz zx xy
x
yz
xyz
⎡⎤
⎛⎞⎛⎞⎛⎞
+++
≥−++
⎢⎥
⎜⎟⎜⎟⎜⎟
⎝⎠⎝⎠⎝⎠
⎣⎦
1,0
22 2
1( ) () ( )
23 23 23
22 2
22
yz zx xy
x
xyyzz
xy z
⎡⎤
⎛⎞
++ +
≥+++++
⎢⎥
⎜⎟
⎝⎠
⎣⎦
()()()
1
2( ) 3 2( ) 3 2( 3
22
yz x zx y xy z≥+++++
⎡⎤
⎣⎦
Suy ra
1 1 2011
()
22
22
VT x y z≥++=
0,5
S GIÁO DC VÀ ĐÀO TO
THANH HÓA
ĐỀ THI CHÍNH THC
KÌ THI CHN HC SINH GII TNH
Năm hc: 2011-2012
Môn thi: TOÁN
Lp 9 THCS
Ngày thi: 23 tháng 3 năm 2012
Thi gian: 150 phút (không k thi gian giao đề)
Đề này có 01 trang, gm 05 câu.
Câu I (4,0 đim)
Cho biu thc
183111
:
10
31 311 1
xx x
P
x
xxxx
⎛⎞
−+ +
=+
⎜⎟
+−
⎝⎠
.
1) Rút gn
P
.
2) Tính giá tr ca P khi
44
322 322
322 322
x
+−
=−
−+
.
Câu II (4,0 đim)
Trong cùng mt h to độ, cho đường thng
:2dy x
=
và parabol
2
():
P
yx
=
. Gi
A
B là giao đim ca
d ()
P
.
1) Tính độ dài
A
B .
2) Tìm
m để đường thng ':dy xm
=
−+ ct ()
P
ti hai đim C
D
sao cho
CD AB= .
Câu III (4,0 đim)
1) Gii h phương trình
2
2
2
1
.
2
x
x
y
y
y
x
+
=
+
=
2) Tìm nghim nguyên ca phương trình
62 3
2 2 320xy xy+− = .
Câu IV (6,0 đim)
Cho tam giác nhn
A
BC
A
BAC> . Gi
M
là trung đim ca BC ; H là trc tâm;
,,
A
DBECF là các đường cao ca tam giác
A
BC . Kí hiu
1
()C
2
()C ln lượt là đưng
tròn ngoi tiếp tam giác
A
EF
D
KE , vi K là giao đim ca EF
BC
. Chng minh
rng:
1)
M
E là tiếp tuyến chung ca
1
()C
2
()C .
2) KH AM .
Câu V (2,0 đim)
Vi
0,, 1
x
yz≤≤. Tìm tt c các nghim ca phương trình:
3
111
xyz
yzx zxy xyz xyz
++=
++ ++ ++ ++
.
------------------------------------------------------ HT-----------------------------------------------------
Thí sinh không được s dng tài liu. Cán b coi thi không được gii thích gì thêm.
S báo danh
…...............……
- 1 -
S GIÁO DC VÀ ĐÀO TO
THANH HÓA
KÌ THI CHN HC SINH GII TNH
Năm hc: 2011-2012
HƯỚNG DN CHM MÔN TOÁN
(Đề chính thc)
Lp 9 THCS
Ngày thi: 23 tháng 3 năm 2012
(Hướng dn gm 03 trang)
CÂU NI DUNG ĐIM
I 1) 2,0 đim
Điu kin xác định: 110x<≠ (*).
Đặt:
1,0 3xaa−= < .
Khi đó:
2
22
9311
:
39 3
aa a
P
aaaaa
⎛⎞
++
⎛⎞
=+
⎜⎟
⎜⎟
+−
⎝⎠
⎝⎠
1,0
2
3( 3) 2 4
:
9(3)
aa
aaa
++
=
−−
3
24
a
a
=
+
31
214
x
x
=
+
.
1,0
2) 2,0 đim
x
()()
22
44
322 322=+ −−
1,0
4,0
đim
()()
44
44
21 21=+−−
(
)
21 21=+ 2
=
.
Suy ra:
3
24
P
=
+
1
2
=−
.
1,0
II 1) 2,0 đim
To độ A và B tho mãn h:
2
2
2
x
x
yx
−=
=−
(; ) (1; 1)
xy=− hoc ( ; ) ( 2; 4)xy=− .
1,0
9932AB =+= .
1,0
2) 2,0 đim
Xét phương trình (hoành độ giao đim ca ( )P
'd
):
2
x
xm
=− +
2
0xxm
+= (1).
Tn ti
CD, khi và ch khi: (1) có 2 nghim
12
,
x
x phân bit
1
4
m
<
(*).
Khi đó, to độ ca
C
D là:
11
(; )Cx y
22
(; )Dx y , trong đó:
11
y
xm
=
−+
22
y
xm
=
−+.
1,0
4,0
đim
222 2 2
12 12 12 12 12
()( )2()2()4CD x x y y x x x x x x
=− + = = +
.
Áp dng định lý Viét đối vi (1), suy ra:
2
2(1 4 )CD m=− .
CD AB= 2(1 4 ) 18m−= 2m
=
, tho mãn (*).
Vy, giá tr cn tìm ca
m là: 2m =− .
1,0
III 1) 2,0 đim
- 2 -
Điu kin xác định: 0xy (*).
Khi đó, h đã cho tương đương vi:
2
2
2
22
x
xy y
yxyx
+=
+=
22
2
23 2
22
x
yxyxy
yxyx
++=+
+=
2
(2)( 1)0
22
xyxy
yxyx
++=
+=
1,0
2
2
0
x
y
yy
=−
−=
hoc
1
1
3
x
y
y
=−
=
(; )
x
y = (0; 0), ( 2; 1) hoc
21
;
33
⎛⎞
⎜⎟
⎝⎠
.
Đối chiếu (*), suy ra nghim ca h đã cho: ( ; )
x
y
=
(2;1)
hoc
21
(; ) ;
33
xy
⎛⎞
=
⎜⎟
⎝⎠
.
1,0
2) 2,0 đim
62 3
2 2 320xy xy+− = (1).
(1)
()
2
332
( ) 320xxy+−= .
Đặt:
3
8
x
u=
3
8
x
yv−= , (1) tr thành:
22
5uv
+
= .
1,0
4,0
đim
H:
3
3
22
8
8
5
,
xu
x
yv
uv
xy
=
−=
+=
suy ra:
( ; ) (2; 8), (2;24), ( 2; 24), ( 2;8)xy =−
.
1,0
IV 1) 3,0 đim
6,0
đim
M
EB CBE= (tam giác
B
EC vuông ti E , có EM là trung tuyến)
=
CAD (hai tam giác vuông
E
BC DAC có chung góc nhn C ).
1,0
M C
D
E
F
H
(C
1
)
A
B
K
(C
2
) L
- 3 -
Mt khác
1
()HC , t đó ta có:
HEM HAE= .
Suy ra,
M
E là tiếp tuyến ca
1
()C .
0,5
M
ED MEC DEC=−
=
M
CE DEC (do tam giác
B
EC vuông ti
E
, có
E
M là trung tuyến)
=
M
CE DHC (t giác HDCE ni tiếp)
=
M
CE FHA (góc đối đỉnh)
1,0
=
M
CE FEA (t giác HEAF ni tiếp)
=
M
CE CEK (góc đối đỉnh)
=
DKE (góc ngoài tam giác), suy ra
M
E là tiếp tuyến ca
2
()C .
Hoàn thành li gii bài toán.
0,5
2) 3,0 đim
Gi
1
()LAM C=∩; theo câu IV.1), ta có:
2
..
M
LMA ME MDMK== .
1.0
Suy ra L thuc đường tròn ngoi tiếp tam giác
A
DK - là đường tròn đường kính
A
K . 1.0
Do đó KL AM .
Mt khác, ta li có
HL AM (vì
1
()LC
- là đường tròn đường kính
A
H ).
Do đó , ,
KLH thng hàng, suy ra điu phi chng minh.
1.0
V
3
111
xyz
y
zx z xy x yz x y z
++=
++ ++ ++ ++
(1).
Gi thiết 0 , , 1
xyz≤< kết hp vi điu kin xác định ca (1), suy ra: 0
x
yz++> (*).
Khi đó, ta có: (1 )(1 ) 0
zx−−
1
zx z x+≥+
1
xx
y
zx x y z
+
+++
.
0.5
Tương t, ta cũng có:
1
yy
zxyxyz
++ ++
1
zz
x
yz x y z
+
+++
.
Suy ra:
3
1
111
xyz
xyz yzx zxy xyz
=++
++ ++ ++ ++
0.5
hay 3xyz++ (1)
Mt khác, t 0,,1
x
yz≤≤, suy ra: 3xyz
+
+≤ (2)
0.5
2,0
đim
T (1) và (2) ta suy ra: 3xyz++=, kết hp vi điu kin 0 , , 1
x
yz
suy ra 1
x
yz===
Vy, phương trình đã cho có nghim duy nht ( ; ; ) (1;1;1)
xyz
=
0.5
----------------------------------------------------HT----------------------------------------------------
SỞ GD&ĐT VĨNH PHÚC
ĐỀ CHÍNH THỨC
KỲ THI CHỌN HSG LỚP 9 NĂM HỌC 2013-2014
ĐỀ THI MÔN: TOÁN
Thời gian làm bài: 150 phút, không kể thời gian giao đề
Câu 1 (3,0 điểm).
a)
Cho biểu thức:
2 16 4 2 1
6 8 2 4
a a a
M
a a a a
. Tìm tất cả các giá trị nguyên của a
để giá trị của M là một số nguyên.
b)
Cho đa thức
2
( )
P x ax bx c
thỏa mãn đồng thời các điều kiện
( ) 0
P x
với mọi s
thực x
b a
. Tìm giá trị nhỏ nhất của biểu thức
a b c
Q
b a
.
Câu 2 (2,0 điểm).
Tìm tất cả các giá trị của tham số m để phương trình sau vô nghiệm:
1
1 2
x x
x m x m
Câu 3 (1,0 điểm).
Cho
p
số nguyên tố lớn hơn 5. Chứng minh rằng s
7
5
1954
1
p
chia
hết cho 60.
Câu 4 (3,0 điểm).
Cho đường tròn
( )O
m O bán kính bằng
R
. Hai điểm phân
biệt
,B C
cố định nằm trên
( )O
sao cho
2BC a R
. Gọi
A
điểm bất thuộc cung
lớn
BC
của
( )O
,
A
không trùng với
,B C
. Gọi
D
chân đường phân giác trong kẻ từ A
của tam giác
ABC
. Hai điểm
,E F
lần lượt tâm đường tròn ngoại tiếp các tam giác
ADB
ADC
.
a) Chứng minh rằng hai tam giác
AEO
ADC
đồng dạng.
b) Tính diện tích tứ giác
AEOF
theo
a
R
.
c) Chứng minh rằng khi điểm A thay đổi thì
E
di chuyển trên mt đường thẳng cố định.
Câu 5 (1,0 điểm).
Trên một đường tròn cho 21 điểm phân biệt. Mỗi một điểm được
bởi một trong 4 màu: xanh, đỏ, tím, vàng. Giữa mỗi cặp điểm nối với nhau bằng một
đoạn thẳng được bởi một trong 2 màu: nâu hoặc đen. Chứng minh rằng luôn tồn tại
một tam giác ba đỉnh được cùng mt màu (xanh, đỏ, tím hoặc vàng) ba cạnh
cũng được tô cùng một màu (nâu hoặc đen).
----------- Hết -----------
Cán bộ coi thi không giải thích gì thêm.
Họ và tên thí sinh:……….………..…….…….….….; Số báo danh……………….
SỞ GD&ĐT VĨNH PHÚC
KỲ THI CHỌN HSG LỚP 9 NĂM HỌC 2013-2014
HƯỚNG DẪN CHẤM MÔN: TOÁN
I. LƯU Ý CHUNG:
- Hướng dẫn chấm chỉ trình bày một cách giải với những ý bản phải có. Khi chấm
bài học sinh làm theo cách khác nếu đúng và đủ ý thì vẫn cho điểm tối đa.
- Điểm toàn bài tính đến 0,25 và không làm tròn.
- Với bài hình học nếu thí sinh không vẽ hình phần nào thì không cho điểm tương ứng
với phần đó.
II. ĐÁP ÁN:
Câu
Ý Nội dung trình bày Điểm
1
a)
2,5
Cho biểu thức:
2 16 4 2 1
6 8 2 4
a a a
M
a a a a
. Tìm tất cả các giá trị
nguyên của a để M là một số nguyên.
ĐKXĐ:
0
4, 16
a
a a
2 16 4 2 1
6 8 2 4
a a a
M
a a a a
2 16 ( 4)( 4) (2 1)( 2)
6 8
a a a a a
a a
2 1
( 2)( 4) 4
a a a
a a a
Từ
1 5
1
4 4
a
M
a a
.
Do M là số nguyên nên
5 ( 4) 4 { 1; 5}
a a
.
TH1.
4 1 25
a a
TH2.
4 1 9
a a
TH3.
4 5 81
a a
TH4.
4 5 1
a a
(loại)
Đối chiếu điều kiện đã đặt, ta suy ra các giá trị cần tìm của a là: 9; 25; 81.
b)
0,5
Cho đa thức
2
( )
P x ax bx c
thỏa mãn đồng thời các điều kiện
( ) 0
P x
với mọi số thực x
b a
. Tìm giá trị nhỏ nhất của biểu thức
a b c
Q
b a
.
- Từ
( ) 0,P x x
ta chứng minh được
2
0
4 0
a
b ac
.
- Do đó:
2 2 2 2
4 4
4 4 4 ( )
b b a b c a ab b
c a b c a b
a a b a a b a
- Lại có:
2 2 2 2 2
4 4 16 8 12 ( ) (4 )
3 3
4 ( ) 4 ( ) 4 ( )
a ab b a ab b a b a a b
a b a a b a a b a
Vậy
min
3 4 0
Q b c a
Học sinh có thể làm theo cách sau:
- Từ giả thiết
( ) 0, ( 2) 0
P x x P
4 2 0 3( ) 0
a b c a b c b a
- Từ đó suy ra
3
a b c
Q
b a
.
Xét đa thức
2
( ) 4 4P x x x
, ta thấy đa thức này thỏa mãn các điều
kiện của giả thiết và khi đó
1 4 4
3
4 1
Q
.
Vậy giá trị nhỏ nhất của
Q
bằng 3.
2 2,0
Tìm tất cả các giá trị của tham số m để phương trình sau nghiệm:
1
1 2
x x
x m x m
(*)
ĐKXĐ:
1
2
x m
x m
Khi đó
2 2
(*) ( 3) 2 (1 ) (2 2) 2
x m x m x m x m x m
(**)
+ Nếu
1
m
,
(**) 0. 1
x
, vô nghiệm, suy ra phương trình (*) vô
nghiệm
+ Nếu
1
m
thì (**) có nghiệm
2
2 2
m
x
m
, do đó phương trình đã
cho vô nghiệm nếu
2
1 (1)
2 2
2
2 (2)
2 2
m
m
m
m
m
m
- TH1 :
2
0
(1) 2 2 2
1
2
m
m m
m
- TH2 :
2 2
2
(2) 2 2 6 4 2 5 2 0
1
2
m
m m m m m
m
Vậy có 4 giá trị của m để phương trình vô nghiệm là :
1
1;0; 2;
2
.
3
1,0
Cho p số nguyên tố lớn hơn 5. Chứng minh rằng số
7
5
1954
1
p
chia
hết cho 60.
Trước hết ta dễ dàng chứng minh
7
5
1954 4m
(với m nguyên dương)
Ta sẽ chứng bài toán tổng quát
4
1
m
p
chia hết cho 60 với mọi số
nguyên tố
5
p
và mọi số nguyên dương m.
Thật vậy, có
4 4 4 2
1 ( ) 1 1 ( 1)( 1)( 1).
m m m
p p p A p p p A
(
A
)
Do p lẻ nên
1, 1p p
là hai số chẵn liên tiếp suy ra
( 1)( 1) 4
p p
(1)
Lại có
( 1) ( 1) 3
p p p
p không chia hết cho 3 nên
( 1)( 1) 3
p p
(2)
Do p không chia hết cho 5 nên p có một trong các dạng
5 1k
;
5 2
k
.
- Nếu
2 2
5 1 25 10 1 5 1p k p k k n
- Nếu
2 2
5 2 25 20 4 5 1p k p k k l
(
, , )
k n l
Suy ra
4
1 5.p q
, hay
2
( 1)( 1)( 1) 5
p p p
(3)
Từ (1), (2), (3) và 3, 5, 4 là các số đôi một nguyên tố cùng nhau nên
2 4
( 1)( 1)( 1) (3.5.4) 1 60
p p p p
.
Vậy
4
1 60
m
p
(điều phải chứng minh).
4
a
1,5
Chứng minh rằng hai tam giác
AEO
ADC
đồng dạng.
Trong đường tròn
( )O
ta có:
1
.
2
AOE AOB ACB
(1)
Trong đường tròn (ADB), ta có
1
2
AEO
ADB
0 0
1
360 2. 180
2
ADB ADB ADC
(2)
Từ (1) và (2) suy ra hai tam giác
AEO
ADC
đồng dạng.
b
1,0
Tính diện tích tứ giác
AEOF
theo
a
R
.
Tương tự phần a), ta có hai tam giác
,
AFO ADB
đồng dạng, do đó
0
, 180
AEO ADC AFO ADB AEO ADB AEOF
là tứ giác nội
tiếp
,E F
nằm hai phía
AO
, suy ra :
1
( . . )
4
AEOF AOE AOF
S S S OE AB OF AC
(3)
(Nếu học sinh không chứng minh (3) trừ 0,25 điểm)
- Lại có:
.OE AO AO CD
OE
CD AC AC
(4)
.OF AO AO BD
OF
BD AB AB
(5)
Thay (4), (5) vào (3) ta được:
. .
4. . .
AEOF
AO CD AO BD
S AB AC
AC AB
(6)
- Vì
AD
là phân giác của tam giác ABC nên ta có:
AB DB
AC DC
(7)
Thế (7) vào (6) ta được
4 ( . . ) ( . . )
AEOF
AB AC BD CD
S AO CD BD AO CD BD
AC AB CD BD
.
( ) . .
4
AEOF
R a
AO BD CD AO BC R a S
(đvdt).
c
0,5
Chứng minh rằng khi A thay đổi thì điểm
E
di chuyển trên một đường
thẳng cố định.
- Đường trung trực của BC cắt cung lớn
BC
tại H , cắt cung nhỏ
BC
tại K.
Khi đó H, K cố định và là điểm chính giữa của các cung tương ứng.
- Gọi M, N tương ứng là trung điểm
,BD AB
suy ra
0
90
BNE BME
Do đó
, , ,B M N E
cùng nằm trên đường tròn đường kính
BE
.
1
4
BEM BNM BAD
BKC
.
1
4
BHK
BKC
, suy ra
BEM BHK
(8)
Lại có EM // HK (cùng vuông góc với BC),
,H E
cùng phía so với
BC
(9)
Kéo dài
/ /
BE HK H BEM BH K
(10)
Từ (8), (9), (10) suy ra
/
H H
, ,B E H
thẳng hàng
E BH
cố định.
5 1,0
A
B
E
C
D
F
- Vì các điểm phân biệt nằm trên một đường tròn nên ba điểm bất kỳ luôn
tạo thành một tam giác.
- Có 21 điểm được tô bằng 4 màu, do đó có ít nhất 6 điểm có cùng màu.
Giả sử có 6 điểm cùng màu đỏ là
, , , , ,A B C D E F
- Nối 5 đoạn
, , , ,AB AC AD AE AF
và tô bằng 2 màu nâu, đen khi đó có ít
nhất 3 đoạn cùng màu, giả sử
, ,
AB AC AD
được tô cùng màu đen.
Xét tam giác
BCD
, xảy ra hai khả năng:
TH1. Nếu ba cạnh
, ,BC BD DC
được tô cùng màu nâu thì tam giác
BCD
ba đỉnh cùng màu đỏ, ba cạnh cùng màu nâu (thỏa mãn)
TH2. Nếu ba cạnh
, ,BC BD DC
có ít nhất một cạnh màu đen, giả sử
BC
đen, khi đó tam giác
ABC
có ba đỉnh cùng màu đỏ, ba cạnh cùng màu đen
(thỏa mãn)
Vậy luôn có một tam giác có ba đỉnh cùng màu và ba cạnh cùng màu.
------------- Hết -------------
UBND HUYN NGHI XUÂN
PHÒNG GIÁO DC- ĐÀO TO
K THI CHN ĐỘI TUYN HC SINH GII LP 9
NĂM HC 2013 -2014
Môn: Toán. Thi gian làm bài: 150 phút
Câu 1: a. Tính giá tr ca biu thc:
6 2 5 14 6 5
A = +
b. Tìm x; y tha mãn:
2 2 4 4 0
x y xy x
+ + =
Câu 2: a. Gii phương trình nghim nguyên:
4 2 2
5 4 85 0
x y x y
+ =
b. Cho x ; y ; z là các s nguyên và
( ) ( ) ( )
5 5 5
2012 2 2013 3 2014
2 3 2013.
P x y z
S x y z
= + + + +
= + + +
Chng minh rng P chia hết cho 30 khi và ch khi S chia hết cho 30.
Câu 3: Cho ba s x, y, z khác 0 và tho mãn:
2 2 2
1
x y z
2
1 1 1 1
4
x y z xyz
1 1 1
0
x y z
+ + =
+ + + =
+ + >
.
Tính giá tr ca biu thc:
(
)
(
)
(
)
2009 2009 2011 2011 2013 2013
P y z z x x y= + + +
Câu 4: a. Cho tam giác nhn ABC trc tâm H, trng tâm I; Giao đim 3 đường trung
trc là O, trung đim ca BC là M.
Tính giá tr biu thc:
2 2
2 2
IO OM
IH HA
+
+
b. Cho góc
xOy
. Mt đường thng d thay đổi luôn ct các tia Ox; Oy ti M N.
Biết giá tr biu thc
1 1
OM ON
+ không thay đổi khi đường thng d thay đổi.
Chng minh rng đường thng d luôn đi qua mt đim c định.
Câu 5: a. Cho các s x; y; z không âm, không đồng thi bng 0 và tha mãn:
1 1 1
1
x 1 y 2 z 3
+ +
+ + +
.
Tìm giá tr nh nht ca biu thc:
1
P x y z
x y z
= + + +
+ +
b. Cho các s dương x, y, z tho mãn điu kin: xy + yz + zx = 671.
Chng minh rng:
2 2 2
1
2013 2013 2013
x y z
x yz y zx z xy x y z
+ +
+ + + + +
-------------------- Hết ----------------------
H và tên thí sinh ............................................................... SBD .............................
1
PHÒNG GD-ĐT NGHI XUÂN
-------------------------- HƯỚNG DN CHM MÔN TOÁN
THI CHN ĐỘI TUYN HC SINH GII LP 9
NĂM HC 2013-2014
Câu 1:(4 đim) . a) 1,5 đim. b) 2,5 đim
BIU ĐIM
a)
( ) ( )
2 2
6 2 5 14 6 5 5 1 3 5 5 1 3 5 2
A
= + = + = + =
1,5
b) ĐKXĐ:
0;
0; 0
x y
x y
=
>
0,5
Xét x = 0. Suy ra y = - 4 ( Tha mãn)
0,75
Xét
0; 0
x y
>
. Biến đổi PT v dng:
(
)
(
)
2 2
2 0
x y x
+ =
Lp lun tính được x = y = 4 ( Tha mãn).
1,0
KL:
(
)
(
)
; 0; 4
x y
=
hoc
(
)
(
)
; 4;4
x y =
0,25
Câu 2: (4,5 đim) a) 2,25 đim. b) 2,25 đim
a) Phương trình đã cho tương đương vi
(
)
2
4 2
85 2
x y x
=
0,5
Lp lun
4 4
85 4
x
<
x Z
Suy ra
4
x
{
4 4 4 4
0 ;1 ;2 ;3
}
1,0
4 4
0
x
=
thì
2
85
y
=
( loi)
4 4
1
x
=
thì
( )
2
2 84
y
=
( loi)
4 4
2
x
=
thì
( )
2
8 71
y
=
( loi)
4 4
3
x
=
thì
( )
2
18 4
y
=
18 2
18 2
y
y
=
=
20
16
y
y
=
=
Khi đó
3
3
x
x
=
=
Vy phương trình có 4 nghim nguyên
(
)
;
x y
là: (3 ; 20); (-3 ; 20); (3 ; 16); (-3 ; 16)
0,75
b) Đặt
2012; 2 2013; 3 2014
a x b y c z
= + = = +
. Ta có:
5 5 5
P a b c
S a b c
= + +
= + +
( a ; b ; c là các s nguyên )
Xét
(
)
(
)
(
)
5 5 5
P S a a b b c c
= + +
0,5
2
Ta có : vi mi s nguyên m thì
5
m m
chia hết cho 30
Tht vy:
5 4 2 2
( 1) ( 1)( 1) ... ( 1)( 1)( 2)( 2) 5 ( 1)( 1)
m m m m m m m m m m m m m m m
= = + = = + + + +
(1)
Vi mi s nguyên m thì
;( 1);( 1);( 2);( 2)
m m m m m
+ +
là 5 s nguyên liên tiếp
nên trong đó có 1 tha s chia hết cho 2; 1 tha s chia hết cho 3;1 tha s chia
hết cho 5 mà 2; 3; 5 nguyên t cùng nhau tng đôi mt nên tích ca chúng chia
hết cho 2.3.5. Hay
( 1)( 1)( 2)( 2)
m m m m m
+ +
chia hết cho 30 (2)
;( 1);( 1)
m m m
+
;( 1);( 1);( 2);( 2)
m m m m m
+ +
là 3 s nguyên liên tiếp nên
trong đó có 1 tha s chia hết cho 2; 1 tha s chia hết cho 3 mà 2; 3 nguyên t
cùng nhau nên tích ca chúng chia hết cho 2.3. Hay
5 ( 1)( 1)
m m m
+
chia hết cho
30 (3)
T (1); (2); (3) Suy ra vi mi s nguyên m thì
5
m m
chia hết cho 30
Do đó
(
)
(
)
(
)
5 5 5
P S a a b b c c
= + +
chia hết cho 30 vi a; b; c là các s nguyên
1,75
Câu 3: (2,5 đim)
T gi thiết suy ra:
2
2 2 2 2 2 2 2 2 2
1 1 1 1 1 1 1 2(x y z) 1 1 1 1 1 1 1 1 1
4 2
x y z xyz x y z xyz x y z xy yz zx x y z
+ +
= + + + = + + + = + + + + + = + +
1 1 1
0
x y z
+ + >
suy ra
1 1 1
2
x y z
+ + =
(1)
Mt khác
1
x y z
2
+ + =
suy ra
1
2
x y z
=
+ +
(2)
T (1) và (2) suy ra
1 1 1 1
x y z x y z
+ + =
+ +
(3)
1,0
Biến đổi (3)
(
)
(
)
(
)
x y y z z x 0
+ + + =
1,0
2013 2013 2013 2013
2009 2009 2009 2009
2011 2011 2011 2011
0 0
0 0
0 0
x y x y x y x y
z y y z y z y z
x z z x z x z x
+ = = = + =
+ = = = + =
+ = = = + =
nên P = 0
0,5
Câu 4 :(5,5 đim) a) 3 đim. b) 2,5 đim
a) Ta có MO // HA (cùng vuông góc vi BC)
OK // BH (cùng vuông góc vi AC)
KOM
=
BHA
(góc có cnh tương ng song song)
MK // AB (M, K là trung đim BC và AC)
HAB
=
OMK
(góc có cnh tương ng song song)
ABH đồng dng vi MKO (1,0)
MO MK 1
AH AB 2
= =
( 0,5)
A
B
C
H
M
K
I
O
3
Xét AIH và MIO có
MO MI 1
AH AI 2
= =
OMI
=
HAI
(so le trong)
AIH đồng dng vi MIO
IO 1
IH 2
=
= =
IO OM 1
IH HA 2
1,0
+
= = =
+
2 2 2 2
2 2 2 2
IO OM IO OM 1
4
IH HA IH HA
2 2
2 2
1
2
IO OM
IH OA
+
=
+
0,5
N
M
D
E
I
O
x
y
d
b) Gi s
1 1 1
OM ON a
+ =
(1) ( a là s dương cho trước). Ly đim D trên Oy sao
cho OD = a thì OD < ON. V DI song song vi Ox ( I
đon MN ). Ly E trên Ox
sao cho OE = ID. Khi đó OEID là hình bình hành.
1,0
Ta có
1
OE OD NI EI NI MI
OM ON NM ON NM MN
+ = + = + =
=>
1 1 1
.
OE
ON OD OM OD a
+ = =
(2)
0,75
T (1) và (2) =>
1
.
OE
OM OD OM
= =>
1
OE
OD
=
=> OE = OD = a không đổi, mà
D
Oy; E
Ox nên D; E c định. Mt khác O c định và OEID là hình bình hành
nên I c định. Vy d luôn đi qua I c định (ĐPCM)
0,75
CÂU 5 (3,5 đim) Câu a) 2 đim. Câu b) 1,5 đim
a) Trước tiên ta chng minh bt đẳng thc: Vi
a, b, c
R và x, y, z > 0 ta
( )
2
2 2 2
a b c
a b c
x y z x y z
+ +
+ +
+ +
(*) Du “=” xy ra
a b c
x y z
= =
Tht vy, vi a, b
R và x, y > 0 ta có
( )
2
2 2
a b
a b
x y x y
+
+
+
(**)
(
)
( ) ( )
2
2 2
a y b x x y xy a b
+ + +
( )
2
0
bx ay
(luôn đúng)
áp dng bt đẳng thc (**) ta có
( ) ( )
2 2
2 2 2 2
a b a b c
a b c c
x y z x y z x y z
+ + +
+ + +
+ + +
Du “=” xy ra
a b c
x y z
= =
Áp dng vi a = b= c = 1 ta có
( )
2
1 1 1
1 1 1 9
1
1 2 3 6 6
x y z x y z x y z
+ +
+ + =
+ + + + + + + + +
=>
6 9
x y z
+ + +
=>
3
x y z
+ +
( Có th chng minh BĐT trên nh áp dng BĐT Bunhicopski )
1
Áp dng BĐT Côsi cho 2 s dương ... ta có:
1 8(x y z) x y z 1 8.3 x y z 1 10
P x y z 2. .
x y z 9 9 x y z 9 9 x y z 3
+ + + + + +
= + + + = + + + =
+ + + + + +
0,75
4
Du “=” xy ra khi và ch khi các s x; y; z không âm và không đồng thi bng 0
tha mãn :
x y z 3
x y z 1
9 x y z
x 1 y 2 z 3
1 1 1
1
x 1 y 2 z 3
+ + =
+ +
=
+ +
+ = + = +
+ + =
+ + +
x 2
y 1
z 0
=
=
=
( Tha mãn)
Vy Min
10
P
3
=
x = 2; y = 1; z = 0.
0,25
b) Áp dng bt đẳng thc (*) ta có
2 2 2
2013 2013 2013
x y z
VT
x yz y zx z xy
= + +
+ + +
( ) ( ) ( )
2 2 2
2 2 2
2013 2013 2013
x y z
x x yz y y zx z z xy
= + +
+ + +
( )
( )
2
3 3 3
3 2013
x y z
x y z xyz x y z
+ +
+ + + + +
(1)
Chú ý: xy + yz + zx = 671 nên
(
)
2
2013
x x yz + =
(
)
2
1342 0
x x xy zx
+ + + >
,
(
)
2
2013 0
y y zx
+ >
(
)
2
2013 0
z z xy
+ >
0,75
Chng minh:
(
)
(
)
3 3 3 2 2 2
3
x y z xyz x y z x y z xy yz zx
+ + = + + + +
( ) ( ) ( )
2
3
x y z x y z xy yz zx
= + + + + + +
(2)
(
)
3 3 3
3 2013
x y z xyz x y z
+ + + + + =
( ) ( ) ( )
2
3 2013
x y z x y z xy yz zx
+ + + + + + +
=
( ) ( )
2
3.671 2013
x y z x y z
+ + + + +
=
( )
3
x y z
+ + (3)
0,5
T (1) và (3) ta suy ra
( )
( )
2
3
1
x y z
VT
x y z
x y z
+ +
=
+ +
+ +
Du “=” xy ra
x = y = z =
2013
3
.
0,25
( Ghi chú: Mi cách gii khác đúng và hp lí đều cho đim ti đa tương ng)
----Hết-----
phßng gd
phßng gdphßng gd
phßng gd-
--
-®t ®øc thä
®t ®øc thä ®t ®øc thä
®t ®øc thä ®Ò thi olympic to¸n 9
®Ò thi olympic to¸n 9 ®Ò thi olympic to¸n 9
®Ò thi olympic to¸n 9 n¨m häc 2012
n¨m häc 2012 n¨m häc 2012
n¨m häc 2012-
--
-2013
20132013
2013
§Ò thi chÝnh thøc Thêi gian lµm bµi 120 phót
Bµi 1: a) Gi¶i ph−¬ng tr×nh
x 2 3 2x 5 x 2 2x 5 2 2
+ + + =
b) Víi gi¸ trÞ nµo cña tham sè a th× ph−¬ng tr×nh sau cã nghiÖm:
(
)
+ + =
2 2
a x 2a 3 1 x x 4 2 3 4
(*)
Lêi gi¶i
: a) Ta cã
(
)
(
)
2
1 1
x 2 3 2x 5 2x 5 6 2x 5 9 2x 5 3 0
2 2
+ + = + + = +
(
)
(
)
2
1 1
x 2 2x 5 2x 5 2 2x 5 1 2x 5 1 0
2 2
= + =
§KX§:
5
2x 5 0 x
2
Ph−¬ng tr×nh t−¬ng ®−¬ng
2x 4 6 2x 5 2x 4 2 2x 5 4
+ + + =
( ) ( )
2 2
2x 5 3 2x 5 1 4 2x 5 3 2x 5 1 4
+ + = + + =
1 2x 5 1 2x 5
=
. Ta cã
1 2x 5 1 2x 5
, do ®ã dÊu “=” x¶y ra khi vµ chØ khi
1 2x 5 0 2x 5 1 2x 5 1 x 3
.
KÕt hîp víi §KX§ ta cã nghiÖm cña ph−¬ng tr×nh lµ
5
x 3
2
b) §KX§:
x 4 0 x 4
Ph−¬ng tr×nh t−¬ng ®−¬ng
(
)
2 2
a x 2a 3 1 x 4 2 3 x 4
+ + =
Ta cã
(
)
(
)
(
)
(
)
2 2
2 2 2 2
a x 2a 3 1 x 4 2 3 a x 2a 3 1 x 3 1 ax 3 1 0
+ + = + + = +
;
x 4 0
. Suy ra
(
)
(
)
2 2
2 2
a x 2a 3 1 x 4 2 3 0
a x 2a 3 1 x 4 2 3 0
x 4
x 4 0
+ + =
+ + =
=
=
§Ó ph−¬ng tr×nh (*) cã nghiÖm th× ph−¬ng tr×nh
(
)
2 2
a x 2a 3 1 x 4 2 3 0
+ + =
cã nghiÖm x = 4
Dã ®ã
( ) ( )
2
2 2
1 3
a 4 2a 3 1 4 4 2 3 0 4a 3 1 0 a
4
+ + = + = =
Bµi 2: a) T×m GTNN cña biÓu thøc
= + + + +
2 2
P 1 4x 4x 4x 12x 9
b) T×m sè thùc a ®Ó ph−¬ng tr×nh sau cã nghiÖm nguyªn
+ + =
2
x ax a 2 0
Lêi gi¶i
: a)
( ) ( )
2 2
2 2
P 1 4x 4x 4x 12x 9 2x 1 2x 3 2x 1 2x 3
= + + + + = + + = + +
2x 1 3 2x 2x 1 3 2x 4
= + + + + =
GTNN cña P lµ 4.
§¹t ®−îc khi
( )( ) ( )
2
2
1 3
2x 1 3 2x 0 4x 4x 3 0 2x 1 4 2x 1 2 x
2 2
+
b) §Ó ph−¬ng tr×nh cã nghiÖm nguyªn th× 0
(
)
2
2
a 4a 8 0 a 2 12 a 2 2 3
a 2 2 3
;
a 2 2 3
+
. Khi ®ã gäi x
1
; x
2
lµ c¸c nghiÖm cña ph−¬ng tr×nh. Theo hÖ thøc Viets ta cã
( ) ( ) ( )( )
1 2
1 2 1 2 1 2 2 1 2
1 2
x x a
x x x x 2 x x 1 x 1 3 x 1 x 1 3
x x a 2
+ =
+ = = =
= +
x
1
– 1 vµ x
2
– 1 lµ −íc cña 3. Gi¶ sö x
1
x
2
th× x
1
– 1 x
2
– 1. Ta cã 2 tr−êng hîp sau:
1 1
2 2
x 1 3 x 4
x 1 1 x 2
= =
= =
khi ®ã a = 6 vµ
1 1
2 2
x 1 1 x 0
x 1 3 x 2
= =
= =
khi ®ã a = -2
§èi chiÕu ®iÒu kiÖn ta cã
{
}
a 2; 6
lµ gi¸ trÞ cÇn t×m
Bµi 3: a) Chøng minh r»ng ®−êng th¼ng (d) ph−¬ng tr×nh
(
)
(
)
m 3 x m 2 y m 1 0
+ =
(m lµ tham sè) lu«n ®i
qua mét ®iÓm cè ®Þnh A. T×m täa ®é A
b) Gi¶i hÖ ph−¬ng tr×nh sau:
2 2
(1)
(2)
x y 3 x 2 4
x y 2xy 4x 4y 5
+ =
+ + =
Lêi gi¶i
: a) Ta cã
(
)
(
)
m 3 x m 2 y m 1 0 mx my m 3x 2y 1 0
+ = + + =
(
)
(
)
m x y 1 2y 3x 1 0
+ + =
®óng víi mäi m khi vµ chØ khi
x y 1 0 2x 2y 2 0
2y 3x 1 0 2y 3x 1 0
+ = + =
= =
x 1 x 1
y x 1 y 2
= =
= + =
. VËy ®−êng th¼ng (d) lu«n ®i qua mét ®iÓm cè ®Þnh A(1; 2)
b) Tõ ph−¬ng tr×nh (2) suy ra
( ) ( ) ( )
2 2
x y 2 3
x y 4 x y 4 9 x y 2 9
x y 2 3
+ =
+ + = + =
+ =
x y 1
x y 5
=
=
Víi x – y = 1 thay vµo ph−¬ng tr×nh (1) ®−îc
x 2 2 x 4
x 2 2
x 2 2 x 0
= =
=
= =
x = 4 y = 3; x = 0 y = -1
Víi x – y = -5 thay vµo ph−¬ng tr×nh (1) ®−îc
x 2 4
=
v« nghm
VËy tËp nghiÖm cña ph−¬ng tr×nh lµ
(
)
(
)
(
)
{
}
x;y 4;3 ; 0; 1
Bµi 4: Cho ABC ®Òu ®Þnh néi tiÕp trong ®−êng trßn (O). §−êng th¼ng d thay ®æi lu«n ®i qua A vµ c¾t cung nhá
AB t¹i ®iÓm E (E A). §−êng th¼ng d c¾t hai tiÕp tuyÕn t¹i B vµ C cña ®−êng trßn (O) lÇn l−ît t¹i M vµ N, MC c¾t BN
t¹i F. Chøng minh r»ng
a) CAN BMA vµ MBC BCN
b) Tø gi¸c BMEF néi tiÕp ®−îc ®−êng trßn
c) Chøng minh r»ng ®−êng th¼ng EF lu«n ®i qua mét ®iÓm cè ®Þnh khi d thay ®æi
Lêi gi¶i
: a) Ta cã
0
1
ACN AC ABC 60
2
= = =
0
1
MBA AB ACB 60 ACN MBA
2
= = = =
(
)
1
ANC EBC AC
2
=
(
)
1 1
EBC BC BE BAM
2 2
= = =
XÐt CAN vµ BMA cã
ACN MBA
ANC BAM
=
=
CAN BMA (g – g)
CA CN BC CN BC BM
BM BA BM CB CN CB
= = =
A
B
C
N
M
O
E
F
K
I
XÐt MBC vµ BCN cã
0
BC BM
CN CB
MBC BCN 120
=
= =
MBC BCN (c – g – c)
b) XÐt MBC vµ BFC cã
BMC CBF (v× MBC BCN)
BCM chung
=
(g – g)
0 0
BFC MBC 120 BFM 60
= = = . MÆt kh¸c
0
BCA AEB 180
+ = ,
0
BEM AEB 180
+ =
0
BEM BCA 60
= = . Suy ra
0
BEM BFM 60
= = , gi¸c BMEF néi tiÕp (E, F cïng nh×n MB d−íi 1 gãc b»ng
nhau)
c) §−êng th¼ng EF c¾t ®−êng trßn (O) t¹i K. Ta
BMF CBF
= (v× MBC BFC);
BMF BEF
=
(gãc néi
tiÕp cïng ch¾n
BF
);
BMF BCK
= (gãc néi tiÕp cïng ch¾n
BK
)
CBF BCK
= BF // CK (1)
Ta l¹i cã
0 0
1
BKC s® BAC 120 KBF 60
2
= = =
0
BFC 120
=
BK // FC (2)
Tõ (1) vµ (2) tø gi¸c BFCK lµ h×nh b×nh hµnh. Do ®ã EF ®i qua trung ®iÓm I cña BC cè ®Þnh
Bµi 5: Cho a, b, c > 0. Chøng minh r»ng
1 1 1 1 1 1
a 3b b 3c c 3a a 2b c b 2c a c 2a b
+ + + +
+ + + + + + + + +
Lêi gi¶i
: Víi x, y > 0 ta cã
1 1 4
x y x y
+
+
. ThËt vËy
( ) ( )
2 21 1 4
x y 4xy x y 0
x y x y
+ +
+
i x, y. DÊu
“=” x¶y ra khi vµ chØ khi x = y
¸p dông bµi to¸n phô trªn ta cã:
1 1 4 2
a 3b b 2c a a 3b b 2c a a 2b c
+ =
+ + + + + + + + +
1 1 4 2
b 3c c 2a b b 3c c 2a b b 2c a
+ =
+ + + + + + + + +
1 1 4 2
c 3a a 2b c c 3a a 2b c c 2a b
+ =
+ + + + + + + + +
Céng theo vÕ 3 B§T trªn ®−îc
1 1 1 1 1 1
a 3b b 3c c 3a a 2b c b 2c a c 2a b
+ + + +
+ + + + + + + + +
DÊu “=” x¶y ra khi vµ chØ khi
a 3b b 2c a
b 3c c 2a b a b c
c 3a a 2b c
+ = + +
+ = + + = =
+ = + +
Lêi gi¶i: NguyÔn Ngäc Hïng – THCS Hoµng Xu©n H·n
S GIÁO DC VÀ ĐÀO TO
BC GIANG
ĐỀ THI CHÍNH THC
Đề thi có 01 trang
K THI CHN HC SINH GII VĂN HOÁ CP TNH
NĂM HC 2012-2013
MÔN THI: TOÁN; LP: 9 PH THÔNG
Ngày thi: 30/3/2013
Thi gian làm bài 150 phút, không k thi gian giao đề
Câu 1. (5,0 đim)
1) Tính giá tr ca biu thc
3 3
26 15 3 26 15 3
A = + .
2) Rút gn biu thc
2 2 2 7 3 2 1 1
. :
3 11
3 2 3 2 2 2
a a a a
P
a
a a a a
+ + +
= +
+
.
Câu 2. (4,0 đim)
1) Gii phương trình:
3 2
3 8 2 3 10
x x x
+ = +
.
2) Gii h phương trình sau:
2 2
2
1 4
( 1)( 2)
x y xy y
x x y y
+ + + =
+ + =
.
Câu 3. (4,0 đim)
1) Cho m s
2
y x
=
. Tìm các giá tr ca
m
để đường thng
phương trình
y x m
=
ct đồ th hàm s ti hai đim phân bit
1 1 2 2
( ; ), ( ; )
A x y B x y
tho mãn:
4 4
2 1 2 1
( ) ( ) 18
x x y y
+ =
.
2) Tìm tt c các b ba s nguyên t
, ,
a b c
đôi mt khác nhau tho mãn điu kin
20 30( ) 21
abc ab bc ca abc
< + + <
Câu 4. (6,0 đim)
Cho tam giác ABC vuông ti A (AB<AC), có đường cao AH O trung đim ca
cnh BC. Đường tròn tâm I đường kính AH ct AB, AC th t ti M N. OA
MN ct nhau ti D.
1) Chng minh t giác BMNC ni tiếp.
2) Chng minh :
1 1 1
AD HB HC
= + .
3) Cho AB=3 và AC=4. Tính bán kính đưng tròn ngoi tiếp tam giác BMN.
Câu 5. (1,0 đim)
Cho ba s dương
,
a b
c
tho mãn
1
abc
=
. Chng minh rng:
2 2 2 2 2 2
1 1 1 1
2 3 2 3 2 3 2
a b b c c a
+ +
+ + + + + +
.
---------------Hết----------------
Cán b coi thi không gii thích gì thêm.
H và tên thí sinh: .................................................................S báo danh:.......................
Giám th 1 (H tên và ký)..............................................................................................................
Giám th 2 (H tên và ký)..............................................................................................................
S GIÁO DC VÀ ĐÀO TO
BC GIANG
HƯNG DN CHM
BÀI THI CHN HC SINH GII VĂN HOÁ CP TNH
NGÀY THI 30 /3/2013
MÔN THI: TOÁN; LP: 9 PH THÔNG
Bn hướng dn chm có 04 .trang
Câu 1
Hướng dn gii
(5 đim)
Ta có
3 3
26 15 3 26 15 3
A = +
2 2 3 2 2 3
3 3
8 3.2 3 3.2.( 3) ( 3) 8 3.2 3 3.2.( 3) ( 3)
= + + + +
0.5
3 3
3 3
(2 3) (2 3)
= +
0.5
(2 3) (2 3)
= +
0.5
1.
(2 đim)
2 3
A = .
KL:
0.5
Điu kin:
2 11
a
<
0.5
Đặt
2
2 (0 3) 2
x a x a x
= < = +
.
0.5
Tính được
2
2 2
( 2) 9 3 1 1
. :
3 3 9 3
x x x x
P
x x x x x
+ + +
= +
+
0.5
2
( 2) 3( 3) 2 4
. :
3 9 ( 3)
x x x
x x x
+ + +
=
0.5
( 2) ( 3)
.
3 2 4 2
x x x x
x x
+
= =
+
0.5
2
(3 đim)
=
2
2
a
KL:
0.5
Câu 2
(4 đim)
ĐK:
2
x
. Vi điu kin biến đổi phương trình đã cho tr thành:
2 2
3. ( 2)( 2 4) 2( 2 4) ( 2)
x x x x x x
+ + = + + +
0.5
Chia c hai vế ca phương trình cho
2
2 4
x x
+
, ta được
2 2
2 2
3 2 0
2 4 2 4
x x
x x x x
+ +
+ =
+ +
(1)
0.5
1
(2 đim)
Đặt
2
2
( 0)
2 4
x
t t
x x
+
=
+
Thay vào (1) ta được
2
3 2 0
t t
+ =
1
t
=
hoc
2
t
=
(t/m)
0.5
ĐỀ CHÍNH THC
+ vi
1
t
=
ta có
2
2
1
2
=1 3 2 0
2
2 4
x
x
x x
x
x x
=
+
+ =
=
+
(t/m).
+ vi
2
t
=
ta có
2
2
2
=2 4 9 14 0
2 4
x
x x
x x
+
+ =
+
(vô nghim).
KL:
0.5
2
2
1 ( ) 4
( 1)( 2)
x y x y y
x x y y
+ + + =
+ + =
+ Vi
0
y
=
Hpt tr thành:
2
2
1 0
( 1)( 2) 0
x
x x
+ =
+ =
(vô nghim)
0.5
+ Vi
0
y
.H tr thành
2
2
1
( ) 4
1
( )( 2) 1
x
x y
y
x
x y
y
+
+ + =
+
+ =
(1)
+ Đặt
2
1
,
x
a b x y
y
+
= = +
thay vào hpt(1) ta được
4
( 2) 1
a b
a b
+ =
=
0.5
+ Gii được:
1, 3
a b
= =
0.5
2
(2 đim)
+ Vi
1, 3
a b
= =
2
1
1
3
x
y
x y
+
=
+ =
.
Gii được nghim ca h:
( ; ) (1; 2) và (x;y)=(-2;5)
x y
=
+ KL:
0.5
Câu 3
(4 đim)
Xét pt hoành độ giao đim:
2
x x m
=
2
0
x x m
+ =
(1)
Đường thng
ct đths đã cho ti hai đim phân bit A, B khi và ch
khi pt(1) có hai nghim phân bit.
0.5
+ Điu kin:
1 4 0
m
= >
1
.
4
m
<
0.5
+ Khi đó
1 1 2 2
( ; ), ( ; )
A x y B x y
+ Theo định lí Viet
1 2 1 2
1,
x x x x m
+ = =
. Ta có
1 1 2 2
,
y x m y x m
= =
+
4 4 4 2 2
1 2 1 2 1 2 1 2 1 2
( ) ( ) 18 ( ) 9 [( ) 4 ] 9
x x y y x x x x x x
+ = = + =
0.5
1
(2 đim)
+ Tìm được
1 (k / )
1
( / )
2
o
m t m
m t m
=
=
KL:
0.5
2
(2 đim)
+ T gi thiết suy ra:
2 1 1 1 7
3 10
a b c
< + + <
. Không gim tính tng quát
gi s
1
a b c
> > >
. Suy ra
2 3
2 9
3
c
c
< <
0.5
Do đó
{2;3}
c
+ Vi
2
c
=
suy ra
2 1 1 1 7 1 1 1 1 1 2 1 1
(1)
3 2 10 6 5 6 5
a b a b b b
< + + < < + < < <
Do đó
{7;11}
b
0.5
+ Vi
7
b
=
t (1) suy ra
1 1 2
{19; 23;29;31;37; 41}
42 35
a
a
< <
+ Vi
11
b
=
t (1) suy ra
5 1 6
13
66 55
a
a
< < =
( do a>b)
0,5
+ Vi
3
c
=
t gi thiết suy ra
1 1 1 11 1 2
(*) 6 5
3 30 3
b b
a b b
< + < < < =
( do b>c)
Thay
5
b
=
vào (*) được
15
6 7
2
a a
< < =
.
Vy có 8 b ba (a;b;c) tho mãn:
(19; 7;2), (23;7;2), (29; 7;2), (31;7;2),(37;
7;2), (41;7; 2),(13;11;2), (7;5;3)
và các hoán v ca nó.
0.5
Câu 4
(6 đim)
+ T giác AMHN ni tiếp nên
AMN AHN
=
0.5
+ Li có
AHN ACH
= (vì cùng ph vi góc
CHN
)
0.5
+ Suy ra
ACB AMN
= , mà
0
180
AMN NMB+ = nên
0
180
ACB NMB+ =
0.5
1
(2 đim)
KL: 0.5
+ Có
AID AOH
= vì cùng bng hai ln
ACB
.
0.5
+ Tam giác
AD AI
AID AOH
AH AO
=
0.5
+ Có
1 1 1 1
( ), AI= .
2 2 2 2
AO BC HB HC AH HB HC
= = + =
0.5
2
(2 đim)
+ Do đó
1 1 1
.
. .
AO HB HC
AD AH AI HB HC HB HC
+
= = = +
0.5
+ Tính được BC=5,
12
5
AH
=
0.5
3
(2 đim)
+ Gi K là tâm ca đường tròn ngoi tiếp tam giác BMN. Khi đó
KI là đường trung trc ca đon MN.
0.5
Do hai tam giac AID và AOH đồng dng nên
0
90
ADI AHO = =
OA MN
Do vy KI//OA.
+ Do t giác BMNC ni tiếp nên
OK BC
. Do đó AH//KO.
+ Dn đến t giác AOKI là hình bình hành.
0.5
Bán kính
2 2 2 2 2 2
1 1 1 769
4 4 4 10
R KB KO OB AI BC AH BC= = + = + = + =
0.5
Câu 5
(1 đim)
Ta có:
2 2 2 2 2
2 3 ( ) ( 1) 2 2 2 2
a b a b b ab b
+ + = + + + + + +
Tương t:
2 2
2 3 2 2 2
b c bc c
+ + + +
,
2 2
2 3 2 2 2
c a ac a
+ + + +
0.5
Suy ra:
2 2 2 2 2 2
1 1 1 1 1 1 1
( )
2 3 2 3 2 3 2 1 1 1
1 1 1 1 1
( ) .
1 1 1
2 1 2
1 1
a b b c c a ab b bc c ac a
ab b
a
a ab b
+ + + +
+ + + + + + + + + + + +
= + + =
+ +
+ + + +
0.5
Đim toàn bài
(20đim)
Lưu ý khi chm bài:
- Trên đây ch sơ lược các bước gii, li gii ca hc sinh cn lp lun cht ch,
hp logic. Nếu hc sinh trình bày cách làm khác đúng thì cho đim các phn theo
thang đim tương ng.
- Vi bài 4, nếu hc sinh v hình sai hoc không v hình thì không chm.
S GIÁO DC & ĐÀO TO
NGH AN
K THI CHN HC SINH GII TNH LP 9 THCS
NĂM HC 2010 - 2011
Môn thi: TOÁN - BNG B
Thi gian: 150 phút (không k thi gian giao đề)
Câu 1 (5,0 đim).
a) Chng minh rng vi mi s nguyên n thì
2
n n 2
+ +
không chia hết cho 3.
b) Tìm tt c các s t nhiên n sao cho
2
n 17
+
là mt s chính phương.
Câu 2 (5,0 đim)
a) Gii phương trình:
2
x 4x+5 = 2 2x+3
+
b) Gii h phương trình:
2
2
2x+y = x
2y+x = y
Câu 3 (3,0 đim).
Tìm giá tr nh nht ca biu thc:
2
4x+3
A
x 1
=
+
Câu 4 (4,5 đim)
Cho tam giác ABC ba góc nhn ni tiếp đường tròn (O). Các đường cao BE, CF ca
tam giác ABC ct nhau ti H.
a) Chng minh rng BH.BE + CH.CF =
2
BC
b) Gi K là đim đối xng vi H qua BC. Chng minh rng K
(O).
Câu 5 (2,5 đim).
Cho tam giác ABC ni tiếp đường tròn m O, mt đim I chuyn động trên cung BC
không cha đim A (I không trùng vi B C). Đường thng vuông góc vi IB ti I ct đưng
thng AC ti E, đưng thng vuông góc vi IC ti I ct đường thng AB ti F. Chng minh
rng đường thng EF luôn đi qua mt đim c định.
- - - Hết - - -
H và tên thí sinh:................................................................................ S báo danh: .....................................
ĐỀ CHÍNH THC
S GD&ĐT NGH AN
K THI CHN HC SINH GII TNH LP 9 THCS
NĂM HC 2010 - 2011
ĐÁP ÁN ĐỀ CHÍNH THC
Môn: TOÁN - Bng B
-------------------------------------------
Câu: Ni dung
1.
*) Nếu
2
n 3 n n 3
+
M M
nên
2
n n 2 3
/
+ +
M
(1)
*) Nếu
2
n 3 n 2 3
/
+
M M
2
n n 2 3
/
+ +
M
(2)
a,
(2,5)
T (1) và (2)
n Z
thì
2
n n 2 3
/
+ +
M
Đặt
2 2
m n 17
= +
(m N)
2 2
m n 17 (m n)(m n) 17 1.17
= + = = =17.1
Do m + n > m - n
m n 17 m 9
m n 1 n 8
+ = =
= =
b,
(2,5)
Vy vi n = 8 ta có
2 2
n 17 64 17 81 9
+ = + = =
2.
Gii phương trình
2
x 4x+5=2 2x+3
+ (1)
Điu kin:
3
2x+3 0 x -
2
(1)
2
x 4x+5-2 2x+3 0
+ =
2
x 2x+1+2x+3-2 2x+3 1 0
+ + =
2 2
(x 1) ( 2x+3 1) 0
+ + =
x 1 0
2x+3 1 0
+ =
=
x 1
2x+3=1
=
a,
(2.5)
x 1
=
tha mãn điu kin
Gii h phương trình
2
2
2x+y=x
2y+x=y
Tr tng vế 2 phương trình ta có:
2 2
x y x y
=
b,
(2.5)
(x y)(x y 1) 0
+ =
(1)
(2)
x y x y
x y 1 0 x 1 y
= =
+ = =
Ta có:
*)
x y x y
x(x 3) 0 x 0
= =
= =
Vy (x; y) = (0;0); (3;3)
*)
2 2 2
x 1 y x 1 y x 1 y
2x+y = x 2 2y y (1 y) y y 1 0
= = =
+ = + =
(*)
Vì phương trình
2
y y 1 0
+ =
vô nghim nên h (*) vô nghim
Vy h đã cho có 2 nghim (x; y) = (0; 0); (3; 3)
3.
Tìmgiá tr nh nht ca
2
4x+3
A
x 1
=
+
Ta có:
2
2 2
4x+3 x 4x+4
A 1
x 1 x 1
+
= = +
+ +
2
2
(x 2)
A 1 1
x 1
+
= +
+
Du "=" xy ra
x 2 0 x 2
+ = =
Vy
min
A 1
=
khi x = -2
4.
H
K
E
I
F
O
B
A
C
Gi I là giao đim ca AH và BC AI BC
Ta có: BHI BCE (g, g)
BH BI
BH.BE BC.BI
BC BE
= = (1)
Ta có: CHI CBF (g, g)
CH CI
CH.CF BC.CI
CB CF
= = (2)
a,
(2,5)
T (1) và (2) suy ra BH.HE + CH.CF = BC(BI + CI) = BC
2
Gi K là đim đối xng ca H qua BC suy ra
HCB KCB
=
b,
(2,0)
FAI HCI
=
(do t giác AFIC ni tiếp)
S
S
hoc x = 3
FAI BCK hay BAK BCK
= =
t giác BACK ni tiếp đưng tròn (O) K (O)
5.
+ Khi
0
BAC 90
=
0
BIC 90
=
.
F trùng vi B, E trùng vi C lúc đó EF là đường kính.
EF đi qua đim O c đnh.
K
F
E
O
A
B
C
I
+ Khi
BAC
< 90
0
BIC
> 90
0
.
Gi K là đim đối xng ca I qua EF.
EIF EAF
=
(cùng bù
BIC
)
EKF EIF
=
(Do I và K đối xng qua EF)
EKF EAF
=
AKFE
ni tiếp
KAB KEF
=
(cung chn
KF
) (1)
IEF KEF
=
(Do K và I đối xng qua EF) (2)
IEF BIK
=
(cùng ph
KIE
) (3)
T (1), (2), (3)
KAB BIK
=
AKBI là t giác ni tiếp
K (O)
Mà EF là đường trung trc ca KI
E, O, F thng hàng.
+ Khi
BAC
> 90
0
BIC
< 90
0
chng minh tương t.
Vy đường thng EF luôn đi qua đim O c định.
- - - Hết - - -
S GIÁO DC & ĐÀO TO ĐỀ THI HC SINH GII VÒNG TNH LP 9 THCS
CÀ MAU NĂM HC 2008-2009
Môn thi: Toán
Ngày thi: 01 – 03 – 2009
Thi gian: 150 phút (Không k thi gian giao đề)
Baøi 1 (3,0 ñieåm):
a) Tính giaù trò cuûa bieåu thöùc: S =
2 + 3 2 - 3
+
2 - 3 2 + 3
b) Ruùt goïn bieåu thöùc: y =
2 2
x - 2x + 1 + x - 4x + 4
Baøi 2 (3,0 ñieåm):
a) Chöùng minh raèng soá a =
(
)
2 3 1 2 - 3
+
laø soá höõu tæ.
b) Cho ña thöùc f(x) = mx
3
+ (m 2)x
2
(3n 5)x 4n. Xaùc ñònh m, n
sao cho ña thöùc f(x) chia heát cho x + 1 vaø x – 3.
Baøi 3 (3,0 ñieåm):
Tìm moät soá töï nhieân goàm ba chöõ soá sao cho khi ta laáy chöõ soá ôû haøng ñôn
ñaët veà beân traùi cuûa soá goàm hai chöõ soá coøn laïi, ta ñöôïc moät soá coù ba chöõ soá
lôùn hôn chöõ soá ban ñaàu 765 ñôn vò.
Baøi 4 (3,0 ñieåm): Cho ña thöùc f(x – 1) = x
2
– (m + 1)x – m
2
+ 2m – 2 .
a) Tìm f(x).
b) Tìm giaù trò nhoû nhaát cuûa f(x) khi m = – 2.
Baøi 5 (3,5 ñieåm):
Cho hình bình haønh ABCD. Goïi I laø trung ñieåm cuûa caïnh CD, E laø giao
ñieåm cuûa AC vaø BI, F laø giao ñieåm cuûa hai tia AB vaø DE. Chöùng minh raèng :
a) B laø trung ñieåm cuûa ñoaïn thaúng AF.
b) Neáu BC = BD thì AC = FD.
c) Neáu AC = FD thì BC = BD.
Baøi 6 (4,5 ñieåm): Cho ù giaùc ABCD noäi tieáp ñöôøng troøn (O) trong ñoù hai
ñöôøng cheùo AC vaø BD caét nhau taïi M. Cho bieát ADB laø tam giaùc caân coù goùc
A > 90
0
.
a) Chöùng minh raèng: AD
2
= AM.AC .
b) Goïi I laø taâm ñöôøng troøn ngoaïi tieáp tam giaùc DCM vaø J laø taâm ñöôøng
troøn ngoaïi tieáp tam giaùc BCM. Chöùng minh raèng:
·
·
IDB = JBD
.
c) Chöùng minh raèng: Toång caùc ñoä daøi cuûa hai ñoaïn thaúng ID vaø JB
khoâng tuyø thuoäc vaøo vò trí cuûa ñieåm C treân cung lôùn BD cuûa ñöôøng troøn (O).
---------- HEÁT ----------
ĐỀ CHÍNH THC
S GIÁO DC VÀ ĐÀO TO K THI CHN HC SINH GII LP 9 CP TNH
LONG AN MÔN THI :TOÁN
NGÀY THI: 07/4/2011
THI GIAN :150 phút (không k thi gian phát đề)
Bài 1:(4 đim)
1/ Không s dng máy tính , thc hin phép tính :
A =
3 5 3 5
2 2 3 5 2 2 3 5
+
+
+ +
2/ Cho biu thc:
B = 2 4
x x
+
(vi
2 4
x
)
Tìm giá tr ln nht ca biu thc B và giá tr x tương ng
Bài 2:( 5 đim)
1/ Cho hàm s y = ax
2
(a
0) có đồ th là (P) đi qua M(-1;2) . Trên (P) ly A và B
hoành độ tương ng là 1 và 2 . Xác định m để đường thng y = mx +5 song song vi
đường thng AB
2/ Tìm x tha mãn :
2 2 3 2
1 1 1
(2 2 1)
4 4 2
x x x x x x
+ + + = + + +
Bài 3: (5 đim)
Cho tam giác ABC có ba góc đều nhn có AB < AC ni tiếp đường tròn O bán kính R.
Ba đường cao AD,BE,CF ct nhau ti H
a/ Chng minh H là tâm đường tròn ni tiếp tam giác DEF
b/ K đường kính AK ca đường tròn O.Gi S là din tích tam giác ABC
Chng minh : S =
. .
4
AB AC BC
R
c/ Gi M là trung đim BC . Chng minh: t giác DFEM là ni tiếp
Bài 4 : (3 đim)
Cho đim M nm trong tam giác ABC có BC = a, AC = b, AB = c. Gi các khong cách
t M đến ba cnh BC, AC, AB tương ng là x,y,z . Hãy xác định v trí M trong tam giác
sao cho biu thc :
a b c
P
x y z
= + +
đạt giá tr nh nht
Bài 5 : (3 đim)
Tìm mt s chính phương có bn ch s , mi ch s nh hơn 9. Biết rng khi tăng mi
ch s thêm mt đơn v thì s mi được to thành cũng là s chính phương.
_______________
ĐỀ CHÍNH THC
S GIÁO DC VÀ ĐÀO TO K THI CHN HC SINH GII LP 9 CP TNH
LONG AN MÔN THI : TOÁN
NGÀY THI : 11/4/2012
THI GIAN : 150 phút (không k thi gian phát đề)
Bài 1: ( 4 đim)
1/ Không s dng máy tính, hãy thc hin phép tính:
A =
2 3 4 15 10
23 3 5
- + - +
-
2/ Cho biu thc B =
3x 6 x x 1 x 2
x x 2 x 2 1 x
+ + +
- +
+ - + -
a/ Tìm điu kin xác định và rút gn B.
b/ Tìm giá tr ln nht ca B và giá tr x tương ng.
Bài 2: (5 đim)
1/ Tìm h s a > 0 sao cho các đường thng y = ax – 1 ; y = 1 ; y = 5 và trc tung to thành hình
thang có din tích bng 8 (đơn v din tích).
2/ Cho các s x, y, z khác 0 tha mãn đồng thi
1 1 1
2
x y z
+ + =
2
2 1
4
xy z
=
. Tính giá tr ca
biu thc P = (x + 2y + z)
2012
.
Bài 3: (5 đim)
Cho tam giác nhn ABC ni tiếp đường tròn (O), các đường cao AD, BE, CF (D
Î
BC, E
Î
AC,
F
Î
AB) ct nhau ti H và ct đường tròn (O) theo th t M, N, K. Chng minh rng:
a/ BH.BE + CH.CF = BC
2
.
b/ AH.AD + BH.BE + CH.CF =
2 2 2
2
AB BC CA
+ +
.
c/
4
AM BN CK
AD BE CF
+ + =
.
Bài 4: (3 đim)
Cho đon thng CD = 6 cm, I mt đim nm gia C D ( IC > ID). Trên tia Ix vuông góc
vi CD ly hai đim M và N sao cho IC = IM, ID = IN, CN ct MD ti K (
)
K MD
, DN ct MC ti L
( )
L MC
. Tìm v trí ca đim I trên CD sao cho CN.NK có giá tr ln nht.
Bài 5: (3 đim)
Tìm các cp s (x; y) nguyên dương tha mãn: xy + 2x = 27 – 3y.
----------------------------------------------------- Hết --------------------------------------------
H và tên thí sinh :………………………………………………….
S báo danh :………………
ĐỀ CHÍNH THC
S GIÁO DC VÀ ĐÀO TO K THI CHN HC SINH GII LP 9 CP TNH
LONG AN MÔN THI : TOÁN
NGÀY THI : 11/4/2012
THI GIAN : 150 phút (không k thi gian phát đề)
HƯỚNG DN CHM
Bài Câu Ni dung Đim
1
A =
2 3 4 15 10
23 3 5
- + - +
-
(
)
( )
2 2 3 4 15 10
2 23 3 5
- + - +
=
-
4 2 3 8 2 15 2 5
46 6 5
- + - +
=
-
( ) ( )
( )
2 2
2
3 1 5 3 2 5
3 5 1
- + - +
=
-
3 1 5 3 2 5
3 5 1
- + - +
=
-
3 5 1
3 5 1
-
=
-
= 1
0,5
0,25
0,75
0,25
0,25
1
(4đ)
2
a/ ĐKXĐ
x 0,x 1
³ ¹
B =
3x 6 x x 1 x 2
x x 2 x 2 1 x
+ + +
- +
+ - + -
( )( )
( )( )
( )( )
( )
( )( )
2
x 1 x 1 x 2
3x 6 x
x 1 x 2 x 1 x 2 x 1 x 2
+ - +
+
= - -
- + - + - +
( )( )
3x 6 x x 1 x 4 x 4
x 1 x 2
+ - + - - -
=
- +
( )( )
x 2 x 3
x 1 x 2
+ -
=
- +
0,25
0,5
0,25
ĐỀ CHÍNH THC
(
)
(
)
( )( )
x 1 x 3
x 1 x 2
- +
=
- +
x 3
x 2
+
=
+
b)
x 3
B
x 2
+
=
+
Vi
x 0,x 1
³ ¹
x 2 2
+ ³
1 1
2
x 2
Û £
+
1 3
1
2
x 2
Û + £
+
Du “ = “ xãy ra khi
x 0 x 0
= Û =
(tmđk)
Vy giá tr ln nht ca B là
3
2
khi x = 0.
0,25
0,25
0,25
0,25
6
5
4
3
2
1
-1
-2
-3
-4
-10
-8 - 6
-4
-2
2
4
6
8
10
D
CB
A
O
y=1
y=5
0,5
2
(5đ)
1
+) Kí hiu hình thang ABCD cn tìm như hình v.
+) Tính được C(
6
;5)
a
; D(
2
;1)
a
BC =
6
a
; AD =
2
a
+)
6 2
.4 : 2 8
ABCD
S
a a
= + =
a = 2 ( Tha ĐK a > 0)
+) Vy phương trình đường thngy = 2x – 1.
0,5
0,5
0,25
0,25
2
+) Ta có
1 1 1
2
x y z
+ + =
2
1 1 1
4
x y z
+ + =
+) Do đó
2
2
1 1 1 2 1
x y z xy z
+ + =
2 2 2 2
1 1 1 2 2 2 2 1
0
x y z xy yz zx xy z
+ + + + + + =
2 2 2 2
1 2 1 1 2 1
0
x xz z y yz z
+ + + + + =
2
2
1 1 1 1
0
x z y z
+ + + =
2
2
1 1
1 1
0
1 1
1 1
0
x z
x z
x y z
y z
y z
+ =
=
= =
=
+ =
Thay vào
1 1 1
2
x y z
+ + =
ta được x = y =
1
2
; z =
1
2
Khi đó P =
2012
2012
1 1 1
2. 1 1
2 2 2
+ + = =
0,25
0.25
0,25
0,5
0,5
0,5
0,5
0,25
H
D
E
F
K
N
M
o
A
B
C
3
(5đ)
a
+) T giác DCEH có
0 0 0
90 90 180
HDC HEC+ = + =
T giác DCEH ni tiếp
HED HCD
=
( cùng chn cung HD)
*
BDE và
BHC có
HED HCD
=
EBC
chung.
BDE đồng dng
BHC (g.g)
0,5
0,25
. .
BD BE
BH BE BC BD
BH BC
= =
(*)
*Chng minh tương t đẳng thc (*)ta được : CH.CF = CD.CB (**)
Cng (*) và (**) theo vế ta được:
BH.BE + CH.CF = BC.BD + CD.CB
= (BD + CD).BC
= BC.BC = BC
2
(1)
0,5
0,25
0,5
b +) Chng minh tương t đẳng thc (1) ta được:
BH.BE + AH.AD = AB
2
(2) và AH.AD + CH.CF = AC
2
(3)
+) Cng (1), (2), (3) theo vế ta được:
2(AH.AD + BH.BE + CH.CF) = AB
2
+ AC
2
+ BC
2
AH.AD + BH.BE + CH.CF =
2 2 2
2
AB BC CA
+ +
.
0,5
0.75
0.25
c
+) Ta có:
MBC MAC
=
( cùng chn cung MC)
MAC CBE
=
( cùng ph
BCA
)
Nên
MBC CBE
=
BC là phân giác
MBE
*
MBH có BC là đường cao đồng thi là đường phân giác nên là tam giác cân ti
B
BC đồng thi là đường trung tuyến ng vi cnh MH.
D là trung đim ca MH.
DM = DH.
*Ta có
1
AM AD DM DM
AD AD AD
+
= = +
(*)
BHC và
ABC có chung đáy BC nên ta có
BHC
ABC
S DH DM
S AD AD
= = (**)
T (*) và (**) suy ra :
1
BHC
ABC
AM S
AD S
= +
(1)
Chng minh tương t đẳng thc (1) ta được:
1
AHC
ABC
BN S
BE S
= +
(2) và
1
AHB
ABC
CK S
CF S
= +
(3)
Công (1) (2) và (3) theo vế ta được :
1 1 1 3 3 1 4
BHC AHC AHB ABC
ABC ABC ABC ABC
AM BN CK S S S S
AD BE CF S S S S
+ + = + + + + + = + = + =
0,25
0,25
0,25
0,25
0,25
0,25
4
(3đ)
x
M
N
D
C
I
K
L
+)
D
IND vuông ti I có IN = ID (gt)
Þ
D
IND vuông cân ti I
0
45
IND IDN = =
* Chng minh tương t ta được
D
IMC vuông cân ti I
0
45
ICM IMC = =
D
LCD có
·
·
0
LCD LDC 45
= =
Þ
D
LCD vuông cân ti L
Þ
DL
^
MC
Mà MI
^
CD (gt)
Þ
DL và MI là hai đường cao ca
D
CDM ct nhau ti N
Þ
N là trc tâm
D
CDM
Þ
CN
^
MD hay CK
^
MD
D
CNI và
D
MNK có:
·
·
0
CIN MKN 90
= =
·
·
INC KNM
=
(đđ)
Þ
D
CNI đồng dng
D
MNK (g-g)
Þ
CN NI
MN NK
=
Þ
CN.NK = MN.NI
Ta có: MN.NI = (MI – NI).NI = ( CI – ID).ID = (CD – ID – ID).ID
Đặt ID = x; x > 0 ta được:
MN.NI = (6 – 2x).x = 6.x – 2x
2
=
2
3 9 9
2 x
2 2 2
æ ö
÷
ç
- - + £
÷
ç
÷
ç
è ø
Du “ = “ xy ra khi x =
3
2
(TMĐK x > 0)
Vy CN. NK có giá tr ln nht là
9
2
khi ID =
3
2
cm.
0.5
0,5
0,5
0,5
0,5
0,5
5
(3đ)
Ta có: xy + 2x = 27 – 3y
xy 2x 3y 27
Û + + =
(
)
(
)
2 3 2 33
x y y
+ + + =
(x 3)(y 2) 33
Û + + =
Û
x 3 1
y 2 33
ì
+ =
ï
ï
í
ï
+ =
ï
î
hoc
x 3 33
y 2 1
ì
+ =
ï
ï
í
ï
+ =
ï
î
hoc
x 3 3
y 2 11
ì
+ =
ï
ï
í
ï
+ =
ï
î
hoc
x 3 11
y 2 3
ì
+ =
ï
ï
í
ï
+ =
ï
î
do x > 0, y > 0.
Û
x 2
y 31
ì
= -
ï
ï
í
ï
=
ï
î
(loi)hoc
x 30
y 1
ì
=
ï
ï
í
ï
= -
ï
î
(loi)hoc
x 0
y 9
ì
=
ï
ï
í
ï
=
ï
î
(loi)hoc
x 8
y 1
ì
=
ï
ï
í
ï
=
ï
î
(tđk)
Vy cp s nguyên dương cn tìm là (x; y) = (8;1)
0,5
0,25
1,0
1,0
0,25
(Nếu HS trình bày bài gii bng cách khác đúng thì chm theo thang đim tương đương)
S GIÁO DC & ĐÀO TO
NGH AN
K THI CHN HC SINH GII TNH LP 9 THCS
NĂM HC 2010 - 2011
Môn thi: TOÁN - BNG A
Thi gian: 150 phút (không k thi gian giao đề)
Câu 1 (4,0 đim).
a) Cho các s nguyên a
1
, a
2
, a
3
, ... , a
n
. Đặt S =
3 3 3
1 2 n
a a ... a
+ + +
1 2 n
P a a ... a
= + + +
.
Chng minh rng: S chia hết cho 6 khi và ch khi P chia hết cho 6.
b) Cho A =
6 4 3 2
n n 2n 2n
+ +
(vi
n N,
n > 1). Chng minh A không phi s
chính phương.
Câu 2 (4,5 đim).
a) Gii phương trình:
3 2
10 x 1 3x 6
+ = +
b) Gii h phương trình:
1
x 3
y
1
y 3
z
1
z 3
x
+ =
+ =
+ =
Câu 3 (4,5 đim).
a) Cho x > 0, y > 0, z > 0 và
1 1 1
4
x y z
+ + =
.
Chng minh rng:
1 1 1
1
2x+y+z x 2y z x y 2z
+ +
+ + + +
b) Cho x > 0, y > 0, z > 0 tha mãn
2011 2011 2011
x y z 3
+ + =
.
Tìm giá tr ln nht ca biu thc:
2 2 2
M x y z
= + +
Câu 4 (4,5 đim).
Cho tam giác ABCba góc nhn ni tiếp đường tròn (O), H là trc tâm ca tam giác.
Gi M là mt đim trên cung BC không cha đim A. (M không trùng vi B và C). Gi N và P
ln lượt là đim đối xng ca M qua các đưng thng AB và AC.
a) Chng minh ba đim N, H, P thng hàng.
b) Khi
0
BOC 120
=
, xác định v trí ca đim M để
1 1
MB MC
+ đạt giá tr nh nht.
Câu 5 (2,5 đim).
Cho tam giác ABC ni tiếp đường tròn m O, mt đim I chuyn động trên cung BC
không cha đim A (I không trùng vi B C). Đường thng vuông góc vi IB ti I ct đưng
thng AC ti E, đưng thng vuông góc vi IC ti I ct đường thng AB ti F. Chng minh
rng đường thng EF luôn đi qua mt đim c định.
- - - Hết - - -
H và tên thí sinh:................................................................................ S báo danh: .....................................
ĐỀ CHÍNH THC
S GD&ĐT NGH AN
K THI CHN HC SINH GII TNH LP 9 THCS
NĂM HC 2010 - 2011
ĐÁP ÁN ĐỀ CHÍNH THC
Môn: TOÁN - Bng A
--------------------------------------------
Câu: Ni dung
1.
Vi
a Z
thì
3
a a (a 1)a(a 1)
= +
tích 3 s
t nhiên liên tiếp nên chia hết
cho 2 và 3. Mà (2.3)=1
3
a a 6
M
3 3 3
1 1 2 2 n n
S P (a a ) (a a ) ... (a a ) 6
= + + +
M
Vy
S 6 P 6
M M
6 4 3 2 2 2 2
n n 2n 2n n (n 1) .(n 2n 2)
+ + = + +
vi
n N
, n > 1 thì
2 2
n 2n 2 (n 1) 1
+ = +
>
2
(n 1)
2 2
n 2n 2 n 2(n 1)
+ =
<
2
n
Vy
2
(n 1)
<
2
n 2n 2
+
<
2
n
2
n 2n 2
+
không s chính ph
ương
đpcm
2.
3 2
10 x 1 3(x 2)
+ = +
2 2
10 (x 1)(x x 1) 3(x 2)
+ + = +
điu kin
x 1
Đặt
x 1 a
+ =
(a 0)
2
x x 1 b
+ =
(b>0)
Ta có:
2 2
10ab = 3a 3b
+
a = 3b
(a 3b)(3a-b) = 0
b 3a
=
Trường hp1: a = 3b
Ta có:
2
x 1 3 x x 1
+ = +
(1)
2
9x 9x+9=x+1
2
9x 10x+8 = 0
'
25 9.8
=
< 0
phương trình (1) vô nghim
Trường hp 2: b = 3a
Ta có:
2
3 x 1 x x 1
+ = +
2
9(x 1) x x 1
+ = +
2
x 10x-8 = 0
1
2
x 5 33 (TM)
x 5 33 (TM)
= +
=
Vy phương trình có 2 nghim
x 5 33
= ±
1
x 3
y
1
y 3
z
1
z 3
x
+ =
+ =
+ =
T (3)
3x-1
z
x
=
thay vào (2)
3xy+3 = 8x+y
(4)
T (1)
xy 1 3y 3xy+3 = 9y
+ =
(5)
T (4) và (5)
8x+y = 9y x y
=
Chng minh tương t : y = z
T đó
x y z
= =
Thay vào (1)
2
1
x 3 x 3x+1 = 0
x
+ =
3 5
x
2
±
=
h có 2 nghim
3 5
x y z
2
±
= = =
3.
Áp dng bt đẳng thc
1 1 4
x y x y
+
+
(vi x,y > 0)
Ta có:
1 1 1 1
( )
2x+y+z 4 2x y z
+
+
;
1 1 1
y z 4y 4z
+
+
Suy ra:
1 1 1 1 1
( )
2x+y+z 4 2x 4y 4z
+ +
(1)
Tương t:
1 1 1 1 1
( )
x+2y+z 4 4x 2y 4z
+ +
(2)
1 1 1 1 1
( )
x+y+2z 4 4x 4y 2z
+ +
(3)
T (1),(2),(3)
1 1 1 1 1 1 1
( )
2x+y+z x+2y+z x+y+2z 4 x y z
+ + + +
1 1 1
1
2x+y+z x+2y+z x+y+2z
+ +
Du "=" xy ra
3
x y z
4
= = =
Áp dng bt đẳng thc CôSy cho
2011 2011
x ,x
và 2009 s 1 ta có:
2011 2011 2 2011
2011
x x 1 1 ... 1 2011 (x )
+ + + + +
2009
2011 2
2x 2009 2011x
+
(1)
Tương t:
2011 2
2y 2009 2011y
+
(2)
2011 2
2z 2009 2011z
+
(3)
T (1), (2), (3)
2011 2011 2011
2 2 2
2(x y z ) 3.2009
x y z
2011
+ + +
+ +
2 2 2
x y z 3
+ +
Giá tr ln nht ca M là 3 khi và ch khi x = y = z = 1
4.
H
P
M
N
F
E
I
O
C
B
A
Gi giao đim ca BH vi AC là E
AH vi BC là F, CH vi AB là I
HECF là t giác ni tiếp.
AHE ACB
=
(1)
ACB AMB
=
( góc ni tiếp cùng chn mt cung)
Ta có:
AMB ANB
=
(Do M, N đối xng AB) (2)
T (1), (2)
AHBN là t giác ni tiếp
NAB NHB
=
(*)
NAB MAB
=
(Do M, N đối xng qua AB (**)
T (*), (**)
NHB BAM
=
Chng minh tương t:
PHC MAC
=
NHB PHC BAM MAC BAC
+ = + =
0
BAC IHE 180
+ =
0
NHB PHC BHC 180
+ + =
( vì
IHE BHC
=
)
N, H, P thng hàng
Gi J là đim chính gia ca cung ln BC
0
BOC 120
=
BJC
đều
Trên đon JM ly K sao cho MK = MB
JKB CMB
=
O
K
B
M
C
J
BM MC JM
+ =
1 1 4
BM MC BM MC
+
+
1 1 4
BM MC JM
+
JM ln nht
JM là đường kính (O) lúc đó M là đim chính gia ca cung nh
BC.
Vy
1 1
BM MC
+
nh nht
M là đim chính gia cung nh BC
5.
+ Khi
0
BAC 90
=
0
BIC 90
=
.
F trùng vi B, E trùng vi C lúc đó EF là đường kính.
EF đi qua đim O c đnh.
K
F
E
O
A
B
C
I
+ Khi
BAC
< 90
0
BIC
> 90
0
.
Gi K là đim đối xng ca I qua EF.
EIF EAF
=
(cùng bù
BIC
)
EKF EIF
=
(Do I và K đối xng qua EF)
EKF EAF
=
AKFE
ni tiếp
KAB KEF
=
(cùng chn
KF
) (1)
IEF KEF
=
(Do K và I đối xng qua EF) (2)
IEF BIK
=
( cùng ph
KIE
) (3)
T (1), (2), (3)
KAB BIK
=
AKBI là t giác ni tiếp
K (O)
Mà EF là đường trung trc ca KI
E, O, F thng hàng.
+ Khi
BAC
> 90
0
BIC
< 90
0
chng minh tương t.
Vy đường thng EF luôn đi qua đim O c định.
- - - Hết - - -
S GIÁO DC ĐÀO TO
THÀNH PH ĐÀ NNG
ĐỀ CHÍNH THC
K THI CHN HC SINH GII LP 9
NĂM HC 2010-2011
Môn thi: TOÁN
Thi gian: 150 phút (không tính thi gian giao đề)
Bài 1. (2,0 đim)
Cho biu thc:
2
a 1 a a 1 a a a a 1
M
a a a a a a
+ +
= + +
vi a > 0, a 1.
a) Chng minh rng
M 4.
>
b) Vi nhng giá tr nào ca a thì biu thc
6
N
M
=
nhn giá tr nguyên?
Bài 2. (2,0 đim)
a) Cho các hàm s bc nht:
y 0,5x 3
= +
,
y 6 x
=
y mx
=
đồ th ln
lượt các đường thng (d
1
), (d
2
) (
m
). Vi nhng giá tr nào ca tham s m t
đường thng (
m
) ct hai đường thng (d
1
) (d
2
) ln lượt ti hai đim A B sao
cho đim A có hoành độ âm còn đim B có hoành độ dương?
b) Trên mt phng ta độ Oxy, cho M N hai đim phân bit, di động ln
lượt trên trc hoành trên trc tung sao cho đường thng MN luôn đi qua đim c
định
I(1; 2)
. Tìm h thc liên h gia hoành độ ca M tung độ ca N; t đó, suy
ra giá tr nh nht ca biu thc
2 2
1 1
.
Q
OM ON
= +
Bài 3. (2,0 đim)
a) Gii h phương trình:
17 2 2011
2 3 .
+ =
=
x y xy
x y xy
b) Tìm tt c các giá tr ca x, y, z sao cho:
1
x y z z x (y 3).
2
+ + = +
Bài 4. (3,0 đim)
Cho đường tròn (C ) vi tâm O và đường kính AB c định. Gi M là đim di động
trên (C ) sao cho M không trùng vi các đim A và B. Ly C là đim đi xng ca O
qua A. Đường thng vuông góc vi AB ti C ct đường thng AM ti N. Đường
thng BN ct đường tròn (C ) ti đim th hai E. Các đường thng BM CN ct
nhau ti F.
a) Chng minh rng các đim A, E, F thng hàng.
b) Chng minh rng tích AMAN không đổi.
c) Chng minh rng A là trng tâm ca tam giác BNF khi và ch khi NF ngn
nht.
Bài 5. (1,0 đim)
Tìm ba ch s tn cùng ca tích ca mười hai s nguyên dương đầu tiên.
---HT---
H và tên thí sinh: ................................................. S báo danh: ........................
Ch ký ca giám th 1: ............................. Ch ký ca giám th 2: ...........................
S GIÁO DC ĐÀO TO
THÀNH PH ĐÀ NNG
KÌ THI CHN SINH HC SINH GII LP 9
NĂM HC 2010-2011
Môn thi: TOÁN
HƯỚNG DN CHM MÔN TOÁN LP 9
Dưới đây sơ lược biu đim ca đề thi Hc sinh gii lp 9. Các Giám kho tho lun
thng nht thêm chi tiết li gii cũng như thang đim ca biu đim đã trình bày. T chm th
phân chia nh thang đim đến 0,25 đim cho tng ý ca đề thi. Tuy nhiên, đim tng bài, tng câu
không được thay đổi. Ni dung tho lun và đã thng nht khi chm được ghi vào biên bn c th để
vic chm phúc kho sau này được thng nht và chính xác.
Hc sinh có li gii khác đúng, chính xác nhưng phi nm trong chương trình được hc thì
bài làm đúng đến ý nào giám kho cho đim ý đó.
Vic làm tròn s đim bài kim tra được thc hin theo quy định ca B Giáo dc Đào
to ti Quyết định s 40/2006/BGD-ĐT.
BÀI-Ý ĐỀ -ĐÁP ÁN ĐIM
Bài 1
Cho biu thc:
2
a 1 a a 1 a a a a 1
M
a a a a a a
+ +
= + +
vi a > 0, a 1.
a) Chng minh rng
M 4.
>
b) Vi nhng giá tr nào ca a thì biu thc
6
N
M
= nhn giá tr nguyên.
2,00
Do a > 0, a 1 nên:
a a 1 ( a 1)(a a 1) a a 1
a a a( a 1) a
+ + + +
= =
0,25
2
a a a a 1 (a 1)(a 1) a (a 1) (a 1)(a a 1) a a 1
a a a a (1 a) a (1 a) a
+ + + +
= = =
0,25
a 1
M 2
a
+
= +
0,25
Do
a 0; a 1
>
nên:
2
( a 1) 0 a 1 2 a
> + >
0,25
1.a
(1,25đ)
2 a
M 2 4
a
> + =
0,25
Ta có
6 3
0 N
M 2
< = <
do đó N ch có th nhn đưc mt giá tr nguyên là 1
0,25
Mà N = 1
6 a
1
a 1 2 a
=
+ +
a 4 a 1 0
+ =
2
( a 2) 3
=
a 2 3 hay a 2 3
= + = (phù hp)
0,25
1.b
(0,75đ)
Vy, N nguyên
2
a (2 3)
= ±
0,25
Bài 2
a) Cho các hàm s bc nht:
y 0,5x 3
= +
,
y 6 x
=
y mx
=
đồ th ln lượt
các đường thng (d
1
), (d
2
) (
m
). Vi nhng giá tr nào ca tham s m thì đường
thng (
m
) ct hai đường thng (d
1
) (d
2
) ln lượt ti hai đim A B sao cho đim
A có hoành độ âm còn đim B có hoành độ dương?
b) Trên mt phng ta độ Oxy, cho M N hai đim phân bit, di động ln lượt
trên trc hoành trên trc tung sao cho đường thng MN luôn đi qua đim c định
I(1 ; 2)
. Tìm h thc liên h gia hoành độ ca M tung độ ca N; t đó, suy ra giá
2,00
tr nh nht ca biu thc
2 2
1 1
.
Q
OM ON
= +
Điu kin để (
m
) là đồ th hàm s bc nht là
m 0
0,25
Phương trình hoành độ giao đim ca (d
1
) và (
m
) là:
0,5x 3 mx
+ =
(m 0,5)x 3
=
Điu kiên để phương trình này có nghim âm là
m 0,5 0 hay m 0,5
< <
0,25
Phương trình hoành độ giao đim ca (d
2
) và (
m
) là:
6 x mx
=
(m 1)x 6
+ =
Điu kiên để phương trình này có nghim dương là
m 1 0 hay m 1
+ > >
2.a
(0,75đ)
Vy điu kin cn tìm là:
1 m 0,5; m 0
< <
0,25
Đặt m = x
M
và n = y
N
mn 0 và m 1 (*)
Nên đường thng qua ba đim M, I, N có dng: y = ax+b
0,25
0 am b
2 a b
n b
= +
= +
=
h thc liên h gia m và n là
2m n mn
+ =
0,25
Chia hai vế cho mn 0 ta được:
1 2
1
m n
+ =
(**)
2 2
2 2 2 2
1 2 1 4 4 1 1 2 1
1 5
m n m n mn m n m n
= + = + + = +
0,25
2 2
1 1 1
Q ;
m n 5
= +
du “=” xy ra khi
2 1
;
m n
= k
ết hp (**): m = 5, n = 2,5 (tha (*))
0,25
2.b
(1,25đ)
Vy giá tr nh nht ca Q là
1
5
0,25
Bài 3
a) Gii h phương trình:
17 2 2011
2 3 .
+ =
=
x y xy
x y xy
(1)
b) Tìm tt c các giá tr ca x, y, z sao cho:
1
x y z z x (y 3)
2
+ + = +
(2)
2,0 đ
Nếu
0
xy
>
thì
17 2
1 1007
9
2011
9
490
(1)
1 2 9
1 490
3
1007
9
x
y x
y
y
y x
x
+ =
=
=
= =
=
(phù hp)
0,50
Nếu
0
<
xy thì
17 2
1 1004
2011
9
(1) 0
1 2
1 1031
3
18
y x
y
xy
y x
x
+ =
=
>
=
=
(loi)
0,25
Nếu
0
xy
=
thì (1)
0
x y
= =
(nhn).
0,25
3.a
(1,25đ)
KL: Hđúng 2 nghim là
(0;0)
9 9
;
490 1007
0,25
Điu kin x 0; y z 0; z x 0 y z x 0 0,25
3.b
(0,75đ)
(2)
2 x 2 y z 2 z x x y z z x 3
+ + = + + +
2 2 2
( x 1) ( y z 1) ( z x 1) 0
+ + =
0,25
x 1
y z 1
z x 1
=
=
=
x 1
y 3
z 2
=
=
=
(tha điu kin)
0,25
Bài 4
Cho đường tròn (C ) vi tâm O đường kính
AB c định. Gi M đim di động trên (C )
sao cho M không trùng vi các đim A B.
Ly C đim đối xng ca O qua A. Đường
thng vuông góc vi AB ti C ct đường thng
AM ti N. Đường thng BN ct đường tròn (C )
ti đim th hai E. Các đường thng BM và
CN ct nhau ti F.
a) Chng minh rng các đim A, E, F thng
hàng.
b) Chng minh rng tích AMAN không
đổi.
c) Chng minh rng A là trng tâm ca tam
giác BNF khi và ch khi NF ngn nht.
C
CC
C
( )
F
E
N
C
O
A
B
M
3,0 đ
MN BF
BC NF
0,25
A là trc tâm ca tam giác BNF
0,25
FA NB
Li có
AE NB
0,25
4.a
(1,00đ)
Nên A, E, F thng hàng
0,25
CAN MAB
= , nên hai tam giác ACN và AMB đồng dng. 0,25
Suy ra:
AN AC
AB AM
=
0,25
4.b
(0,75đ)
Hay
2
AM AN AB AC 2R
= = không đổi (vi R là bán kính đường tròn (C ))
0,25
Ta có
2
BA BC
3
= nên A là trong tâm tam giác BNF C là trung đim NF (3)
0,25
Mt khác:
CAN CFM
= , nên hai tam giác CNA và CBF đồng dng
2
CN AC
CN CF BC AC 3R
BC CF
= = =
0,25
Áp dng bt đẳng thc Cô-si, ta có:
NF CN CF 2 CN CF 2R 3
= + = không đổi
0,25
Nên: NF ngn nht CN =CF C là trung đim NF (4) 0,25
4.c
(1,25đ)
(3) và (4) cho ta: A là trong tâm tam giác BNF NF ngn nht
0,25
Bài 5
Tìm ba ch s tn cùng ca tích ca mười hai s nguyên dương đầu tiên.
0,75
Đặt: S = 123456789101112
100
S
=
3467891112 (1) là mt s nguyên
hai ch s tn cùng ca S là 00
0,50
Mt khác, trong sut quá trình nhân liên tiếp các tha s vế phi ca (1), nếu ch đ ý
đến ch s tn cùng, ta thy
100
S
ch s tn cùng 6 (vì 34=12; 26=12; 27=14;
48=32; 29=18; 811=88; 812=96) 0,25
(1,00đ)
Vy ba ch s tn cùng ca S là 600 0,25
--- Hết ---
Điu kin x 0; y z 0; z x 0 y z x 0 0,25
Theo BĐT Cauchy:
x 1 y z 1 z x 1
x ; y z ; z x
2 2 2
+ + +
1
VP x y z z x (y 3) VT
2
= + + + =
0,25
3.b
(0,75đ)
Do đó
x 1
y z 1
z x 1
=
=
=
x 1
y 3
z 2
=
=
=
tha điu kin
0,25
S GIÁO DC VÀ ĐÀO TO
TP. ĐÀ NNG
Đề thi chính thc
ĐỀ THI CHN HC SINH GII LP 9
NĂM HC 2012 – 2013
MÔN THI: TOÁN – LP 9 THCS
(Thi gian làm bài 150 phút không k thi gian giao đề)
Bài 1. (2,5 đim)
Cho biu thc vi
a/ Rút gn biu thc vi
b/ Tìm tt c các giá tr sao cho P là mt s nguyên t.
Bài 2. (2,0 đim)
a/ Tìm x, biết:
b/ Gii h phương trình:
Bài 3. (2,0 đim)
a/ Cho hàm s bc nht y = ax + b có đồ th đi qua đim M(1;4). Biết rng đồ th ca hàm s đã cho ct trc Ox ti đim
P có hoành độ dương và ct trc Oy ti đim Q có tung độ dương. Tìm a và b sao cho OP + OQ nh nht (vi O là gc
ta độ)
b/ Tìm s t nhiên có 2 ch s. Biết rng nếu ly tng ca 2 ch s y cng vi 3 ln tích ca 2 ch s y thì bng 17.
Bài 4. (2,0 đim)
Cho tam giác ABC. Gi I là tâm đường tròn ni tiếp tam giác ABC, qua I v đường thng vuông góc vi đường thng
CI, đường thng này ct các cnh AC, BC ln lượt ti M và N.
a/ Chng minh rng hai tam giác IAM và BAI đồng dng.
b/ Chng minh rng
Bài 5. (1,5 đim)
Cho tam giác ABC có là góc tù. V các đường cao CD và BE ca tam giác ABC (D nm trên đường thng AB, E
nm trên đường thng AC). Gi M,N ln lượt là chân đường vuông góc ca các đim B và C trên đường thng DE. Biết
rng là din tích tam giác ADE, là din tích tam giác BEM và là din tích tam giác CDN. Tính din tích tam
giác ABC theo .
S GIÁO DC VÀ ĐÀO TO
TNH ĐĂK NÔNG
K THI CHN HC SINH GII CP TNH LP 9 THCS
NĂM HC 2010-2011
Khóa thi ngày: 10/3/2011
Môn thi: TOÁN
Thi gian: 150 phút ( không k thi gian giao đề)
Bài 1: (4,0 đim)
1) Cho biu thc
2 2 4 3
: .
4
2 2 2
x x x x
A
x
x x x x
+
=
+
Tìm điu kin ca x để A > 0.
2) Cho
2
1 1
2 1 1 2 1 1
x =
+ + +
Tính giá tr ca biu thc:
4 3 2 2011
( 2 1)
B x x x x= +
Bài 2: (4,0 đim)
1) Gii phương trình: 322323
22
++=+++ xxxxxx .
2) Cho x, y z là nghim ca h phương trình:
2
2
2
2 1 0
2 1 0
2 1 0.
x y
y z
z x
+ + =
+ + =
+ + =
Tính giá tr ca biu thc:
10 3 2011
C x y z
= + + .
Bài 3: (4,0 đim)
1) Tìm các cp s ( a, b) tha mãn h thc:
2011 2011
a b a b+ = + .
2) Tìm tt c các s t nhiên n sao cho: n
2
– 14n + 38 là mt s chính phương.
Bài 4: (5,0 đim)
Cho đưng tròn tâm O, hai đường kính AB CD vuông góc vi nhau. E mt đim
nm trên cung nh
AD
. Ni CE ct OA ti M và ni BE ct OD ti N.
1) Chng minh:
. 2 .
AM ED OM EA
=
2) Chng minh tích
OM ON
AM DN
mt hng s. T đó, suy ra giá tr nh nht ca tng
OM ON
AM DN
+ , khi đó cho biết v trí ca đim E?
Bài 5: (3,0 đim)
Cho
, ,
a b c
là ba s thc dương. Chng minh bt đẳng thc:
3 3 3 2 2 2 2 2 2
2 2 2
9
.
2a 2
a b c a b b c c a
bc ab c bc a ca b
+ + + + +
+ + +
+ + +
--------HT--------
ĐỀ CHÍNH THC
S GD&ĐT VĨNH PHÚC
——————
ĐỀ CHÍNH THC
K THI CHN HSG LP 9 NĂM HC 2011-2012
ĐỀ THI MÔN: TOÁN
Thi gian làm bài: 150 phút, không k thi gian giao đề
————————————
Câu 1 (3,0 đim).
1. Cho
( )
3
2
1 3 3
x
f x
x x
=
+
. Hãy tính giá tr ca biu thc sau:
1 2 2010 2011
...
2012 2012 2012 2012
A f f f f
= + + + +
2. Cho biu thc
2
2 1 1 2 2
1
x x x x x
P
x x x x x x x x
+ +
= + +
+ +
Tìm tt c các giá tr ca
x
sao cho giá tr ca P là mt s nguyên.
Câu 2 (1,5 đim).
Tìm tt c các cp s nguyên dương
(
)
;
x y
tha mãn
( ) ( )
3 2
6
x y x y+ = .
Câu 3 (1,5 đim).
Cho
, , ,
a b c d
là các s thc tha mãn điu kin:
2012
abc bcd cda dab a b c d+ + + = + + + +
Chng minh rng:
(
)
(
)
(
)
(
)
2 2 2 2
1 1 1 1 2012
a b c d+ + + + .
Câu 4 (3,0 đim).
Cho ba đường tròn
(
)
(
)
1 2
,
O O
(
)
O
(kí hiu
(
)
X
ch đường tròn tâm đim X). Gi s
(
)
(
)
1 2
,
O O
tiếp xúc ngoài vi nhau ti đim I
(
)
(
)
1 2
,
O O
ln lượt tiếp xúc trong vi
(
)
O
ti
1 2
,
M M
. Tiếp tuyến ca đường tròn
(
)
1
O
ti đim I ct đường tròn
(
)
O
ln lượt ti các đim
, '
A A
. Đường thng
1
AM
ct li đường tròn
(
)
1
O
ti đim
1
N
, đường thng
2
AM
ct li đường
tròn
(
)
2
O
ti đim
2
N
.
1. Chng minh rng t giác
1 1 2 2
M N N M
ni tiếp đường thng
OA
vuông góc vi đường
thng
1 2
N N
.
2. K đường kính
PQ
ca đường tròn
(
)
O
sao cho
PQ
vuông góc vi
AI
(đim
P
nm trên
cung
1
AM
không cha đim
2
M
). Chng minh rng nếu
1 2
,
PM QM
không song song thì các
đường thng
1
,
AI PM
2
QM
đồng quy.
Câu 5 (1,0 đim)
Tt c các đim trên mt phng đều được màu, mi đim được bi mt trong 3 màu xanh,
đỏ, tím. Chng minh rng khi đó luôn tn ti ít nht mt tam giác cân, 3 đỉnh thuc các đim
ca mt phng trên mà 3 đỉnh ca tam giác đó cùng màu hoc đôi mt khác màu.
—Hết—
Cán b coi thi không gii thích gì thêm.
H và tên thí sinh:……….………..…….…….….….; S báo danh……………….
S GD&ĐT VĨNH PHÚC
———————
K THI CHN HSG LP 9 THCS NĂM HC 2011-2012
HƯỚNG DN CHM MÔN: TOÁN
———————————
I. LƯU Ý CHUNG:
- Hướng dn chm ch trình bày mt cách gii vi nhng ý cơ bn phi có. Khi chm bài hc sinh
làm theo cách khác nếu đúng và đủ ý thì vn cho đim ti đa.
- Đim toàn bài tính đến 0,5 và không làm tròn.
- Vi bài hình hc nếu thí sinh không v hình phn nào thì không cho đim tương ng vi phn đó.
II. ĐÁP ÁN:
Câu
Ý Ni dung trình bày Đim
1 1,5 đim
Nhn xét. Nếu
1
x y
+ =
thì
(
)
(
)
1
f x f y
+ =
.
Tht vy, ta có
( )
( )
( ) ( )
( )
( )
3
3
3 3
3 3
1
1
1 1
x
x
f x f y f x
x x x x
= = =
+ +
0,5
suy ra
( ) ( ) ( ) ( )
( )
( )
( )
3
3
3 3
3 3
1
1 1
1 1
x
x
f x f y f x f x
x x x x
+ = + = + =
+ +
.
Vy, nhn xét được chng minh. Ta có
1 1
2 2
f
=
.
0,5
Theo nhn xét trên ta có:
1 2011 2 2010
...
2012 2012 2012 2012
1005 1007 1006 1
1005 1005,5
2012 2012 2012 2
A f f f f
f f f f
= + + + + +
+ + = + =
0,5
2 1,5 đim
Điu kin:
0, 1
x x
>
. Khi đó ta có
Rút gn biu thc ta được
2
1
x
P
x x
+
=
+ +
0,5
Ta
(
)
1 2 0
Px P x P
+ + =
, ta coi đây phương trình bc hai ca
x
.
Nếu
0 2 0
P x
= =
vô lí, suy ra
0
P
nên để tn ti
x
thì phương trình
trên có
( ) ( )
2
1 4 2 0
P P P
=
( )
2
2 2
4 4
3 6 1 0 2 1 1
3 3
P P P P P
+ + +
0,5
1
Do P nguyên nên
( )
2
1
P
bng 0 hoc 1
+) Nếu
( )
2
1 0 1 1
P P x
= = =
không tha mãn.
+) Nếu
( )
2
2
1 1 2 2 0 0
0
P
P P x x x
P
=
= = + = =
=
không tha mãn
Vy không có giá tr nào ca x tha mãn.
0,5
1,5 đim
2
Nếu
6 ( 6) 1
x y x y x y
+ + > +
phương trình nghim. Do đó
6
x y
< +
2 6 3
x y y x x
+ < + <
{1;2}
x
0,5
Vi
1
x
=
thay vào phương trình ban đầu ta được:
( ) ( )
(
)
3
2 2
1 ( 5) 3 5 8 0 3
y y y y y y
+ = + + + = =
suy ra phương trình có
nghim
(
)
; (1; 3)
x y = .
0,5
Vi
2
x
=
thay vào phương trình ban đầu ta được:
( )
3
2 3 2
2 ( 4) 5 4 8 0
y y y y y
+ = + + + =
phương trình này nghim do
1
y
.
Vy phương trình đã cho có nghim
(
)
; (1; 3)
x y = .
0,5
1,5 đim
Ta có:
( )
2
2012
abc bcd cda dab a b c d
= + + +
( )( ) ( )( )
(
)
2
1 1
ab c d cd a b
= + + +
0,5
( ) ( ) ( ) ( )
2 2 2 2
1 1ab a b cd c d
+ + + +
0,5
3
(
)
(
)
(
)
(
)
(
)
(
)
2 2 2 2 2 2 2 2 2 2 2 2
1 1 1 1 1 1
a b a b c d c d a b c d
= + + + + + + = + + + +
Suy ra
(
)
(
)
(
)
(
)
2 2 2 2
1 1 1 1 2012
a b c d+ + + +
0,5
S
N
2
N
1
I
O
2
O
1
M
2
M
1
O
Q
P
A'
A
1 2,0 đim
+) Ta có
2
1 1 2 2
. .AM AN AM AN AI
= =
1 2
AN N
đồng dng vi
2 1
AM M
0,5
suy ra
0
1 2 2 1 1 1 2 2 1
180
AN N AM M M N N AM M= + = hay t giác
1 1 2 2
M N N M
ni tiếp.
0,5
4
+) Ta
1 2 2 1
AN N AM M
=
1
1
2
AOM
= tam giác
1
AOM
cân ti
O
nên
0
1
1
180
2
AOM
M AO
=
0,5
Do đó ta được
0
1 2 1 1 2
90 .
AN N M AO OA N N
+ =
0,5
2 1,0 đim
Gi
S
là giao đim ca
1
PM
2
QM
.
Ta
2 2
, ,
O O M
thng hàng và
2
O I
song song vi
OP
2 2 2
IO M POM
=
(1). Mt khác tam giác
2 2
O IM
cân ti
2
O
, tam giác
2
OPM
cân ti
O
kết
hp vi (1) ta được
2 2 2
O IM OPM
= suy ra
2
, ,
P I M
thng hàng. Tương t ta
1
, ,
Q I M
thng hàng.
0,5
Do
PQ
đường kính ca đường tròn
(
)
O
suy ra
0
1 2
90
PM Q PM Q= =
I
là trc tâm ca tam giác
SPQ
suy ra
AI
đi qua
S
hay ba đường thng
1 2
, ,
AI PM QM
đồng quy.
0,5
1,0 đim
E
D
C
B
A
Xét ngũ giác đều ABCDE, ta nhn thy ba đỉnh bt ca ngũ giác luôn to
thành mt tam giác cân.
Do đó khi tô 5 đỉnh A, B, C, D, E bng 3 màu xanh, đỏ và tím s xy ra hai kh
năng sau:
+) Nếu tô 5 đỉnh A, B, C, D, E bi đủ ba loi màu đã cho thì tn ti 3 đỉnh
màu khác nhau và to thành mt tam giác cân.
0,5
5
+) Nếu tô 5 đỉnh A, B, C, D, E bi nhiu nht 2 màu thì ít nht 3 đỉnh cùng
màu và to thành mt tam giác cân.
Vy, trong mi trường hp luôn tn ti ít nht mt tam giác cân, 3 đỉnh
được tô bi cùng mt màu hoc đôi mt khác màu.
0,5
UBND tnh Thái Nguyên Cng hoà xã hi ch nghĩa vit nam
S Giáo dc & Đào to Độc lp - T do - Hnh phúc
K THI CHN HC SINH GII CP TNH LP 9 THCS
Tháng 3 / 2012
MôN: Toán
(Thi gian làm bài: 150 phút không k thi gian giao đề)
Đề chính thc
Bài 1. Chng minh rng tng bình phương ca 5 s nguyên liên tiếp không s chính
phương.
Bài 2. Gii phương trình và h phương trình sau:
a,
3
2 x
+
1x
= 1
b,
=+
=+
=+
2
2
2
2
2
2
yxz
xyz
zxy
Bài 3. Cho
ABC có 3 góc đu nhn. Gi O tâm đường tròn ngoi tiếp
ABC; R, r
theo th t độ dài bán kính đường tròn ngoi tiếp đường tròn ni tiếp
ABC; M, N, P ln lượt là hình chiếu vuông góc ca O trên AB, BC và AC.
a, Chng minh: BN . OM + BM . ON = BO . MN
b, Đặt ON = d
1
; OM = d
2
;
OP = d
3
.
Tính R + r theo d
1
, d
2
, d
3
?
Bài 4. Ly mt s t nhiên 2 ch s chia cho s 2 ch s viết theo th t ngược
li thì được thương 4 dư 15. Nếu ly s đó tr đi 9 tđược mt s bng
tng bình phương ca 2 ch s to thành s đó. Tìm s t nhiên y?
-------------- Hết ---------------
H tn th sinh:..........................................................S bo danh:.........................
áp án Đ1
UBND tnh Thái Nguyên Cng hoà xã hi ch nghĩa vit nam
S Giáo dc & Đào to Độc lp - T do - Hnh phúc
K THI CHN HC SINH GII CP TNH LP 9 THCS
Tháng 3 / 2012
hng dn chm toán 9
Bài 1: 3,5 đim
C1: Gi 5 s nguyn lin tiếp là n-2, n-1, n, n+1, n+2 vi n nguyn, d thy tng cc
bnh phương ca 5 s đó là 5(n
2
+ 2) chia hết cho 5 nhưng không chia hết cho 25 nên
không th là s chính phương.
C2: Xét tính chn l ca 5 s nguyên liên tiếp đó.
Bài 2: a. 3,5 đim
Đặt a =
3
x2
b = 1x 0
Ta có :
( )
I
1ba
1
2
b
3
a
=+
=+
a
3
+ a
2
- 2a = 0
a ( a
2
+ a -2) = 0
=+
=
02a
2
a
0a
H ( I ) có ba nghim : ( 0 ; 1) ; ( 1 ; 0) ; ( -2 ; 3)
nên phương trình đã cho có nghim : 2 ; 1 ; 10
b, 3,5 đim
T (1) ; (2) ta có : (x – z)(x – y + z) = 0 (4)
T (2) và (3) ta có: ( y - x)(x + y –z) = 0 (5)
T (3) ; (4) ; (5) ta có h :
( )
( )
( )( )
=+
=+
=+
2
2
yxz
0zyxxy
0zyxzx
( )
( )
+ =
+ =
+ =
2
xy z 2 1
2
yz x 2 2
2
xz y 2 (3)
Để gii h trên ta gii 4 h
( ) ( )
B
2
2
yxz
0zyx
0zx
A
2
2
yxz
0xy
0zx
=+
=+
=
=+
=
=
( ) ( )
D
2
2
yxz
0zyx
0zyx
C
2
2
yxz
0zyx
0xy
=+
=+
=+
=+
=+
=
Gii 4 h trên ta được 8 b nghim ca h phương trình :
(1; 1; 1) ; ( -1;-1; -1 ) ;
(
)
2;0;2 ;
(
)
2;0;2
(
)
0;2;2 ;
(
)
0;2;2 ;
(
)
2;2;0 ;
(
)
2;2;0
Bài 3: 6 đim
a, Ta có BMO = BNO = 90
0
=> OMBN là t giác ni tiếp
Trên BO ly E sao cho BME = OMN
=>
BME
NMO
=>
NO
NM
BE
BM
=
=> BM . NO = BE . NM
Chng minh tương t BN. OM = OE .MN
Cng theo tng vế BM .ON +BN . ON = MN . BO
b. Đặt a , b , c là đ dài các cnh BC , AC , AB ca
ABC
theo câu a ta có d
1
.
2
a
+ d
2
2
c
= R .
2
b
áp dng câu a đối vi các t giác OMAP , ONCD ta có
d
1
.
2
b
+ d
3
.
2
c
= R.
2
a
d
3
.
2
a
+ d
2
.
2
b
= R.
2
c
Cng theo tng vế :
2
R
. ( a+b+c) =
2
1
. ( d
1
b + d
2
b + d
3
c + d
3
a + d
1
a + d
2
c)
mt khác S
ABC
=
2
r
. ( a+b +c ) =
2
1
.( d
1
c + d
3
b + d
2
a
)
Do đó ( R + r )( a+b+c) = ( a+b+c)( d
1
+d
2
+d
3
)
hay R + r = d
1
+ d
2
+ d
3
Bài 4: 3,5 đim
A
0
B
N
O
d1
M
P
E
d3
d2
C
Gi s phi tm (a , b N; 1 a, b 9)
Ta cú h
2 2
4. 15(1)
9 (2)
ab ba
ab a b
= +
= +
C1 : T (1) ta thy nếu
=> a = b = 9 khng thamón (1) và (2)
Vy b = 1 thay b = 1 vào (2) ta đưc:
9 = a
2
+ 1
10a + 1 – 9 = a
2
+ 1
a
2
– 10a + 9 = 0
a
1
= 1; a
2
= 9
(*) a = 1 => a = b loi
(*) a = 9 => = 91 tha món (1)
91 = 4 * 19 + 15
Vy: S phi tm là 91
C2: T h trn cú th dng PP thế để gii. Rút 1 n t PT (1) thế vào PT (2) ta s
được mt PT bc 2. Gii PT bc 2 đó s tm được nghim.
Ch ý: - Hc sinh làm theo cách khác mà đúng vn cho đim ti đa.
- GK có th bàn để thng nht đim cho tng phn nh ca mi bài.
------------------------------------------------------------------------------
S GD&ĐT PHÚ TH
ĐỀ CHÍNH THC
ĐỀ THI CHN HC SINH GII CP TNH
NĂM HC 2012 - 2013
MÔN: TOÁN - LP 9
Thi gian làm bài 150 phút không k thi gian giao đề
Câu1( 3,0 đim)
1) Gii phương trình nghim nguyên
2
8 3x 5 25
x y y
=
2)Tìm tt c s nguyên dương n sao cho A=
.4 3 7
n n
n +
M
Câu 2( 4,0 đim)
1) Rút gn biu thc: A=
2 10 30 2 2 6 2
:
2 10 2 2 3 1
+
2) Cho các s thc dương a,b,c,x,y,z khác 0 tho mãn .
2 2 2
x
x yz y z z xy
a b c
= =
Chng minh rng
2 2 2
a bc b ca c ab
x y z
= =
Câu 3( 4,0 đim)
1) Cho phương trình:
2
6x 0
x m
=
(Vi m là tham s). Tìm m để phương trình đã
cho có hai nghim x
1
x
2
tho mãn
2 2
1 2
12
x x
=
2) Gii h phương trình:
3 3 3
2 2
8x 27 18
4x 6x
y y
y y
+ =
+ =
Câu 4( 7,0 đim)
1) Cho đường tròn (O) đường kính BD=2R, dây cung AC ca đường tròn (O) thay đổi
nhưng luôn vuông góc và ct BD ti H. Gi P,Q,R,S ln lượt là chân các đường vuông
góc h t H xung AB,AD,CD,CB.
a) CMR:
2 2 2 2
D
HA HB HC H
+ + +
không đổi.
b) CMR :
RS
PQ
là t giác ni tiếp.
2) Cho hình vuông ABCD và MNPQ có bn đỉnh M,N,P,Q ln lượt thuc các cnh
AB,BC,CD,DA ca hình vuông. CMR:
D
ABC
S
4
MN NP PQ QM
AC
+ + +
Câu 5( 2,0 đim)
Cho a,b,c là các s thc dương. CMR:
3 2 3 2a 3 2 6
ab bc ca a b c
a b c b c c a b
+ +
+ +
+ + + + + +
---Hêt—
Hướng dn
Câu1.1)
2
8 3x 5 25
x y y
=
Z
x
xy
x
x
yxxy
+
=
+
==+
5
3
25
40249
5
3
258
258)53(
2
2
Khi 3x+5 là ước 25 t đó tìm được
{
}
)5;0();7;2();31;10();( yx
( cách khac nhân 2 vế vi 9 đưav tích)
1.2) Vi n chn n=2k thì
( )
Nmmtn
t
kkkkA
kkkkk
+==
=+++=+= 614114
2
17
7127)916(4).12(34.2
222
MM
Vi n l n=2k+1
(
)
NmmntkkkkA
kkkkk
+==++=++=
+++++
1147727)34(4.234).12(
1212121212
MM
Vy
614
+
=
mn
hoc
114
+
=
mn
( vi mi n
)N
thì A chia hết cho 7
Câu2.1)
2 10 30 2 2 6 2
:
2 10 2 2 3 1
+
=
2
1
2
13
.
2
13
2
13
.
4
324
2
13
.
2
32
2
13
.
)15(22
)15(6)15(22
=
+
=
+
=
+
=
+
2.2)
2 2 2
x
x yz y z z xy
a b c
= =
)3(
)3(2
:
)2(
)3(2
:
)1(
)3(2
333
2
233222224
2
333
2
233222224
2
333
2
233222224
2
222
xyzzyxz
abc
xyzzyzxyx
ab
yxxyzZ
c
Tuongtu
xyzzyxy
acb
zxyyzyxzx
ac
zxxzyy
b
Tuongtu
xyzzyxx
bca
yzxxzxyzy
bc
zyyzxx
a
xyz
c
xzy
b
yzx
a
++
=
+
=
+
++
=
+
=
+
++
=
+
=
+
=
=
T (1) (2) (3) ta co ĐPCM
Câu 3.1) Để phương trình có nghim
90
/
m
(*)
Mt khác ta phi
8
2
.
4
2
.
6
12
.
6
2
21
1
21
21
21
2
2
2
1
21
21
=
=
=
=
=
=
=+
=
=
=+
m
x
mxx
x
xx
mxx
xx
xx
mxx
xx
TM ĐK (*)
3.2)Gii h phương trình
=+
=+
22
333
64
18278
yxyx
yyx
HD y =0 không nghim ca h chia 2 vế PT(1) cho y
3
PT(2) cho y
2
Ta
h
=+
=+
164
18
27
8
2
2
3
3
y
x
y
x
y
x
Đ
t
=
=
b
y
ax
3
2
ta có h
=
=+
=+
=+
1
3
3
18
22
33
ab
ba
abba
ba
H có 2 nghim
+
+
53
6
;
4
53
;
53
6
;
4
53
),( yx
Câu 4.1)
O
H
R
S
P
Q
D
C
B
A
a) theo Pitago
;;;;
222222222222
ADHDHACDHDHCBCHBHCABHBHA =+=+=+=+
suy ra đpcm
b)T giác HPBS ni tiếp
DBCHBSHPS
=
=
T giác HPAQ là hình ch nht
CBDCADHAQHPQ
=
=
=
Do đó
CBCHPQHPSSPQ
=
+
=
2
Tương t
BDCSQR
=
2
Do đó
00
180180 =+=+ SRQSPQBDCDBC
nên t giác PQRS ni tiếp ( đ/lí
đảo)
4.2)
L
K
P
Q
I
C
N
D
M
A
B
Cách 1 Gi T, K, L là trung đim MQ, MP, NP theo t/c đường trung bình và trung tuyến tam
giác vuông ta có
ACAIIKCLKLQMPQNPMN 2)(2
+
+
+
=
+
+
+
t đó suy ra đpcm
Cách 2 Ta có theo Pitago
2
2
)(
2
222
BNBM
MN
BNBM
BMBNMN
+
+
+=
( áp dng BĐT Bunhiacoopsky
Tương T
2
;
2
;
2
AMAQ
MQ
DQDP
PQ
NPCN
NP
+
+
+
Nên
( )
dpcmaQMPQNPMN
a
a
aAMQADQPDCPNCNBBM
QMPQNPMN
=+++
==
+
+
+
+
+
+
+
+++
2
4
2
22
2
4
2
Du “=” xy ra khi MNPQ là hình ch nht
Câu 5
Cho a,b c>0 .Chng minh rng:
6
2
3
3
2
2
3
cba
c
b
a
ca
c
b
a
bc
c
b
a
ab
+
+
++
+
++
+
++
D đoán a=b=c tách mu để a+c=b+c=2b
Tacó áp dng BĐT
++
++
++++
zyxzyxzyx
zyx
111
9
11
9
111
)(
1 1 1 1
(1)
3 2 ( ) ( ) 2 9 2 9 2
ab ab ab ab ab a
a b c a c b c b a c b c b a c b c
= + + = + +
+ + + + + + + + + +
Tương t
1 1 1 1
(2)
2 3 ( ) ( ) 2 9 2 9 2
1 1 1 1
(2)
3 2 ( ) ( ) 2 9 2 9 2
bc bc bc bc bc b
a b c a b a c c a c b c b a b b c
ac ac ac ac ac c
a b c a b b c a a b b c a a b b c
= + + = + +
+ + + + + + + + + +
= + + = + +
+ + + + + + + + + +
T (1) (2) (3)
629
1 cbacba
ca
abbc
cb
acab
ba
bcac
P
++
=
++
+
+
+
+
+
+
+
+
+
Du “=” xy ra khi a=b=c
GV Nguyn Minh Sang THCS Lâm Thao-Phú Th
S GD&ĐT K THI CHN HC SINH GII CP TNH LP 9 THCS
QUNG BÌNH NĂM HC 2010 - 2011
Môn thi: Toán
ĐỀ THI CHÍNH THC (Khóa ngày 30 tháng 3 năm 2011)
S BÁO DANH:…………….. Thi gian làm bài: 150 phút (không k thi gian giao đề)
Câu 1:(2.5 đim) Cho biu thc
2
4 4 4 4
8 16
1
x x x x
A
x x
+ +
=
+
vi
4 8
x
<
a) Rút gn biu thc A.
b) Tìm x nguyên để A có giá tr nguyên.
Câu 2:(2.5 đim) S đo hai cnh góc vuông ca mt tam giác là nghim ca phương
trình bc hai
2
( 2) 2( 1) 0
m x m x m
+ =
. Xác định m đ s đo đường cao ng
vi cnh huyn ca tam giác đã cho là
2
5
Câu 3:(3.0 đim) Cho hai đường tròn (O) và (O’) ct nhau ti hai đim A và B. Tiếp
tuyến chung gn B ca hai đường tròn ln lưt tiếp xúc (O) (O’) ti C D.
Qua A k đường thng song song CD ct (O) và (O’) ln lượt ti M N. Các
đường thng BC, BD ln lượt ct MN ti P Q. Các đưng thng CM, DN ct
nhau ti E. Chng minh rng:
a) Các đường thng AE và CD vuông góc nhau.
b) Tam giác EPQ cân.
Câu 4:(1.0 đim) Cho
, , 0
x y z
>
tha mãn:
2 2 2
3
x y z
+ + =
. Chng minh:
z x
3
x y
xy y z
z
+ +
Câu 5:(1.0 đim) Cho a, b, c, d là các s nguyên tha mãn :
5 5 5 5
4( )
a b c d
+ = +
Chng minh rng :
a b c d
+ + +
chia hết cho 5.
--------------------HT----------------------
S GD&ĐT K THI CHN HC SINH GII CP TNH LP 9 THPT
QUNG BÌNH NĂM HC 2012- 2013
Môn thi: Toán
ĐỀ THI CHÍNH THC (Khóa ngày 27 tháng 3 năm 2013)
S BÁO DANH:…………….. Thi gian làm bài: 150 phút (không k thi gian giao đề)
Câu 1:(2.0 đim)
Cho biu thc:
26 19 2 3
2 3 1 3
x x x x x
P
x x x x
+
= +
+ +
a) Rút gn P.
b) Tìm x để P đạt giá tr nh nht.
Câu 2:(2.0 đim)
Cho phương trình
2
2 4 0
x mx m
+ =
a) Tìm m để phương trình có hai nghim phân bit
1 2
,
x x
tha mãn
3 3
1 2
26
x x m
+ =
b) Tìm m nguyên để phương trình có hai nghim nguyên.
Câu 3:(3,5 đim)
Cho tam giác ABC đều c định ni tiếp trong đường tròn (O). Đường thng d
thay đổi nhưng luôn đi qua A ct cung nh AB ti đim th hai E (E
A).
Đường thng d ct hai tiếp ti B C ca đưng tròn (O) ln lượt ti M N. MC
ct BN ti F. Chng minh rng:
a) Tam giác CAN đồng dng vi tam giác BMA, tam giác MBC đồng dng vi tam
giác BCN.
b) T giác BMEF là t giác ni tiếp.
c) Chng minh đường thng EF luôn đi qua mt đim định khi d thay đổi nhưng
luôn đi qua A.
Câu 4:(1,5 đim)
Cho c¸c sè thùc d−¬ng a, b, c tho¶ mn a + b + c =6. Chng minh rng:
5 4 3
6
1 2 3
b c c a a b
a b c
+ + + + + +
+ +
+ + +
. DÊu ®¼ng thøc x¶y ra khi nµo?
Câu 5:(1,0 đim)
Cho n là s t nhiên ln hơn 1. Chng minh rng
n4
4
n
+
là hp s.
--------------------HT----------------------
Trang:
1
-
Đáp án Toán 11
S GD&ĐT K THI CHN HC SINH GII CP TNH LP 9 THPT
QUNG BÌNH NĂM HC 2012 - 2013
Môn thi: Toán
(Khóa ngày 27 tháng 3 năm 2013)
HƯỚNG DN CHM
(Đáp án, hướng dn này có 4 trang)
yªu cÇu chung
* Đáp án ch trình bày mt li gii cho mi bài. Trong bài m ca hc sinh yêu cu phi lp
lun lô gic cht ch, đầy đủ, chi tiết và rõ ràng.
* Trong mi bài, nếu hc sinh gii sai bước gii trước thì cho đim 0 đối vi nhng bước
gii sau có liên quan. câu 3 nếu hc sinh không v hình hoc v hình sai thì cho đim 0.
* Đim thành phn ca mi bài nói chung phân chia đến 0,25 đim. Đối vi đim thành phn
là 0,5 đim thì tu t giám kho thng nht để chiết thành tng 0,25 đim.
* Hc sinh li gii khác đáp án (nếu đúng) vn cho đim ti đa tu theo mc đim ca
tng bài.
* Đim ca toàn bài là tng (không làm tròn s) ca đim tt c các bài.
Câu
Ni dung Đim
a) ĐK:
0 1
x
.Ta có:
26 19 2 3
( 1)( 3) 1 3
26 19 2 ( 3) ( 3)( 1)
( 1)( 3)
26 19 2 6 4 3
( 1)( 3)
16 16 ( 1)( 16) 16
( 1)( 3) ( 1)( 3) 3
x x x x x
P
x x x x
x x x x x x x
x x
x x x x x x x
x x
x x x x x x x
x x x x x
+
= +
+ +
+ + +
=
+
+ + +
=
+
+ + +
= = =
+ + +
1,0 đim
0,25
0,25
0,25
0,25
1
b)
16 25 25
3 3 6
3 3 3
25
2 ( 3) 6 10 6 4
3
x
P x x
x x x
x
x
+
= = + = + +
+ + +
+ = =
+
Vy GTNN ca P = 4 khi
25
3 4
3
x x
x
+ = =
+
1,0 đim
0,5
0,25
0,25
Trang:
2
-
Đáp án Toán 11
2
a)
2
2 4 0
x mx m
+ =
Ta có:
2
2
1 15
' 4 0
2 4
m m m m
= + = + >
Vy phương trình luôn có 2 nghim phân bit vi mi m.
Theo định lý Viet:
1 2 1 2
2 ; 4
x x m x x m
+ = =
(
)
3
3 3
1 2 1 2 1 2 1 2
3 2
26 3 ( ) 26
8 6 ( 4) 26 (8 6 2) 0
1
0; 1;
4
x x m x x x x x x m
m m m m m m m
m m m
+ = + + =
= =
= = =
1,0 đim
0,25
0,25
0,25
0,25
b) Gi
1 2 1 2
, ( )
x x x x
< là hai nghim nguyên ca phương trình.
Ta có:
1 2 1 2
2 ; 4
x x m x x m
+ = =
.
Suy ra
1 2 1 2 1 2 1 2 1 2
2 8 2( ) 4 1 15 (2 1)(2 1) 15
x x x x x x x x x x
+ = + = =
.
TH1:
1 1
2 2
2 1 1 0
4
2 1 15 8
x x
m
x x
= =
=
= =
TH2:
1 1
2 2
2 1 5 2
0
2 1 3 2
x x
m
x x
= =
=
= =
TH3:
1 1
2 2
2 1 15 7
3
2 1 1 1
x x
m
x x
= =
=
= =
TH4:
1 1
2 2
2 1 3 1
1
2 1 5 3
x x
m
x x
= =
=
= =
Th li m=0, m=1, m=-3,m=4 tha mãn điu kin bài toán.
1,0 đim
0,25
0,5
0,25
3
C
N
I
F
M
O
B
A
E
3,5 đim
0,5
Trang:
3
-
Đáp án Toán 11
a) Ta có: AC//BM suy ra
BMA CAN
=
AB//CN suy ra
BAM CNA
=
Do đó tam giác CAN đồng dng vi tam giác BMA
Suy ra:
MB AB MB BC
AC NC BC CN
= =
Mt khác
0
120
MBC BCN = =
Suy ra tam giác MBC đồng dng vi tam giác BCN.
0,5
0,25
0,25
0,25
b)
0 0
180 60
BFM BCM NBC BCM BMC MBC = + = + = =
Mt khác
0
60
BEM BCA = =
(do t/c góc ngoài ca t giác ni tiếp)
Suy ra
0
60
BFM BEM = =
. Do đó t giác BMEF ni tiếp.
0,5
0,25
0,25
c) Gi I là giao đim EF vi BC.
Ta có
IBF BMF
=
(câu a), suy ra IB là tiếp tuyến ca đường tròn ngoi
t giác BMEF.
Tương t chng minh được IC là tiếp tuyến ca đường tròn ngoi t giác
CNEF.
T đó:
2 2
. ; .
IB IE IF IC IE IF IB IC
= = = hay I là trung đim BC.
Vy d luôn đi qua đim c định là I.
0,25
0,25
0,25
4
Đặt
1; 2; 3
x a y b z c
= + = + = +
. (x, y, z >0)
2 . 2 . 2 . 6
y z z x x y y x x z y z
VT
x y z x y z x z y
y x z x y z
x y x z z y
+ + +
= + + = + + + + +
+ + =
Du bng xy ra khi x=y=z, suy ra a=3, b=2, c=1
1,5 đim
0,5
0,5
0,25
0,25
5
n s t nhiên ln hơn 1 nên n có dng n = 2k hoc n = 2k + 1, vi k là
s t nhiên ln hơn 0.
- Vi n = 2k, ta
k24n4
4)k2(4n +=+
ln hơn 2 chia hết cho 2. Do
đó
n4
4
n
+
là hp s.
-Vi n = 2k+1, tacó
( )( )
( )( )
4 4 2 4 2 2 2 2
2 2
2 2
4 4 .4 (2.4 ) ( 2.4 ) (2. .2 )
2.4 2. .2 2.4 2. .2
( 2 ) 4 ( 2 ) 4
n k k k k
k k k k
k k k k
n n n n n
n n n n
n n
+ = + = + = +
= + + +
= + + +
Mi tha s đều ln hơn hoc bng 2. Vy n
4
+ 4
n
là hp s
1,0 đim
0,25
0,25
0,25
0,25
NGUYN VĂN HÂN TRƯỜNG THCS NGUYN CÁT – TNH HÒA – SƠN
TNH – QUNG NGÃI.
S GIÁO DC VÀ ĐÀO TO K THI CHN HC SINH GII LP 9 CP TNH NĂM HC 2013-2014
QUNG NGÃI Ngày thi : 22/3/2014
Môn : TOÁN
Thi gian làm bài: 150 phút
Bài 1:(4 đim)
a) Cho a;b là hai s nguyên dương khác nhau, tho mãn 2a
2
+a = 3b
2
+b.
Chng minh
a b
2a+2b+1
là phân s ti gin.
b) Tìm các cp s nguyên dương (x; y) tho mãn: 15x
2
− 7y
2
= 9
Bài 2: (4 đim)
a) Cho
3 3
x
2 2
; x0
3 2x 3 2x a
+ =
.
Tính giá tr biu thc
2
6 2 9 4x
P
x
+
= theo a.
b) Cho a,b,c là 3 s dương tho mãn
1 1 1
2
1 a 1 b 1 c
+ + =
+ + +
.
Tìm giá tr ln nht ca Q=abc
Bài 3: (4 đim)
a) Gii phương trình:
( )( ) ( )
x 2
x 1 x 2 4 x 1 12
x 1
+
+ + =
.
b) Gii h phương trình:
1
2 x 1 3
x y
+ =
+
1
2 y 1 1
x y
=
+
.
Bài 4: (6 đim)
Cho na đường tròn tâm O đường kính AB c định. EF lày cung di động trên na đường tròn
đó, sao cho E thuc cung AF và
AB
EF= R
2
=
. Gi H là giao đim ca AF và BE; C là giao đim ca AE
và BF; I là giao đim ca CH và AB.
a) Tính s đo
·
CIF
b) Chng minh rng biu thc AE.AC+BF.BC có giá tr không đổi khi EF di động trên na đường
tròn.
c) Xác định v trí ca EF trên na đường tròn để t giác ABFE có din tích ln nht. Tính din tích
ln nht đó theo R.
Bài 5: (2 đim)
Tìm cnh ca hình vuông nh nht, biết rng: hình vuông đó cha 5 đường tròn có bán kính bng 1
và 5 đường tròn này đôi mt không có quá 1 đim chung.
--------------Hết-------------
ĐỀ CHÍNH THC
NGUYN VĂN HÂN TRƯỜNG THCS NGUYN CÁT – TNH HÒA – SƠN
TNH – QUNG NGÃI.
BÀI GII ĐỀ THI CHN HC SINH GII LP 9 TNH QUNG NGÃI NĂM HC 2013-2014
Môn : TOÁN Ngày thi : 22/3/2014
Câu 1:
1) 2a
2
+a = 3b
2
+b 2a
2
+a −2b
2
−b = b
2
(a−b)(2a+2b+1) = b
2
Gi (a−b,2a+2b+1) = d
Ta có: a – b d, 2a+2b+1d (a−b) (2a+2b+1) d
2
b
2
d
2
bd
Mà a – b d ad
ad; bd mà 2a+2b+1 d nên 1d d=1
Vy phân s đã cho ti gin.
2) Gi s cp s nguyên dương (x; y) là nghim ca phương trình:
15x
2
− 7y
2
= 9 (1) =>15x
2
− 9 =7y
2
=>7y
2
3 => y
2
3 => y 3
Đặt y = 3z và thay vào (1) ta có 15x
2
− 63z
2
= 9 =>5x
2
− 21z
2
= 3(2) => x 3
Đặt x = 3t và thay vào (2) ta có 45t
2
− 21z
2
= 3=>15x
2
− 7z
2
= 1(3)
Nếu z 0(mod3) => VP 0(mod3). VT 1(mod3). Vô lí
Nếu z 1(mod3) => z
2
1(mod3) => − 7z
2
2(mod3) .
VP 2(mod3). VT 1(mod3). Vô
Nếu z 2(mod3) => z
2
1(mod3) =>− 7z
2
2(mod3)
VP 2(mod3). VT 1(mod3). Vô
Vy không tìm được cp s nguyên dương (x; y) nào là nghim ca phương trình đã cho.
Câu 2:
a) Cho
3 3
x
2 2
; x0 và
3 2x 3 2x a
+ =
.Tính giá tr biu thc
2
6 2 9 4x
P
x
+
= theo a.
( ) ( )
( )
( )
2
3 2x 3 2x
3 2x+2 3 2x 3 2x 3 2x
3 2x 3 2x
P
x x x
4x 4
.
a
x 3 2x 3 2x
+ +
+ + +
+ +
= = =
= =
+
b) Cho ba s dương a , b , c và tha mãn điu kin :
1 1 1
2
1 1 1
a b c
+ + =
+ + +
.Tìm giá tr ln
nht ca Q = a.b.c
Gii :Ta có :
1 1 1
1 1 2
1 1 1 1 1 (1 )(1 )
b c bc
a b c b c b c
= + = +
+ + + + + + +
Tương t :
1 1
2 , 2
1 (1 )(1 ) 1 (1 )(1 )
ca ab
b c a c a b
+ + + + + +
Nhân các bt đẳng thc va nhn được ta có :
1 1 1
. . 8
1 1 1 (1 )(1 )(1 )
abc
a b c a b c
+ + + + + +
Hay : abc
1
8
. Du = xãy ra khi a = b = c =
1
2
. Vy maxQ =
1
8
Bài 3: (4 đim)
NGUYN VĂN HÂN TRƯỜNG THCS NGUYN CÁT – TNH HÒA – SƠN
TNH – QUNG NGÃI.
a) Gii phương trình
( ) ( ) ( )
x 2
x 1 x 2 4 x 1 12
x 1
+
+ + =
. ĐK : x - 2 ; x > 1.
( ) ( ) ( )( )
x 1 x 2 4 x 2 x 1 12 0
=> + + + =
.
Đặt
( )( )
t x 2 x 1
= + -
ta có phương trình t
2
+ 4t – 12 = 0 => t =2 hoc t = - 6 (loi)
(x+2)(x-1) = 2 => x
2
+ x – 6 = 0 => x = 2(nhn) hoc x = - 3 (nhn)
b)Gii h phương trình:
1
2 x 1 3
x y
1
2 y 1 1
x y
+ =
+
=
+
.
( )( )
2 2
3 1
1 1 3
2 (Công vê)
2 x 1 3 1
x y x y 2 x 2 y
2 x
1 1
2 3 1
1
1
( tru vê)
2 y 1 1
x y
2 y x y
2 x 2 y
x y
4 9 1
( Nhân vê) =>x 8xy-9y 0 x y x 9y 0
x y 4x 4y
x y;x 9y(loai)
= +
+ = + =
+ +
=> =>
=
=
=
+
+
+
=> = + = => + =
+
=> = =
=
3 1 2
2 2 x 1.
2 x 2 x x
> = + => = => =
Vy nghim ca h là x = y = 1.
Bài 4: (6 đim)
a)
Tính s đo
·
CIF
T giác BFHI ni tiếp =>
·
·
º
0
1
HIF HBF sd EF 30
2
= = =
(tam giác OEF đều)
b) Chng minh rng biu thc AE.AC+BF.BC có giá tr không đổi khi EF di động trên na
đường tròn.
Ta có AE.AC = AC(AC –CE) = AC
2
– AC.AE
BF.BC = BC(BC –CF) = BC
2
– BC.CF
AE.AC+BF.BC = AC
2
+ BC
2
– AC.AE – BC .CF
MÀ AC.AE = BC.CF =CO
2
– R
2
2 2 2
2
2AC 2BC AB
CO
4
+ -
=
=> AC
2
+ BC
2
=2CO
2
+
2
AB
4
Suy ra : AE.AC+BF.BC = 2CO
2
+
2
AB
4
– CO
2
+ R
2
– CO
2
+ R
2
= 3R
2
AE.AC+BF.BC= 3R
2
C định.
c) Xác định v trí ca EF trên na đường tròn để t giác ABFE
có din tích ln nht. Tính din tích ln nht đó theo R.
Ta có S
ABEF
= S
AOF
+ S
FOE
+ S
EOB
I
H
C
A
O
B
E
F
OQ
N
P
M
F
E
C
B
A
NGUYN VĂN HÂN TRƯỜNG THCS NGUYN CÁT – TNH HÒA – SƠN
TNH – QUNG NGÃI.
S
FOE
=
2
R 3
4
(Vì tam giác FOE là tam giác đều cnh R)
S
AOF
+ S
EOB
=
1
2
OA.FM+
1
2
OB.EN = R.
FM EN
2
+
= R.PQ (PQ là đường trung bình ca hình thang
EFMN)
S
ABEF
=
2
R 3
4
+ R.PQ mà PQ OP =
R 3
2
.
Do đó S
ABEF
=
2
R 3
4
+
2
R 3
2
=
2
3R 3
4
khi Q trùng vi O hay EF // AB.
Bài 5: (2 đim)
Gi cnh hình vuông ABCD nh nht cha bên trong 5 đường tròn có bán kính bng 1cm và
đôi mt không có quá 1 đim trong chung là x (cm).
T đây suy ra các tâm ca 5 đường tròn này nm trong hình vuông MNPQ có cnh bng x – 2
cm. (vì tâm ca các đường tròn các đường tròn cách cnh hình vuông ít nht 1cm).
Chia hình vuông MNPQ thành 4 hình vuông nhđộ dài mi cnh là
x 2
(cm)
2
-
. (hình v)
Theo nguyên lí Dirichlet có ít nht hai tâm đường tròn cùng thuc mt hình vuông. Gi s hai
tâm đó là O
1
.O
2
.
Vì hai đường tròn này có không quá 1 đim chung nên O
1
O
2
không nh hơn hai ln bán kính và không ln hơn độ dài đường chéo
ca hình vuông cnh
x 2
(cm)
2
-
.
Hay 2 O
1
.O
2
(x 2) 2
2
-
=>
(x 2) 2
2 x 2 2 2 x 2 2 2
2
-
³ Þ - ³ Þ ³ +
Vy cnh hình vuông nh nht cha 5 đưng tròn có bán kính
bng 1 và 5 đường tròn này đôi mt không có quá 1 đim chung
2 2 2
+
Q
P
N
M
D
C
B
A
1cm
1cm
X cm
O1
O4
O2
O5
O3
S GIÁO DC VÀ ĐÀO TO
THANH HOÁ
ĐỀ THI CHÍNH THC
S báo danh
........................
K THI CHN HC SINH GII CP TNH
Năm hc 2013 - 2014
Môn thi: TOÁN - Lp 9 THCS
Thi gian: 150 phút (không k thi gian giao đề)
Ngày thi: 21/03/2014
(Đề thi có 01 trang, gm 05 câu)
Câu I (4,0 đim): Cho biu thc
xy x xy x
x 1 x 1
A 1 : 1
xy 1 1 xy xy 1 xy 1
+ +
+ +
= + +
+ +
.
1. Rút gn biu thc A.
2. Cho
1 1
6
x y
+ =
. Tìm giá tr ln nht ca A.
Câu II (5,0 đim).
1.Cho phương trình
(
)
04222
22
=+++ mmxmx . Tìm
m
để phương trình
có hai nghim thc phân bit
1
x ,
2
x tha mãn
mxxxx 15
112
21
2
2
2
1
=
+
.
2. Gii h phương trình
4 4 4
1x y z
x y z xyz
+ + =
+ + =
.
Câu III (4,0 đim).
1. Tìm tt c các cp s nguyên dương (a; b) sao cho (a + b
2
) chia hết cho (a
2
b – 1).
2. Tìm Nzyx
,, tha mãn zyx +=+ 32 .
Câu IV (6,0 đim) : Cho na đường tròn tâm O đưng kính AB. Mt đim C c định thuc đon thng
AO (C khác A C khác O). Đường thng đi qua C vuông góc vi AO ct na đường tròn đã cho
ti D. Trên cung BD ly đim M (M khác B M khác D). Tiếp tuyến ca na đường tròn đã cho ti
M ct đường thng CD ti E. Gi F là giao đim ca AM và CD.
1. Chng minh tam giác EMF là tam giác cân.
2. Gi I là tâm đường tròn ngoi tiếp tam giác FDM. Chng minh ba đim D, I, B thng hàng.
3. Chng minh góc ABI có s đo không đổi khi M di chuyn trên cung BD.
Câu V (1,0 đim) : Cho x, y là các s thc dương tho mãn x + y = 1.
Tìm giá tr nh nht ca biu thc
3 3
1 1
B
xy
x y
= +
+
.
----- H
T -----
Thí sinh không được s dng tài liu. Cán b coi thi không gii thích gì thêm
Câu Ý Li gii (vn tt) Đim
Điu kin:
xy 1
.
0,25
(
)
(
)
(
)
(
)
(
)
(
)
( )( )
x 1 1 xy xy x xy 1 xy 1 1 xy
A :
xy 1 1 xy
+ + + + + +
=
+
(
)
(
)
(
)
(
)
(
)
(
)
( )( )
xy 1 1 xy xy x xy 1 x 1 1 xy
xy 1 1 xy
+ + + + +
=
+
0,50
(
)
(
)
(
)
(
)
(
)
(
)
( )( ) ( )( )
( )
( )
x 1 1 xy xy x xy 1 xy 1 1 xy
xy 1 1 xy xy x xy 1 x 1 1 xy
+ + + + + +
= =
+ + + + +
0,50
1
(2,5đ)
1 x
1
x y xy xy
+
= =
+
.
1,25
Theo Côsi, ta có:
1 1 1 1
6 2 9
x y xy xy
= +
.
0,50
Du bng xy ra
1 1
x y
= x = y =
1
9
.
0,50
I
(4,0đ)
2
(1,5đ)
Vy: maxA = 9, đạt được khi : x = y =
1
9
.
0,50
PT đã cho có hai nghim phân bit có điu kin:
0'
>
(
)
(
)
00422
2
2
<>+ mmmm (*)
0,50
Vi 0
<
m theo Vi-et ta có:
+=
=+
42.
24
2
21
21
mmxx
mxx
.
0,25
Ta có
( )
mxx
xxxx
mxx
xx
15
11
2
2
15
112
21
21
2
21
21
2
2
2
1
=
+
=
+
(1)
0,50
m
m
m
m
m
15
1
4
2
1
4
6
1
22
=
+
+
0,50
1
(2,5đ)
15
1
2
4
1
6
4
1
=
+
+
m
m
m
m
. Đặt t
m
m =+
4
do 0
<
m 0
<
t
0,50
Ta cos (1) tr thành 4
12
4
15
1
2
1
6
1
=
=
=
=
t
t
t
tt
( do 0
<
t )
0,50
Vi 4
=
t ta có 24
4
==+ m
m
m tha mãn (*)
0,25
S GIÁO DC VÀ ĐÀO TO
THANH HOÁ
HƯỚNG DN CHM
ĐỀ THI CHÍNH THC
K THI CHN HC SINH GII CP TNH
Năm hc 2013 - 2014
Môn thi: TOÁN - Lp 9 THCS
Thi gian: 150 phút (không k thi gian giao đề)
Ngày thi: 21/03/2014
(Hướng dn chm gm 04 trang)
Ta có:
4 4 4 4 4 4
4 4 4
2 2 2
x y y z z x
x y z
+ + +
+ + = + +
2 2 2 2 2 2
x y y z z x
+ + =
=
2 2 2 2 2 2 2 2 2 2 2 2
2 2 2
x y y z y z z x z x x y
xyyz yzzx zxxy
+ + +
+ + + + =
= xyz (x + y + z) = xyz ( vì x + y + z = 1).
0,50
0,50
0,50
2
(2,5đ)
Du bng xy ra
1
1
3
x y z
x y z
x y z
= =
= = =
+ + =
Vy nghim ca h phương trình là:
1 1 1
; ;
3 3 3
x y z
= = =
0,50
Gi s (a + b
2
) M (a
2
b – 1), tc là: a + b
2
= k(a
2
b – 1), vi k N
*
a + k = b(ka
2
– b) a + k = mb (1)
đó m Z mà: m = ka
2
– b m + b = ka
2
(2)
0,50
T (1) và (2) suy ra: (m – 1)(b – 1) = mb – b – m + 1
(m – 1)(b – 1) = (a + 1)(k + 1 – ka) (3)
Do m > 0 (điu này suy ra t (1) do a, k, b > 0) nên m 1 (vì m Z).
Do b > 0 nên b – 1 0 (do b Z) (m – 1)(b – 1) 0.
Vì thế t (3) suy ra: (a + 1)(k + 1 – ka) 0. 0,50
Li do a > 0 nên suy ra: k + 1 – ka 0 k + 1 ka 1 k(a – 1) (4)
a 1 0 (do a Z, a > 0) k Z, k > 0 nên t (4) có:
a 1
k(a 1) 0
a 2
k(a 1) 1
k 1
=
=
=
=
=
0,25
- Vi a = 1. Thay vào (3) ta được: (m 1)(b 1) = 2
m 1 2
b 1 1
b 2
b 3
m 1 1
b 1 2
=
=
=
=
=
=
Vy, trường hp này ta có: a = 1, b = 2 hoc a = 1, b = 3. 0,25
- Vi a = 2 (vì k = 1). Thay vào (3) ta có: (m – 1)(b – 1) = 0
b 1
m 1
=
=
.
Khi b = 1, ta được: a = 2, b = 1.
Khi m = 1: T (1) suy ra a + k = b b = 3. Lúc này được: a = 2, b = 3. 0,25
1
(2,0đ)
Tóm li, có 4 cp s (a; b) tha mãn bài toán là: (1; 2), (1; 3), (2; 3), (2; 1). 0,25
III
(4,0đ)
2
(2,0đ)
Ta có zyx +=+ 32 yzzyx 232 ++=+
(
)
(
)
(
)
yzzyxzyxyzzyx 41234232
2
=++=+ (1)
0,50
TH1. Nếu 0
zyx Ta có
(
)
( )
zyx
zyxyz
=
4
124
3
2
(2) vô lý
( do Nzyx
,, nên vế phi ca (2) là s hu t ).
0,50
TH2. 0
=
zyx khi đó
( )
=
=
3
0
1
yz
zyx
(3)
0.50
Gii (3) ra ta được
=
=
=
3
1
4
z
y
x
hoc
=
=
=
1
3
4
z
y
x
th li tha mãn
0,50
D
E
M
I
H
F
C O
BA
Ta có M thuc đường tròn tâm O đường kính AB (gi thiết) nên
0
AMB 90
= (góc ni tiếp chn na đường tròn)
hay
0
FMB 90
= .
Mt khác
0
FCB 90
= (gi thiết).Do đó
0
FMB FCB 180
+ = .
Suy ra BCFM là t giác ni tiếp
( )
CBM EFM 1
= (vì cùng bù vi
CFM
).
Mt khác
( )
CBM EMF 2
= (góc ni tiếp; góc to bi tiếp tuyến và dây
cung cùng chn
AM
). T (1) và (2)
EFM EMF
= .
Suy ra tam giác EMF là tam giác cân ti E.
(thnhn ra ngay
EMF MBA MFE
= =
nên suy ra EMF cân)
0,50
0,50
0,50
0,50
0,50
IV
(6,0đ)
1
(2.5đ)
2
(2.5đ)
Gọị H là trung đim ca DF. Suy ra
IH DF
( )
DIF
DIH 3
2
= .
Trong đường tròn
(
)
I
ta có:
DMF
DIF
ln lượt là góc ni tiếp và góc
tâm cùng chn cung DF. Suy ra
1
DMF DIF
2
= (4).
0,50
0,50
T (3) và (4) suy ra
DMF DIH
=
hay
DMA DIH
=
.
Trong đường tròn
(
)
O
ta có:
DMA DBA
=
(góc ni tiếp cùng chn
DA
)
Suy ra
DBA DIH
=
.
Vì IH và BC cùng vuông góc vi EC nên suy ra IH // BC. Do đó
o
DBA HIB 180
+ =
o
DIH HIB 180
+ =
Ba đim D, I, B thng
hàng.
0,50
0,50
0,50
3(1đ)
Vì ba đim D, I, B thng hàng
ABI ABD
= =
1
2
sđ
AD
.
Mà C c định nên D c định
1
2
sđ
AD
không đổi.
Do đó góc ABI có s đo không đổi khi M thay đổi trên cung BD.
0,50
0,50
Ta có:
3
1 2xy
1 1 1 1
B
xy 1 3xy xy xy(1 3xy)
(x y) 3xy(x y)
= + = + =
+ +
.
Theo Côsi:
2
(x y)
1
xy
4 4
+
=
.
0.25
Gi B
o
là mt giá tr ca B, khi đó, x, y để:
o
1 2xy
B
xy(1 3xy)
=
3B
o
(xy)
2
– (2 + B
o
)xy + 1 = 0 (1)
Để tn ti x, y thì (1) phi nghim xy = B
o
2
8B
o
+ 4 0
o
o
B 4 2 3
B 4 2 3
+
0.25
Để ý rng vi gi thiết bài toán thì B > 0. Do đó ta có:
o
B 4 2 3
+ .
Vi
( ) ( )
o
o
o
2 B
3 3 3 3
B 4 2 3 xy x(1 x)
6B
6 2 3 6 2 3
+
+ +
= + = = =
+ +
( )
2
2 3 2 3
1 1 1 1
3 3
x ,x
3 3
x x 0
3
2
6 2
2
+
+
+ =
+
= = .
0.25
V(1đ)
Vy,
min
B 4 2 3
= + , đạt được khi
2 3 2 3
1 1 1 1
3 3
x , y
2 2
+
= =
hoc
2 3 2 3
1 1 1 1
3 3
x , y
2 2
+
= = .
0.25
Chú ý:
1) Nếu hc sinh làm bài không theo cách nêu trong đáp án nhưng đúng thì cho đủ s đim tng
phn như hướng dn quy định.
2) Vic chi tiết hóa (nếu có) thang đim trong hướng dn chm phi bo đảm không làm sai lch
hướng dn chm và phi được thng nht thc hin trong t chm.
3) Đim bài thi là tng đim không làm tròn.
S GIÁO DC VÀ ĐÀO TO
TNH NINH BÌNH
ĐỀ THI CHN HC SINH GII LP 9 THCS
Năm hc 2012 – 2013
MÔN: TOÁN
(Thi gian làm bài 150 phút, không k thi gian giao đề)
Đề thi gm 05 câu, trong 01 trang
Câu 1 (4 đim). Cho phương trình
2
x + (4m + 1)x + 2(m - 4) = 0
(1)
(x là n s, m là tham s).
1. Chng minh rng phương trình (1) luôn có hai nghim phân bit vi mi m.
2. Gi x
1
, x
2
là hai nghim ca (1). Tìm m để
1 2
x x 17
=
.
Câu 2 (4 đim). Cho biu thc: P =
2
x - x 2x + x 2(x - 1)
- + (x > 0, x 1).
x + x + 1 x x - 1
1. Rút gn P.
2. Tìm giá tr ca x để P = 3.
Câu 3 (4 đim).
1. Gii h phương trình:
3 3 2 2
x + y + xy = 7
x + y + 3(x + y ) + 3(x + y) = 70
2. Gii phương trình:
2
( x + 5 - x + 2)(1 + x + 7x + 10) = 3
.
Câu 4 (5 đim). Cho đường tròn m O đường kính AB c định. Ax Ay hai tia thay đổi
luôn to vi nhau góc 60
0
, nm v hai phía ca AB, ct đường tròn (O) ln lượt ti M N.
Đường thng BN ct Ax ti E, đường thng BM ct Ay ti F. Gi K trung đim ca đon
thng EF.
1. Chng minh rng
EF
3
AB
= .
2. Chng minh OMKN là t giác ni tiếp.
3. Khi tam giác AMN đều, gi C đim di động trên cung nh AN
(C A, C N).
Đường thng qua M vuông góc vi AC ct NC ti D. Xác định v trí ca đim C để din
tích tam giác MCD là ln nht.
Câu 5 (3 đim).
1. Cho các s thc m, n, p tho mãn: n
2
+ np + p
2
= 1 -
2
3m
2
. Tìm giá tr ln nht
nh nht ca biu thc S = m + n + p.
2. Cho các s thc dương a, b, c tho mãn abc = 1. Chng minh rng:
2 2 2
a b c 1
(ab a 1) (bc b 1) (ca c 1) a b c
+ +
+ + + + + + + +
.
Đẳng thc xy ra khi nào?
-----HT-----
H
và tên thí sinh :....................................................... S báo danh .............................................
H và tên, ch ký: Giám th 1:.......................................................................................................
H và tên, ch ký: Giám th 2:.......................................................................................................
Đ
THI CHÍNH TH
C
S GIÁO DC & ĐÀO TO K THI CHN HC SINH GII CP TNH
LÂM ĐỒNG NĂM HC 2010-2011
Môn : TOÁN – THCS
ĐỀ CHÍNH THC Thi gian : 150 phút ( không k thi gian giao đề)
(Đề thi gm có 1 trang) Ngày thi : 18/02/2011
Câu 1: (2,0 đim ) Rút gn
A 127 48 7 127 48 7
= +
.
Câu 2:(2,0 đim) Cho hàm s y = f(x) = (3m
2
– 7m +5) x – 2011 (*) . Chng minh hàm s (*)
luôn đồng biến trên R vi mi m.
Câu 3:( 2,0 đim) Cho hai đường tròn (O) và (O’) ct nhau ti hai đim A và B . Trên đường
thng AB ly đim M sao cho A nm gia M và B . T M k cát tuyến MCD
vi đường tròn (O) và tiếp tuyến MT vi đường tròn (O’) (T là tiếp đim)
Chng minh MC.MD = MT
2
.
Câu 4: (2,0 đim ) Cho hai s dương x, y tha mãn điu kin 3x + y – 1 = 0 .
Tìm giá tr nh nht ca biu thc B = 3x
2
+ y
2
.
Câu 5: (1,5 đim) Chng minh tng C = 1 + 2 + 2
2
+ … + 2
2011
chia hết cho 15 .
Câu 6: (1,5 đim ) Phân tích đa thc x
3
– x
2
– 14x + 24 thành nhân t .
Câu 7: (1,5 đim) Gii h phương trình
2
x y z 2
2xy z 4
+ + =
=
Câu 8: (1,5 đim ) Chng minh D = n(n + 1)(n + 2)(n + 3) không phi là s chính phương
vi mọi n
N*
.
Câu 9: (1,5 đim ) Cho hai s dương a và b . Chng minh
1 1 4
a b a b
+
+
.
Câu 10:(1,5 đim ) Tìm nghim t nhiên ca phương trình : 2x
2
– xyy
2
– 8 = 0
Câu 11: (1,5 đim ) Cho hình thang vuông ABCD (
0
A D 90
= =
) , có DC = 2AB . K DH vuông
góc vi AC (H
AC)
, gi N là trung đim ca CH .
Chng minh BN vuông góc vi DN .
Câu 12: (1,5 đim). Cho tam giác MNP cân ti M (
0
M 90
<
) . Gi D là giao đim các đường
phân giác trong ca tam giác MNP . Biết DM =
2 5
cm , DN = 3 cm .
Tính độ dài đon MN .
---------- HT---------
H và tên thí sinh :……………………………………………...S báo danh : ………………………
Giám th 1 :……………………………………………………..Ký tên : …………………………….
Giám th 2 :……………………………………………………..Ký tên : …………………………….
(Thí sinh không được s dng máy tính )
S GIÁO DC &ĐÀO TO K THI CHN HC SINH GII CP TNH
LÂM ĐỒNG NĂM HC 2010-2011
HƯỚNG DN CHM ĐỀ THI CHÍNH THC
Môn : TOÁN – THCS
Ngày thi 18/02/2011
Câu Hướng dn chm Đim
Câu 1
(2 đim )
A 127 48 7 127 48 7
= +
=
2 2
(8 3 7) (8 3 7)
+
=
|8 3 7 | | 8 3 7 |
+
8 3 7 8 3 7 (8>3 7)
6 7
=
=
0,5 đim
0,5 đim
0,5 đim
0,5 đim
Câu 2
(2 đim )
3m
2
– 7m + 5 = 3
2
7 5
m m
3 3
+
2
2
7 49 60
3 m
6 36 36
7 11
3 m 0 m
6 36
= +
= + >
Vây f(x) đồng biến trên R vi mi m
0,5 đim
0,5 đim
0,5 đim
0,5 đim
Câu 3
(2 đim)
Chng minh MC. MD = MA. MB
Chng minh MT
2
= MA. MB
Suy ra MC.MD = MT
2
0,75 đim
0,75 đim
0,5 đim
Câu 4
(2 đim )
3x + y – 1 = 0
y = 1 – 3x
2 2
2
2
2
B 3x (1 3x)
12x 6x 1
1 1
12 x
4 48
1 1 1
12 x
4 4 4
= +
= +
= +
= +
Vây GTNN ca B là
1 1 1
khi x = và y =
4 4 4
0,5 đim
0,5 đim
0,5 đim
0,5 đim
Câu 5 C = 1 + 2 + 2
2
+ … + 2
2011
(1,5 đim ) = (1 + 2 + 2
2
+ 2
3
) + (2
4
+ 2
5
+ 2
6
+ 2
7
) + …+ ( 2
2008
+ 2
2009
+2
2010
+ 2
2011
)
= (1 + 2 + 2
2
+ 2
3
)+ 2
4
(1 + 2 + 2
2
+ 2
3
)+ …+2
2008
(1 + 2 + 2
2
+ 2
3
)
= 15 ( 1 + 2
4
+ …+ 2
2008
) chia hết cho 15
0,5 đim
0,5 đim
0,5 đim
Câu 6
(1,5 đim )
x
3
– x
2
– 14x +24
= x
3
+ 4x
2
– 5x
2
– 20x + 6x + 24
= (x + 4) (x
2
– 5x + 6 )
= (x + 4) (x – 2) (x – 3)
0,5 đim
0,5 đim
0,5 đim
Câu 7
(1,5 đim )
2
2 2
2 2
x y z 2 z 2 x y
(2 x y) 2xy 4
2xy z 4 z 2xy 4 z 2 x y
(x 2) (y 2) 0
z 2 x y
x y 2
z 2
+ + = =
=
= = =
+ =
=
= =
=
0,5 đim
0,5 đim
0,5 đim
Câu 8
(1,5 đim )
D = n(n + 1) (n + 2) (n + 3)
= (n
2
+ 3n) (n
2
+ 3n + 2 )
= (n
2
+ 3n)
2
+2 (n
2
+ 3n)
(n
2
+ 3n)
2
< D < (n
2
+ 3n)
2
+2 (n
2
+ 3n) +1
(n
2
+ 3n)
2
< D < (n
2
+ 3n +1)
2
Nên D không phi là s chính phương vì (n
2
+ 3n)
2
và (n
2
+ 3n +1)
2
là 2 s
chính phương liên tiếp
0,5 đim
0,5 đim
0,5 đim
Câu 9
(1,5 đim )
Ta có (a – b)
2
0
2 2
2
a b 2ab
(a b) 4ab
a b 4
( vì (a+b)ab >0 )
ab a b
1 1 4
a b a b
+
+
+
+
+
+
Du “ = ” xy ra khi a = b ( thiếu câu y không trừ đim)
0,5 đim
0,5 đim
0,5 đim
Câu 10
(1,5 đim)
2x
2
– xy – y
2
– 8 = 0
(2x + y) (x – y) = 8
2x y 8
x y 1
+ =
=
hoc
2x y 4
x y 2
+ =
=
x 3
y 2
=
=
hoc
x 2
y 0
=
=
0,5 đim
0,5 đim
0,5 đim
Câu 11
(1,5 đim )
Gi M là trung đim ca DH
Chng minh t giác ABNM là hình bình hành
AM // BN
(1)
Chng minh MN
AD
Suy ra M là trc tâm ca
ADN
AM DN
(2)
T (1) và (2)
BN DN
0,25 đim
0,5 đim
0,25 đim
0,25 đim
0,25 đim
Câu 12
(1,5 đim )
Qua M k tia Mx vuông góc vi MN ct ND ti E , k MF
ND
Chng minh
1
D E
=
MD ME 2 5
= =
cm và EF =DF
ME
2
= EF .EN = EF .(2EF + DN )
2
2
(2 5) EF(2EF 3)
2EF 3EF 20 0
= +
+ =
(EF 4)(2EF 5) 0
EF 2,5 (vì EF >0)
+ =
=
MN 2 11
=
cm
0,5 đim
0,5 đim
0,5 đim
(Nếu hc sinh gii bng cách khác đúng , giám kho da theo biu đim để cho đim tương ng )
S GIÁO DC ĐÀO TO
HI DƯƠNG
KÌ THI CHN HC SINH GII TNH
LP 9 NĂM HC 2013-2014
MÔN THI: TOÁN
Thi gian làm bài: 150 phút
Ngày thi 20 tháng 03 năm 2014
(đề thi gm 01 trang)
Câu 1 (2 đim).
a) Rút gn biu thc
(
)
2 3 3
2
1 1 . (1 ) (1 )
2 1
x x x
A
x
+ +
=
vi
1 1
x
.
b) Cho ab là các s tha mãn a > b > 0 và
3 2 2 3
6 0
a a b ab b
+ =
.
Tính giá tr ca biu thc
4 4
4 4
4
4
a b
B
b a
=
.
Câu 2 (2 đim).
a) Gii phương trình
2 2 2
( 2) 4 2 4.
x x x x
+ = +
b) Gii h phương trình
3
3
2
2
x x y
y y x
= +
= +
.
Câu 3 (2 đim).
a) Tìm các s nguyên dương x, y tha mãn phương trình
2
2 32
xy xy x y
+ + =
.
b) Cho hai s t nhiên a, b tha mãn
2 2
2 3
a a b b
+ = +
.
Chng minh rng
2 2 1
a b
+ +
là s chính phương.
Câu 4 (3 đim).
Cho tam giác đều ABC ni tiếp đường tròn (O, R). H là mt đim di động trên
đon OA (H khác A). Đường thng đi qua H vuông góc vi OA ct cung nh AB
ti M. Gi K là hình chiếu ca M trên OB.
a) Chng minh
=
HKM 2AMH.
b) Các tiếp tuyến ca (O, R) ti A B ct tiếp tuyến ti M ca (O, R) ln lượt
ti D và E. OD, OE ct AB ln lượt ti F và G. Chng minh OD.GF = OG.DE.
c) Tìm giá tr ln nht ca chu vi tam giác MAB theo R.
Câu 5 (1 đim).
Cho a, b, c là các s thc dương tha mãn
2 6 2 7
ab bc ac abc
+ + =
. Tìm giá tr
nh nht ca biu thc
4 9 4
2 4
ab ac bc
C
a b a c b c
= + +
+ + +
.
----------------------Hết------------------------
H và tên thi sinh…………………………………………..s báo danh…………...
Ch ký ca giám th 1………………………..ch ký ca giám th 2………………
ĐỀ THI CHÍNH THC
S GIÁO DC ĐÀO TO
HI DƯƠNG
---------------------------
HƯỚNG DN CHM ĐỀ THI CHN
HC SINH GII TNH
LP 9 NĂM HC 2013-2014
MÔN THI: TOÁN
Ngày thi 20 tháng 03 năm 2014
(Hướng dn chm gm có 03 trang)
Lưu ý: Nếu hc sinh làm theo cách khác mà kết qu đúng thì giám kho vn
cho đim ti đa.
Câu Ni dung Đim
(
)
(
)
2 2
2
1 1 . 1 1 2 1
2 1
x x x x
A
x
+ +
=
0.25
(
)
2
1 1 . 1 1
x x x
= + +
0.25
(
)
( )
(
)
(
)
2
2 2 2
1 1 1 1 1 1 2 2 1
x x x x x
= + + = +
0.25
Câu
1a:
(1,0 đ)
2
2
x
= =
2
x
0.25
3 2 2 3 2 2
6 0 ( 2 )( 3 ) 0 (*)
a a b ab b a b a ab b + = + + =
0.25
Vì a > b > 0
2 2
3 0
a ab b + + > nên t (*) ta có a = 2 b
0.25
Vy biu thc
4 4 4 4
4 4 4 4
4 16 4
4 64
a b b b
B
b a b b
= =
0.25
Câu
1b:
(1,0 đ)
4
4
12 4
63 21
b
B
b
= =
0.25
Đặt
(
)
2 2 4 2
2 4 2 2
t x x t x x
= + = +
( )
2
2 2
2
2
t
x x
+ =
0.25
ta được phương trình
2
2
4
4 2 8 0
2
2
t
t
t t t
t
=
= + =
=
0.25
Vi t = -4 ta có
( )
2
4 2
4 2
0
0
2 4 4
2 2 16
2 8 0
x
x
x x
x x
x x
<
<
+ =
+ =
+ =
2
0
2
2
x
x
x
<
=
=
0.25
Câu
2a:
(1,0 đ)
Vi t =2 ta có
( )
2
4 2
4 2
0
0
2 4 2
2 2 4
2 2 0
x
x
x x
x x
x x
>
>
+ =
+ =
+ =
2
0
3 1
3 1
x
x
x
>
=
=
. Kết lun nghim ca phương trình.
0.25
T h ta có
(
)
3 3 2 2 2 2
(2 ) (2 ) ( ) 2 0
x y x y x y x y xy x y
+ = + + + =
0.25
3
( ) ( ) 0
x y
x y x y
x y
=
+ =
=
0.25
Câu
2b:
(1,0 đ)
* Vi x = y ta tìm đưc (x ; y) = (0; 0); (
3; 3
);(
3; 3
)
0.25
* Vi x = - y ta tìm được (x ; y) = (0; 0); (
1; 1
);(
1;1
)
Vy h phương trình có nghim
(x ; y) = (0; 0);
3; 3
);(
3; 3
);(
1;1
);(
1; 1
)
0.25
2
2 32
xy xy x y
+ + =
2
( 1) 32
x y y
+ =
Do y nguyên dương
2
32
1 0
( 1)
y
y x
y
+ =
+
0.25
2
( , 1) 1 ( 1)
y y y
+ = +
(32)
U
0.25
5
32 2
=
2 2
( 1) 2
y
+ =
2 4
( 1) 2
y
+ =
(Do
2
( 1) 1
y
+ >
)
0.25
Câu
3a:
(1,0 đ)
*Nếu
2 2
( 1) 2 1; 8
y y x
+ = = =
*Nếu
2 4
( 1) 2
y
+ =
3; 6
y x
= =
Vy nghim nguyên dương ca phương trình là:
8
1
x
y
=
=
6
3
x
y
=
=
0.25
2 2
2 3
a a b b
+ = +
2
( )(2 2 1)
a b a b b
+ + =
(*)
0.25
Gi d là ước chung ca (a - b, 2a + 2b + 1) (
*
d
). Thì
( )( )
2
2 2
( )
2 2 1
(2 2 1)
a b d
a b a b d
a b d
b d b d
+ +
+ +
M
M
M
M M
0.25
( ) (2 2 )
a b d a d a b d
+
M M M
(2 2 1) 1 1
a b d d d
+ + =
M M
0.25
Câu
3b:
(1,0 đ)
Do đó (a - b, 2a + 2b + 1) = 1. T (*) ta được
a b
2 2 1
a b
+ +
s chính
phương =>
2 2 1
a b
+ +
là s chính phương.
0.25
Qua A k tia tiếp tuyến Ax ca (O). Ta
= =
1 1
1 1
A O
2 2
sđ
AM
(1)
0.25
Có Ax // MH (cùng vuông góc vi OA)
=
1 1
A M
(2)
0.25
T giác MHOK ni tiếp
=
1 1
O K
(cùng chn
MH
) (3)
0.25
Câu
4a:
(1,0 đ)
T (1), (2), (3) ta có
=
1 1
1
M K
2
hay
=
HKM 2AMH.
0.25
Câu
4b:
(1,0 đ)
Có t giác AOMD ni tiếp (4)
0.25
x
1
1
1
1
H
K
O
A
B
C
M
1
2
1
1
2
1
F
G
E
D
H
O
A
B
C
M
=
1
1
A
2
sđ
BM
;
= =
1 2
1
O O
2
sđ
BM
=
1 1
A O
t giác AMGO ni tiếp (5)
0.25
T (4), (5) ta có 5 đim A, D, M, G, O cùng nm trên mt đường tròn
= =
1 2 1
G D D
0.25
OGF
ODE
đồng dng
=
OG GF
OD DE
hay OD.GF = OG.DE.
0.25
Trên đon MC ly đim A’ sao cho
MA’ = MA
AMA'
đều
(
)
= =
0
1 2
A A 60 BAA'
= =
MAB A 'AC MB A'C
0.25
+ =
MA MB MC
Chu vi tam giác MAB là
+ + = + +
MA MB AB MC AB 2R AB
0.25
Đẳng thc xy ra khi MC là đường kính ca (O) => M là đim chính gia
cung AM => H là trung đim đon AO
Vy giá tr ln nht ca chu vi tam giác MAB là 2R + AB
0.25
Câu
4c:
(1,0 đ)
Gi I là giao đim ca AO và BC = = =
3 AB 3
AI R AB R 3
2 2
Giá tr ln nht ca chu vi tam giác MAB là 2R + AB = +
(2 3)R
0.25
T gt :
2 6 2 7
ab bc ac abc
+ + =
và a,b,c > 0
Chia c hai vế cho abc > 0
2 6 2
7
c a b
+ + =
đặt
1 1 1
, ,x y z
a b c
= = =
, , 0
2 6 2 7
x y z
z x y
>
+ + =
Khi đó
4 9 4
2 4
ab ac bc
C
a b a c b c
= + +
+ + +
4 9 4
2 4
x y x z y z
= + +
+ + +
0.25
4 9 4
2 4 (2 4 )
2 4
C x y x z y z x y x z y z
x y x z y z
= + + + + + + + + + + + + +
+ + +
0.25
2 2
2
2 3 2
2 4 17 17
2 4
x y x z y z
x y x z y z
= + + + + + +
+ + +
0.25
Câu 5:
(1,0 đ)
Khi
= = =
1
x ,y z 1
2
thì C = 17
Vy GTNN ca C là 17 khi a =2; b =1; c = 1
0.25
2
1
A'
I
H
O
A
B
C
M
S GD&ĐT HI DƯƠNG KÌ THI CHN HC SINH GII TNH
LP 9 THCS NĂM HC 2012 – 2013
MÔN THI: TOÁN
Thi gian làm bài: 150 phút (không k thi gian giao đ)
Ngày thi: 27/03/2013
( Đề thi gm có 01 trang )
Câu 1 (2,0 đim):
a) Rút gn biu thc:
(
)
2
A = x 50 x + 50 x + x 50
vi
x 50
b) Cho
x + 3 = 2
. Tính giá tr ca biu thc: B = x
5
– 3x
4
– 3x
3
+ 6x
2
– 20x + 2018
Câu 2 (2,0 đim):
a) Gii phương trình
2 2
4x 3x
+ = 6
x 5x + 6 x 7x + 6
b) Gi¶i hÖ ph−¬ng tr×nh sau:
x + y + 4 xy = 16
x + y =10
Câu 3 (2,0 đim):
a) Vi a, b là các s nguyên. Chng minh rng nếu
2 2
4a + 3ab 11b
chia hết cho 5
thì
4 4
a b
chia hết cho 5.
b) Cho phương trình
2
ax +bx+1 0
=
vi a, b là các s hu t. Tìm a, b biết
5 3
x =
5+ 3
là nghim ca phương trình.
Câu 4 (3,0 đim):
Cho 3 đim A, B, C c định nm trên mt đường thng d (B nm gia A và C). V
đường tròn tâm O thay đi nhưng luôn đi qua B C (O không nm trên đưng thng
d). K AM AN các tiếp tuyến vi đường tròn tâm O ti M N. Gi I trung
đim ca BC, AO ct MN ti H và ct đường tròn ti các đim P và Q (P nm gia A và
O), BC ct MN ti K.
a) Chng minh 4 đim O, M, N, I cùng nm trên mt đường tròn.
b) Chng minh đim K c định khi đường tròn tâm O thay đổi.
c) Gi D là trung đim HQ, t H k đường thng vuông góc vi MD ct đường thng
MP ti E. Chng minh P là trung đim ME.
Câu 5 (1,0 đim):
Cho
n
1
A =
(2n +1) 2n 1
vi n
*
.
Chng minh rng:
1 2 3 n
A + A + A + ... + A <1
.
------------- HT ------------
H và tên thí sinh: ……………………………… ….. S báo danh …………….
Ch kí giám th 1 ………………….. Ch kí giám th 2 …………………..
ĐỀ THI CHÍNH THC
S GD&ĐT HI DƯƠNG ĐÁP ÁN VÀ HƯỚNG DN CHM
ĐỀ THI HC SINH GII TNH
MÔN TOÁNLP 9 THCS NĂM HC 2012 – 2013
Lưu ý: Thí sinh làm theo các khác đúng vn cho đim ti đa. Đim bài thi làm tròn đến 0,25 đim
CÂU PHN NI DUNG ĐIM
a)
1,0
đim
Ta có :
(
)
(
)
(
)
(
)
(
)
(
)
( )
2
2 2
2 2 2
2 2 2
2 2 2
2
A = x - 50 - x + 50 x + x - 50
A = x - 50 + x + 50 - 2 x -50 x + x -50
A = 2x - 2 x -50 x + x -50
A = 2 x - x + 50
A = 100
Nhưng do theo gi thiết ta thy
(
)
2
A = x - 50 - x + 50 x + x -50
<0
A= -10
0,25
0,25
0,25
0,25đ
Câu 1
2,0
đim
b)
1,0
đim
x + 3 = 2
=>
2
2 3 ( 2) 3
= =
x x
2
4 1 0
x x
+ =
B = x
5
– 3x
4
– 3x
3
+ 6x
2
– 20x + 2018
B = (x
5
– 4x
4
+ x
3
) + ( x
4
– 4x
3
+ x
2
) + 5( x
2
– 4x + 1) + 2013
B = x
3
( x
2
– 4x + 1) +x
2
( x
2
– 4x + 1) +5(x
2
– 4x + 1) + 2013
B = 2013
0,25
0,25
0,25
0,25
Câu 2
2,0
đim
a)
1.0
đim
Nhn xét x = 0 không là nghim ca phương trình
Vi
x 0
, phương trình đã cho tương đương vi:
4 3
+ = 6
6 6
x 5 + x 7 +
x x
Đặt
6
t = x 7 +
x
phương trình tr thành
( )( )
( )
2 2
4 3
+ =6 1 t 0;t 2
t+2 t
1 4t 3t 6 6t 12t 6t 5t 6 0
+ + = + + =
Gii phương trình ta được
1 2
3 2
t ; t
2 3
= =
( tha mãn )
Vi
1
3
t
2
=
ta có
2
6 3
7 2 11 12 0
2
x x x
x
+ = + =
Gii phương trình ta được
1 2
3
x ;x 4
2
= =
( tha mãn )
V
i
2
2
t
3
=
ta có
2
6 2
7 3 23 18 0
3
x x x
x
+ = + =
0,25
0,25
0,25
Gii phương trình ta được
3 4
23 313 23 313
x ;x
6 6
+
= =
(tha mãn)
Vy phương trình đã cho có bn nghim là :
1 2
3
x ;x 4
2
= =
;
3 4
23 313 23 313
x ;x
6 6
+
= =
0,25
b)
1,0
®iÓm
x + y + 4 xy = 16
x + y =10
(I) (
x;y 0
)
Đặt S=
x y
+ ; P =
xy
(
S 0;P 0
) h (I) có dng
2
S+ 4P =16
S - 2P =10
( II)
Gii h ( II) và đối chiếu điu kin ta được
S = 4
P = 3
Khi đó
x; y
là 2 nghim ca phương trình t
2
– 4t + 3 =0
Gii phương trình ta được t
1
= 3; t
2
= 1
T đó suy ra h phương trình đã cho có hai nghim
x = 9 x =1
;
y =1 y = 9
0,25
0,25
0,25
0,25
a)
1.0
đim
(
)
(
)
( )
+ + +
+ +
+
M M
M
M
2 2 2 2 2 2
2 2
2
4a 3ab 11b 5 5a 5ab 10b 4a 3ab 11b 5
a 2ab b 5
a b 5
+
M
a b 5
( Vì 5 là s nguyên t)
(
)
(
)
(
)
4 4 2 2
a b a b a b a b 5
= + +
M
0.25
0,25
0,25
0,25
Câu 3
2,0
đim
b)
1,0
®iÓm
5 3
5 3
x
=
+
=
(
)
( )( )
2
5 3
4 15
5 3 5 3
=
+
5 3
5 3
x
=
+
là nghim ca phương trình nên ta có
(
)
(
)
( ) ( )
2
4 15 4 15 1 0
31 8 15 4 15 1 0
15(8 ) 31 4 1 0
a b
a b
a b a b
+ + =
+ + =
+ + + + =
,
a b Q
nên
(8 ), (31 4 1)
a b a b Q
+ + +
Do đó nếu
8 0
a b
+
thì
15
31 4 1
8
a b
Q
a b
+ +
=
+
(Vô lí)
Suy ra
8 0 1
31 4 1 0 8
a b a
a b b
+ = =
+ + = =
0,25
0,25
0,25đ
0,25
d
K
E
D
A
B
C
M
N
P
Q
I
H
O
a)
1,0
®iÓm
I là trung đim ca BC ( dây BC không đi qua O )
0
90
OI BC OIA =
Ta có
0
90
AMO =
( do AM là hai tiếp tuyến (O) )
0
90
ANO =
( do AN là hai tiếp tuyến (O) )
Suy ra 4 đim O, M, N, I cùng thuc đường tròn đường kính OA
0,25
0,25
0,25
0.25
b)
1,0
®iÓm
AM, AN là hai tiếp tuyến (O) ct nhau ti A nên OA là tia phân giác
MON
OMN cân ti O nên
OA MN
ABN đồng dng vi ANC ( vì
1
ANB=ACN=
2
sđ
NB
CAN
chung ) suy ra
2
AB AN
= AB.AC=AN
AN AC
ANO vuông ti N đường cao NH nên ta có AH.AO = AN
2
Suy ra AB.AC = AH.AO
AHK đồng dng vi AIO ( vì
0
AHK=AIO=90
OAI
chung )
AH AK
= AI.AK=AH.AO
AI AO
AI.AK=AB.AC
AB.AC
AK=
AI
Ta có A,B,C c định nên I c định suy ra AK c định mà A c định,
K là giao đim ca dây BC và dây MN nên K thuc tia AB suy ra K
c định
0,25
0,25
0,25
0,25
Câu 4
3,0
đim
c)
1,0
®iÓm
Ta có
0
PMQ=90
( góc ni tiếp chn na đường tròn ).
Xét MHE và QDM có
MEH=DMQ
( cùng ph vi
DMP
),
EMH=MQD
( cùng ph vi
MPO
)
ME MH
MQ DQ
=
PMH đồng dng vi MQH
0,25
2
1
2
MP MH MH
MQ HQ DQ
MP ME
MQ MQ
= =
=
ME = 2 MP P là trung đim ME.
0,25
0,25
0,25
Câu 5
1,0
đim
( )
1 2 1
(2 1) 2 1
(2 1) 2 1
n
A
n
n n
n n
= =
+
+
2 1 1 1 2 1 1 1 1 1
2 2 1 2 1 2
2 1 2 1 2 1 2 1
n n
A
n
n n
n n n n
= = +
+
+ +
1 1
0
2 1 2 1n n
>
+
1 1 2
2 1 2 1 2 1
n n n
+ <
+
nên
A
n
<
1 1
( *)
2 1 2 1
n
n n
+
Do đó:
1 2 3
1 1 1 1 1
... 1
3 3 5 2 1 2 1
n
A A A A
n n
+ + + + < + + +
+
1 2 3
1
... 1 1
2 1
n
A A A A
n
+ + + + < <
+
0,25
0,25
0,25
0,25
Hết
S GD&ĐT HÒA BÌNH
Đ chính thc
Đ THI CHN HC SINH GII CP TNH
NĂM HC 2012 - 2013.
MÔN: TOÁN LP 9
(Thi gian 150 phút không k thi gian giao đ).
Bài 1: (4,5 đim): Cho biu thc A=
2
3
:
2
2
2
2
4
4
2
2
+
+
+
+
x
x
x
x
x
x
x
x
1. Rút gn A.
2. Tìm giá tr ca A khi
13 =x
Bài 2 (4 đim):
1. Gii phương trình
9132 =+ xx
2. Cho các s a,b,c tha mãn điukin
ab bc ca 1
+ + =
. Tính giá tr nh nht ca biu
thc
2 2 2
P (a 2bc 1)(b 2ca 1)(c 2ab 1)
= + + +
Bài 3 (4 đim):
a, Mt ca nô xuôi mt khúc sông i 52km ri ngưc dòng tr li 40km mt tng cng 4
gi Biết vn tc ca dòng nưc là 3km/h. Tìm vn tc ca ca nô lúc dòng nưc yên lng.
b, Trong mt phng ta đ Oxy, cho đưng thng (d) : y= ax +b. Tìm a,b biết (d) đi qua
M(-1; 3) và (d) ct Ox ti đim N tha mãn ON = 2
Bài 4 (6 đim): Cho đưng tròn tâm O và hai đim B,C thuc đưng tròn, các tiếp tuyến vi
đưng tròn ti B và C ct nhau A. Mt M là mt đim thuc cung nh BC. Tiếp tuyến vi
đưng tròn ti M ct AB, AC theo th t D, E . Gi giao đim ca OD, OE vi BC theo th t
là I và K . Chng Minh rng
a) BD.OE = OD.BI
b) T giác DIKE ni tiếp
c) OM, DK, EI đng quy
Bài 5 (2 đim): Cho dãy s sau:
1 1 2 1 2 3 1 2 3 4 1 2 3 4 5 1 2 3
; ; ; ; ; ; ; ; ; ; ; ; ; ; ; ; ; ;....
1 2 1 3 2 1 4 3 2 1 5 4 3 2 1 6 5 4
Tìm s hng th 2013 trong y s trên
(Ghi chú: Cán b coi thi không gii thích gì thêm)
Së GD & §T Hoµ B×nh kú thi chän häc sinh giái cÊp tØnh
Líp 9 tHCS n¨m häc 2010 - 2011
§Ò chÝnh thøc §Ò thi m«n : To¸n
Ngµy thi: 22 th¸ng 3 n¨m 2011
Thêi gian lµm bµi: 150 phót (kh«ng kÓ thêi gian giao ®Ò)
(§Ò thi gåm cã 01 trang)
Bµi 1: (4 ®iÓm)
1. Ph©n tÝch thµnh nh©n tö c¸c biÓu thøc sau:
a/
3 2 2 3
3 4 12
A x x y xy y
= + b/
3 2 2 3
4 2 8
B x y xy x y
= + + +
2. Cho
11 6 2 11 6 2
a = + + . Chøng minh r»ng
a
lµ mét sè nguyªn.
Bµi 2: (6 ®iÓm)
1. Gi¶i ph−¬ng tr×nh:
2 2
12 3
1
4 2
x x x x
=
+ + + +
2. Cho hµm sè
2
( 1) 1
y m x m
= +
(m: tham sè). T×m m ®Ó ®å thÞ hµm sè lµ ®−êng
th¼ng c¾t hai trôc to¹ ®é t¹i hai ®iÓm A, B sao cho tam gi¸c OAB c©n.
3. T×m x ®Ó biÓu thøc
1
1
x
A
x
=
+
®¹t gi¸ trÞ nhá nhÊt.
Bµi 3: (4 ®iÓm)
1. Cho tam gi¸c ABC nhän néi tiÕp trong ®−êng trßn t©m O, cã b¸n kÝnh b»ng 2.
BiÕt
0
60
BAC = , ®−êng cao AH = 3. TÝnh diÖn tÝch tam gi¸c ABC.
2. Đội c vua ca trường A thi đấu vi đội c vua ca trường B, mi đấu th ca
trường này thi đấu vi mi đấu th ca trường kia mt trn. Biết rng tng s trn đu bng
bn ln tng s cu th ca c hai đội s cu th ca trường B s l. Tìm s cu th
ca mi đội.
Bµi 4: (5 ®iÓm) Cho na đường tròn tâm O bán kính R, đường kính AB. Hai đim E, F thay
đổi trên na đường tròn sao cho s đo cung AE khác không và nh hơn s đo cung AF, biết
EF = R . Gi s AF ct BE ti H, AE ct BF ti I.
1. Chng minh rng t giác IEHF ni tiếp được trong mt đường tròn.
2. Gi EG FQ các đưng cao ca tam giác IEF, chng minh rng độ dài QG
không đổi.
3. Chng minh rng QG song song vi AB.
Bµi 5: (1 ®iÓm) Gi¶i ph−¬ng tr×nh:
2
2 7 2 1 8 7 1
x x x x x
+ = + + +
--------------------HÕt-----------------
Hä vµ tªn thÝ sinh:................................ .................. SBD: ..........
Gi¸m thÞ 1 (hä vµ tªn, ch÷ ký): ...................................................
Gi¸m thÞ 2 (hä vµ tªn, ch÷ ký): ....................................................
Së GD&§T Hoµ B×nh H−íng dÉn chÊm m«n to¸n
Kú thi chän häc sinh giái cÊp tØnh cÊp THCS
N¨m häc 2010-2011
Bµi ý Néi dung
§iÓm
1.
(4®)
1
2
a/ A = ( x + 3y ).( x - 2y ).( x + 2y ).
b/ B = ( x + 2y + 1 ).( x
2
- 2xy + 4y
2
).
2 2
11 6 2 11 6 2 (3 2) (3 2) 6
a
= + + = + + =
Tõ ®ã
a
lµ sè nguyªn.
1,0
1,0
1,5
0,5
2
(6 ®)
1.
2.
3.
+ HS lËp luËn ®−îc x
2
+ x + 4 vµ x
2
+ x + 2 kh¸c 0 råi ®−a PT vÒ d¹ng
9( x
2
+ x ) + 12 = ( x
2
+ x + 4 ) ( x
2
+ x + 2 )
+HS biÕn ®æi PT vÒ d¹ng ( x
2
+ x - 4 ) ( x
2
+ x + 1 ) = 0
+HS gi¶i PT tÝch t×m ®−îc 2 nghiÖm lµ x =
1 17
2
±
+ HS lËp luËn ®−îc ®Ó ®å thÞ hµm sè lµ ®−êng th¼ng c¾t 2 trôc täa ®é t¹i 2 ®iÓm
A vµ B sao cho tam gi¸c OAB c©n th× ®å thÞ hµm sè ®Z cho song song víi ®−êng
th¼ng y = x ( hoÆc y = - x )
+ Tõ ®ã dÉn ®Õn
2
1 1
1 0
m
m
=
hoÆc
2
1 1
1 0
m
m
=
gi¶i 2 hÖ PT ®ã t×m ®−îc
m = 2 hoÆc m = 0 vµ tr¶ lêi bµi to¸n.
+ HS viÕt ®−îc
2
1
1
A
x
=
+
+ HS lËp luËn vµ t×m ®−îc gi¸ trÞ nhá nhÊt cña biÓu thøc A b»ng - 1 khi x = 0.
1,0
0,5
0,5
1,0
1,0
0,5
1,5
3
(4 ®)
1.
2.
C
B
O
A
K
Gäi K lµ trung ®iÓm cña BC, dÔ cã
0
60
KOC = .
XÐt tam gi¸c vu«ng OKC cã OC = 2
TÝnh ®−îc
0
.sin 60 3
KC OC= = ,
TÝnh ®−îc
2 3
BC = , suy ra diÖn tÝch
tam gi¸c ABC lµ
3 3
S = (§vdt)
Chó ý: Thùc chÊt tam gi¸c ABC ®Òu
nh−ng kh«ng yªu cÇu HS vÏ h×nh ®óng.
+ Gi s cÇu thñ ®éi tr−êng A lµ x; Sè cÇu thñ ®éi tr−êng B lµ y đặt đk vµ lËp
®−îc PT: xy = 4( x + y )
( 4)( 4) 16
x y
=
+ HS
lp lun vµ t×m ®−îc x = 20 ; y= 5, KL
1,0
1,0
1,0
1,0
Chó ý: Mäi lêi gi¶i ®óng kh¸c ®Òu ®−îc cho ®iÓm t−¬ng ®−¬ng
4
(5 đ)
1.
2.
3.
1. Chng minh được t giác IEHF ni tiếp
được trong mt đường tròn.
2. Chng minh được
IFE
IQG
(g.g),
t đó có
1
EF IE 2
QG IG
= =
;
1 1
EF =
2 2
QG R
= (đpcm).
3. Chng minh được
IFE
IAB
(g.g), kết hp
vi (2) ta
IAB
IQG
, suy ra
IA IB
IQ IG
= dn
đến QG song song vi AB.
2,0
1,0
1,0
1,0
5
(1®)
+ HS t×m ®−îc §K
1 7
x
vµ biÕn ®æi PT vÒ d¹ng tÝch
(
1 2
x
).( 1 7
x x
) = 0
+ HS gi¶i PT tÝch t×m ®−îc x = 5 hoÆc x = 4 ®Òu tháa mZn vµ tr¶ lêi.
0,5
0,5
H
Q
G
I
F
B
A
O
E
S GIÁO DC ĐÀO TO
HÀ NAM
ĐỀ THI CHÍNH THC
K THI CHN HC SINH GII LP 9 THCS
NĂM HC 2012-2013
Môn thi: TOÁN
Thi gian làm bài: 150 phút, không k thi gian giao đề
Bài 1. (4,0 đim)
Cho biu thc:
( )(1 ) ( )( 1) ( 1)(1 )
x y xy
P
x y y x y x x y
=
+ + + +
1. Rút gn biu thc P.
2. Tìm các giá tr x, y nguyên tha mãn P = 2.
Bài 2. (4,0 đim)
1. Cho hai s thc a, b không âm tha mãn
18 4 2013
a b
+
. Chng minh rng
phương trình sau luôn có nghim:
2
18 4 671 9 0
ax bx a
+ + =
.
2. Tìm tt c các nghim nguyên x, y ca phương trình
3 2 3
2 3 2
x x x y
+ + + =
.
Bài 3. (4,5 đim)
1. Cho p 2p + 1 hai s nguyên t ln hơn 3. Chng minh rng 4p + 1
mt hp s.
2. Gii phương trình:
2 3 2
4 3 3 4 3 2 2 1
+ + = + +
x x x x x
Bài 4. (6,0 đim)
Cho góc xOy s đo bng 60
o
. Đường tròn tâm K nm trong góc xOy tiếp
xúc vi tia Ox ti M tiếp xúc vi tia Oy ti N. Trên tia Ox ly đim P tha mãn
OP = 3OM. Tiếp tuyến ca đường tròn (K) qua P ct tia Oy ti Q khác O. Đường
thng PK ct đường thng MN E. Đường thng QK ct đường thng MN F.
1. Chng minh tam giác MPE đồng dng vi tam giác KPQ.
2. Chng minh t giác PQEF ni tiếp được trong đường tròn.
3. Gi D là trung đim ca đon PQ. Chng minh tam giác DEF là mt tam giác
đều.
Bài 5. (2,0 đim)
Cho a, b, c là ba s thc dương tha mãn:
3
a b c
+ + =
. Chng minh rng:
2 2 2
1 1 1
3
1 1 1
a b c
b c a
+ + +
+ +
+ + +
------HT------
Thí sinh không được s dng máy tính cm tay.
H và tên thí sinh: ................................................. S báo danh: ............................
Ch ký ca giám th 1: ............................. Ch ký ca giám th 2: ...............................
S GIÁO DC VÀ ĐÀO TO
HÀ NAM
ĐÁP ÁN CHÍNH THC
K THI CHN HC SINH GII LP 9 THCS
NĂM HC 2012-2013
Môn thi: TOÁN
ĐÁP ÁN-BIU ĐIM
(Đáp án biu đim này gm 3 trang)
Câu
Ni dung Đim
§iÒu kiÖn ®Ó P x¸c ®Þnh lµ :
0;1;0;0
+
yxyyx
.
0,5
(
)
( )( )( )
(1 ) (1 )
1 1
x x y y xy x y
P
x y x y
+ +
=
+ +
(
)
(
)
( )( )( )
( )
1 1
x y x x y y xy x y
x y x y
+ + +
=
+ +
0,5
(
)
(
)
( )( ) ( )
1 1
x y x y x xy y xy
x y x y
+ + +
=
+ +
(
)
(
)
(
)
(
)
( )( )
1 1 1 1
1 1
x x y x y x x
x y
+ + + +
=
+
0,5
( )
1
x y y y x
y
+
=
(
)
(
)
(
)
( )
1 1 1
1
x y y y y
y
+
=
0,5
Câu
1.1
(2,5 đ)
= +
x xy y
0,5
P = 2
x xy y
+
= 2 vi
0;1;0;0
+
yxyyx
(
)
(
)
(
)
(
)
1 1 1 1 1 1
x y y x y
+ + = + =
0,5
Ta cã: 1 +
1
y
1 1
x
0 4
x
x = 0; 1; 2; 3 ; 4
0,5
Câu
1.2
(1,5 đ)
Thay vµo P ta cã c¸c cÆp gi¸ trÞ (4; 0) vµ (2 ; 2) tho¶ mn
0,5
Cho hai s thc a, b tha mãn
18 4 2013
a b
+
(1)
Chng minh rng phương trình sau có nghim:
2
18 4 671 9 0
ax bx a
+ + =
(2)
TH1 : Vi a = 0 thì (2)
4 671 0
bx
+ =
T (1)
0
b
. Vy (2) luôn có nghim
671
4
x
b
=
0,5
TH2 : Vi
0
a
, ta có :
2 2 2
' 4 18 (671 9 ) 4 6 .2013 162
b a a b a a
= = +
0,5
2 2 2 2 2 2
4 6 (18 4 ) 162 4 24 54 (2 6 ) 16 0, ,
b a a b a b ab a b a a a b
+ + = + = +
0,5
Câu
2.1
(2,0 đ)
Vy pt luôn có nghim 0,5
Tìm các s nguyên x, y tha mãn phương trình:
3 2 3
2 3 2
x x x y
+ + + =
Ta có
2
3 3 2
3 7
2 3 2 2 0
4 8
= + + = + + > <
y x x x x x y
(1)
0,5
2
3 3 2
9 15
( 2) 4 9 6 2 0 2
4 16
+ = + + = + + > < +
x y x x x y x
(2)
0,5
T (1) và (2) ta có x < y < x+2 mà x, y nguyên suy ra y = x + 1 0,5
Câu
2.2
(2,0 đ)
Thay y = x + 1 vào pt ban đầu và gii phương trình tìm được x = -1; x = 1 t đó
tìm được hai cp s (x, y) tha mãn bài toán là (1 ; 2), (-1 ; 0)
0,5
Do p là s nguyên t ln hơn 3 nên p có dng
3 1
p k
= ±
0,5
Câu
3.1
(2,0đ)
*) Nếu
3 1
p k
= +
thì
2 1 6 3 3(2 1)
p k k
+ = + = +
0,5
2 1
p
+
là hp s (Vô lý)
*) Nếu
3 1, 2
p k k
=
thì
4 1 12 3 3(4 1)
p k k
+ = =
0,5
Do
4 1 7
k
nên
4 1
p
+
là mt hp s.
0,5
Điu kin:
1
2
x
0,5
PT
2
4 3 3 4 3 2 2 1
x x x x x
+ + = + +
(
)
(
)
2
4 4 3 3 1 2 2 1 2 1 0
x x x x x x
+ + + + + =
0,5
(
)
(
)
2 2
2 3 1 2 1 0
x x x
+ + =
0,5
2 3
1 2 1
x x
x
= +
=
0,5
Câu
3.2
(2,5 đ)
2
4 3
1
1 2 1
x x
x
x
= +
=
=
(tmđk)
0,5
Câu 4
Câu
4.1
(2,5 đ)
Hình v đúng.
+PK là phân giác góc
QPO
=
MPE KPQ
(*) .
+ Tam giác OMN đều
0
120
=EMP .
+ QK cũng là phân giác
OQP
(
)
0
QKP 180 KQP KPQ
= +
0 0 0
2KQP 2KPQ 180 60 120
+ = =
0
120
QKP = . Do đó:
( )
EMP QKP **
=
.
T (*) và (**), ta có
MPE KPQ
0,5
0,5
0,5
0,5
0,5
Do hai tam giác MPE và KPQ đồng dng nên:
MEP KQP
=
0,5
Câu
4.2
(1,0 đ)
hay:
FEP FQP
= Suy ra, t giác PQEF ni tiếp được trong đường tròn.
0,5
Gi D là trung đim ca đon PQ. Chng minh tam giác DEF là mt tam giác đều.
Do hai tam giác MPE và KPQ đồng dng nên:
PM
PK
=
PE
PQ
. Suy ra:
PM
PE
=
PK
PQ
.
Ngoài ra:
MPK EPQ
= . Do đó, hai tam giác MPK và EPQ đồng dng.
0,5
T đó:
0
PEQ PMK 90
= = .
0,5
Suy ra, D là tâm ca đường tròn ngoi tiếp t giác PQEF.
Vì vy, tam giác DEF cân ti D.
0,5
Ta có:
FDP 2FQD OQP
= = ;
EDQ 2EPD OPQ
= = .
0,5
Câu
4.3
(2,5 đ)
(
)
0 0
FDE 180 FDP EDQ POQ 60
= + = =
T đó, tam giác DEF là tam giác đều.
0,5
Câu 5
Cho a, b, c là ba s thc dương tha mãn:
3
a b c
+ + =
. Chng minh rng:
K
E
F
D
N
P
Q
y
M
O
x
2 2 2
1 1 1
3
1 1 1
a b c
b c a
+ + +
+ +
+ + +
Theo bt đẳng thc Cauchy ta có:
2
1 2
b b
+
nên:
2 2
2 2
1 ( 1) ( 1)
( 1) ( 1) 1
1 1 2 2
a b a b a ab b
a a a
b b b
+ + + +
= + + = +
+ +
2
1
1
1 2
a ab b
a
b
+ +
+
+
Tương t ta có:
2
1
1
1 2
b bc c
b
c
+ +
+
+
(2)
2
1
1
1 2
c ca a
c
a
+ +
+
+
(3)
0,5
(2,0 đ)
Cng vế theo vế (1), (2) và (3) ta được:
2 2 2
1 1 1
3
1 1 1 2
a b c a b c ab bc ca
b c a
+ + + + +
+ + +
+ + +
(*)
Mt khác:
( )
2
3( ) 9 0
2
a b c ab bc ca
ab bc ca a b c
+ +
+ + + + =
Nên (*)
2 2 2
1 1 1
3
1 1 1
a b c
b c a
+ + +
+ +
+ + +
(đpcm)
Du "=" xy ra khi và ch khi
1
a b c
= = =
0,5
0,5
---------------HT--------------
Lưu ý: - Các cách gii đúng khác cho đim tương đương vi biu đim
- Đim toàn bài không làm tròn
S GIÁO DC ĐÀO TO
HÀ NAM
ĐỀ CHÍNH THC
K THI CHN HC SINH GII TNH
LP 9 THCS NĂM 2011
Môn Toán
Thi gian: 150 phút (không k thi gian giao đề)
Bài 1.(6 đim)
1. Cho biu thc
6 2 5 13 48
A
3 1
+ +
=
+
a) Rút gn biu thc A.
b) Tìm nghim nguyên ca phương trình:
2 2
y A x(A x)(A x )
= + +
2. Gi
1 2
d ,d
là các đường thng ln lượt có phương trình:
1
d :y 2x 3m 2
= + +
2
2
d : y (m m)x 4
= +
a) Tìm m để hai đường thng
1 2
d ,d
song song.
b) Tu theo giá tr ca m, tìm giá tr nh nht ca biu thc:
( )
2
2
2
B 2x y 3m 2 (m m)x y 4
= + + + +
Bài 2.(6 đim)
1. Gii phương trình:
(
)
2 3
2(x 2) 3 x 8 2x
+ = + +
2. Tìm m để phương trình sau có 4 nghim phân bit:
(
)
4 3 2 2
x 3x (2m 1)x 3m 1 x m m 0
+ + + + =
Bài 3.(1 đim)
Gii h phương trình :
(
)
(
)
3 3
x 2 y 1 x y 2 x 1 y 0
x y 16
+ =
+ =
Bài 4.(6 đim)
Cho 3 đim c định A, B, C phân bit thng hàng theo th t đó. Đường tròn (O) đi
qua B C (O không thuc BC). Qua A k các tiếp tuyến AE AF đến đường tròn (O) (E
F là các tiếp đim). Gi I là trung đim ca đon thng BC, N là trung đim ca đon thng EF.
1. Chng minh rng: E F nm trên mt đường tròn c định khi đường tròn (O) thay
đổi.
2. Đường thng FI ct đường tròn (O) ti E’(khác F). Chng minh t giác BCE’E
hình thang.
3. Chng minh rng: m đường tròn ngoi tiếp tam giác ONI nm trên mt đường
thng c định khi đường tròn (O) thay đổi.
Bài 5.(1 đim)
Cho tam giác ABC. Xác định v trí ca đim M nm trong tam giác ABC sao cho
AM.BC BM.CA CM.AB
+ +
đạt giá tr nh nht.
-----------------------------HT---------------------------
(Giám th không gii thích gì thêm )
H và tên: ................................................. S báo danh:.................................................
Ch kí ca giám th 1.................................Ch kí ca giám th 2....................................
PHÒNG GIÁO DC – ĐÀO TO ĐỨC TH
ĐỀ THI CHN ĐỘI TUYN HC SINH GII CP HUYN NĂM HC 2013-2014
MÔN TOÁN 9
Thi gian làm bài: 150 phút
Bài 1: Rút gn các biu thc sau:
a)
A 4 10 2 5 4 10 2 5 5
= + + + +
b)
( )
( )
( )
( )
2 2
2 2
2 2
x y x x y y
x y
B
xy x x y y x y
= +
vi xy > 0; x y
Bài 2: Tìm các s nguyên x, y tha mãn
2
y 2xy 7x 12 0
+ =
Bài 3: Gii các phương trình
a)
5 x 5 x
x x 6
x 1 x 1
+ =
+ +
b)
( ) ( )
10 14
x 2013 x 2014 1
+ =
Bài 4: Cho ABC vuông ti A (AC > AB), đường cao AH (H BC). Trên tia HC ly đim D sao cho HD =
HA. Đường vuông góc vi BC ti D ct AC ti E.
a) Chng minh rng BEC ADC. Tính BE theo m = AB
b) Gi M là trung đim ca BE. Chng minh rng BHM BEC. Tính
AHM
c) Tia AM ct BC ti G. Chng minh rng
GB HD
BC AH HC
=
+
Bài 5: a) Cho
(
)
(
)
3 3 2 2
x y 3 x y 4 x y 4 0
+ + + + + + =
và xy > 0
Tìm GTLN ca
1 1
M
x y
= +
b) Vi a, b, c là các s thc dương. Chng minh rng
5 5 5 3 3 3
2 2 2 2 2 2
a b c a b c
a ab b b bc c c ca a 3
+ +
+ +
+ + + + + +
Bài gii ca Nguyn Ngc Hùng – THCS Hoàng Xuân Hãn
Bài 1: a) Đặt
( )
= + + + + = + = + = +
2
x 4 10 2 5 4 10 2 5 x 8 2 6 2 5 8 2 5 1 6 2 5
x 5 1
= +
. Do đó A = 1
b)
(
)
( )
(
)
( )
= +
x y x x y y
B 1
x x y y x y
Xét các trường hp x < y < 0; y < x < 0; x > y > 0 và y > x > 0 ta đều được
=
B 1
Bài 2: Cách 1:
(
)
(
)
(
)
+ = + = + +
2
2
y 2xy 7x 12 0 x y x 3 x 4
(x + 3)(x + 4) là tích ca 2 s nguyên liên tiếp nên không th là 1 s chính phương
đó
x 3 0 x 3
x 4 0 x 4
+ = =
+ = =
T đó ta tìm được (x; y) {(-3; 3); (-4; 4)}
Cách 2:
(
)
+ = + = + =
2 2 2
y 2xy 7x 12 0 4y 8xy 28x 48 0 4y 49 4x 2y 7 1
(
)
(
)
2y 7 2y 7 4x 1
+ + =
ta có
2y 7 1 x 4
2y 7 4x 1 y 4
= =
+ + = =
2y 7 1 x 3
2y 7 4x 1 y 3
= =
+ + = =
Bài 3: a) Cách 1: ĐKXĐ: x -1. Đặt
=
+
5 x
x a
x 1
+ =
+
5 x
x b
x 1
.
Ta có
+ + +
+ = + + = =
+ + +
2 2
5 x 5 x 5x x x x 5 x
a b x x 5
x 1 x 1 x 1
Do đó
a 2
b 3
ab 6
a b 5
a 3
b 2
=
=
=
+ =
=
=
. Vi
2
2
2
5 x
x 2
a 2 x 3x 2 0
x 1
x 3x 2 0
b 3
x 3x 2 0
5 x
x 3
x 1
=
= + =
+
+ =
=
+ =
+ =
+
( )( )
x 1
x 1 x 2 0
x 2
=
=
=
Vi
( )
2
2
2
2
5 x
x 3
a 3 x 2x 3 0
x 1
x 2x 3 0 x 1 2 0
b 2
x 2x 3 0
5 x
x 2
x 1
=
= + =
+
+ = + =
=
+ =
+ =
+
, vô nghim
Vy phương trình có tp nghim S = {1; 2}
Cách 2:
( ) ( )
( )
+ = + = + + + =
+ +
2
2 2 4 3 2
5 x 5 x
x x 6 5x x x 5 6 x 1 x 5x 11x 13x 6 0
x 1 x 1
(
)
(
)
+ + = + + =
4 3 2 2 2
x 5x 11x 13x 6 0 x 3x 2 x 2x 3 0
T đó ta tìm được tp nghim S = {1; 2}
b)
( ) ( )
+ = + =
10 14 5 7
x 2013 x 2014 1 x 2013 x 2014 1
Ta có x = 2013, x = 2014 là 2 nghim ca phương trình. Ta chng minh 2 nghim này là duy nht
Xét x < 2013
< > > + >
7 5 7
x 2014 1 x 2014 1 x 2014 1 x 2013 x 2014 1
Xét 2013 < x < 2014
5
7
0 x 2013 1 x 2013 x 2013
0 x 2013 1
1 x 2014 0
0 x 2014 1
x 2014 x 2014
< < <
< <
< <
< <
<
5 7
x 2013 x 2014 x 2013 x 2014 x 2013 2014 x 1
+ < + = + =
Xét x > 2014
< > > + >
5 5 7
x 2014 1 x 2013 1 x 2013 1 x 2013 x 2014 1
Vy phương trình có nghim x = 2013, x = 2014
Bài 4: a) Xét EDC và BAC có
0
EDC BAC 90 (gt)
C chung
= =
EDC BAC (g – g)
EC BC
DC AC
=
Xét BEC và ADC có
A
B
C
H
D
E
M
G
m
EC BC
DC AC
C chung
=
BEC ADC (c – g - c)
BEC ADC
=
. Mt khác AH = HD (gt) nên
0 0 0 0
ADH 45 ADC 135 BEC 135 AEB 45
= = = =
AEB vuông cân ti A.
Do đó
BE m 2
=
b) Xét AHB và CAB có
0
AHB CAB 90 (gt)
B chung
= =
AHB CAB (g – g)
2 2 2
AB BH BE BH BM BH
AB BH.BC 2AB 2BH.BC BE 2BH.BC
BC AB 2BC BE BC BE
= = = = = =
(Vì BE = 2BM). Xét BHM và BEC có
BM BH
BC BE
MBH chung
=
BHM BEC (c – g - c)
0 0
BHM BEC 135 AHM 45
= = =
c) Xét AHC và BAC có
0
AHC BAC 90 (gt)
C chung
= =
AHC BAC (g – g)
AH AB
HC AC
= (1)
Mt khác AEB vuông cân ti A AM trung tuyến thì AM cũng phân giác hay AG đường
phân giác ca ABC. Suy ra
GB AB
GC AC
=
(2). T (1) và (2) ta có:
( )
GB AH
GB.HC AH.GC GB.HC AH. BC GB GB.HC AH.BC AH.GB
GC HC
= = = =
AH.GB GB.HC HD.BC
+ =
(Vì HD = AH)
(
)
GB. AH HC HD.BC
+ =
GB HD
BC AH HC
=
+
Bài 5: a)
(
)
(
)
3 3 2 2
x y 3 x y 4 x y 4 0
+ + + + + + =
(
)
(
)
(
)
(
)
(
)
2 2 2 2 2 2
x y x xy y 2 x xy y x 2xy y 4 x y 4 0
+ + + + + + + + + + =
( )
( ) ( ) ( )
( )
2
2 2 2 2
1
x xy y x y 2 x y 2 0 x y 2 2x 2xy 2y 2x 2y 4 0
2
+ + + + + + = + + + + + + =
( ) ( ) ( ) ( )
2 2 2
1
x y 2 x y x 1 y 1 2 0 x y 2 0 x y 2
2
+ + + + + + + = + + = + =
Mà xy > 0 do đó x, y < 0
Áp dng BĐT CauChy ta có
( )( )
(
)
(
)
x y
x y 1
2
+
=
nên xy 1, do đó
2
2
xy
Vy
1 1 x y
M 2
x y xy
+
= + =
, GTLN ca M là -2. Đạt được khi x = y = -1
b) Cách 1: Ta có:
( )
( )
( )
3
3 2 2 3 3
2 2
a 2a b
3a 2a b a ab b a b ab a b
a ab b 3
+ + + +
+ +
( )
2
2 2
a ab b ab a b 0
+
luôn đúng.
Do đó
3 5 3 2
2 2 2 2
a 2a b a 2a a b
a ab b 3 a ab b 3
+ + + +
. Chng minh tương t ta được
5 5 5 3 3 3 3 3 3 2 2 2
2 2 2 2 2 2
a b c a b c a b c a b b c c a
a ab b b bc c c ca a 3 3
+ + + +
+ + +
+ + + + + +
Mt khác: Vai trò a, b, c như nhau nên gi s
a b c 0
>
(
)
(
)
(
)
3 3 3 2 2 2 2 2 2
a b c a b b c c a a a b b b c c c a
+ + = + +
( ) ( ) ( ) ( ) ( ) ( ) ( ) ( )
2
2 2 2
a a b b b a a c c c a a b a b a c b c b c 0
= + + + = + + +
T đó suy ra
5 5 5 3 3 3
2 2 2 2 2 2
a b c a b c
a ab b b bc c c ca a 3
+ +
+ +
+ + + + + +
. Du “=” xy ra khi a = b = c
Cách 2: Áp dng BĐT Bunhia m rng ta
5 5 5 6 6 6
2 2 2 2 2 2 3 2 2 3 2 2 3 2 2
a b c a b c
a ab b b bc c c ca a a a b ab b b c bc c c a ca
+ + = + +
+ + + + + + + + + + + +
(
)
2
3 3 3
3 3 3 2 2 2 2 2 2
a b c
a b c a b ab b c bc c a ca
+ +
+ + + + + + + +
Mt khác
( ) ( )
2
2 2 3 3
a b 0 a ab b ab a b ab a b
+ + +
tương t
(
)
3 3
b c bc b c
+ +
(
)
3 3
c a ca c a
+ +
. Suy ra
(
)
(
)
(
)
(
)
3 3 3
2 a b c ab a b bc b c ca c a
+ + + + + + +
(
)
(
)
(
)
(
)
3 3 3 3 3 3
3 a b c a b c ab a b bc b c ca c a
+ + + + + + + + + +
(
)
2
3 3 3
3 3 3
3 3 3 2 2 2 2 2 2
a b c
a b c
a b c a b ab b c bc c a ca 3
+ +
+ +
+ + + + + + + +
D đoán: Mi câu 1 đ theo thang đim 10 và mi câu 2 đ theo thang đim 20
PHÒNG GD&ĐT BÌNH GIANG
ĐỀ THI CHN HC SINH GII VÒNG I
NĂM HC 2012-2013
MÔN: TOÁN - LP 9
(Thi gian làm bài: 150 phút)
Câu I (2,0 đim).
Cho biu thc:
x 2 x 1 1
A
x x 1 x x 1 1 x
+ +
= + +
+ +
vi
x 0, x 1
1) Rút gn A
2) Chng t rng:
1
A
3
<
Câu II (2,0 đim).
1) Gii phương trình:
=
x x 15 17
2) Tìm x, y sao cho:
(
)
2
5x 2 x 2 y y 1 0
+ + + =
Câu III (2,0 đim).
1) Tìm s nguyên x, sao cho :
2
x x p 0
+ =
vi p là s nguyên t.
2) Tìm m để hàm s bc nht
+
=
+
2
2
m 2013m 2012
y x 2011
m 2 2m 3
hàm s
nghch biến.
Câu IV (3,0 đim).
1) Cho tam giác ABC có ba góc nhn ni tiếp đường tròn (O ; R), hai đường
cao BE và CF ca tam giác ct nhau ti H. K đường kính AK ca đường tròn (O ;
R), gi I là trung đim ca BC.
a) Chng minh AH = 2.IO.
b) Biết
0
BAC 60
=
, tính độ dài dây BC theo R.
2) Cho
0
ABC(A 90 )
= , BC = a. Gi bán kính ca đường tròn ni tiếp
ABC
là r. Chng minh rng:
r 2 1
a 2
.
Câu V (1,0 đim).
Cho
x 3y 1
+
. Tìm giá tr nh nht ca biu thc:
2 2
C x y
= +
–––––––– Hết ––––––––
ĐỀ CHÍNH THC
HƯỚNG DN CHM ĐỀ THI CHN HSG VÒNG I NĂM HC 2012-2013
MÔN: TOÁN - LP 9
Câu Phn Ni dung Đim
1
(1,0 đ)
( )( )
x 2 x 1 1
A
x x 1 x 1
x 1 x x 1
+ +
= +
+ +
+ +
( )( )
( )( )
x 2 x 1 x x 1
A
x 1 x x 1
x x
A
x 1 x x 1
+ +
=
+ +
=
+ +
(
)
( )( )
x x 1
x
A
x x 1
x 1 x x 1
= =
+ +
+ +
, vi
x 0, x 1
0.25
0.25
0.25
0.25
Câu I
(2,0 đim)
2
(1,0 đ)
Xét
(
)
2
x 1
1 1 x
A
3 3
x x 1 3(x x 1)
= =
+ + + +
Do
x 0, x 1
( )
2
2
1 3
x 1 0 và x x 1 x 0
2 4
> + + = + + >
1
A 0
3
>
1
A
3
<
0.50
0.25
0.25
1
(1,0 đ)
ĐKXĐ:
x 15
x x 15 17 x 15 x 15 2 0
= =
Đặt
2
t x 15 (t 0) t t 2 0
= =
( )( )
(
)
( )
=
+ =
=
t 2 TM§K
t 2 t 1 0
t 1 lo¹i
Vi
t 2 x 15 2 x 15 4 x 19
= = = =
(TMĐK)
0.25
0.25
0.25
0.25
Câu II
(2,0 đim)
2
(1,0 đ)
ĐKXĐ:
x 0
(
)
2
5x 2 x 2 y y 1 0
+ + + =
2
4x 4 x 1 x 2y x y 0
+ + + =
(
)
(
)
2 2
2 x 1 x y 0
+ =
(1)
(
)
(
)
2 2
2 x 1 0, x y 0 x 0, y
(
)
(
)
2 2
2 x 1 x y 0
+
.
Để (1) xy ra thì
1
x
2 x 1 0
4
(TM)
1
x y 0
y
2
=
=
=
=
0.25
0.25
0.25
0.25
Câu III
(2,0 đim)
1
(1,0 đ)
Theo bài ra:
(
)
2
p x x x x 1
= + = +
x, x + 1 s nguyên liên tiếp
nên
(
)
+
x x 1
là s chn
p là s chn.
Mt khác p là s nguyên t nên p = 2
2
x x 2 0
+ =
(
)
(
)
x 2 x 1 0
+ =
x = 1 hoc x = - 2 (TM)
0.25
0.25
0.50
2
(1,0 đ)
Để m s
2
2
m 2013m 2012
y x 2011
m 2 2m 3
+
=
+
nghch biến thì
2
2
m 2013m 2012
0
m 2 2m 3
+
<
+
(1).
(
)
2
2
m 2 2m 3 m 2 1 0 m
+ = + >
(1)
(
)
(
)
2
m 2013m 2012 0 m 1 m 2012 0
+ < <
m 1 0 m 1
m 2012 0 m 2012
m 1 0 m 1
m 2012 0 m 2012
> >
< <
< <
> >
1 m 2012
< <
0.25
0.25
0.25
0.25
1a
(1,0 đ)
K
H
F
E
O
I
B
C
A
B, C thuc đường tròn đường kính
AK
0
ABK ACK 90
= =
KB AB, KC AC
CH AB,BH AC
(gt)
BK // CH,CK // BH
BHCK
là hình bình hành
I là trung đim ca BC (gt)
I
là trung đim ca HK
O là trung đim ca AK (gt)
OI
đường trung bình ca
KAH
1
OI AH AH 2.IO
2
= =
0.25
0.25
0.25
0.25
1b
(1,0 đ)
OA OC OAC
=
cân ti O
OAC OCA
=
KOC OAC OCA
= + (T/c góc ngoài ca tam giác)
KOC 2.OAC
=
Chng minh tương t:
KOB 2.OAB
=
(
)
0
KOC KOB 2 OAC OAB BOC 2.BAC 120
+ = + = =
OB OC OBC
=
cân ti O
(
)
0 0 0
OCI 180 120 : 2 30
= =
Vì I là trung đim ca BC (gt)
OI BC
Trong
(
)
0
OIC I 90
=
$
:
0
3
IC OC.cos30 R. BC R 3
2
= = =
0.25
0.25
0.25
0.25
Câu IV
(3,0 đim)
2
(1,0 đ)
r
D
E
F
O
A
B
C
r 2 1
2r a 2 a 2r a a 2
a 2
+
C/m được AB + AC = 2r + a
AB AC BC 2
+
2 2 2
AB 2AB.AC AC 2BC
+ +
( ) ( )
2 2 2 2
2
AB 2AB.AC AC 2AB 2AC
AB AC 0 1
+ + +
B
ĐT (1) đúng
r 2 1
a 2
, du “=” xy ra khi
ABC
v/cân ti A.
0.25
0.25
0.25
0.25
Câu V
(1,0 đim)
(1,0 đ)
Do
x 3y 1
+
, đặt
x 3y 1 a
+ = +
vi
a 0
x = 1 + a 3y, thay vào
biu thc C:
2 2
C 10y 6ay 6y a 2a 1
= + + +
( )
( )
2
2
3 1 1 1
C 10 y a 1 a 2a
10 10 10 10
= + + + +
.
1
min C
10
=
khi:
( )
3
3 3 3
y
y a 1 0 y y
10
10 10 10
1
a 0 a 0 x 3y 1
x
10
=
+ = = =
= = + =
=
0.25
0.50
0.25
* Hc sinh làm bng cách khác đúng vn cho đim ti đa.
| 1/159

Preview text:

SỞ GIÁO DỤC VÀ ĐÀO TẠO
KÌ THI CHỌN HỌC SINH GIỎI LỚP 9 THCS THÀNH PHỐ CẦN THƠ
CẤP THÀNH PHỐ-NĂM HỌC 2012-2013 Khóa ngày 11/04/2013 Đề chính thức MÔN THI: TOÁN
Thời gian làm bài: 150 phút, không kể thời gian phát đề. Câu 1 (5,0 điểm) √ √ 2m + 16m + 6 m − 2 3 1. Cho biểu thức P = √ + √ + √ − 2 m + 2 m − 3 m − 1 m + 3 a) Rút gọn P .
b) Tìm giá trị tự nhiên của m để P là số tự nhiên. √ √ 2013
2. Tính giá trị (a3 + 15a − 25) với a = 3 p13 − 7 6 + 3p13 + 7 6. Câu 2 (5,0 điểm) √ √ √ 1. Giải phương trình: x + 5 + 3 − x − 2 15 − 2x − x2 + 1 = 0.
2. Tìm giá trị của m để hệ phương trình sau có nghiệm: 2x2 + mx − 1 = 0 mx2 − x + 2 = 0 Câu 3 (5,0 điểm) 1 1 1
1. Tìm tất cả các số nguyên dương x, y, z thỏa + + = 2. x y z x + y ≤ 2
2. Cho hai số x, y thỏa mãn: x2 + y2 + xy = 3
Tìm giá trị lớn nhất, giá trị nhỏ nhất của biểu thức T = x2 + y2 − xy. Câu 4 (2,0 điểm)
Cho đường tròn (O; R) và hai điểm A, B nằm ngoài đường tròn sao cho OA = 2R. Tìm điểm M
trên đường tròn để M A + 2M B đạt giá trị nhỏ nhất. Câu 5 (3,0 điểm)
Cho tam giác ABC có ba góc nhọn nội tiếp đường tròn (O; R). Gọi P là một điểm di động trên cung BC không chứa A.
1. Gọi M, N lần lượt là hình chiếu vuông góc hạ từ A xuống P B, P C. Chứng minh rằng đường
thẳng M N luôn đi qua một điểm cố định.
2. Gọi I, D, E là chân các đường cao lần lượt hạ từ A, B, C xuống các cạnh BC, CA, AB.
Chứng minh rằng chu vi tam giác IDE không đổi khi A, B, C thay đổi trên đường tròn
(O; R) sao cho diện tích của tam giác ABC luôn bằng a2. —–HẾT—–
Ghi chú: Giám thị coi thi không giải thích gì thêm.
SỞ GIÁO DỤC VÀ ĐÀO TẠO
KÌ THI CHỌN HỌC SINH GIỎI LỚP 9 THCS THÀNH PHỐ CẦN THƠ
CẤP THÀNH PHỐ-NĂM HỌC 2012-2013 Khóa ngày 11/04/2013 Đề chính thức MÔN THI: TOÁN
Thời gian làm bài: 150 phút, không kể thời gian phát đề. HƯỚNG DẪN CHẤM
(Hướng dẫn chấm này có 03 trang.) CÂU NỘI DUNG ĐIỂM 1. (3,5 điểm)
a) Điều kiện: m ≥ 0, m 6= 1 0,5đ √m + 1 P = √ 2,0đ m − 1 2 b) P = 1 + √ 0,5đ 1(5,0đ) m − 1
Để P ∈ N =⇒ m ∈ {4; 9} 0,5đ 2.(1,5 điểm) √ √ a = 3
p13 − 7 6 + 3p13 + 7 6 =⇒ a3 = 26 − 15a 1,0đ 2013
a3 + 15a − 25 = 1 =⇒ (a3 + 15a − 25) = 1 0,5đ 1. (2,5 điểm)
Điều kiện: −5 ≤ x ≤ 3 0,5đ √ √ √ √ Đặt t = x + 5 +
3 − x, t2 = 8 + 2 15 − 2x − x2 =⇒ t ≥ 2 2 t = 3
Phương trình đã cho có dạng: t2 − t − 6 = 0 ⇐⇒ 1,0đ t = −2 (loại) √ √ t = 3 ⇐⇒ x + 5 + 3 − x = 3 √ 1,0đ 2(5,0đ)  −2 + 3 7 x =
⇐⇒ 4x2 + 8x − 59 = 0 ⇐⇒  2 √  −2 − 3 7 x = 2 2. (2,5 điểm) mx + 2y = 1
Đặt x2 = y ≥ 0. Hệ trở thành: 0,5đ −x + my = −2  m + 4  x =   m2 + 2 Hệ luôn có nghiệm: 0,5đ 1 − 2m 1   y = ≥ 0 (m ≤ )  m2 + 2 2 m + 4 2 1 − 2m Ta có: x2 = y ⇐⇒ = 0,5đ m2 + 2 m2 + 2
⇐⇒ (m + 1) (m2 − m + 7) = 0 ⇐⇒ m = −1 1,0đ 3(5,0đ) 1. (3,0 điểm) Tiếp CÂU NỘI DUNG ĐIỂM
Không mất tính tổng quát giả sử: 1 ≤ x ≤ y ≤ z 1,0đ 1 1 1 3 =⇒ 2 = + + ≤ =⇒ x = 1 x y z x 1 1 2 y = 1 (vô lý) =⇒ + = 1 ≤ =⇒ 1,0đ y z y y = 2 =⇒ z = 2
Vậy (1; 2; 2) và các hoán vị của chúng là nghiệm của phương trình đã cho 1,0đ 2. (2,0 điểm) ( x + y ≤ 2 ( x + y = 2 − a (a ≥ 0) Hệ ⇐⇒ 0,5đ x2 + y2 + xy = 3 x2 + y2 + xy = 3 ( x + y = 2 − a Do đó:
, ∆ = S2 − 4P ≥ 0 =⇒ 0 ≤ a ≤ 4 0,5đ xy = (2 − a)2 − 3
T = x2 + y2 + xy − 2xy = 9 − 2(2 − a)2 0,5đ
min T = 1 khi x = 1, y = 1 hoặc x = −1, y = −1 √ √ √ √ 0,5đ max T = 9 khi x =
3, y = − 3 hoặc x = − 3, y = 3 B M 0 M C O A 4(2,0đ) R
Gọi C là điểm trên đoạn thẳng OA sao cho OC =
, ta có điểm C cố định 0,5đ 2
Dễ thấy ∆OCM đồng dạng ∆OM A =⇒ M A = 2M C 0,5đ
Ta có M A + M B ≥ BC (không đổi) 0,5đ
M A + 2M B = 2(M B + M C) ≥ 2BC
Dấu “=” xảy ra khi và chỉ khi M nằm giữa B và C
Vậy khi điểm M là giao điểm của đoạn BC và đường tròn (O) thì M A+2M B 0,5đ đạt giá trị nhỏ nhất 5(3,0đ) 1. (2,0 điểm) Tiếp CÂU NỘI DUNG ĐIỂM A D E O C M I B N A0 P
Kẻ AI ⊥ BC, I ∈ BC cố định. Ta có \ BM A = [ BIA = 90◦ nên tứ giác 1,0đ AM BI nội tiếp hay [ AIM = \ ABM
Ta lại có tứ giác ABP C nội tiếp nên \ ABM = [ ACP Do đó [ AIM = [ ACP (1) Mặt khác [ AIC = \
AN C = 90◦ nên tứ giác AIN C nội tiếp, suy ra 0,5đ [ ACP + [ AIN = 180◦ (2) Từ (1) và (2) suy ra [ AIM + [ AIN = 180◦ 0,5đ
Vậy đường thẳng M N luôn đi qua điểm cố định I 2. (1,0 điểm)
Tứ giác BCDE nội tiếp suy ra \ AED = [ ACB 0,5đ
Kéo dài AO cắt (O; R) tại điểm A0. Ta có: [ EAO + \ AED = \ BAA0 + [ ACB = 90◦ 1 1
=⇒ AO ⊥ DE =⇒ SAEOD = AO.DE = R.DE 2 2 1 1
Tương tự ta cũng có: SBEOI = R.EI, SCDOI = R.ID 0,5đ 2 2 1
Vậy: SABC = SAEOD + SBIOE + SCDOI = R.(DE + EI + ID) 2 2S 2a2 =⇒ DE + EI + ID = ABC = (không đổi) R R —–HẾT—– Ghi chú:
• Mọi cách giải đúng khác đáp án đều cho điểm tối đa.
SỞ GIÁO DỤC VÀ ĐÀO TẠO
KỲ THI CHỌN HỌC SINH GIỎI LỚP 9
THÀNH PHỐ ĐÀ NẴNG NĂM HỌC 2010-2011 ĐỀ CHÍNH THỨC Môn thi: TOÁN
Thời gian: 150 phút (không tính thời gian giao đề)
Bài 1. (2,0 điểm) 2      Cho biểu thức: a 1 a a 1 a a a a 1 M    với a > 0, a  1. a a  a a  a a
a) Chứng minh rằng M  4.
b) Với những giá trị nào của a thì biểu thức 6 N  nhận giá trị nguyên? M
Bài 2. (2,0 điểm)
a) Cho các hàm số bậc nhất: y  0,5x  3, y  6  x và y  mx có đồ thị lần
lượt là các đường thẳng (d
). Với những giá trị nào của tham số m thì 1), (d2) và (m
đường thẳng ( ) cắt hai đường thẳng (d
) lần lượt tại hai điểm A và B sao m 1) và (d2
cho điểm A có hoành độ âm còn điểm B có hoành độ dương?
b) Trên mặt phẳng tọa độ Oxy, cho M và N là hai điểm phân biệt, di động lần
lượt trên trục hoành và trên trục tung sao cho đường thẳng MN luôn đi qua điểm cố
định I(1 ; 2) . Tìm hệ thức liên hệ giữa hoành độ của M và tung độ của N; từ đó, suy 1 1
ra giá trị nhỏ nhất của biểu thức   . Q 2 2 OM ON
Bài 3. (2,0 điểm) 1  7x  2y  2011 xy
a) Giải hệ phương trình:  x  2y  3x . y
b) Tìm tất cả các giá trị của x, y, z sao cho: 1 x  y  z  z  x  (y  3). 2
Bài 4. (3,0 điểm)
Cho đường tròn (C) với tâm O và đường kính AB cố định. Gọi M là điểm di
động trên (C) sao cho M không trùng với các điểm A và B. Lấy C là điểm đối xứng
của O qua A. Đường thẳng vuông góc với AB tại C cắt đường thẳng AM tại N.
Đường thẳng BN cắt đường tròn (C ) tại điểm thứ hai là E. Các đường thẳng BM và CN cắt nhau tại F.
a) Chứng minh rằng các điểm A, E, F thẳng hàng.
b) Chứng minh rằng tích AMAN không đổi.
c) Chứng minh rằng A là trọng tâm của tam giác BNF khi và chỉ khi NF ngắn nhất.
Bài 5. (1,0 điểm)
Tìm ba chữ số tận cùng của tích của mười hai số nguyên dương đầu tiên. ---HẾT---
Họ và tên thí sinh: ................................................. Số báo danh: ........................
Chữ ký của giám thị 1: ............................. Chữ ký của giám thị 2: ...........................
SỞ GIÁO DỤC VÀ ĐÀO TẠO
KÌ THI CHỌN SINH HỌC SINH GIỎI LỚP 9
THÀNH PHỐ ĐÀ NẴNG NĂM HỌC 2010-2011 Môn thi: TOÁN
HƯỚNG DẪN CHẤM MÔN TOÁN LỚP 9
Dưới đây là sơ lược biểu điểm của đề thi Học sinh giỏi lớp 9. Các Giám khảo thảo luận
thống nhất thêm chi tiết lời giải cũng như thang điểm của biểu điểm đã trình bày. Tổ chấm có thể
phân chia nhỏ thang điểm đến 0,25 điểm cho từng ý của đề thi. Tuy nhiên, điểm từng bài, từng câu
không được thay đổi. Nội dung thảo luận và đã thống nhất khi chấm được ghi vào biên bản cụ thể để
việc chấm phúc khảo sau này được thống nhất và chính xác.

Học sinh có lời giải khác đúng, chính xác nhưng phải nằm trong chương trình được học thì
bài làm đúng đến ý nào giám khảo cho điểm ý đó.
Việc làm tròn số điểm bài kiểm tra được thực hiện theo quy định của Bộ Giáo dục và Đào
tạo tại Quyết định số 40/2006/BGD-ĐT. BÀI-Ý ĐỀ -ĐÁP ÁN ĐIỂM 2      Cho biểu thức: a 1 a a 1 a a a a 1 M    với a > 0, a  1. a a  a a  a a Bài 1
a) Chứng minh rằng M  4.
b) Với những giá trị nào của a thì biểu thức 6 N  nhận giá trị nguyên. M 2,00 a a 1 ( a 1)(a  a 1) a  a 1 Do a > 0, a  1 nên:   và a  a a ( a 1) a 0,25 2 a  a a  a 1
(a 1)(a 1)  a (a 1) (a 1)(a  a 1) a   a 1    a  a a a (1 a) a (1 a) a 0,25 1.a (1,25đ) a 1  M   2 a 0,25 Do a  0; a  1 nên: 2
( a 1)  0  a 1  2 a 0,25  2 a M   2  4 a 0,25 6 3 Ta có 0  N 
 do đó N chỉ có thể nhận được một giá trị nguyên là 1 M 2 0,25 1.b 6 a Mà N = 1 
1  a  4 a 1  0  2 ( a  2)  3 (0,75đ) a 1 2 a  a  2  3 hay a  2  3 (phù hợp) 0,25 Vậy, N nguyên  2 a  (2  3) 0,25
a) Cho các hàm số bậc nhất: y  0,5x  3 , y  6  x và y  mx có đồ thị lần lượt
là các đường thẳng (d1), (d2) và (m). Với những giá trị nào của tham số m thì đường
thẳng (m) cắt hai đường thẳng (d1) và (d2) lần lượt tại hai điểm A và B sao cho điểm Bài 2
A có hoành độ âm còn điểm B có hoành độ dương?
b) Trên mặt phẳng tọa độ Oxy, cho M và N là hai điểm phân biệt, di động lần lượt
trên trục hoành và trên trục tung sao cho đường thẳng MN luôn đi qua điểm cố định
I(1 ; 2) . Tìm hệ thức liên hệ giữa hoành độ của M và tung độ của N; từ đó, suy ra giá 2,00
trị nhỏ nhất của biểu thức 1 1   . Q 2 2 OM ON Điều kiện để ( 
m) là đồ thị hàm số b ậc nhất là m 0 0,25
Phương trình hoành độ giao điểm của (d1) và (m) là:
0,5x  3  mx  (m  0,5)x  3 2.a
Điều kiên để phương trình này có nghiệm âm là m  0,5  0 hay m  0,5 0,25
(0,75đ) Phương trình hoành độ giao điểm của (d2) và (m) là:
6  x  mx  (m 1)x  6
Điều kiên để phương trình này có nghiệm dương là m 1  0 hay m  1 
Vậy điều kiện cần tìm là: 1   m  0,5; m  0 0,25
Đặt m = xM và n = yN  mn  0 và m  1 (*)
Nên đường thẳng qua ba điểm M, I, N có dạng: y = ax+b 0,25 0  am  b  
2  a  b  hệ thức liên hệ giữa m và n là 2m  n  mn n  b  0,25 2.b
Chia hai vế cho mn  0 ta được: 1 2  1 (**) (1,25đ) m n 2 2   1 2  1 4 4  1 1   2 1  1       5          2 2 2 2  m n  m n mn  m n   m n  0,25  1 1 1 Q  
 ; dấu “=” xảy ra khi 2 1
 ; kết hợp (**): m = 5, n = 2,5 (thỏa (*)) 2 2 m n 5 m n 0,25
Vậy giá trị nhỏ nhất của Q là 1 5 0,25 1  7x  2y  2011 xy
a) Giải hệ phương trình:  (1) x  2y  3x . y Bài 3 1
b) Tìm tất cả các giá trị của x, y, z sao cho: x  y  z  z  x  (y  3) (2) 2 2,0 đ 1  7 2 1 1007  9   2011   x  y x      Nếu y 9 490 xy  0 thì (1)       (phù hợp) 1 2 1 490 9  3      y  y x x 9  1007 0,50 1  7 2 1 1  004 3.a   2011     (1,25đ)  y x  Nếu y 9 xy  0 thì (1)      xy  0 (loại) 1 2 1 1031  3      y x x 18 0,25
Nếu xy  0 thì (1)  x  y  0 (nhận). 0,25  
KL: Hệ có đúng 2 nghiệm là 9 9 (0;0) và ;    490 1007  0,25
Điều kiện x ≥ 0; y  z ≥ 0; z  x ≥ 0  y ≥ z ≥ x ≥ 0 0,25 3.b
(2)  2 x  2 y  z  2 z  x  x  y  z  z  x  3 (0,75đ)  2 2 2
( x 1)  ( y  z 1)  ( z  x 1)  0 0,25  x 1    x 1  
  y  z  1  y  3 (thỏa điều kiện)    z  x  1  z 2  0,25
Cho đường tròn (C ) với tâm O và đường kính F
AB cố định. Gọi M là điểm di động trên (C )
sao cho M không trùng với các điểm A và B. M
Lấy C là điểm đối xứng của O qua A. Đường
thẳng vuông góc với AB tại C cắt đường thẳng
AM tại N. Đường thẳng BN cắt đường tròn (C
) tại điểm thứ hai là E. Các đường thẳng BM và C A B O Bài 4 CN cắt nhau tại F.
a) Chứng minh rằng các điểm A, E, F thẳng hàng. C ( )
b) Chứng minh rằng tích AM E AN không đổi.
c) Chứng minh rằng A là trọng tâm của tam
giác BNF khi và chỉ khi NF ngắn nhất. N 3,0 đ MN  BF và BC  NF 0,25
 A là trực tâm của tam giác BNF 0,25 4.a  FA  NB
(1,00đ) Lại có AE  NB 0,25 Nên A, E, F thẳng hàng 0,25
CAN  MAB , nên hai tam giác ACN và AMB đồng dạng. 0,25 4.b AN AC (0,75đ) Suy ra:  AB AM 0,25 Hay 2
AM  AN  AB AC  2R không đổi (với R là bán kính đường tròn (C )) 0,25 2 Ta có BA 
BC nên A là trong tâm tam giác BNF  C là trung điểm NF (3) 3 0,25
Mặt khác: CAN  CFM , nên hai tam giác CNA và CBF đồng dạng CN AC 4.c  2 
 CNCF  BCAC  3R (1,25đ) BC CF 0,25
Áp dụng bất đẳng thức Cô-si, ta có: NF  CN  CF  2 CNCF  2R 3 không đổi 0,25
Nên: NF ngắn nhất  CN =CF  C là trung điểm NF (4) 0,25
(3) và (4) cho ta: A là trong tâm tam giác BNF  NF ngắn nhất 0,25 Bài 5
Tìm ba chữ số tận cùng của tích của mười hai số nguyên dương đầu tiên. 0,75
Đặt: S = 123456789101112  S
 3467891112 (1) là một số nguyên 100
 hai chữ số tận cùng của S là 00 0,50
(1,00đ) Mặt khác, trong suốt quá trình nhân liên tiếp các thừa số ở vế phải của (1), nếu chỉ để ý
đến chữ số tận cùng, ta thấy S có chữ số tận cùng là 6 (vì 34=12; 26=12; 27=14; 100
48=32; 29=18; 811=88; 812=96) 0,25
Vậy ba chữ số tận cùng của S là 600 0,25 --- Hết ---
Điều kiện x ≥ 0; y  z ≥ 0; z  x ≥ 0  y ≥ z ≥ x ≥ 0 0,25      Theo BĐT Cauchy: x 1 y z 1 z x 1 x  ; y  z  ; z  x  2 2 2 1
 VP  x  y  z  z  x  (y  3)  VT 3.b 2 0,25 (0,75đ)  x 1    x 1  
Do đó  y  z  1  y  3 thỏa điều kiện    z  x  1  z 2  0,25
PHÒNG GD-ĐT CẨM THỦY KỲ THI CHỌN HỌC SINH GIỎI TOÁN 9 (ĐỀ SỐ 3)
năm học : 2011 - 2012
Môn : TOÁN
(Thời gian làm bài: 150 phút: Vòng 2) Bài 1 ( 3,0 điểm) 2ab a  x  a  x 1
Cho các số dương: a; b và x = . Xét biểu thức P =  2 b  1 a  x  a  x b 3
1. Chứng minh P xác định. Rút gọn P.
2. Khi a và b thay đổi, hãy tìm giá trị nhỏ nhất của P. Bài 2 (3,0 điểm)
Tìm x; y; z thoả mãn hệ sau:  x3  x 3  2  2  y  y3  y 3  2  4  2z  3 z  z 3  2  6  x 3 Bài 3 ( 3,0 điểm) 3  5
Với mỗi số nguyên dương n ≤ 2008, đặt Sn = an +bn , với a = ; b 2 3  5 = . 2
1. Chứng minh rằng với n ≥ 1, ta có Sn + 2 = (a + b)( an + 1 + bn + 1) – ab(an + bn)
2. Chứng minh rằng với mọi n thoả mãn điều kiện đề bài, Sn là số nguyên. 2  n   5  n 1  5 1 3. Chứng minh S  
 . Tìm tất cả các số n để S n – 2 =     n – 2 2        2 là số chính phương. Bài 4 (5,0 điểm)
Cho đoạn thẳng AB và điểm E nằm giữa điểm A và điểm B sao cho AE < BE.
Vẽ đường tròn (O ) đường kính AE và đường tròn (O ) đường kính BE. Vẽ tiếp 1 2
tuyến chung ngoài MN của hai đường tròn trên, với M là tiếp điểm thuộc (O1) và N
là tiếp điểm thuộc (O2).
1. Gọi F là giao điểm của các đường thẳng AM và BN. Chứng minh rằng đường
thẳng EF vuông góc với đường thẳng AB.
2. Với AB = 18 cm và AE = 6 cm, vẽ đường tròn (O) đường kính AB. Đường
thẳng MN cắt đường tròn (O) ở C và D, sao cho điểm C thuộc cung nhỏ AD. Tính
độ dài đoạn thẳng CD.
Bài 5: (4đ): Cho ABC đường thẳng d cắt AB và AC và trung tuyến AM theo thứ tự . Là E , F , N . AB AC 2AM a) Chứng minh :   AE AF AN
b) Giả sử đường thẳng d // BC. Trên tia đối của tia FB lấy điểm K, đường thẳng KN cắt
AB tại P đường thẳng KM cắt AC tại Q. Chứng minh PQ//BC. Bài 6: (2 điểm)
Cho 0 < a, b,c <1 .Chứng minh rằng : a3 2  b3 2  c3 2
 3 a2b b2c c2a
------------- HẾT-------------
HƯỚNG DẪN CHẤM: ĐỀ SỐ 3 Câu 1. (3,0 điểm)
Tóm tắt lời giải Điểm 1. (2.0 điểm) 0,25
Ta có: a; b; x > 0  a + x > 0 (1) a(b  ) 1 2 0,25 Xét a – x =  0 (2) 2 b 1
Ta có a + x > a – x ≥ 0  a  x  a  x  0 (3) 0,25
Từ (1); (2); (3)  P xác định Rút gọn: 2ab a(b  ) 1 2 a 0,25 Ta có: a + x = a    a  x  (b  ) 1 2 b 1 2 b 1 2 b 1 2ab a(b  ) 1 2 a a - x = a    a  x  b 1 0,25 2 b 1 2 b 1 2 b 1 a a (b  ) 1  b 1 2 2     1 1 1 b 1 b b b 1 1 P =    a a b 3 b 1 b 1 b 3 0,25 (b  ) 1  b 1 1 b2 1 b2 2 1 4 Nếu 0 < b < 1  P =   2b b 3 b 3 0,25 1 b 3 2 1 Nếu b  1  P = b   b 3 b 3 2. (1.0 điểm) 0,25 Xét 2 trường hợp: 4 4
Nếu 0 < b < 1, a dương tuỳ ý thì P =  P  b 3 3 1  b 1  2b
Nếu b  1, a dương tuỳ ý thì P = b       3b  3 3b  3 b 1 2 0,25 Ta có: 
 , dấu bằng xảy ra khi và chỉ khi b = 1 3 3b 3 0,25 Mặt khác: 2b 2
 , dấu bằng xảy ra khi và chỉ khi b = 1 3 3 Vậy P 2 2 4
   , dấu bằng xảy ra khi và chỉ khi b = 1 3 3 3
KL: Giá trị nhỏ nhất của P = 4 3 0,25 0,25 Câu 2 (3,0 điểm)
Tóm tắt lời giải Điểm
Biến đổi tương đương hệ ta có  (x  )( 2 x  ) 1 2  2  y (y  )( 2 y  ) 1 2  ( 2 2  z)  1,00 2 (z  )( 2 z  ) 1  ( 3 2  x)
Nhân các vế của 3 phương trình với nhau ta được:
(x - 2)(y - 2) (z - 2)(x+1)2(y+1)2(z+1)2= - 6(x - 2)(y - 2) (z - 2) 0,50
 (x - 2)(y - 2) (z - 2) (x  ) 1 2 (y  ) 1 2 (z  ) 1 2   6 = 0  0,25 (x - 2)(y - 2) (z - 2) = 0
 x = 2 hoặc y = 2 hoặc z = 2 0,25
Với x = 2 hoặc y = 2 hoặc z = 2 thay vào hệ ta đều có x = y = z = 2 0,25
Vậy với x = y = z = 2 thoả mãn hệ đã cho 0,50 0,25 Câu 3 (3,0 điểm)
Tóm tắt lời giải Điểm 1. (1,0 điểm) Với n ≥ 1 thì S 0,25 n + 2 = an+2 + bn +2 (1)
Mặt khác: (a + b)( an + 1 +bn + 1) – ab(an +bn) = an+2 + bn+2 (2) 0,50
Từ (1); (2) ta có điều phải chứng minh 0,25
2. (1.0 điểm) Ta có: S 0,25 1 = 3; S2 = 7
Do a + b =3; ab =1 nên theo 1 ta có: với n ≥ 1 thì S 0,25 n+2 = 3Sn+1 - Sn Do S   0,25 1, S2  Z nên S3
Z; do S2, S3  Z nên S4 Z
Tiếp tục quá trình trên ta được S 0,25
5 ; S6;...; S2008  Z
3. (1.0 điểm) n n 2 2      5 1   5 1  Ta có S         n – 2 = 2      2 2     2 2        0,25 2 2 n n n      5 1  5 1  5 1   5 1 =       2     2      2            2   2  2  n   5  n 1  5 1 =    đpcm     2 2        0,25 Đặt a 5  1 5 1 1 = ; b1 =  a1 + b1 = 5 ; a1b1 = 1 2 2 Xét U n n  n= a b 1 1 Với n ≥ 1 thì U n+1 n + 1 n n n+2 = (a1 + b1)(a1 - b1
) – a1b1(a1 - b1 )  Un+2 = 5 Un+1 – Un 0,25 Ta có U
1 = 1  Z; U2 =
5  Z; U3 = 4  Z; U4 = 3 5  Z;...
Tiếp tục quá trình trên ta được U n nguyên  n lẻ Vậy S  0,25
n – 2 là số chính phương  n = 2k+1 với k  Z và 0 k  1003 Câu 4 (5,0 điểm)
Tóm tắt lời giải Điểm F D N I M C S B A O E O O2 1 0,25 0.25 0,25 1. (2,5 điểm) O 0,25 1M; O2N  MN  O1M/ / O2N
Do O1; E; O2 thẳng hàng nên  MO1E =  NO2B 0,50 Các tam giác O NB lần lượt cân tại O 1ME; O2
1 và O2 nê n t a có:  MEO1=  NBO2 (1) 0,25
Mặt khác ta có:  AME = 900   MAE +  MEO 0,25 1= 900 (2) 0,25   MAE +  NBO 0,25 2 = 900   AFB = 900
 Tứ giác FMEN có 3 góc vuông  Tứ giác FMEN là hình chữ nhật   NME =  FEM 0,25 (3) 0,25 Do MN  MO
1   MNE +  EMO1 = 900 (4) 0,5 Do tam giác O ME cân tại O 1 1   MEO1 =  EMO1 (5) 0,25
Từ (3); (4); (5) ta có:  FEM +  MEO 0,25
1= 900 hay  FEO1 = 900 (đpcm) 2. (2,5 điểm) 0,5 Ta có EB = 12 cm  O 1M = 3 cm < O2 N = 6 cm
 MN cắt AB tại S với A nằm giữa S và B. 0,25
Gọi I là trung điểm CD  O M SO CD  OI  OI// O 1 1   1M //O2 N O N SO 2 2  0,25 SO  2 = 2SO1  SO1+O1O2 = 2SO1 SO1= O1O2
Do O1O2 = 3 + 6 = 9 cm  SO1= O1O2 = 9 cm  SO =SO1 + O1O = 15cm Mặt khác: OI SO   OI = 5 cm O M SO 1 1
Xét tam giác COI vuông tại I ta có: CI2 + OI2= CO2  CI2 + 25 = CO2
Ta có: CO = 9 cm  CI2 + 25 = 81  CI = 56  CD = 4 14 cm Câu 5 (2,0 điểm) Điểm A E E N I M B C S a)
Kẻ BI ,CS // EF (I , S AM ) AB AI AC AS Ta có:  ,  AE AN AF AN
AB AC AI AS ( )  AE AF AN AN 1,0 Ta có: BIM CSM (cgc)  IM MS
Vậy: AI AS AI AI IM MS  2AM
Thay vào (*) ta được (đpcm) 0,5
Khi d // BC EF // BC N là trung điểm của EF 0,5
+Từ F kẻ đường thẳng song song với AB cắt KP tại L Ta có: NFP NFL c
( gc)  EP LF 0,5 Do đó : A K EP LF KF   ) 1 ( L 0,5 PB PB KB N E F
+Từ B kẻ đường thẳng song song với AC cắt P Q KM tại H Ta có BMH CM ( Q cgc)  BH QC B M C 0,5 Do đó: FQ FQ KF   ( ) 2 QC BH KB Từ FP FQ (1) va (2)  
PQ // BC (đpcm) 0,5 PB QC Bài 6: 2 điểm) Do a <1  2 a <1 và b <1 Nên  2
a   b 2 2 1 . 1
 0 1 a b a b  0 Hay  a2 1
b a2  b (1) 0,5 Mặt khác 0 3 a a ; 3 b b  2 3 3
b a a b
Vậy a3  b3  1 a2b 0,5 Tương tự ta có
b3  c3  1 b2c 0,25
a3  c3  1 c2a  0,25 a3 2  b3 2  c3 2
 3 a2b b2c c2a 0,5 UBND HUYỆN
ĐỀ THI CHỌN HỌC SINH GIỎI HUYỆN
PHÒNG GIÁO DỤC - ĐÀO TẠO NĂM HỌC 2013-2014 MÔN: TOÁN LỚP 9 ĐỀ CHÍNH THỨC
Thời gian làm bài 150 phút không kể thời gian giao đề  x  y x  y   x  y  2xy 
Bài 1: (4 điể m) Cho biểu thức: P     : 1    . 1  xy 1  xy    1  xy 
a) Rút gọn biểu thức P. 2
b) Tính giá trị của P với x  . 2  3
Bài 2: (4 điể m) Trong mặt phẳng tọa độ Oxy, gọi (D) và (L) lần lượt là đồ thị của hai hàm 1 3 số: y   x  và y  x . 2 2
a) Vẽ đồ thị (D) và (L).
b) (D) và (L) cắt nhau tại M và N. Chứng minh OMN là tam giác vuông.
Bài 3: (4 điể m) Giải phương trình: 4 3 2
6x  5x  38x  5x  6  0.
Bài 4: (2 điể m) Qua đỉnh A của hình vuông ABCD cạnh là a, vẽ một đường thẳng c ắt cạnh
BC ở M và cắt đường thẳng DC ở I. 1 1 1 Chứng minh rằng:   . 2 2 2 AM AI a Bài 5: (6 điể m)
Cho hai đường tròn ( O ) và ( O/ ) ở ngoài nhau. Đường nối tâm OO/ cắt đường tròn ( O )
và ( O/ ) tại các điểm A, B, C, D theo thứ tự trên đường thẳng. Kẻ tiếp tuyến chung ngoài EF, E (
O ) và F  ( O/ ). Gọi M là giao điểm của AE và DF; N là giao điểm của EB và FC. Chứng minh rằng:
a) Tứ giác MENF là hình chữ nhật. b) MN  AD. c) ME.MA = MF.MD.
---------- Hết ---------- UBND HUYỆN
ĐÁP ÁN VÀ HƯỚNG DẪN CHẤM THI
PHÒNG GIÁO DỤC - ĐÀO TẠO
KỲ THI CHỌN HỌC SINH GIỎI HUYỆN
NĂM HỌC 2013-2014-MÔN: TOÁN LỚP 9 Bài Đáp án Điểm 1
ĐKXĐ: x  0; y  0;xy  1. 0,5 đ a)
Mẫu thức chung là 1 – xy
( x  y)(1  xy)  ( x  y)(1  xy) 1 xy  x  y  2xy P  : 0,5 đ 1 xy 1  xy 0,5 đ x  x y  y  y x  x  x y  y  y x 1  xy  . 1  xy 1  x  y  xy 0,5 đ 2( x  y x) 2 x (1  y) 2 x    (1  x)(1  y) (1  x)(1  y) 1  x b) 2 2(2  3) 2 x  
 3  2 3 1 ( 3 1) 0,5 đ 2  3 4  3 0,5 đ 2 x  ( 3  1)  3  1  3  1 0,5 đ 2( 3  1) 2 3  2 P    2 1  ( 3  1) 1  3  2 3  1 0,5 đ 2( 3  1) 6 3  2 P   5  2 3 13 2  3 a) 1 3 x  0  y  0,5 đ Đồ thị y   x  có :  2 2 2  y  0  x  3  x khi x  0 Đồ thị y  x   0,5 đ x khi x  0 Đồ thị như hình vẽ: 1 đ b)
Đồ thị (D) và (L) cắt nhau tại hai điểm có tọa độ M(1; 1) và N( - 3; 3) 0,5 đ 2 2 Ta có: OM = 1  1  2  OM2 = 2 2 2 ON = 3  ( 3  )  3 2  ON2 = 18 0,5 đ MN = 2 2
(1  3)  (1  3)  20  MN2 = 20 0,5 đ Vì: OM2 + ON2 = MN2 0,5 đ
Vậy: tam giác OMN vuông tại O 3
Ta thấy x = 0 không phải là nghiệm của phương trình
Chia cả 2 vế của phương trình cho x2 ta được: 5 6 2 6x  5x  38    0 2 x x 1 1 2 1 đ  6(x  )  5(x  )  38  0 2 x x 1 1 Đặt y  x  2 2 thì: x   y  2 x 2 x
Ta được pt: 6y2 – 5y – 50 = 0 <=> (3y – 10)(2y + 5) = 0 1 đ 10 5 Do đó: y  và y   3 2 10 1 10 * Với y  thì: 2 x    3x 10x  3  0 3 x 3  1 1 đ x  
<=> (3x – 1)(x – 3) = 0 <=> 1 3  x  3  2 5 1 5 * Với y   2 thì: x 
   2x  5x  2  0 2 x 2 1 đ  1 x   
<=> (2x + 1)(x + 3) = 0 <=> 3 2  x  2   4 4 A B M J D C I
Vẽ Ax  AI cắt đường thẳng CD tại J. 0,5 đ
Ta có  AIJ vuông tại A, có AD là đường cao thuộc cạnh huyền IJ, nên: 1 1 1   (1) 0,5 đ 2 2 2 AD AJ AI
Xét hai tam giác vuông ADJ và ABM, ta có:
AB = AD = a; DAJ  BAM (góc có cạnh tương ứng vuông góc) 0,5 đ  A  DJ = A  BM. Suy ra: AJ = AM 1 1 1 1 0,5 đ Thay vào (1) ta được:    (đpcm) 2 2 2 2 AD AM AI a 5 M E I F H A D O B C O / N a) 0
Ta có AEB  CFD  90 (góc nội tiếp chắn nữa đường tròn)
Vì EF là tiếp tuyến chung của hai đường tròn (O) và (O/ ), nên: 0,5 đ
OE  EF và OF  EF => OE // O/F => /
EOB  FO D (góc đồng vị) => / EAO  FCO 0,5 đ
Do đó MA // FN, mà EB  MA => EB  FN 0,5 đ 0 Hay ENF  90 . Tứ giác MENF có O
E  N  F  90 , nên MENF là hình chữ nhật 0,5 đ b)
Gọi I là giao điểm của MN và EF; H là giao điểm của MN và AD 0,5 đ
Vì MENF là hình chữ nhật, nên IFN  INF 1
Mặt khác, trong đường tròn (O/): IFN  FDC  sđ FC 2 0,5 đ 0,5 đ => FDC  HNC 0,5 đ Suy ra F  DC đồng dạng HNC  (g – g) O
=> NHC  DFC  90 hay MN  AD c) 0,5 đ
Do MENF là hình chữ nhật, nên MFE  FEN 1
Trong đường tròn (O) có: FEN  EAB  sđ EB 2 0,5 đ => MFE  EAB 0,5 đ Suy ra M  EFđồng dạng M  DA (g – g) 0,5 đ ME MF =>  , hay ME.MA = MF.MD MD MA
Lưu ý: Nếu học sinh giải theo cách khác, nếu đúng và phù hợp với kiến thức trong chương
trình đã học thì hai Giám khảo chấm thi thống nhất việc phân bố điểm của cách giải đó, sao cho
không làm thay đổi tổng điểm của bài (hoặc ý) đã nêu trong hướng dẫn này./.

ĐỀ THI HỌC SINH GIỎI TOÁN 9
Thời gian: 150 phút( không kể thời gian giao đề) Câu1: ( 5đ) 2 x  9 2 x  1 x  3 Cho biÓu thøc M =   x  5 x  6 x  3 2  x
a. T×m ®iÒu kiÖn cña x ®Ó M cã nghÜa vµ rót gän M b. T×m x ®Ó M = 5
c. T×m x  Z ®Ó M  Z.
Câu: 2(2đ). Cho 4a2+b2=5ab với 2a>b>0.
Tính giá trị của biểu thức: ab P  2 2 4a b Câu 3(4đ) 3 2 x  8x  6
a. Tìm giá trị nhỏ nhất của biểu thức A  2 x  2x  1 2  2  2   
b. Chứng minh rằng với mọi số thực a,b,c ta có a b c ab bc ca Câu: 4 (4đ)
a. Phân tích đa thức sau thành nhân tử: x3+y3+z3-3xyz
b. Giải phương trình : x4+2x3-4x2-5x-6=0
Câu: 5 (5đ) Cho hình bình hành ABCD có đường chéo AC lớn hơn đường chéo BD. Gọi E, F
lần lượt là hình chiếu của B và D xuống đường thẳng AC.
1) Tứ giác BEDF là hình gì vì sao?
2) Gọi CH và CK lần lượt là đường cao của tam giác ACB và tam giác ACD.Chứng minh rằng.
a. Tam giác CHK và tam giác ABC đồng dạng .
b. AB.AH+AD.AK=AC2 ĐÁP ÁN Câu: 1(5đ) a) ĐK x  ; 0 x  ; 4 x  9 0,5đ
2 x  9   x   3  x   3  2 x   1  x  2 Rút gọn M =  0,5đ
x  2 x   3 x x  2
Biến đổi ta có kết quả: =  0,5đ
x  2 x   3
x  1 x 2 x 1 =  1đ x    3  x  2 x  3 x 1 M  5   5 b) x  3 1đ
x  4  x  ( 16 TM ) x  1 x  3  4 4 c) M =   1 0,5đ x  3 x  3 x  3 Do M  z nên
x  3 là ước của 4 
x  3 nhận các giá trị: -4;-2;-1;1;2;4 0,5đ  x   49 ; 25 ; 16 ; 4 ; 1
do x  4  x   49 ; 25 ; 16 ; 1 0,5đ Câu: 2 (2đ)
Phân tích được 4a2+b2=5ab thành (a-b)(4a-b)=0 0,5đ
<=> a=b hoặc 4a=b 0,5đ
Lập luận chỉ ra a=b (nhận) 4a=b (loại) 0,5đ 2 Tính được ab a 1 P    0,5đ 4 2 2 a b 3 2 a 3 Câu: 3 (4đ) 2 2 2
x x   x x x  a. Viết được 2 4 2 4 4 ( ) 2 A   2   2 1,5đ 2 x  2x  1 (x  ) 1 2
Lập luận min A = 2 khi x-2= 0 => x= 2 0,5đ 2  2  2    b. biến đổi a b c ab bc ca
<=> 2a2+2b2+2c2≥2ab+2bc+2ca 0,5đ
<=> a2-2ab+b2+b2-2bc +c2 +c2 -2ca+a2 ≥0 0,5đ
<=> (a-b)2+(b-c)2+(c-a)2 ≥ 0 0,5đ
Lập luận => khẳng định 0,5đ Câu: 4 (4đ) a. x3+y3+z3-3xyz
= x3+3x2 y+3xy2+y3+z3-3x2y-3xy2 -3xyz 0,5đ
= (x+y)3+z3 –3xyz(x+y+z) 0,5đ
= (x+y+z)(x2+2xy+y2+z2-xz-yz)-3xy(x+y+z) 0,5đ
=(x+y+z)(x2+y2+z2-xy-yz- zx) 0,5đ
b. Giải phương trình : x4+2x3-4x2-5x-6=0
<=> x4-2x3+4x3-8x2+4x2-8x + 3x-6=0 0,5đ
<=> x3(x-2)+4x2(x-2)+4x(x-2)+3(x-2)=0 0,5đ
<=> (x-2)(x3+4x2+4x+3)=0 0,25đ
<=> (x-2)(x3+3x2+x2+3x+x+3) =0 0,25đ
<=> (x-2)[x2(x+3)+x(x+3)+(x+3)]=0 0,25đ
<=> (x-2)(x+3)(x2+x+1) =0 0,25đ Câu: 5 (5đ) H C B F E
1. Chỉ ra Tam giác ABE = Tam giác CDF 0,5đ A K D
=>BE=DF . BE//DF cùng vuông góc với AC 0,25đ
=> BEDF là hình bình hành 0,25đ
2.a. Chỉ ra góc CBH = góc CDK 0,5đ
=> tam giác CHB đồng dạng với Tam giác CDK (g,g) 0,25đ CH CK   0,25đ CB CD
Chỉ ra CB//AD,CK vuông góc CB=> CK vuông góc CB 0,25đ
Chỉ ra góc ABC = góc HCK ( cùng bù với BAD) 0,25đ CH CK CH CK Chỉ ra   hay   vì AB=CD 0,25đ CB CD CB AB
Chỉ ra tam giác CHK đồng dạng tam giác BCA (c- g-c) 0,25đ
b. chỉ ra tam giác AFD = tam giác CEB => AF=CE 0,5đ
chỉ ra tam giác AFD đồng dạng với tam giác AKC 0,25đ
=> AD.AK=AF.AC => AD.AK=CE.AC (1) 0,5đ
Chỉ ra tam giác ABE đồng dạng với tam giác ACH 0,25đ => AB.AH=AE.AC (2) 0,25đ
Công theo vế (1) và (2) ta được
AD.AK+ AB.AH =CE.AC+ AE.AC =(CE+AE)AC=AC2 0,25đ
Lưu ý: Học sinh làm cách khác đúng vẫn cho điể m tối đa
PHÒNG GIÁO DỤC VÀ ĐÀO
ĐỀ THI CHỌN HỌC SINH GIỎI HUYỆN
TẠO HUYỆN KIM THÀNH
NĂM HỌC 2012 – 2013 Môn: Toán 9
Thời gian làm bài: 120 phút Đề gồm 01 trang Bài 1: (4,0 điểm)
a) Rút gọn biểu thức A = 2 x 9 x  3 2 x 1   x  5 x  6 x  2 3  x
b) Cho x, y, z thoả mãn: xy + yz + xz = 1. 2 2 2 2 2 2
(1 y )(1 z )
(1 z )(1 x )
(1 x )(1 y )
Hãy tính giá trị biểu thức: A = xyz 2 2 2 (1 x ) (1 y ) (1 z ) Bài 2: (3,0 điểm)
a) Cho hàm số : f(x) = (x3 + 12x – 31)2012 Tính f(a) tại a = 3 3 16  8 5  16  8 5
b) Tìm số tự nhiên n sao cho n2 + 17 là số chính phương? Bài 3: (4,0 điểm)
Giải các phương trình sau:
a) 1 x  4  x  3 b) 2
x  4x  5  2 2x  3 Bài 4: (3,0 điểm)
a) Tìm x; y thỏa mãn: 2x y 4  y x 4  xy
b) Cho a; b; c là các số thuộc đoạn  1
 ;2 thỏa mãn: a2 + b2 + c2 = 6 hãy chứng minh rằng: a + b + c  0 Bài 5: (6,0 điểm)
Cho tam giác ABC nhọn; các đường cao AK; BD; CE cắt nhau tại H. 2 2 2
a) Chứng minh: KC AC CB BA  2 2 2 KB
CB BA AC
b) Giả sử: HK = 1 AK. Chứng minh rằng: tanB.tanC = 3 3
c) Giả sử SABC = 120 cm2 và BÂC = 600. Hãy tính diện tích tam giác ADE?
TRƯỜNG THCS THƯỢNG VŨ
HƯỚNG DẪN GIẢI ĐỀ THI HSG HUYỆN KIM THÀNH Tổ KHTN
NĂM HỌC 2012 – 2013 Môn: Toán 9 Thời gian: 120’
Câu 1: (4 điểm)
a/ Rút gọn biểu thức A = 2 x 9 x  3 2 x 1   x  5 x  6 x  2 3  x ĐKXĐ: x  4; x  9 2 x  9 x  3 2 x 1
2 x  9  x  9  2x  3 x  2 x x  2 A =     
x  2 x  3 x  2 x  3
x 2 x 3
x 2 x 3
x  1 x 2 x 1 =  
x  2 x  3 x  3
b/ Cho x, y, z thoả mãn: xy + yz + xz = 1. 2 2 2 2 2 2
(1 y )(1 z )
(1 z )(1 x )
(1 x )(1 y ) Hãy tính: A = xyz 2 2 2 (1 x ) (1 y ) (1 z )
Gợi ý: xy + yz + xz = 1  1 + x2 = xy + yz + xz + x2 = y(x + z) + x(x + z) = (x + z)(x + y)
Tương tự: 1 + y2 = …; 1 + z2 = ….
Câu 2: (3 điểm)
a/ Cho hàm số : f(x) = (x3 + 12x – 31)2012 Tính f(a) tại a = 3 3 16  8 5  16  8 5
b/ Tìm số tự nhiên n sao cho n2 + 17 là số chính phương? Giải a/Từ a= 3 3 16  8 5  16  8 5 3    a        3 3 3 32 3 16 8 5 16 8 5
16  8 5  16  8 5  32 12a   nên a3 + 12a = 32 Vậy f(a) = 1    b/ Giả sử: n2 k n
+ 17 = k2 (k  ) và k > n (k – n)(k + n) = 17  1   n  8 k n 17
Vậy với n = 8 thỏa mãn yêu cầu bài toán.
Câu 3: (4 điểm)
Giải các phương trình sau:
a/ 1 x  4  x  3 b/ 2
x  4x  5  2 2x  3 Giải a/ ĐK: 4   x 1
Bình phương 2 vế: 1 x  4  x  2 (1 x)(4  x)  9  (1 x)(4  ) x  2 x  0 2
 4  3x x  4  x(x  3)  0   (thỏa mãn) x  3 
Vậy phương trình có 2 nghiệm: x = 0; x = -3  b/ 2
x  4x  5  2 2x  3 ĐKXĐ: x 3  2   2 x  2x  
1  2x  3 2 2x  3   1  0 x 1  0    x  
1   2x  3  2 2 1  0    x  1
 vậy phương trình có nghiệm duy  2x  3 1 nhất x = -1
Câu 4: (3 điểm)
a/ Tìm x; y thỏa mãn: 2x y 4  y x 4  xy
b/ Cho a; b; c là các số thuộc đoạn  1
 ;2 thỏa mãn: a2 + b2 + c2 = 6 hãy chứng minh rằng: a + b + c  0 Giải
a/ 2x y  4  y x  4  xy  .x2. y  4  .
y 2. x  4  xy Xét VP = .
x 2. y  4  .
y 2. x  4 theo BĐT cosi: 4  y  4 y 4  x  4 x 2 y  4   ;2 x  4   vậy VP  xy = VT 2 2 2 2    Dấu = xảy ra khi: x 4 2 
x y  8  y  4  2 
b/ Do a; b; c thuộc đoạn  1
 ;2 nên a + 1  0; a – 2  0 nên (a + 1)(a – 2)  0
Hay: a2 – a – 2  0  a2  a + 2
Tương tự: b2  b + 2; c2  c + 2
Ta có: a2 + b2 + c2  a + b + c + 6 theo đầu bài: a2 + b2 + c2 = 6 nên: a + b + c  0
Câu 5: (6 điểm)
Cho tam giác ABC nhọn; các đường cao AK; BD; CE cắt nhau tại H. 2 2 2
a/ Chứng minh: KC AC CB BA  2 2 2 KB
CB BA AC
b/ Giả sử: HK = 1 AK. Chứng minh rằng: tanB.tanC = 3 3
c/ Giả sử SABC = 120 cm2 và BÂC = 600. Hãy tính diện tích tam giác ADE? Giải
a/ Sử dụng định lý pytago: A 2 2 2 2 2 2 2
AC CB BA
AK KC  (BK CK )  AB  2 2 2 2 2 2
CB BA AC
(BK CK )  BA  (AK KC) 2
2CK  2BK.CK
2CK (CK BK ) CK =   D 2
2BK  2BK.CK
2BK (BK CK ) BK E H AK AK b/ Ta có: tanB = ; tanC = BK CK 2 AK B K Nên: tanBtanC = (1) C BK.CK Mặt khá KC
c ta có: B HKC mà: tanHKC = KH KC K . B KC Nên tanB =
tương tự tanC = KB  tan . B tan C  (2) KH KH 2 KH 2 Từ (1)(2)     B C 2 AK tan . tan     KH  1 Theo gt: HK = AK  tan . B tan C 3  3 2 c/ Ta chứng minh được: SAB ABC  và A
DE đồng dạng vậy: ABC    (3) SAD ADE Mà BÂC = 600 nên 0
ABD  30  AB = 2AD(4)
Từ (3)(4) ta có: SABC 2  4  S  30(cm ) ADE SADE
SỞ GIÁO DỤC VÀ ĐÀO TẠO KỲ THI CHỌN HỌC SINH GIỎI TỈNH
THANH HÓA NĂM HỌC 2011 - 2012
§Ò CHÝNH THøC MÔN: TOÁN Lớp 9 thcs
Thời gian làm bài 150 phút không kể thời gian phát đề
Ngày thi: 23 tháng 3 năm 2012 Câu I (4đ)
æ ö æ ö
Cho biểu thức P = x - 1 x + 8 ç ÷ 3 x - 1 + 1 1 ç ÷ ç + : ÷ ç - ÷ ç ÷ çè + - - ÷ ç ÷ 3 x 1 10 x÷
ø çèx- 3 x- 1- 1 x - 1÷ ø 1) Rút gọn P 3  2 2 3  2 2
2) Tính giá trị của P khi x = 4  4 3  2 2 3  2 2 Câu II (4đ)
Trong cùng một hệ toạ độ, cho đường thẳng d: y = x – 2 và parabol (P): y = - x2. Gọi A và B là
giao điểm của d và (P).
1) Tính độ dài AB.
2) Tìm m để đường thẳng d’: y =- x = m cắt (P) tại hai điểm C và D sao cho
CD = AB. Câu III (4đ)  2 x x  2  y
1) Giải hệ phương trình  2 y 1   y  .  x 2
2) Tìm nghiệm nguyên của phương trình 2x6 + y2 –2 x3y = 320 Câu IV (6đ)
Cho tam giác nhọn ABC có AB > AC. Gọi M là trung điểm của BC; H là trực tâm; AD, BE, CF
là các đường cao của tam giác ABC. Kí hiệu (C1) và (C2) lần lượt là đường tròn ngoại tiếp tam

giác AEF và DKE, với K là giao điểm của EF và BC. Chứng minh rằng:
1) ME là tiếp tuyến chung của (C1) và (C2).
2) KH
AM. Câu V (2đ) Với 0  ; x ;
y z  1. Tìm tất cả các nghiệm của phương trình: x y z 3    1  y zx 1  z xy 1  x yz
x y z
SỞ GIÁO DỤC VÀ ĐÀO TẠO
KỲ THI CHỌN HỌC SINH GIỎI CẤP TỈNH LỚP 9 THANH HÓA
NĂM HỌC 2011-2012 Môn : TOÁN
Ngày thi :18/02/2012

Câu 1:ĐK
1< x ¹ 10 1) 3 x 1 9 é 1 2 x 1 4ù - + - + P : ê . ú = 10 x ê ú - ê x - 1 x - 1- 3 ë úû x - 1 x (. x- 1- - + ) 3 3( 1 3) P = . 10- x 2 x - 1 + 4
3 x - 1(x - 10)( x - 1- 2) 3(x - 2) P = = -
2(10- x)(x - 1- 4) 2(x - 5) 3+ 2 2 3- 2 2 b) 4 2 4 2 4 4 x = - = (3+ 2 2) - (3- 2 2) = 3+ 2 2 - 3- 2 2 3- 2 2 3+ 2 2 => x=1+
2 - ( 2 - 1) = 2 vì x>1 Vậy P=0 Câu II:
1) Hoành độ giao điểm là nghiệm phương trình
x2+x-2=0 => x=1 hoặc x=2
Vậy A(1,-1) và B(-2;-4) hoặc A(-2;-4) vàB(1;-1)

2)Để (d’) cắt (P) tại 2 điểm phân biệt thì phương trình x2-x+m=0 (1) 1
có hai nghiệm phân biệt <=> D > 0 <=> m < 4
Ta có khoảng cách AB2 =18
để CD = AB <=> (x1-x2)2+(y1-y2)2=18 <=>(x1-x2)2=9 <=>(x1+x2)2-4x1x2=9
<=>1-4m-9=0=> m=-2(TM)
Vậy C(-1,-3) và D(2;0) hoặc D(-1;-3) hoặc C(2;0
Câu III
1,ĐK x
¹ 0, y ¹ 0
Đặt x=ky ( k ¹ 0)  2 x 2 ì 
ï (k + k) y = 2 x  2 ï  ï y í  <=> 1 1 ï ( + 1)y = ï (1)  2 y 1 ïî k 2   y  .  x 2
Nếu k=-1 thì hệ phương trình (1) vô nghiệm nên hệ phương trình đã cho vô nghiệm
Nếu k
¹ -1 2 (k + k)k từ (1) => = 4 k + 1
=> k=2 hoặc k = -2 Nếu k=2 => 2 1
(x, y) = ( ; ) 3 3
Nếu k = -2 => (x;y)=(-2;1)
2, Từ 2x6 + y2 – x3y = 320 <=>(x3-y)2 +(x3)2=320
=> (x3)2 £ 320
mà x nguyên nên x £ 2
Nếu x=1 hoặc x=-1 thì y không nguyên (loại)
Nếu x=2=> y=-2 hoặc y=6
Nếu x=-2 => y=-6 hoặc y=2
Vậy phương trình đã cho có 4 cặp nghiệm (x;y) là(2;-2);(2;6);(-2;-6);(-2;2)

Câu IV: 1) Ta có µ µ 0
E = F = 90 nên tứ giác AEHF nội tiếp một đường tròn tâm chính là
(C1) là trung điểm AH · 1 ¼ EAH = sd EH (1) 2 · ·
EAH = CBE (2) ( cùng phụ với góc ACD) · ·
MEB = CBE (3)( do đương trung tuyến ứng với cạng huyền)
Từ (1), (2) và (3) ta có · 1 ¼ MEH = sd EH 2
=> ME là tiếp tuyến đường tròn tâm (C1) A F E N B K C D M C
2, gọi giao điểm AM với KH là N trước tiên chứng minh 5 điểm A,E,H,N,F cùng thuộc một đường tròn Ta thấy · · · · · · AFE = AC ;
B ANE = AFE = > ANE = ACB
=> nghĩa là C,M,N, F cùng thuộc một đường tròn
chứng minh A,E,N, B nội tiếp do đó
· 0
KNM = 90 KH AM
Câu V:: do vai trò x,y,z như nhau nên
0 £ x £ y £ z £ 1 y z 3 + = 1+ z 1+ zy y + z
Nếu x= 0 => y 1 z 1 1 = > ( - ) + ( - ) = 1+ z y + z 1+ zy y + z y + z 2
( y - 1)( y + 1+ z) z - 1 1 = > + =
(1+ z)( y + z)
(1+ yz)( y + z) y + z
Ta có VT ³ 0 mà VP < 0 nên trong trường hợp này không có nghiệm
Nếu x khác 0 mà 0 £ x £ y £ z £ 1  z  
1 1 x  0 <=>
1 zx x z >0
x z zx 1  0
x zx z 1  0
đúng với mọi 0  ;
x z  1.
Dấu “=” xảy ra khi: x=z=1.
+ Ta có:
1 zx x z  1 y zx x y z x x   1  y zx
x y z + Tương tự: y y 1  z xy
x y z z z 1  x yz
x y z x y z
x y z
VT     1. (1) 1  y zx 1  z xy 1  x yz
x y z
+ Mặt khác, vì: 0  ; x ;
y z  1 x y z  3 3 3
VP
 1 Dấu “=” xảy ra khi : x=y=z=1. (2)
x y z 3
+ Từ (1) và (2) VT VP chỉ đúng khi: VT VP 1. Khí đó x=y=z=1.
* Vậy phương trình có nghiệm duy nhất:
 ; x ; y z   ; 1 ; 1  1 .
SỞ GIÁO DỤC VÀ ĐÀO TẠO KỲ THI CHỌN HỌC SINH GIỎI VÒNG TỈNH LỚP 9 THCS
TỈNH KIÊN GIANG NĂM HỌC 2012-2013
HƯỚNG DẪN CHẤM MÔN TOÁN 9 Câu Đáp án Điểm
Câu 1a - Hàm số y = (m2 – 2m)x + m2 – 1 nghịch biến
(1,25đ) ⇔ m2 – 2m < 0 ⇔ m(m – 2) < 0 0,25 ⎡⎧m > 0 ⎡⎧m > 0 ⎢⎨ ⎢⎨ ⎩m − 2 < 0 m < 2 ⇔ ⎢ ⎢⎩ ⇔ ⎢ ⎢ ⇔ 0 < m < 2 (1) ⎧m < 0 m < 0,25 ⎢ ⎢⎧ 0 ⎨ ⎨ (loai) ⎢⎣⎩m − 2 > 0 ⎢⎣⎩m > 2 0,25
- Cắt trục tung : m2 – 1 = 3 ⇔ m = ±2 (2)
Từ (1) và (2) ⇒ m ∈∅ 0,5
Câu 1b Tìm giá trị nhỏ nhất của : (1,5đ)
M = 5x2 + y2 + z2 - z – 4x – 2xy – 1
M = x2 - 2xy + y2 + 4x2 – 4x + 1 + z2 - z + 1 9 − 4 4 0,25 2
= (x – y)2 + (2x – 1)2 + ⎛ 1 ⎞ 9 9
z − ⎟ – ≥ - ⎝ 2 ⎠ 4 4 0,5
Giá trị nhỏ nhất của M = 9 − 4 0,25 ⎧ ⎪x y = 0 ⎪ ⇔ 1
⎨2x −1 = 0 ⇔ x = y = z = 0,5 2 ⎪ 1 ⎪z − = 0 ⎩ 2
Câu 1c Cho x + y = - 5 và x2 + y2 = 11. Tính x3 + y3 (1,25đ) 0,25
Ta có : x3 + y3 = (x+y)(x2 + y2 – xy) = -5(11 – xy) (1)
Mà x + y = -5 ⇒ x2 + y2 +2xy = 25 0,5
⇒ 11 + 2xy = 25 ⇒ xy = 7 (2)
Từ (1) và (2) ⇒ x3 + y3 = -5(11- 7) = -20 0,5 Câu 2a
x2 + 5x + 6 + x 9 − x2 2x (2,0đ) Rút gọn : A = : . 2 1 +
3x x2 + (x + 2) 9 − 2 3 − x x ĐK : -3 < x < 3 0,25 ( x + 3)(x + 2) A =
+ x 3 + x. 3 − x 3 − x 2x : 2 +
x(3 − x) + (x + 2) 3 + x. 3 − x 3 − x 3 − x 0,5
3 + x ([x + 2) 3 + x + x 3 − x] = 3 + : 2 3 − x[ x
x 3 − x + (x + 2) 3 + x ]. 3 − x 0,5 = 3 + x 3 + x : 2 3 − x 3 − x 0,5 = 1 2 0,25
Câu 2b Cho a, b c thỏa mãn : 1 + 1 + 1 = 1 (2,0đ) a b c a + b + c
Tính giá trị biểu thức Q = (a27 + b27)(b41 + c41)(c2013+ a2013) Ta có : 1 + 1 + 1 = 1 ⇒ 1 1 1 1 + = − 0,25 a b c a + b + c a b a + b + c c a + b − (a + b) ⇒ = 0,25 ab
c(a + b + c) ⇒ (a+b)c(a+b+c) = -ab(a+b)
⇒ (a+b)[c(a+b+c) +ab] = 0 ⇒ (a+b)[c(a+c)+bc +ab] = 0
⇒ (a+b)[c(a+c) +b(a+c)] = 0⇒ (a+b)(a+c)(b+c) = 0 0,5 ⎡a + b = 0 ⎡a = −b ⇒ ⎢ ⎢ b + c = 0 ⇒ ⎢ ⎢b = −c 0,25 ⎢c + a = 0 ⎢ ⎣ ⎣c = −a
- Thế vào tính được Q = 0 0,75
Giải phương trình : 3 x +10 3 + 17 − x = 3 Câu 3a (
3 x +10 + 17 − x )3 3 3 (2,0đ) = 3 0,5
x + 10 + 17 – x + 3. 3 (x + 17 )( 10 − x) .3 = 27 0,5 (x+10)(17 – x) = 0 0,5 x = -10 , x = 17 0,5 Câu 3b ⎧ 2x − 3 y + 5 2,0đ + = 2
Giải hệ phương trình : ⎪⎨ y + 5 2x − 3 ⎪ 3x + 2 y = 19 ⎩ (với 3
x > , y > −5 ) 2 0,5
Đặt 2x − 3 = m > 0 y + 5 0,5 1 2
m − 2m +1 = 0 ⇔ (m − )2 ⇒ m + = 2
1 = 0 ⇔ m = 1(nhận) m 0,5 2x − 3 ⇒
= 1 ⇔ 2x − 3 = y + 5 ⇔ 2x y = 8 y + 5 ⎧2x y = 8
⎧4x − 2y = 16 ⎧x = 5 0,5 Giải hệ ⎨ ⇔ ⎨ ⇔ ⎨ ⎩3x + 2y = 19 ⎩3x + 2y = 19 ⎩y = 2 Câu 4 (4,0đ) Hình 0,5đ Câu a
a) Chứng minh : KD = CI và EF//AB. (1,0đ)
– Cminh ABID, ABCK là hình bình hành 0,5
⇒ DI = CK (cùng bằng AB) 0,25
⇒ DI + IK = CK + IK ⇒ DK = CI 0,25
(1,25đ) - C/m : ΔAEB đồng dạng Δ KED (g.g) AE AB = EK KD 0,5
Δ AFB đồng dạng Δ CFI (g.g) ⇒ AF AB = FC CI 0,25 Mà KD = CI (cmtrên) AE AF ⇒ =
⇒ EF / /KC (Đlí Talet đảo trong Δ AKC) EK FC 0,5 Câu b b) Chứng minh AB2 = CD. EF.
(1,25đ) Ta có : Δ KED đồng dạng ΔAEB (cmtrên) DK DE DK + AB DE + EB ⇒ = ⇒ = AB EB AB EB DK + KC DB ⇒ = AB EB DC DB 0,5 ⇒ = (1) AB EB
Do EF//DI (theo CMT: EF//KC, I ∈ KC) 0,25 DB DI DB AB ⇒ = ⇒ = (2) (Vì DI = AB) EB EF EB EF DC AB 0,5 Từ (1) và (2) ⇒ =
AB2 = DC.EF AB EF Câu 5 A Hình 0,5đ 4,0đ xO Q D B K C H M Câu a
a) Chứng minh MC + MB = MA ? (1,75đ)
- Trên MA lấy D sao cho MD = MB 0,25 ⇒ ΔMBD cân tại M
góc BMD = góc BCA = 600 (cùng chắn cung AB) ⇒ ΔMBD đều 0,5 - Xét ΔMBC và ΔDBA Ta có : MB = BD (vì ΔMBD đều) BC = AB (vì ΔABC đều)
Góc MBC = góc DBA (cùng cộng góc DBC bằng 600) ⇒ ΔMBC = ΔDBA (c-g-c) 0,5 ⇒ MC = DA Mà MB = MD (gt) ⇒ MC + MB = MA 0,5 Câu b b)
Xác định vị trí của điểm M để tổng MA + MB + (075đ)
MC đạt giá trị lớn nhất.
Ta có : MA là dây cung của (O;R) ⇒ MA ≤ 2R 0,25
⇒ MA + MB + MC ≤ 4R (không đổi)
Dấu “ = “ xảy ra ⇔ MA là đường kính 0,25
⇔ M là điểm chính giữa của cung BC 0,25 2 3 (S + 2S ') Câu c
c) CMR : MH + MK + MQ = (1,0đ) 3R
Ta có MH.AB MK.BC MQ.AC + + = S + S + S 0,25 MAB MBC MAC 2 2 2
⇒ AB.(MH + MK + MQ ) = 2 (S + 2S’) 0,25
Tính hoặc nói AB là cạnh tam giác đều nội tiếp (O;R) ⇒ AB = R 3 0,25 2 3(S + 2S') ⇒ MH + MK + MQ = 3R 0,25
Lưu ý : Học sinh giải cách khác đúng cho trọn số điểm
Họ tên TS:...........................................
SốBD:...................... Chữ ký GT1:..................
SỞ GIÁO DỤC VÀ ĐÀO TẠO
KỲ THI CHỌN HỌC SINH GIỎI CẤP TỈNH NINH THUẬN NĂM HỌC 2012 – 2013
Khóa ngày: 18 / 11 / 2012
(Đề thi chính thức)
Môn thi: TOÁN - Cấp: THCS
Thời gian làm bài: 150 phút
(Không kể thời gian phát đề) ĐỀ:
(Đề thi có 01 trang)
Bài 1 (5,0 điểm):
Tìm tất cả các cặp số thực x y thỏa mãn bất đẳng thức sau: 2 2
x y  2x  2y  2  0
Bài 2 (4,0 điểm):
Tìm tất cả các nghiệm nguyên dương của phương trình: 1 1 1   x y 7
Bài 3 (5,0 điểm):
Cho hình thang ABCD (AD // BC). Hai đường phân giác trong của góc A
và góc B cắt nhau tại điểm E, hai đường phân giác trong của góc C và góc D cắt nhau tại điểm F.
a) Chứng minh rằng: EF // AD.
b) Tính độ dài đoạn EF thông qua các cạnh của hình thang ABCD.
Bài 4 (3,0 điểm): Cho số thực A = 2 + 2 2
28n  1 , với n nguyên. Chứng minh rằng nếu A
là số nguyên thì A là một số chính phương (bằng bình phương của một số nguyên).
Bài 5 (3,0 điểm):
Trong hình vuông có độ dài cạnh bằng 1 cho 151 điểm bất kỳ. Chứng minh 1
rằng có ít nhất 7 điểm đã cho nằm trong một hình tròn có bán kính bằng . 7 ------- HẾT -------
SỞ GIÁO DỤC VÀ ĐÀO TẠO
KỲ THI CHỌN HỌC SINH GIỎI CẤP TỈNH QUẢNG NINH
LỚP 9 NĂM HỌC 2012 – 2013
ĐỀ THI CHÍNH THỨC MÔN: TOÁN Họ và tên, chữ ký (Bảng A) của giám thị số 1: Ngày thi: 20/3/2013
Thời gian làm bài: 150 phút
...............................
(không kể thời gian giao đề)
...............................
(Đề thi này có 01 trang)
Bài 1. (4,5 điểm) 1 2 4
a) Chứng minh đẳng thức: 3 3 − = 3 − 3 + 3 2 1 . 9 9 9 2
x (2013y − 2012) = 1
b) Giải hệ phương trình :  . 2
x( y + 2012) = 2013
Bài 2. (3,5 điểm)
Cho hàm số bậc nhất y = mx + m - 1 (*) (với m là tham số).
a) Tìm các giá trị của m để đồ thị của hàm số (*) tạo với các trục tọa độ Oxy một
tam giác có diện tích bằng 2.
b) Chứng minh rằng đồ thị của hàm số (*) luôn đi qua một điểm cố định với mọi giá trị của m.
Bài 3.
(4,0 điểm)
Cho x, y, z là ba số thực dương thoả mãn xyz = 1. 1 1 1
Tìm giá trị lớn nhất của biểu thức A = + + .
x3 + y3 +1 y3 + z3 +1 z3 + x3 +1
Bài 4. (6,0 điểm)
Cho tam giác ABC có ba góc nhọn nội tiếp đường tròn tâm O. Gọi I là một điểm
trên cung nhỏ AB (I không trùng với A và B). Gọi M, N, P theo thứ tự là hình chiếu của
điểm I trên các đường thẳng BC, AC, AB.
a) Chứng minh rằng ba điểm M, N, P thẳng hàng.
b) Xác định vị trí của điểm I để đoạn thẳng MN có độ dài lớn nhất.
Bài 5.
(2,0 điểm)
Giải phương trình sau: (x+3) (4 − x)(12 + x) + x = 28 .
.......................Hết.....................
Họ và tên thí sinh:.............................................................Số báo danh:...............
SỞ GIÁO DỤC VÀ ĐÀO TẠO
KỲ THI CHỌN HỌC SINH GIỎI CẤP TỈNH QUẢNG NINH
LỚP 9 NĂM HỌC 2012 – 2013
ĐỀ THI CHÍNH THỨC MÔN: TOÁN Họ và tên, chữ ký (Bảng B) của giám thị số 1: Ngày thi: 20/3/2013
Thời gian làm bài: 150 phút
..............................
(không kể thời gian giao đề)
...............................
(Đề thi này có 01 trang)
Câu 1. (4,0 điểm)   − +  2(x − 2 xx x x x + ) 1 1 1 Cho biểu thức P =   :   − 
với x>0; x ≠ 1. x x x + x   x −1     
a) Rút gọn biểu thức P.
b) Tìm x nguyên để P nhận giá trị nguyên.
Câu 2. (4,0 điểm) a + b + c = 6
Cho ba số thực a, b, c thỏa mãn đồng thời:  . 2 2 2 a + b + c = 12 
Tính giá trị của biểu thức P = 2013 2013 2013 (a - 3) + (b - 3) + (c - 3) .
Câu 3
. (4,0 điểm) Giải phương trình: 2 2
2(x − 4x) + x − 4x − 5 −13 = 0 .
Câu 4
. (6,0 điểm)
Cho đường tròn (O) và BC là một dây cung không đi qua tâm O. Điểm A bất kì
nằm trên cung lớn BC của đường tròn (O) sao cho điểm O luôn nằm trong tam giác
ABC (A ≠ B; C). Các đường cao AD, BE, CF cắt nhau tại H.
a) Chứng minh tứ giác BFEC nội tiếp.
b) Đường cao AD cắt đường tròn (O) tại I. Chứng minh I đối xứng với H qua BC.
c) Gọi M là trung điểm của BC. Chứng minh AH = 2OM.
Câu 5. (2,0 điểm) 1 1 1
Cho ba số thực dương x, y, z thỏa mãn + + ≥ 2.
1+ x 1+ y 1+ z
Tìm giá trị lớn nhất của biểu thức P = xyz.
-----------------Hết----------------
Họ và tên thí sinh :……………………………………………..Số báo danh :………...
`SỞ GD&ĐT QUẢNG NINH HƯỚNG DẪN CHẤM THI CHỌN HỌC SINH GIỎI CẤP TỈNH
LỚP 9 NĂM HỌC 2012 – 2013
ĐỀ THI CHÍNH THỨC Môn: TOÁN (BẢNG A)
(Hướng dẫn chấm này có 04 trang) Bài
Sơ lược bài giải Điểm Đặt 3 3
2 = a ⇔ 2 = a . 2 − +
Đẳng thức cần chứng minh tương đương với: 1 a a 3 a −1 = 0,5 3 9 Câu a ⇔ ( 9 a − ) 1 2 3
= a a +1 ⇔ ( 2 a a + ) 1 3 = ( 9 a − ). 1 0,5 2,5 Biến đổi vế trái: điểm 2 3 2 2 2
(a a +1) = (a a +1) (a a +1) 2 2 2
= 3(a −1)(a a +1) = 3(a −1)(a +1)(a a +1) 1,5 3
= 3(a −1)(a +1) = 3(a −1)(2 +1) = 9(a −1)
Vậy đẳng thức được chứng minh.
2. ta thấy x = 0 không là nghiệm. hệ phương trình tương đương với:  1 Bài 1 2013y − 2012 =  0,5 2  4,5đ x  (*) 2013  2 y + 2012 =  x 2 t
 − 2013y + 2012 = 0 2 2
Câu b Đặt: 1 = t , hệ (*) ⇒ 
t − 2013y = y − 2013t x 2
 y − 2013t + 2012 = 0 2,0 0,5 y = t điểm
⇔ (t y)(t + y + 2013) = 0 ⇒  y = t − −  2013 * Trường hợp 2
y = t t − 2013t + 2012 = 0,
Giải PT được :t =1;t = 2012 0,5 1 2 * Trường hợp 2 2 y = t
− − 2013 ⇒ t + 2013t + 2013 + 2012 = 0 , PT vô nghiệm 0,5 Vậy hệ có nghiêm ( 1
(x = 1; y = 1);(x = ; y = 2012) 1 1 2 2 2012
Câu a Vì (*) là hàm số bậc nhất nên m ≠ 0. (1) 0,25 2,0
Điều kiện để đồ thị của (*) tạo với các trục tọa độ Oxy một tam điểm giác là m ≠ 1. (2) 0,25
Gọi A là giao điểm của đường thẳng (*) với trục tung Bài 2 0,25 3,5đ
⇒ A(0; m-1) nên độ dài OA = | m - 1|.
Gọi B là giao điểm của đường thẳng (*) với trục hoành 1− m 1− m 0,25 ⇒ B( ; 0) nên độ dài OB = | |. m m 1 S 1 OA.OB = 2⇔ OA.OB = 4. 0,25 ABC = 2 ⇔ 2 ⇔ (m - 1)2 = 4|m|
*Với m > 0 thì m2 - 2m + 1 = 4m 0,25 ⇔ m2 - 6m + 1 = 0
⇔ m1 = 3 – 2 2 ; m2 = 3 + 2 2 .
*Với m < 0 thì m2 - 2m + 1 = - 4m ⇔ m2 + 2m +1 = 0 ⇔ m = -1 0,25
Vậy m ∈{ -1; 3 - 2 2 ; 3 + 2 2 } thỏa mãn điều kiện (1) và (2). 0,25
Gọi M(x0; y0) là điểm cố định thuộc đồ thị (*) khi và chỉ khi: y0 = mx0 + m – 1 m ∀ ∈R 0,75
⇔ (x0 + 1)m – (y0 + 1) = 0 m ∀ ∈R Câu b x +1 = 0 x = −1 1,5 ⇔ 0 0  ⇔ 
Vậy đồ thị của (*) luôn đi qua một điểm y +1 = 0 y = −   1 Điểm 0 0
cố định M(-1; -1) ∀mR 0,75
Ta có (x - y)2 ≥ 0 với ∀ x, y ∈R ⇔ x2 - xy + y2 ≥xy. 0,5
Mà x; y > 0 nên x + y > 0. 0,5
Mà x3 + y3 = (x + y)(x2 - xy + y2 ) ≥(x + y)xy. 0,5
⇒ x3 + y3 +1 = x3 + y3 + xyz ≥ (x + y)xy + xyz.
⇒ x3 + y3 +1 ≥ xy(x + y + z) > 0. 0,5
Tương tự chứng minh được:y3 + z3 +1 ≥ yz(x + y + z) > 0. Bài 3 4
z3 + x3 +1 ≥ zx(x + y + z) > 0. 0,5 điểm 1 1 1 ⇒ A ≤ + + 0,5 xy(x + y + z) yz(x + y + z) xz(x + y + z) x + y + z 1 ⇔ A ≤ = ⇔ A ≤ 1 . 0,5 xyz(x + y + z) xyz
Vậy giá trị lớn nhất của A là 1 khi x = y = z = 1. 0,5
Từ giả thiết ta có: ∠IPA + ∠INA = 1800⇒ tứ giác IPAN nội tiếp 0,75
⇒ ∠IPN = ∠IAN ( cùng chắn cung IN) (1)
Lại có ∠IPB = ∠IMB = 900 ⇒ tứ giác IPMB là tứ giác nội tiếp 0,75
Câu a ⇒ ∠MPI + ∠IBM = 1800 (2)
Bài 4 3 điểm Vì I ∈(O) ⇒ ∠ CAI + ∠IBM = 1800 (3) 0,5
Từ (2) và (3) ⇒ ∠MPI = ∠CAI (4) 0,5
Từ (4) và (1) ⇒ ∠MPI +∠IPN = ∠CAI + ∠IAN = 1800 0,5 Suy ra M, P, N thẳng hàng.
Câu b Tứ giác IPMB là tứ giác nội tiếp nên ∠IBA = ∠IMN 0,5
3 điểm ( cùng chắn cung IP) (5) 2
Tứ giác INAP là tứ giác nội tiếp nên ∠INM = ∠IAB 0,5 ( cùng chắn cung IP) (6)
Từ (5) và (6) ⇒ tam giác IMN đồng dạng với tam giác IBA 0,5 MN IM IN ⇒ = = ≤ 1 ⇒ MN ≤ AB 0,5 BA IB IA M B
Dấu “ =’’xảy ra ⇔  ⇔ ∠IAC = ∠IBC = 900 N A 0,5
⇔ CI là đường kính của (O).
Vậy MN lớn nhất bằng AB ⇔ I đối xứng với C qua O. 0,5
(x+3). (4 − x)(12 + x) + x = 28 (*) 0,25
Điều kiện xác định: - 12 ≤ x ≤ 4
Đặt x + 3 = u; (4 − x)(12 + x) = v 0,25
⇒ u2 + v2 = x2 + 6x + 9 + 48 - 8x – x2 = 57 - 2x 0,25
⇒ u2 + v2 - 1 = 2(28 - x) (1)
Theo đề bài ta có uv = 28 - x (2) 0,25 Bài 5
Từ (1) và (2) ta có u2 + v2 - 1 = 2uv ⇔ (u - v)2 = 1 2 điểm u − v = 1 u = v +1 ⇔  ⇔  0,5 u − v = −1 u = v −1
i) Với u = v +1 ⇒ (4 − x)(12 + x) = x + 2 (điều kiện: x ≥ 2 − ) 0,25
Giải phương trình được x = - 3 + 31 ( thỏa mãn).
ii) Với u = v - 1 ⇒ (4 − x)(12 + x) = x + 4(điều kiện: x≥ 4 − )
Giải phương trình được x = - 4 + 4 2 ( thỏa mãn) 0,25
=> S = {-4 +4 2 ; -3 + 31 }. 3 n i A p o C m B Hình vẽ bài 4
Các lưu ý khi chấm:
1. Hướng dẫn chấm này chỉ trình bày sơ lược một cách giải. Bài làm của học sinh
phải chi tiết, lập luận chặt chẽ, tính toán chính xác mới được điểm tối đa.
2. Các cách giải khác nếu đúng vẫn cho điểm. Tổ chấm thống nhất cho điểm
thành phần của câu nhưng không vượt quá số điểm của câu hoặc phần đó.
3. Bài 4 không vẽ hình không cho điểm cả bài. Bài 4 câu b tìm được vị trí điểm
I không chứng minh không cho điểm.
4. Mọi vấn đề phát sinh trong quá trình chấm phải được trao đổi trong tổ chấm
và chỉ cho điểm theo sự thống nhất của cả tổ.
5. Điểm toàn bài là tổng số điểm đã chấm. Không làm tròn.
.......................Hết..................... 4
`SỞ GD&ĐT QUẢNG NINH HƯỚNG DẪN CHẤM THI CHỌN HỌC SINH GIỎI CẤP TỈNH
LỚP 9 NĂM HỌC 2012 – 2013
ĐỀ THI CHÍNH THỨC Môn: TOÁN (BẢNG B)
(Hướng dẫn chấm này có 03 trang) Cho Câu
Tóm tắt lời giải điểm  ( x − )( 1 x + x + ) 1 ( x + )(
1 x x +   ) 1 2( x − 2  a, P =  ) 1 −      :   0,5  x( x − ) 1 x( x + ) 1   ( x − )( 1 x + ) 1 
x + x +1 x x +   1 2( x −  =  ) 1 −      :   0,5  x x   x + 1 
= x + x +1− x + x −1 x + 1 . 0,5 x 2( x − ) 1 x x + x + = 2 1 1 . = . 0,5 x 2( x − ) 1 x −1 Câu 1 x +1 2
(4điểm ) b, P = = 1+ 0,5 x −1 x −1
Để P nhận giá trị nguyên thì x −1∈Ư(2). 0,25
* x −1 = 1 ⇒ x = 2 ⇒ x = 4 0,5
* x −1 = 2 ⇒ x = 3 ⇒ x = 9 * x −1 = 1
− ⇒ x = 0 ⇒ x = 0 (loại). * x −1 = 2 − ⇒ x = 1 − (loại). 0,5
Vậy x nhận các giá trị nguyên 4 ; 9 thì P nhận các giá trị nguyên lần 0,25 lượt là 3; 2.
a + b + c = 6
4a + 4b + 4c = 24  ⇔  . 1,0 2 2 2
a + b + c =  12 2 2 2
a + b + c =  12
Từ hai phương trình ta suy ra: 2 2 2
a + b + c − 4a − 4b − 4c +12 = 0 . 0,75 2 2 2
⇔ (a − 2) + (b − 2) + (c − 2) = 0 vì 2 (a − 2) ≥ 0 ; 2 (b − 2) ≥ 0 ; 2 (c − 2) ≥ 0 với Câu 2 1,0 mọi số thực a, b, c. ( 4điểm) 2 (a − 2) = 0 (a − 2) = 0 a = 2    2
⇔ (b − 2) = 0 ⇔ (b − 2) = 0 ⇔ b  = 2 . 0,75  2   (c − 2) = 0 − = =  (c 2) 0 c 2 Vậy P = 2013 2013 2013 (a − 3) + (b − 3) + (c − 3) = 2013 2013 2013 ( 1 − ) + ( 1 − ) + (−1) = −3 . 0,5 2 2 1,0 Câu 3
2(x − 4x) + x − 4x − 5 −13 = 0 (4điểm) 2 2
⇔ 2(x − 4x − 5) + x − 4x − 5 − 3 = 0 1 Điều kiện 2
x − 4x − 5 ≥ 0 ⇔ x ≤ −1 hoặc x ≥ 5 (*) 0,25 Đặt : 2
t = x − 4x − 5 ; ( t ≥ 0) 2 2
t = x − 4x − 5 0,75
Phương trình đã cho trở thành: 2
2t + t − 3 = 0 ⇔ (t −1)(2t + 3) = 0 3
⇔ t = 1 hoặc t = - (loại). 1,0 2 Với t = 1 ta có : 2
x − 4x − 5 = 1 2 ⇔
x − 4x − 6 = 0 0,75 ⇔ x = 2 ± 10
Vậy phương trình có nghiệm là x = 2 ± 10 ( thỏa mãn điều kiện (*)). 0,25
a, Có∠BFC=900 (vì CF là đường cao của tam giác ABC)
∠BEC = 900 (vì BE là đường cao của tam giác ABC) 1,0
Như vậy từ hai đỉnh F và E cùng nhìn cạnh BC dưới một góc vuông 0,75
Suy ra hai điểm E và F cùng nằm trên đường tròn đường kính BC
Vậy tứ giác BFEC nội tiếp. 0,25
b, Tứ giác ABDE nội tiếp do có∠BDA = ∠ BEA = 900
⇒ ∠DBE = ∠DAE (hai góc nội tiếp cùng chắn cung DE) hay
Câu 4 ∠CBE=∠IAC (1) 1,0
(6 điểm) ∠ IBC=∠ IAC ( góc nội tiếp chắn cung IC) (2)
Từ (1) và (2) ⇒ ∠CBE = ∠IBC ⇒ BC là tia phân giác góc IBH .
Ta lại có BC ⊥ HD nên tam giác IBH cân tại B
Suy ra BC cũng là trung trực của HI 1,0
Vậy I và H đối xứng nhau qua BC
c,
Kẻ đường kính AK suy ra : KB // CH ( cùng vuông góc với AB ) 1,0
KC//BH ( cùng vuông góc với AC )
⇒ tứ giác BHCK là hình bình hành và M là giao điểm hai đường chéo.
⇒ M là trung điểm của HK 0,5
⇒ OM là đường trung bình của tam giác AHK ⇒ AH = 2OM ( đpcm) 0,5 1 1 1 y z yz ≥ (1− ) + (1− ) = + ≥ 2 (1) 0,75 1+ x 1+ y 1+ z) 1+ y 1+ z (1+ y)(1+ z) Câu 5 zx
(2 điểm) Tương tự : 1 ≥ 2 (2) + + + 1 y (1 x)(1 z) 0,5 1 xy ≥ 2 (3) 1+ z (1+ x)(1+ y) 2
Nhân ba bất đẳng thức cùng chiều (1), (2), (3) với nhau ta được xyz 1 ≤ 0,5 8
Suy ra giá trị lớn nhất của P = 1 khi x = y = z = 1 . 0,25 8 2 A E F O H C B M D K I Hình vẽ bài 4 Các chú ý khi chấm
1. Hướng dẫn chấm này chỉ trình bày sơ lược một cách cách giải. Bài làm của học
sinh phải chi tiết, lập luận chặt chẽ, tính toán chính xác mới được điểm tối đa.
2. Các cách giải khác nếu đúng vẫn cho điểm. Tổ chấm trao đổi và thống nhất điểm
chi tiết nhưng không vượt quá số điểm dành cho câu hoặc phần đó.
3. Với bài 4 không cho điểm nếu không có hình vẽ.Có thể chia nhỏ điểm thành
phần nhưng không dưới 0,25 điểm và phải thống nhất trong tổ chấm.
………………… Hết ………………. 3
SỞ GIÁO DỤC VÀ ĐÀO TẠO
KỲ THI CHỌN HỌC SINH GIỎI TỈNH THANH HOÁ Năm học 2010- 2011 Đề chính thức Môn thi: Toán Lớp: 9 THCS Số báo danh
Thời gian: 150 phút (không kể thời gian giao đề) Ngày thi: 24/03/2011
(Đề thi có 01 trang, gồm 05 câu).
Câu I
. (5,0 điểm). 1) Cho phương trình: 2
x − 2m x + 2m −1= 0. Chứng minh phương trình luôn có hai nghiệm 2x x + 3
x , x với mọi m. Tìm giá trị lớn nhất của biểu thức 1 2 P = khi m thay đổi. 1 2 2 2
x + x + 2(1+ x x ) 1 2 1 2 1 1 1
2) (a). Cho ba số hữu tỉ a, b, c thoả mãn + = . Chứng minh rằng 2 2 2
A = a + b + c a b c là số hữu tỉ. (b). Cho ba số hữu tỉ ,
x y, z đôi một phân biệt. Chứng minh rằng: 1 1 1 B = + + là số hữu tỉ. 2 2 2 (x − ) y (y − ) z (z − ) x 2 2 ⎛ x ⎞ ⎛ x ⎞ 10
Câu II. (5,0 điểm).1) Giải phương trình: + = . ⎜ ⎟ ⎜ ⎟ x −1⎠ ⎝ x +1⎠ 9 ⎧ 1 ⎛ 1 ⎞ 2 ⎪x + x + 1+ = 4 ⎜ ⎟ ⎪ y y
2) Giải hệ phương trình: ⎠ ⎨ 2 ⎪ x x 1 3 x + + + = 4. 2 3 ⎪⎩ y y y
Câu III. (2,0 điểm). Cho tam giác đều ABC, các điểm D, E lần lượt thuộc các cạnh AC, AB,
sao cho BD, CE cắt nhau tại P và diện tích tứ giác ADPE bằng diện tích tam giác BPC. Tính BPE.
Câu IV. (4,0 điểm). Cho đường tròn tâm O và dây cung AB cố định ( O AB ). P là điểm di động
trên đoạn thẳng AB ( P ≠ ,
A B và P khác trung điểm AB). Đường tròn tâm C đi qua điểm
P tiếp xúc với đường tròn (O) tại A. Đường tròn tâm D đi qua điểm P tiếp xúc với đường
tròn (O) tại B. Hai đường tròn (C) và (D) cắt nhau tại N ( N P ). 1) Chứng minh rằng ANP =
BNP và bốn điểm O, D, C, N cùng nằm trên một đường tròn.
2) Chứng minh rằng đường trung trực của đoạn ON luôn đi qua điểm cố định khi P di động.
Câu V. (4,0 điểm).
1) Cho a ,a ,....,a là 45 số tự nhiên dương thoả mãn a < a < .... < a ≤ 130. Đặt 1 2 45 1 2 45 d = a
a , ( j = 1,2,...,44). Chứng minh rằng ít nhất một trong 44 hiệu d xuất hiện ít j j 1 + j j nhất 10 lần.
2) Cho ba số dương a,b,c thoả mãn: 2 2 2 2 2 2
a + b + b + c + c + a = 2011. 2 2 2 a b c 1 2011 Chứng minh rằng: + + ≥ .
b + c c + a a + b 2 2
............................................................. HẾT ........................................................
Thí sinh không được sử dụng tài liệu.
Cán bộ coi thi không giải thích gì thêm.
SỞ GD & ĐT THANH HOÁ
KỲ THI CHỌN HỌC SINH GIỎI TỈNH HƯỚNG DẪN CHẤM NĂM HỌC 2010 - 2011 ĐỀ CHÍNH THỨC MÔN THI: TOÁN LỚP: 9 THCS (Gồm có 3 trang) Ngày thi: 24 - 3 - 2011 Câu Ý Hướng dẫn chấm Điểm Câu I 1) Ta có 2
Δ ' = (m −1) ≥ 0, m
∀ nên phương trình có hai nghiệm với mọi m. 0,5 6 đ 2,5đ 4m +1 1,0
Theo định lí viet, ta có x + x = 2 ,
m x x = 2m −1, suy ra P = 1 2 1 2 2 4m + 2 2 (2m −1) 1,0 = 1 1−
≤ 1. Max P = 1, khi m = . 2 4m + 2 2
2a) Từ giả thiết suy ra 2ab − 2bc − 2ca = 0 0,5 1,5đ Suy ra 2
A = (a + b c) = a + b c là số hữu tỉ 1,0 2b) 1 1 1 1 1 1 0,5 Đặt a = , b = ,c = suy ra + = . 1,0đ x y y z x z a b c 1 1 1 0,5
Áp dụng câu 2a) suy ra B = + + là số hữu tỉ. 2 2 2 (x y) (y z) (z − ) x Câu II 1) Đk: x ≠ 1.
± Phương trình tương đương với 1,0 2 2 6 đ 2,5đ 2 2 2 ⎛ x x x 10 ⎛ 2x ⎞ 2x 10 + − 2 = ⇔ − − = 0. ⎜ ⎟ 2 ⎜ 2 ⎟ 2
x +1 x −1⎠ x −1 9 ⎝ x −1⎠ x −1 9 2 2x 2 − 0,5 Đặt t =
, ta được phương trình 2 10 5 t t
= 0 ⇔ t = hoặct = 2 x −1 9 3 3 5 2 2x 5 0,5
Với t = , ta được = (vô nghiệm) 3 2 x −1 3 2 2 2x 2 1 0,5
Với t = − , ta được = − suy ra x = ± . 3 2 x −1 3 2 2) ⎧ 0,5 2 1 1 x + + x + = 4 2,5đ ⎪ 2 y y
Đk: y ≠ 0. Hệ tương đương với ⎨ ⎛ ⎞ ⎪ 3 1 x 1 x + + x + = 4. 3 ⎜ ⎟ ⎪ y y ⎩ ⎝ y ⎠ ⎧ 1 1,0 u = x + ⎪⎪ y 2 2 u
⎧⎪ + u − 2v = 4 u ⎧⎪ − 4u + 4 = 0 u ⎧ = 2 Đặt ⎨ ta được hệ ⎨ ⇔ ⎨ ⇔ ⎨ x ⎪ 3 2 u ⎪⎩ − 2uv = 4 u
⎪⎩ + u − 4 = 2vv = 1. v = , ⎪⎩ y ⎧ 1 1,0 x + = 2 ⎧u = 2 ⎪⎪ yx = 1 Với ⎨ ta được ⎨ ⇔ ⎨ (thoả mãn điều kiện) ⎩v = 1, x ⎪ ⎩y = 1. = 1 ⎪⎩ y Câu
Kẻ EF AC tại F, DG BC tại G. 0,5 III Theo giả thiết S = S ( ADPE ) (BPC ) 2đ ⇒ S = S . ( ACE ) (BCD)
AC = BC EF = DG A = C 0,5 Suy ra A
Δ EF = ΔCDG AE = CG. Do đó AEC Δ = C
Δ DB(c g c) ⇒ DBC = ECA 0,5 ⇒ BPE = PBC + PCB = PCD + 0 PCB = 60 0,5 Câu 1)
Gọi Q là giao điểm của các tiếp tuyến 1,0 IV 3,0đ
chung của (O) với (C), (D) tại A, B 4,0đ tương ứng. Suy ra ANP = QAP = QBP = B . NP N H O C D Ta có A B P 0,5 ANB = ANP + BNP = QAP + QBP 0 = 180 −
AQB , suy ra NAQB nội tiếp (1). E
Dễ thấy tứ giác OAQB nội tiếp (2)
Từ (1) và (2) suy ra 5 điểm O, N, A, Q, B 0,5
cùng nằm trên một đường tròn. Q
Suy ra các điểm O, N, A, B cùng nằm trên 0,5 một đường tròn. Ta có OCN = 2 OAN = 2 OBN = ODN ,
suy ra bốn điểm O, D, C, N cùng nằm 0,5 trên một đường tròn. 2)
Gọi E là trung điểm OQ, suy ra E cố định và E là tâm đường tròn đi qua 1,0
1,0đ các điểm N, O, D, C. Suy ra đường trung trực của ON luôn đi qua điểm E cố định. Câu V 1)
d +d +. .+d = (a a )+(a a )+. .+(a a ) = a a ≤130 1 − =129. (1) 0,5 1 2 44 2 1 3 2 45 44 45 1 2đ
2,0 Nếu mỗi hiệu d ( j =1,2,....,44) xuất hiện không quá 10 lần thì đ j
d + d + ... + d ≥ 9(1+ 2 + 3 + 4) + 8.5 = 130 mâu thuẫn với (1). 1 2 44 1,5
Vậy phải có ít nhất một hiêụ (
d j =1,...,44) xuất hiện không ít hơn 10 lần j 2) Ta có 2 2 2
2(a + b ) ≥ (a + b) . 0,5 2,0đ 2 2 2 2 2 2 a b c a b c Suy ra + + ≥ + +
b + c c + a a + b ( 2 2 b + c ) ( 2 2 c + a ) ( 2 2 2 2 2 c + a ) Đặt 2 2 2 2 2 2
x = b + c , y = c + a , z = a + b , 2 2 2 2 2 2 2 2 2
y + z x
z + x y
x + y z suy ra VT ≥ + + 1,0 2 2x 2 2y 2 2z 2 2 2
1 ⎡⎛ ( y + z) ⎞ ⎛ (z + x) ⎞ ⎛ (x + y) ⎞⎤ ≥ ⎢ − x + − y + − z ⎜ ⎟ ⎜ ⎟ ⎜ ⎟⎥ 2 2 2x 2y 2z ⎣⎝ ⎠ ⎝ ⎠ ⎝ ⎠⎦ 2 2 2
1 ⎡⎛ ( y + z) ⎞ ⎛ (z + x) ⎞ ⎛ (x + y) ⎞⎤ ≥ ⎢ + 2x − 3x + + 2y − 3y + + 2z − 3z ⎜ ⎟ ⎜ ⎟ ⎜ ⎟⎥ 2 2 2x 2y 2z ⎣⎝ ⎠ ⎝ ⎠ ⎝ ⎠⎦ 0,5 1 ≥
⎡(2(y + z) − 3x) + (2(z + x) − 3y) + (2(x + y − 3z)⎤ ⎣ ⎦ 2 2 1 1 2011 Suy ra VT
(x + y + z) = 2 2 2 2
GHI CHÚ: Nếu học sinh giải cách khác mà đúng thì vẫn cho điểm tối đa.
SỞ GIÁO DỤC VÀ ĐÀO TẠO
KÌ THI CHỌN HỌC SINH GIỎI TỈNH THANH HÓA Năm học: 2011-2012 ĐỀ THI CHÍNH THỨC Môn thi: TOÁN Số báo danh Lớp 9 THCS
Ngày thi: 23 tháng 3 năm 2012
…...............……
Thời gian: 150 phút (không kể thời gian giao đề)
Đề này có 01 trang, gồm 05 câu. Câu I (4,0 điểm)x −1
x + 8 ⎞ ⎛ 3 x −1 +1 1 ⎞
Cho biểu thức P = ⎜ + ⎟ : ⎜ − ⎟. 3 + x −1 10 − x
x − 3 x −1 −1 x −1 ⎝ ⎠ ⎝ ⎠ 1) Rút gọn P . 3 + 2 2 3 − 2 2
2) Tính giá trị của P khi 4 4 x = − . 3 − 2 2 3 + 2 2 Câu II (4,0 điểm)
Trong cùng một hệ toạ độ, cho đường thẳng d : y = x − 2 và parabol 2
(P) : y = − x . Gọi A
B là giao điểm của d và (P) .
1) Tính độ dài AB .
2) Tìm m để đường thẳng d ' : y = − x + m cắt (P) tại hai điểm C D sao cho CD = AB . Câu III (4,0 điểm) 2 ⎧ x + x = 2 ⎪⎪ y
1) Giải hệ phương trình ⎨ 2 ⎪ y 1 + y = . ⎪⎩ x 2
2) Tìm nghiệm nguyên của phương trình 6 2 3
2x + y − 2x y = 320 . Câu IV (6,0 điểm)
Cho tam giác nhọn ABC AB > AC . Gọi M là trung điểm của BC ; H là trực tâm;
AD, BE, CF là các đường cao của tam giác ABC . Kí hiệu (C ) và (C ) lần lượt là đường 1 2
tròn ngoại tiếp tam giác AEF DKE , với K là giao điểm của EF BC . Chứng minh rằng:
1) ME là tiếp tuyến chung của (C ) và (C ) . 1 2 2) KH AM . Câu V (2,0 điểm)
Với 0 ≤ x, y, z ≤ 1. Tìm tất cả các nghiệm của phương trình: x y z 3 + + = . 1+ y + zx 1+ z + xy 1+ x + yz x + y + z
------------------------------------------------------ HẾT-----------------------------------------------------
Thí sinh không được sử dụng tài liệu. Cán bộ coi thi không được giải thích gì thêm.
SỞ GIÁO DỤC VÀ ĐÀO TẠO
KÌ THI CHỌN HỌC SINH GIỎI TỈNH THANH HÓA Năm học: 2011-2012
HƯỚNG DẪN CHẤM MÔN TOÁN (Đề chính thức) Lớp 9 THCS
Ngày thi: 23 tháng 3 năm 2012
(Hướng dẫn gồm 03 trang) CÂU NỘI DUNG ĐIỂM I 1) 2,0 điểm 4,0
Điều kiện xác định: 1 < x ≠ 10 (*).
điểm Đặt: x −1 = a, 0 < a ≠ 3. 1,0 2 ⎛ a
a + 9 ⎞ ⎛ 3a +1 1 ⎞ Khi đó: P = ⎜ + ⎟ : − 2 ⎜ 2 ⎟ ⎝ 3 + a
9 − a ⎠ ⎝ a − 3a a ⎠ 3(a + 3) 2a + 4 − − − = 3 3 x 1 : a = = . 1,0 2 9 − a a(a − 3) 2a + 4 2 x −1 + 4 2) 2,0 điểm 2 2 x 4 = ( + ) 4 3 2 2 − (3− 2 2) 1,0 = ( + )4 − ( − )4 4 4 2 1 2 1 = 2 +1− ( 2 − ) 1 = 2 . −3 1 1,0 Suy ra: P = = − . 2 + 4 2 II 1) 2,0 điểm 2 4,0
⎧− x = x − 2
Toạ độ A và B thoả mãn hệ: điểm ⎨ ⎩y = x − 2 1,0 ⇔ ( ;
x y) = (1; −1) hoặc ( ;
x y) = (− 2; − 4) . AB = 9 + 9 = 3 2 . 1,0 2) 2,0 điểm
Xét phương trình (hoành độ giao điểm của (P) và d ' ): 2
x = − x + m ⇔ 2
x x + m = 0 (1).
Tồn tại CD, khi và chỉ khi: (1) có 2 nghiệm x , 1 2 x phân biệt 1,0 ⇔ 1 m < (*). 4
Khi đó, toạ độ của C D là: C(x ; y ) và D(x ; y ) , trong đó: = − + và = − + . 1 1 2 2 1 y 1 x m y2 2 x m 2 2 2 2 2
CD = (x x ) + ( y y ) = 2(x x ) = 2 ⎡(x + x ) − 4 ⎤ 1 2 1 2 1 2 ⎣ 1 2 1 x 2 x ⎦ .
Áp dụng định lý Viét đối với (1), suy ra: 2
CD = 2(1− 4m) . 1,0
CD = AB ⇔ 2(1− 4m) = 18 ⇔ m = − 2 , thoả mãn (*).
Vậy, giá trị cần tìm của m là: m = − 2 . III 1) 2,0 điểm - 1 - 4,0
Điều kiện xác định: xy ≠ 0 (*). điểm 2
⎧⎪ x + xy = 2y 2 2
⎧⎪x + 2y + 3xy = x + 2y
Khi đó, hệ đã cho tương đương với: ⎨ ⇔ ⎨ 2 ⎪
⎩2y + 2xy = x 2
⎪⎩2y + 2xy = x
⎧(x + 2y)(x + y − 1) = 0 ⇔ 1,0 ⎨ 2
⎩2y + 2xy = x ⎧ ⎧x = 1− y x = − 2 y ⇔ ⎪ ⎨ hoặc ⎨ 1 2 ⎩y y = 0 y = ⎪⎩ 3 ⇔ ⎛ 2 1 ⎞ 1,0 ( ;
x y) = (0; 0), (− 2; 1) hoặc ; ⎜ ⎟ . ⎝ 3 3 ⎠ ⎛ 2 1 ⎞
Đối chiếu (*), suy ra nghiệm của hệ đã cho: ( ;
x y) = (− 2; 1) hoặc ( ; x y) = ; ⎜ ⎟ . ⎝ 3 3 ⎠ 2) 2,0 điểm 6 2 3
2x + y − 2x y = 320 (1). (1) ⇔ (x )2 3 3 2
+ (x y) = 320 . Đặt: 3 x = 8u và 3
x y = 8v , (1) trở thành: 2 2 u + v = 5 . 1,0 3 ⎧x = 8u ⎪ 3
x y = 8v Hệ: ⎨ suy ra: ( ;
x y) = (2; − 8), (2; 24), ( 2 − ;− 24), ( 2 − ;8) . 1,0 2 2 u ⎪ + v = 5 ⎪⎩x, yIV 1) 3,0 điểm 6,0 điểm A (C1) F E L (C2) H 1,0 B M C D K MEB =
CBE (tam giác BEC vuông tại E , có EM là trung tuyến) =
CAD (hai tam giác vuông EBC DAC có chung góc nhọn C ). - 2 -
Mặt khác H ∈ (C ) , từ đó ta có: HEM = . 1 HAE 0,5
Suy ra, ME là tiếp tuyến của (C ) . 1 MED = MEC DEC = MCE
DEC (do tam giác BEC vuông tại E , có EM là trung tuyến) 1,0 = MCE
DHC (tứ giác HDCE nội tiếp) = MCE
FHA (góc đối đỉnh) = MCE
FEA (tứ giác HEAF nội tiếp) = MCE
CEK (góc đối đỉnh) 0,5 =
DKE (góc ngoài tam giác), suy ra ME là tiếp tuyến của (C ) . 2
Hoàn thành lời giải bài toán. 2) 3,0 điểm
Gọi L = AM ∩ (C ) ; theo câu IV.1), ta có: 2 M .
L MA = ME = M . . 1.0 1 D MK
Suy ra L thuộc đường tròn ngoại tiếp tam giác ADK - là đường tròn đường kính AK . 1.0
Do đó KL AM .
Mặt khác, ta lại có HL AM (vì L ∈ (C ) - là đường tròn đường kính 1 AH ). 1.0 Do đó ,
K L, H thẳng hàng, suy ra điều phải chứng minh. V 2,0 x y z 3 + + = (1).
điểm 1+ y + zx 1+ z + xy 1+ x + yz x + y + z
Giả thiết 0 ≤ x, y, z < 1 kết hợp với điều kiện xác định của (1), suy ra: x + y + z > 0 (*). Khi đó, ta có: (1 − 0.5
z)(1 − x) ≥ 0 ⇔ 1 + x x
zx z + x ⇔ ≤ . 1 + y + zx x + y + z Tương tự, ta cũng có: y y ≤ và z z ≤ . 1 + z + xy x + y + z 1+ x + yz x + y + z 0.5 3 Suy ra: x y z = + + ≤ 1 x + y + z 1+ y + zx 1+ z + xy 1+ x + yz hay 3
x + y + z ≥ (1) 0.5
Mặt khác, t ừ 0 ≤ x, y, z ≤ 1, suy ra: x + y + z ≤ 3 (2)
Từ (1) và (2) ta suy ra: x + y + z = 3, kết hợp với điều kiện 0 ≤ x, y, z ≤ 1 suy ra x = y = z = 1 0.5
Vậy, phương trình đã cho có nghiệm duy nhất ( ;
x y; z) = (1;1;1)
----------------------------------------------------HẾT---------------------------------------------------- - 3 - SỞ GD&ĐT VĨNH PHÚC
KỲ THI CHỌN HSG LỚP 9 NĂM HỌC 2013-2014 ĐỀ THI MÔN: TOÁN ĐỀ CHÍNH THỨC
Thời gian làm bài: 150 phút, không kể thời gian giao đề Câu 1 (3,0 điểm). 2 a 16 a  4 2 a 1
a) Cho biểu thức: M   
. Tìm tất cả các giá trị nguyên của a a  6 a  8 a  2 4  a
để giá trị của M là một số nguyên. b) Cho đa thức 2
P(x)  ax bx c thỏa mãn đồng thời các điều kiện P(x)  0 với mọi số
a b c
thực xb a . Tìm giá trị nhỏ nhất của biểu thức Q  . b a
Câu 2 (2,0 điểm). Tìm tất cả các giá trị của tham số m để phương trình sau vô nghiệm: x 1 xx m 1 x m  2 7 5 1954
Câu 3 (1,0 điểm). Cho p là số nguyên tố lớn hơn 5. Chứng minh rằng số p 1 chia hết cho 60.
Câu 4 (3,0 điểm). Cho đường tròn (O) có tâm là O và bán kính bằng R . Hai điểm phân
biệt B,C cố định nằm trên (O) sao cho BC a  2R . Gọi A là điểm bất kì thuộc cung lớn 
BC của (O) , A không trùng với B,C . Gọi D là chân đường phân giác trong kẻ từ A
của tam giác ABC . Hai điểm E, F lần lượt là tâm đường tròn ngoại tiếp các tam giác ADB ADC .
a) Chứng minh rằng hai tam giác AEO ADC đồng dạng.
b) Tính diện tích tứ giác AEOF theo a R .
c) Chứng minh rằng khi điểm A thay đổi thì E di chuyển trên một đường thẳng cố định.
Câu 5 (1,0 điểm). Trên một đường tròn cho 21 điểm phân biệt. Mỗi một điểm được tô
bởi một trong 4 màu: xanh, đỏ, tím, vàng. Giữa mỗi cặp điểm nối với nhau bằng một
đoạn thẳng được tô bởi một trong 2 màu: nâu hoặc đen. Chứng minh rằng luôn tồn tại
một tam giác có ba đỉnh được tô cùng một màu (xanh, đỏ, tím hoặc vàng) và ba cạnh
cũng được tô cùng một màu (nâu hoặc đen). ----------- Hết -----------
Cán bộ coi thi không giải thích gì thêm.
Họ và tên thí sinh:……….………..…….…….….….; Số báo danh………………. SỞ GD&ĐT VĨNH PHÚC
KỲ THI CHỌN HSG LỚP 9 NĂM HỌC 2013-2014
HƯỚNG DẪN CHẤM MÔN: TOÁN I. LƯU Ý CHUNG:
- Hướng dẫn chấm chỉ trình bày một cách giải với những ý cơ bản phải có. Khi chấm
bài học sinh làm theo cách khác nếu đúng và đủ ý thì vẫn cho điểm tối đa.
- Điểm toàn bài tính đến 0,25 và không làm tròn.
- Với bài hình học nếu thí sinh không vẽ hình phần nào thì không cho điểm tương ứng với phần đó. II. ĐÁP ÁN: Câu Ý Nội dung trình bày Điểm 1 a) 2 a 16 a  4 2 a 1
Cho biểu thức: M   
. Tìm tất cả các giá trị 2,5 a  6 a  8 a  2 4  a
nguyên của a để M là một số nguyên.a  0 ĐKXĐ: 
a  4, a  16  2 a 16 a  4 2 a 1 M    a  6 a  8 a  2 a  4
2 a 16  ( a  4)( a  4)  (2 a 1)( a  2)  a  6 a  8 a a  2 a 1  
( a  2)( a  4) a  4 a 1 5 Từ . M   1 a  4 a  4
Do M là số nguyên nên 5( a  4)  a  4 { 1;  5}.
TH1. a  4  1  a  25
TH2. a  4  1  a  9
TH3. a  4  5  a  81 TH4. a  4  5   a  1  (loại)
Đối chiếu điều kiện đã đặt, ta suy ra các giá trị cần tìm của a là: 9; 25; 81. b) Cho đa thức 2
P(x)  ax bx c thỏa mãn đồng thời các điều kiện
0,5 P(x)  0 với mọi số thực x và b a . Tìm giá trị nhỏ nhất của biểu thức
a b c Q . b aa  0
- Từ P(x)  0, x
   ta chứng minh được  . 2
  b  4ac  0  2 2 2 2 b b
a b c
4a  4ab b - Do đó: c
a b c a b    4a 4a b a
4a(b a) 2 2 2 2 2
4a  4ab b
16a  8ab b 12a(b a) (4a b) - Lại có:   3   3
4a(b a)
4a(b a)
4a(b a) Vậy Q
 3  b c  4a  0 min
Học sinh có thể làm theo cách sau:
- Từ giả thiết P(x)  0, x
    P(2)  0
 4a  2b c  0  a b c  3(b a)  0
a b c
- Từ đó suy ra Q   3 . b a Xét đa thức 2
P(x)  x  4x  4 , ta thấy đa thức này thỏa mãn các điều 1 4  4
kiện của giả thiết và khi đó Q   3 . 4 1
Vậy giá trị nhỏ nhất của Q bằng 3. 2
2,0 Tìm tất cả các giá trị của tham số m để phương trình sau vô nghiệm: x 1 x (*) x m 1 x m  2 x m 1 ĐKXĐ:  x  m  2  Khi đó 2 2
(*)  x  (m  3)x m  2  x  (1 m)x  (2m  2)x  m  2 (**)
+ Nếu m  1, (**)  0.x  1, vô nghiệm, suy ra phương trình (*) vô nghiệm m  2 + Nếu m  1
 thì (**) có nghiệm x  
, do đó phương trình đã 2m  2  m  2   m 1 (1)  cho vô nghiệm nếu 2m  2  m  2   m  2 (2)  2m  2 m  0 - TH1 : 2 (1) m 2 2m 2        1 m    2 m  2  - TH2 : 2 2 (2) m 2 2m 6m 4 2m 5m 2 0              1 m    2 1
Vậy có 4 giá trị của m để phương trình vô nghiệm là : 1;0; 2  ;  . 2 3 1,0 7
Cho p là số nguyên tố lớn hơn 5. Chứng minh rằng số 5 1954 p 1 chia hết cho 60.
Trước hết ta dễ dàng chứng minh 7 5 1954
 4m (với m nguyên dương)
Ta sẽ chứng bài toán tổng quát 4m p
1 chia hết cho 60 với mọi số
nguyên tố p  5 và mọi số nguyên dương m. Thật vậy, có 4m 4 m m p   p    4 p   2 1 ( ) 1
1 A  ( p 1)( p 1)( p 1).A ( A   )
Do p lẻ nên p 1, p 1 là hai số chẵn liên tiếp suy ra ( p 1)( p 1)4 (1)
Lại có ( p 1) p( p 1)3 mà p không chia hết cho 3 nên ( p 1)( p 1)3 (2)
Do p không chia hết cho 5 nên p có một trong các dạng 5k 1; 5k  2 . - Nếu 2 2
p  5k 1  p  25k 10k 1  5n 1 - Nếu 2 2
p  5k  2  p  25k  20k  4  5l 1 ( k, n,l  ) Suy ra 4
p 1  5.q , hay 2
( p 1)( p 1)( p 1)5 (3)
Từ (1), (2), (3) và 3, 5, 4 là các số đôi một nguyên tố cùng nhau nên 2 4
( p 1)( p 1)( p 1)(3.5.4)  p 160 . Vậy 4m p
160 (điều phải chứng minh). 4 a
Chứng minh rằng hai tam giác AEO và ADC đồng dạng. 1,5 1
Trong đường tròn (O) ta có:    AOE
.AOB ACB (1) 2
Trong đường tròn (ADB), ta có  1 1 AEO  sđ  ADB 0   360 2.ADB 0  
 180  ADB ADC (2) 2 2
Từ (1) và (2) suy ra hai tam giác AEO ADC đồng dạng. b
1,0 Tính diện tích tứ giác AEOF theo a và R .
Tương tự phần a), ta có hai tam giác AFO, ADB đồng dạng, do đó       0
AEO ADC, AFO ADB AEO ADB  180  AEOF là tứ giác nội
tiếp  E, F nằm hai phía AO , suy ra : 1 SSS
(OE.AB OF.AC) (3) AEOF AOE AOF 4
(Nếu học sinh không chứng minh (3) trừ 0,25 điểm) OE AO A . O CD - Lại có:   OE  (4) CD AC AC OF AO A . O BD   OF  (5) BD AB AB A . O CD A . O BD
Thay (4), (5) vào (3) ta được: 4.S  .AB  .AC (6) AEOF AC AB AB DB
- Vì AD là phân giác của tam giác ABC nên ta có:  (7) AC DC
Thế (7) vào (6) ta được AB AC BD CD 4SAO(C . DB . D )  AO(C . DB . D ) AEOF AC AB CD BD . R a
AO(BD CD)  A . O BC  . R a S  (đvdt). AEOF 4 c
Chứng minh rằng khi A thay đổi thì điểm E di chuyển trên một đường
0,5 thẳng cố định.
- Đường trung trực của BC cắt cung lớn 
BC tại H , cắt cung nhỏ  BC tại K.
Khi đó H, K cố định và là điểm chính giữa của các cung tương ứng.
- Gọi M, N tương ứng là trung điểm BD, AB suy ra   0
BNE BME  90
Do đó B, M , N , E cùng nằm trên đường tròn đường kính BE .    1
BEM BNM BAD  sđ  BKC . 4  1 BHK  sđ  BKC , suy ra  
BEM BHK (8) 4
Lại có EM // HK (cùng vuông góc với BC), H , E cùng phía so với BC (9) /   Kéo dài /
BE HK H BEM BH K (10) Từ (8), (9), (10) suy ra /
H H B, E, H thẳng hàng  E BH cố định. 5 1,0 B E A C D F
- Vì các điểm phân biệt nằm trên một đường tròn nên ba điểm bất kỳ luôn tạo thành một tam giác.
- Có 21 điểm được tô bằng 4 màu, do đó có ít nhất 6 điểm có cùng màu.
Giả sử có 6 điểm cùng màu đỏ là ,
A B, C, D, E, F
- Nối 5 đoạn AB, AC, AD, AE, AF và tô bằng 2 màu nâu, đen khi đó có ít
nhất 3 đoạn cùng màu, giả sử AB, AC, AD được tô cùng màu đen.
Xét tam giác BCD , xảy ra hai khả năng:
TH1. Nếu ba cạnh BC, BD, DC được tô cùng màu nâu thì tam giác BCD
ba đỉnh cùng màu đỏ, ba cạnh cùng màu nâu (thỏa mãn)
TH2. Nếu ba cạnh BC, BD, DC có ít nhất một cạnh màu đen, giả sử BC
đen, khi đó tam giác ABC có ba đỉnh cùng màu đỏ, ba cạnh cùng màu đen (thỏa mãn)
Vậy luôn có một tam giác có ba đỉnh cùng màu và ba cạnh cùng màu.
------------- Hết -------------
UBND HUYN NGHI XUÂN
K THI CHN ĐỘI TUYN HC SINH GII LP 9
PHÒNG GIÁO DC- ĐÀO TO
NĂM HC 2013 -2014
Môn: Toán. Thi gian làm bài: 150 phút
Câu 1: a. Tính giá trị của biểu thức: A = 6 − 2 5 + 14 − 6 5
b. Tìm x; y thỏa mãn: 2x + y − 2 xy − 4 x + 4 = 0
Câu 2: a. Giải phương trình nghiệm nguyên: 4 2 2
5x + y − 4x y − 85 = 0 5 5 5
P = x + 2012 + 2y − 2013 + 3z + 2014
b. Cho x ; y ; z là các số nguyên và ( ) ( ) ( ) 
S = x + 2y + 3z + 2013.
Chứng minh rằng P chia hết cho 30 khi và chỉ khi S chia hết cho 30.
Câu 3: Cho ba số x, y, z khác 0 và thoả mãn:  1 x + y + z =  2  1 1 1 1  + + + = 4 . 2 2 2 x y z xyz   1 1 1  + + > 0  x y z 2009 2009 2011 2011 2013 2013
Tính giá trị của biểu thức: P = ( y + z )(z + x )(x + y )
Câu 4: a. Cho tam giác nhọn ABC có trực tâm H, trọng tâm I; Giao điểm 3 đường trung
trực là O, trung điểm của BC là M. 2 2 Tính giá tr IO + OM ị biểu thức: 2 2 IH + HA
b. Cho góc xOy . Một đường thẳng d thay đổi luôn cắt các tia Ox; Oy tại M và N. 1 1
Biết giá trị biểu thức +
không thay đổi khi đường thẳng d thay đổi. OM ON
Chứng minh rằng đường thẳng d luôn đi qua một điểm cố định.
Câu 5: a. Cho các số x; y; z không âm, không đồng thời bằng 0 và thỏa mãn: 1 1 1 + + ≤ 1. x +1 y + 2 z + 3 1
Tìm giá trị nhỏ nhất của biểu thức: P = x + y + z + x + y + z
b. Cho các số dương x, y, z thoả mãn điều kiện: xy + yz + zx = 671. x y z 1 Chứng minh rằng: + + ≥ 2 2 2 x yz + 2013 y zx + 2013 z xy + 2013 x + y + z
-------------------- Hết ----------------------
Họ và tên thí sinh ............................................................... SBD .............................
PHÒNG GD-ĐT NGHI XUÂN
-------------------------- H
ƯỚNG DN CHM MÔN TOÁN
THI CHN ĐỘI TUYN HC SINH GII LP 9
NĂM HC 2013-2014
Câu 1:(4 điểm) . a) 1,5 điểm. b) 2,5 điểm
BIU ĐIM 2 2
a) A = 6 − 2 5 + 14 − 6 5 = ( 5 − )
1 + (3− 5) = 5 −1+ 3− 5 = 2 1,5  x = 0; y ∀ b) ĐKXĐ:  0,5
x > 0; y ≥ 0
Xét x = 0. Suy ra y = - 4 ( Thỏa mãn) 0,75 2 2
Xét x > 0; y ≥ 0 . Biến đổi PT về dạng: ( x y ) + ( x − 2) = 0 1,0
Lập luận tính được x = y = 4 ( Thỏa mãn). KL: ( ;
x y ) = (0; −4) hoặc ( ; x y ) = (4; 4) 0,25
Câu 2: (4,5 điểm) a) 2,25 điểm. b) 2,25 điểm
a) Phương trình đã cho tương đương với x = − ( y x )2 4 2 85 2 0,5 Lập luận 4 4
x ≤ 85 < 4 Mà x Z Suy ra 4 x ∈ { 4 4 4 4 0 ;1 ; 2 ;3 } 1,0 4 4 x = 0 thì 2 y = 85 ( loại) 4 4
x = 1 thì ( y − )2 2 = 84 ( loại) 4 4
x = 2 thì ( y − )2 8 = 71 ( loại) 0,75  y −18 = 2  y = 20 x = 3 4 4
x = 3 thì ( y − )2 18 = 4 ⇔  ⇔  Khi đó   y −18 = −2  y = 16 x = −3
Vậy phương trình có 4 nghiệm nguyên ( ;
x y ) là: (3 ; 20); (-3 ; 20); (3 ; 16); (-3 ; 16) 5 5 5 b)
P = a + b + c
Đặt a = x + 2012;b = 2 y − 2013; c = 3z + 2014 . Ta có:
S = a + b + c
( a ; b ; c là các số nguyên ) 0,5
Xét − = ( 5 − ) + ( 5 − ) + ( 5 P S a a b b c c) 1
Ta có : với mọi số nguyên m thì 5
m m chia hết cho 30 Thật vậy: 5 4 2 2 m m = ( m m 1 − ) = ( m m 1 − )(m 1 + ) =. . = ( m m 1 − )(m 1
+ )(m−2)(m+ 2) +5 ( m m 1 − )(m 1 + ) (1)
Với mọi số nguyên m thì ;
m (m −1); (m +1); (m − 2); (m + 2) là 5 số nguyên liên tiếp
nên trong đó có 1 thừa số chia hết cho 2; 1 thừa số chia hết cho 3;1 thừa số chia
hết cho 5 mà 2; 3; 5 nguyên tố cùng nhau từng đôi một nên tích của chúng chia
hết cho 2.3.5. Hay m(m −1)(m +1)(m − 2)(m + 2) chia hết cho 30 (2) Và ;
m (m −1); (m +1) ;
m (m −1); (m +1); (m − 2); (m + 2) là 3 số nguyên liên tiếp nên 1,75
trong đó có 1 thừa số chia hết cho 2; 1 thừa số chia hết cho 3 mà 2; 3 nguyên tố
cùng nhau nên tích của chúng chia hết cho 2.3. Hay 5m(m −1)(m +1) chia hết cho 30 (3)
Từ (1); (2); (3) Suy ra với mọi số nguyên m thì 5
m m chia hết cho 30
Do đó − = ( 5 − ) + ( 5 − ) + ( 5 P S a a b b
c c) chia hết cho 30 với a; b; c là các số nguyên
Câu 3: (2,5 điểm) Từ giả thiết suy ra: 2 1 1 1 1 1 1 1 2(x + y + z) 1 1 1  1 1 1   1 1 1  4 = + + + = + + + = + + + 2 + +  =  + +  2 2 2 2 2 2 2 2 2 x y z xyz x y z xyz x y z  xy yz zx   x y z  1 1 1 1 1 1
Mà + + > 0 suy ra + + = 2 (1) x y z x y z 1,0 1 1
Mặt khác x + y + z = suy ra = 2 (2) 2 x + y + z 1 1 1 1 Từ (1) và (2) suy ra + + = (3) x y z x + y + z
Biến đổi (3) ⇔ (x + y)(y + z)(z + x) = 0 1,0 2013 2013 2013 2013 x + y = 0 x = − yx = − yx + y = 0     ⇔ 2009 2009 2009 2009
z + y = 0 ⇔ y = −z y = −zy + z = 0     nên P = 0 0,5 2011 2011 2011 2011  x + z = 0  z = −xz = −xz + x = 0  
Câu 4 :(5,5 đim) a) 3 đim. b) 2,5 đim A
a) Ta có MO // HA (cùng vuông góc với BC)
OK // BH (cùng vuông góc với AC)
⇒ KOM = BHA (góc có cạnh tương ứng song song) K
MK // AB (M, K là trung điểm BC và AC) H
⇒ HAB = OMK (góc có cạnh tương ứng song song) I O
⇒ ∆ABH đồng dạng với ∆MKO (1,0) B C MO MK 1 M ⇒ = = ( 0,5) AH AB 2 2 MO MI 1 Xét ∆AIH và ∆MIO có = = và OMI = HAI (so le trong) AH AI 2 1,0 IO 1 IO OM 1
⇒ ∆AIH đồng dạng với ∆MIO ⇒ = ⇒ = = IH 2 IH HA 2 2 2 2 2 IO OM IO OM 1 2 2 IO + OM 1 ⇒ + = = = ⇒ = 0,5 2 2 2 IH HA IH + 2 HA 4 2 2 IH + OA 2 d x M E I O N D y b) Giả sử 1 1 1 + =
(1) ( a là số dương cho trước). Lấy điểm D trên Oy sao OM ON a 1,0
cho OD = a thì OD < ON. Vẽ DI song song với Ox ( I∈đoạn MN ). Lấy E trên Ox
sao cho OE = ID. Khi đó OEID là hình bình hành. Ta có OE OD NI EI NI MI OE + = + = + = 1=> 1 1 1 + = = (2) OM ON NM ON NM MN ON O . D OM OD a 0,75 T OE OE ừ (1) và (2) => 1 = =>
= 1 => OE = OD = a không đổi, mà OM O . D OM OD 0,75
D∈ Oy; E∈ Ox nên D; E cố định. Mặt khác O cố định và OEID là hình bình hành
nên I cố định. Vậy d luôn đi qua I cố định (ĐPCM)
CÂU 5 (3,5 đim) Câu a) 2 đim. Câu b) 1,5 đim
a) Trước tiên ta chứng minh bất đẳng thức: Với ∀ a, b, c ∈ R và x, y, z > 0 ta a b c
(a + b + c)2 2 2 2 có + + ≥
(*) Dấu “=” xảy ra ⇔ a b c = = x y z x + y + z x y z a b (a + b)2 2 2
Thật vậy, với a, b ∈ R và x, y > 0 ta có + ≥ (**) x y x + y ⇔ ( + )( + ) ≥ ( + )2 2 2 a y b x x y
xy a b ⇔ (bx ay)2 ≥ 0 (luôn đúng)
áp dụng bất đẳng thức (**) ta có 1 a b c ( + )2 c ( + + )2 2 2 2 2 a b a b c + + ≥ + ≥
Dấu “=” xảy ra ⇔ a b c = = x y z x + y z x + y + z x y z 1 1 1 ( + + )2 1 1 1 9
Áp dụng với a = b= c = 1 ta có 1 ≥ + + ≥ = x +1 y + 2 z + 3
x + y + z + 6
x + y + z + 6
=> x + y + z + 6 ≥ 9 => x + y + z ≥ 3
( Có thể chứng minh BĐT trên nhờ áp dụng BĐT Bunhicopski )
Áp dụng BĐT Côsi cho 2 số dương ... ta có: 1 8(x + y+z) x + y+z 1 8.3 x + y+z 1 10 0,75 P = x + y+z + = + + ≥ + 2. . = x + y+z 9 9 x + y+z 9 9 x + y+z 3 3
Dấu “=” xảy ra khi và chỉ khi các số x; y; z không âm và không đồng thời bằng 0 x + y + z = 3  x + y + z 1  = x = 2  9 x + y + z  thỏa mãn :  ⇔ y = 1 ( Thỏa mãn) x +1 = y + 2 = z + 3   0,25  z = 0 1 1 1  + + = 1  x +1 y + 2 z + 3 10 Vậy Min P = ⇔ x = 2; y = 1; z = 0. 3
b) Áp dụng bất đẳng thức (*) ta có x y z VT = + + 2 2 2 x yz + 2013 y zx + 2013 z xy + 2013 2 2 2 x y z = + + x ( 2 x yz + ) y( 2 y zx + ) z ( 2 2013 2013 z xy + 2013) ( 0,75
x + y + z )2 ≥ (1) 3 3 3
x + y + z − 3xyz + 2013( x + y + z )
Chú ý: xy + yz + zx = 671 nên x ( 2
x yz + 2013) = x ( 2
x + xy + zx +1342) > 0 , y ( 2
y zx + 2013) > 0 và z ( 2
z xy + 2013) > 0 Chứng minh: 3 3 3
x + y + z xyz = ( x + y + z ) ( 2 2 2 3
x + y + z xy yz zx)
( x y z) ( x y z)2 3( xy yz zx) = + + + + − + +   (2) 0,5 3 3 3 2
x + y + z −3xyz + 2013( x + y + z) = ( x y z) (
x y z) 3(xy yz zx) 2013 + + + + − + + +  
= ( x y z) ( x y z)2 3.671 2013 + + + + − +   = ( + + )3 x y z (3) Từ (1) và (3) ta suy ra
( x + y + z)2 1 2013 VT ≥ =
Dấu “=” xảy ra ⇔ x = y = z = . 0,25
( x + y + z)3 x + y + z 3
( Ghi chú: Mi cách gii khác đúng và hp lí đều cho đim ti đa tương ng) ----Hết----- 4 ph p ßng g gd g -®t ®øc ø th t ä ä ® Ò Ò th t i ol o y l m y p m i p c to¸ o n ¸ 9 n¨m ¨ m häc ä 20 2 1 0 2-20 2 13 1 §Ò thi chÝnh thøc
Thêi gian lµm bµi 120 phót
Bµi 1: a) Gi¶i ph−¬ng tr×nh x + 2 + 3 2x − 5 + x − 2 − 2x − 5 = 2 2
b) Víi gi¸ trÞ nµo cña tham sè a th× ph−¬ng tr×nh sau cã nghiÖm: 2 2 a x + 2a( 3 − ) 1 x + x − 4 = 2 3 − 4 (*) Lêi gi¶i: a) Ta cã + + − = ( − + − + ) = ( − + )2 1 1 x 2 3 2x 5 2x 5 6 2x 5 9 2x 5 3 ≥ 0 2 2 − − − = ( − − − + ) = ( − − )2 1 1 x 2 2x 5 2x 5 2 2x 5 1 2x 5 1 ≥ 0 2 2 5
§KX§: 2x − 5 ≥ 0 ⇔ x ≥ 2
Ph−¬ng tr×nh t−¬ng ®−¬ng 2x + 4 + 6 2x − 5 + 2x − 4 − 2 2x − 5 = 4 ⇔ ( − + )2 + ( − − )2 2x 5 3
2x 5 1 = 4 ⇔ 2x − 5 + 3 + 2x − 5 −1 = 4
⇔ 1− 2x − 5 = 1− 2x − 5 . Ta cã 1− 2x − 5 ≥ 1− 2x − 5 , do ®ã dÊu “=” x¶y ra khi vµ chØ khi
1− 2x − 5 ≥ 0 ⇔ 2x − 5 ≤ 1⇔ 2x − 5 ≤ 1⇔ x ≤ 3 . 5
KÕt hîp víi §KX§ ta cã nghiÖm cña ph−¬ng tr×nh lµ ≤ x ≤ 3 2
b) §KX§: x − 4 ≥ 0 ⇔ x ≥ 4
Ph−¬ng tr×nh t−¬ng ®−¬ng 2 2
a x + 2a( 3 − )1x + 4 −2 3 = − x − 4 2 2 Ta cã 2 2 + ( − ) 2 2 a x 2a
3 1 x + 4 − 2 3 = a x + 2a( 3 − )1x + ( 3 − ) 1 = (ax + 3 − )1 ≥ 0 ;  2 2  + ( − )  2 2 a x 2a 3 1 x + 4 − 2 3 = 0 a x + 2a( 3 − ) 1 x + 4 − 2 3 = 0 − x − 4 ≤ 0 . Suy ra  ⇔   x − 4 = 0 x = 4
§Ó ph−¬ng tr×nh (*) cã nghiÖm th× ph−¬ng tr×nh 2 2 a x + 2a( 3 − )
1 x + 4 − 2 3 = 0 cã nghiÖm x = 4 Dã ®ã + ( − ) + − = ⇔ ( + − )2 2 2 1− 3 a 4 2a 3 1 4 4 2 3 0 4a 3 1 = 0 ⇔ a = 4
Bµi 2: a) T×m GTNN cña biÓu thøc = + + 2 + 2 P 1 4x 4x 4x −12x + 9
b) T×m sè thùc a ®Ó ph−¬ng tr×nh sau cã nghiÖm nguyªn 2 x − ax + a + 2 = 0 Lêi gi¶i: a) = + + + − + = ( + )2 + ( − )2 2 2 P 1 4x 4x 4x 12x 9 2x 1 2x 3 = 2x +1 + 2x − 3
= 2x + 1 + 3 − 2x ≥ 2x + 1+ 3 − 2x = 4 GTNN cña P lµ 4.
§¹t ®−îc khi ( + )( − ) ≥ ⇔ − − ≤ ⇔ ( − )2 2 1 3 2x 1 3 2x 0 4x 4x 3 0
2x 1 ≤ 4 ⇔ 2x −1 ≤ 2 ⇔ − ≤ x ≤ 2 2
b) §Ó ph−¬ng tr×nh cã nghiÖm nguyªn th× ∆ ≥ 0 ⇒ − − ≥ ⇔ ( − )2 2 a 4a 8 0
a 2 ≥ 12 ⇔ a − 2 ≥ 2 3
⇔ a ≤ 2 − 2 3 ; a ≥ 2 + 2 3 . Khi ®ã gäi x ; x lµ c¸c nghiÖm cña ph−¬ng tr×nh. Theo hÖ thøc Viets ta cã 1 2 x + x = a 1 2 
⇒ x x − x + x = 2 ⇔ x x −1 − x −1 = 3 ⇔ x −1 x −1 = 3 1 2 1 2 1 ( 2 ) ( 2 ) ( 1 )( 2 ) x x = a + 2 1 2
x – 1 vµ x – 1 lµ −íc cña 3. Gi¶ sö x th× x – 1
– 1. Ta cã 2 tr−êng hîp sau: 1 2 1 ≥ x2 1 ≥ x2 x −1= 3 x = 4 x −1= 1 − x = 0 1 1  ⇔  khi ®ã a = 6 vµ 1 1  ⇔  khi ®ã a = -2 x −1 = 1 x = 2 x −1 = 3 − x = −2 2 2 2 2
§èi chiÕu ®iÒu kiÖn ta cã a ∈{ 2 − ; } 6 lµ gi¸ trÞ cÇn t×m
Bµi 3: a) Chøng minh r»ng ®−êng th¼ng (d) cã ph−¬ng tr×nh (m − 3)x − (m − 2)y + m −1= 0 (m lµ tham sè) lu«n ®i
qua mét ®iÓm cè ®Þnh A. T×m täa ®é A
 x − y − 3 + x − 2 = 4 (1)
b) Gi¶i hÖ ph−¬ng tr×nh sau:  2 2
x + y − 2xy + 4x − 4y = 5 (2)
Lêi gi¶i: a) Ta cã (m − 3)x − (m − 2)y + m −1= 0 ⇔ mx − my + m − 3x + 2y −1= 0 x − y +1= 0 2x − 2y + 2 = 0 ⇔ m( x − y + ) 1 + (2y − 3x − )
1 = 0 ®óng víi mäi m khi vµ chØ khi  ⇔  2y − 3x −1 = 0 2y − 3x −1= 0 x = 1 x = 1 ⇔  ⇔ 
. VËy ®−êng th¼ng (d) lu«n ®i qua mét ®iÓm cè ®Þnh A(1; 2) y = x + 1 y = 2  − + =
b) Tõ ph−¬ng tr×nh (2) suy ra ( − )2 + ( − ) + = ⇔ ( − + )2 x y 2 3 x y 4 x y 4 9 x y 2 = 9 ⇔  x − y + 2 = −3 x − y = 1 ⇔  x − y = 5 − x − 2 = 2 x = 4
Víi x – y = 1 thay vµo ph−¬ng tr×nh (1) ®−îc x − 2 = 2 ⇔ ⇔  x 2 2  − = − x = 0
x = 4 ⇒ y = 3; x = 0 ⇒ y = -1
Víi x – y = -5 thay vµo ph−¬ng tr×nh (1) ®−îc x − 2 = −4 v« nghiÖm
VËy tËp nghiÖm cña ph−¬ng tr×nh lµ (x;y)∈ ( { 4;3);(0;− ) 1 }
Bµi 4: Cho ∆ABC ®Òu cè ®Þnh néi tiÕp trong ®−êng trßn (O). §−êng th¼ng d thay ®æi lu«n ®i qua A vµ c¾t cung nhá
AB t¹i ®iÓm E (E ≠ A). §−êng th¼ng d c¾t hai tiÕp tuyÕn t¹i B vµ C cña ®−êng trßn (O) lÇn l−ît t¹i M vµ N, MC c¾t BN t¹i F. Chøng minh r»ng
a) ∆CAN ∼ ∆BMA vµ ∆MBC ∼ ∆BCN
b) Tø gi¸c BMEF néi tiÕp ®−îc ®−êng trßn
c) Chøng minh r»ng ®−êng th¼ng EF lu«n ®i qua mét ®iÓm cè ®Þnh khi d thay ®æi 1 Lêi gi¶i: a) Ta cã 0 ACN = s® AC = ABC = 60 2 N 1 0
MBA = s® AB = ACB = 60 ⇒ ACN = MBA 2 A 1 ANC = s® (EBC − AC) E 2 1 M = ( − ) 1 s® EBC BC = s® BE = BAM 2 2 F O XÐt ∆CAN vµ ∆BMA cã ACN = MBA 
⇒ ∆CAN ∼ ∆BMA (g – g) I B C ANC = BAM CA CN BC CN BC BM ⇒ = ⇒ = ⇒ = BM BA BM CB CN CB K BC BM  =
XÐt ∆MBC vµ ∆BCN cã CN CB
⇒ ∆MBC ∼ ∆BCN (c – g – c)  0 M  BC = BCN = 120 B  MC = CBF (v× ∆MBC B ∆ CN)
b) XÐt ∆MBC vµ ∆BFC cã  ⇒ (g – g) B  CM chung 0 0
⇒ BFC = MBC = 120 ⇒ BFM = 60 . MÆt kh¸c 0 BCA + AEB = 180 , 0 BEM + AEB = 180 ⇒ 0 BEM = BCA = 60 . Suy ra 0
BEM = BFM = 60 , tø gi¸c BMEF néi tiÕp (E, F cïng nh×n MB d−íi 1 gãc b»ng nhau)
c) §−êng th¼ng EF c¾t ®−êng trßn (O) t¹i K. Ta cã BMF = CBF (v× ∆MBC ∼ ∆BFC); BMF = BEF (gãc néi
tiÕp cïng ch¾n BF ); BMF = BCK (gãc néi tiÕp cïng ch¾n BK ) ⇒ CBF = BCK ⇒ BF / CK (1) 1 Ta l¹i cã 0 0
BKC = s® BAC = 120 ⇒ KBF = 60 mµ 0 BFC = 120 ⇒ BK // FC (2) 2
Tõ (1) vµ (2) ⇒ tø gi¸c BFCK lµ h×nh b×nh hµnh. Do ®ã EF ®i qua trung ®iÓm I cña BC cè ®Þnh 1 1 1 1 1 1
Bµi 5: Cho a, b, c > 0. Chøng minh r»ng + + ≥ + +
a + 3b b + 3c c + 3a a + 2b + c b + 2c + a c + 2a + b 1 1 4 1 1 4
Lêi gi¶i: Víi x, y > 0 ta cã 2 2 + ≥ . ThËt vËy + ≥
⇔ ( x + y) ≥ 4xy ⇔ ( x − y) ≥ 0 víi ∀x, y. DÊu x y x + y x y x + y
“=” x¶y ra khi vµ chØ khi x = y 1 1 4 2
¸p dông bµi to¸n phô trªn ta cã: + ≥ =
a + 3b b + 2c + a a + 3b + b + 2c + a a + 2b + c 1 1 4 2 + ≥ =
b + 3c c + 2a + b b + 3c + c + 2a + b b + 2c + a 1 1 4 2 + ≥ = c + 3a a + 2b + c c + 3a + a + 2b + c c + 2a + b 1 1 1 1 1 1
Céng theo vÕ 3 B§T trªn ®−îc + + ≥ + +
a + 3b b + 3c c + 3a a + 2b + c b + 2c + a c + 2a + b a + 3b = b + 2c + a 
DÊu “=” x¶y ra khi vµ chØ khi b
 + 3c = c + 2a + b ⇔ a = b = c c + 3a = a + 2b +  c
Lêi gi¶i: NguyÔn Ngäc Hïng – THCS Hoµng Xu©n H·n
S GIÁO DC VÀ ĐÀO TO
K THI CHN HC SINH GII VĂN HOÁ CP TNH BC GIANG
NĂM HC 2012-2013
MÔN THI: TOÁN; LP: 9 PH THÔNG
ĐỀ THI CHÍNH THC Ngày thi: 30/3/2013 Đề thi có 01 trang
Thi gian làm bài 150 phút, không k thi gian giao đề
Câu 1.
(5,0 đim)
1) Tính giá trị của biểu thức 3 3 A = 26 +15 3 − 26 −15 3 .  − +   − +   − +  2) a 2 2 a 2 a 7 3 a 2 1 1
Rút gọn biểu thức P =  . +  :  −        . 3 3 + a − 2 11− a
a − 3 a − 2 − 2 a −      2 
Câu 2. (4,0 đim)
1) Giải phương trình: 3 2
3 x + 8 = 2x − 3x +10 . 2 2  + + + = 2) x y xy 1 4 y
Giải hệ phương trình sau:  . 2
(x +1)(x + y − 2) = y
Câu 3. (4,0 đim) 1) Cho hàm số 2
y = x . Tìm các giá trị của m để đường thẳng ∆ có phương trình
y = x m cắt đồ thị hàm số tại hai điểm phân biệt (
A x ; y ), B(x ; y ) thoả mãn: 1 1 2 2 4 4
(x x ) + ( y y ) = 18 . 2 1 2 1
2) Tìm tất cả các bộ ba số nguyên tố a,b,c đôi một khác nhau thoả mãn điều kiện
20abc < 30(ab + bc + ca) < 21abc
Câu 4. (6,0 đim)
Cho tam giác ABC vuông tại A (ABcạnh BC. Đường tròn tâm I đường kính AH cắt AB, AC thứ tự tại M và N. OA và MN cắt nhau tại D.
1) Chứng minh tứ giác BMNC nội tiếp. 2) 1 1 1 Chứng minh : = + . AD HB HC
3) Cho AB=3 và AC=4. Tính bán kính đường tròn ngoại tiếp tam giác BMN.
Câu 5. (1,0 đim)
Cho ba số dương a,b c thoả mãn abc = 1. Chứng minh rằng: 1 1 1 1 + + ≤ . 2 2 2 2 2 2 a + 2b + 3 b + 2c + 3 c + 2a + 3 2
---------------Hết----------------
Cán b coi thi không gii thích gì thêm.
Họ và tên thí sinh: .................................................................Số báo danh:.......................
Giám thị 1 (H tên và ký)..............................................................................................................
Giám thị 2 (H tên và ký)..............................................................................................................
S GIÁO DC VÀ ĐÀO TO
HƯỚNG DN CHM BC GIANG
BÀI THI CHN HC SINH GII VĂN HOÁ CP TNH NGÀY THI 30 /3/2013
ĐỀ CHÍNH THC
MÔN THI: TOÁN; LP: 9 PH THÔNG
Bn hướng dn chm có 04 .trang Câu 1 Hướng dẫn giải
(5 đim) Ta có 3 3 A = 26 +15 3 − 26 −15 3 3 2 2 3 3 2 2 3 = 8 + 3.2
3 + 3.2.( 3) + ( 3) − 8 − 3.2 3 + 3.2.( 3) − ( 3) 0.5 1. 3 3 3 3 = (2 + 3) − (2 − 3) (2 đim) 0.5 = (2 + 3) − (2 − 3) 0.5 A = 2 3 . 0.5 KL:
Điều kiện: 2 < a ≠ 11 0.5 Đặt 2 x =
a − 2 (0 < x ≠ 3) ⇒ a = x + 2 . 0.5 2 (x + 2)  x
x + 9   3x +1 1  Tính được P = . + : −     0.5 2 2 3  3 + x
9 − x   x − 3x x 2
(x + 2)  3(x + 3)   2x + 4 
(3 đim) = .  :   0.5 2 3
 9 − x   x(x − 3) 
(x + 2) x(x − 3) x = . = − 0.5 3 − x 2x + 4 2 a − 2 = − 2 0.5 KL: Câu 2
(4 đim)
ĐK: x ≥ −2 . Với điều kiện biến đổi phương trình đã cho trở thành: 1 0.5 2 2
(2 đim) 3. (x + 2)(x − 2x + 4) = 2(x − 2x + 4) + (x + 2)
Chia cả hai vế của phương trình cho 2
x − 2x + 4 , ta được x + 2 x + 2 − 3 + 2 = 0 (1) 0.5 2 2 x − 2x + 4 x − 2x + 4 x + 2 Đặt t = (t ≥ 0) 2 x − 2x + 4 0.5 Thay vào (1) ta được 2
t − 3t + 2 = 0 ⇔ t = 1 hoặc t = 2 (t/m) x + 2 x = 1 + với t = 1ta có 2
=1 ⇔ x − 3x + 2 = 0 ⇔  (t/m). 2 x − 2x + 4 x = 2 x + 2 0.5 + với t = 2 ta có 2
=2 ⇔ 4x − 9x +14 = 0 (vô nghiệm). 2 x − 2x + 4 KL: 2
x +1+ y(x + y) =  4 y  2
(x +1)(x + y − 2) = y 0.5 2 x +1 =  0
+ Với y = 0 Hpt trở thành:  (vô nghiệm) 2
(x +1)(x − 2) = 0 2
x +1 + (x + y) =  4  y
+ Với y ≠ 0 .Hệ trở thành  (1) 2 x +  1 (
)(x + y − 2) = 1 2  0.5  y
(2 đim) 2 x +1 a + b = 4 + Đặt a =
, b = x + y thay vào hpt(1) ta được  y a(b − 2) =  1
+ Giải được: a = 1, b = 3 0.5 2  x +1 =  1
+ Với a = 1, b = 3 ⇒  y . x + y =  3 0.5
Giải được nghiệm của hệ: ( ;
x y) = (1; 2) và (x;y)=(-2;5) + KL: Câu 3
(4 đim)
Xét pt hoành độ giao điểm: 2
x = x m 2
x x + m = 0 (1) 0.5
Đường thẳng ∆ cắt đths đã cho tại hai điểm phân biệt A, B khi và chỉ
khi pt(1) có hai nghiệm phân biệt.
+ Điều kiện: ∆ = 1− 4m > 0 1 0.5 ⇔ m < . 1 4
(2 đim) + Khi đó (
A x ; y ), B(x ; y ) 1 1 2 2
+ Theo định lí Viet x + x = 1, x x = m . Ta có y = x − ,
m y = x m 1 2 1 2 1 1 2 2 0.5 + 4 4 4 2 2
(x x ) + ( y y ) = 18 ⇔ (x x ) = 9 ⇔ [(x + x ) − 4x x ] = 9 1 2 1 2 1 2 1 2 1 2  = 1 (ko m t / m)  + Tìm được 1  m = − (t / m) 0.5  2 KL: 2 1 1 1 7 + Từ giả thiết suy ra: < + + <
. Không giảm tính tổng quát 2 3 a b c 10
(2 đim) 2 3 0.5
giả sử a > b > c > 1. Suy ra < ⇒ 2c < 9 3 c Do đó c ∈{2;3} + Với c = 2 suy ra 2 1 1 1 7 1 1 1 1 1 2 1 1 < + + < ⇒ < + < (1) ⇒ < và < 0.5 3 2 a b 10 6 a b 5 6 b b 5 Do đó b ∈{7;11} 1 1 2
+ Với b = 7 từ (1) suy ra < <
a ∈{19;23;29;31;37;41} 42 a 35 0,5 5 1 6
+ Với b = 11 từ (1) suy ra < <
a = 13 ( do a>b) 66 a 55
+ Với c = 3 từ giả thiết suy ra 1 1 1 11 1 2 < + < (*) ⇒ <
b < 6 ⇒ b = 5 ( do b>c) 3 a b 30 3 b 15
Thay b = 5 vào (*) được 6 < a < ⇒ a = 7 . 0.5 2
Vậy có 8 bộ ba (a;b;c) thoả mãn:
(19; 7; 2), (23; 7; 2), (29; 7; 2), (31; 7; 2), (37; 7; 2), (41; 7; 2), (13;11; 2), (7;5;3)
và các hoán vị của nó. Câu 4
(6 đim) 1
(2 đim)
+ Tứ giác AMHN nội tiếp nên AMN = AHN 0.5
+ Lại có AHN = ACH (vì cùng phụ với góc CHN ) 0.5
+ Suy ra ACB = AMN , mà 0
AMN + NMB = 180 nên 0 ACB + NMB = 180 0.5 KL: 0.5
+ Có AID = AOH vì cùng bằng hai lần ACB . 0.5 AD AI + Tam giác AID AOH ⇒ = 0.5 AH AO 2 1 1 1 1 (2 điểm) + Có AO = BC = (HB + HC), AI= AH = H . B HC 0.5 2 2 2 2 1 AO HB + HC 1 1 0.5 + Do đó = = = + . AD AH.AI H . B HC HB HC 3 12
+ Tính được BC=5, AH = 0.5 (2 điểm) 5
+ Gọi K là tâm của đường tròn ngoại tiếp tam giác BMN. Khi đó
KI là đường trung trực của đoạn MN. 0.5
Do hai tam giac AID và AOH đồng dạng nên 0
ADI = AHO = 90 ⇒ OA MN Do vậy KI//OA.
+ Do tứ giác BMNC nội tiếp nên OK BC . Do đó AH//KO.
+ Dẫn đến tứ giác AOKI là hình bình hành. 0.5 Bán kính 1 1 1 769 0.5 2 2 2 2 2 2 R = KB = KO + OB = AI + BC = AH + BC = 4 4 4 10 Câu 5
(1 đim) Ta có: 2 2 2 2 2
a + 2b + 3 = (a + b ) + (b +1) + 2 ≥ 2ab + 2b + 2 0.5 Tương tự: 2 2
b + 2c + 3 ≥ 2bc + 2c + 2 , 2 2
c + 2a + 3 ≥ 2ac + 2a + 2 Suy ra: 1 1 1 1 1 1 1 + + ≤ ( + + ) 2 2 2 2 2 2 a + 2b + 3 b + 2c + 3 c + 2a + 3 2 ab + b +1 bc + c +1 ac + a +1 1 1 1 1 1 0.5 = ( + + ) = . 2 ab + b +1 1 1 1 2 + + 1 + a +1 a ab b
Điểm toàn bài (20đim)
Lưu ý khi chm bài:
- Trên đây ch là sơ lược các bước gii, li gii ca hc sinh cn lp lun cht ch,
hp logic. Nếu hc sinh trình bày cách làm khác mà đúng thì cho đim các phn theo
thang
đim tương ng.
- Vi bài 4, nếu hc sinh v hình sai hoc không v hình thì không chm.
S GIÁO DC & ĐÀO TO
K THI CHN HC SINH GII TNH LP 9 THCS NGH AN
NĂM HC 2010 - 2011
ĐỀ CHÍNH THC
Môn thi: TOÁN - BNG B
Thời gian: 150 phút (không kể thời gian giao đề) Câu 1 (5,0 điểm).
a) Chứng minh rằng với mọi số nguyên n thì 2
n + n + 2 không chia hết cho 3.
b) Tìm tất cả các số tự nhiên n sao cho 2
n + 17 là một số chính phương. Câu 2 (5,0 điểm) a) Giải phương trình: 2 x + 4x+5 = 2 2x+3 2 2x+y = x
b) Giải hệ phương trình:  2 2y+x = y Câu 3 (3,0 điểm). 4x+3
Tìm giá trị nhỏ nhất của biểu thức: A = 2 x + 1 Câu 4 (4,5 điểm)
Cho tam giác ABC có ba góc nhọn nội tiếp đường tròn (O). Các đường cao BE, CF của
tam giác ABC cắt nhau tại H.
a) Chứng minh rằng BH.BE + CH.CF = 2 BC
b) Gọi K là điểm đối xứng với H qua BC. Chứng minh rằng K∈(O). Câu 5 (2,5 điểm).
Cho tam giác ABC nội tiếp đường tròn tâm O, một điểm I chuyển động trên cung BC
không chứa điểm A (I không trùng với B và C). Đường thẳng vuông góc với IB tại I cắt đường
thẳng AC tại E, đường thẳng vuông góc với IC tại I cắt đường thẳng AB tại F. Chứng minh
rằng đường thẳng EF luôn đi qua một điểm cố định.
- - - Hết - - -
H và tên thí sinh:................................................................................ S báo danh: .....................................
S GD&ĐT NGH AN
K THI CHN HC SINH GII TNH LP 9 THCS
NĂM HC 2010 - 2011
ĐÁP ÁN ĐỀ CHÍNH THC
Môn: TOÁN - Bng B
------------------------------------------- Câu: Ni dung 1. *) Nếu 2 nM3 ⇒ n + nM3 nên 2 n + n + 2 /M 3 (1) a, *) Nếu 2 n /M 3 ⇒ n + 2M3 (2,5) 2 ⇒ n + n + 2 /M3 (2) Từ (1) và (2) ⇒ n ∀ ∈ Z thì 2 n + n + 2 /M 3 Đặt 2 2 m = n + 17 (m ∈ N) 2 2
⇒ m − n = 17 ⇒ (m − n)(m + n) = 17 = 1.17=17.1 b, Do m + n > m - n (2,5) m + n = 17 m = 9 ⇒  ⇒  m − n = 1 n =   8 Vậy với n = 8 ta có 2 2 n + 17 = 64 + 17 = 81 = 9 2. Giải phương trình 2 x + 4x+5=2 2x+3 (1) 3
Điều kiện: 2x+3 ≥ 0 ⇒ x ≥ - 2 (1) 2 ⇔ x + 4x+5-2 2x+3 = 0 2
⇔ x + 2x+1+2x+3-2 2x+3 + 1 = 0 a, 2 2
⇔ (x + 1) + ( 2x+3 − 1) = 0 (2.5) x + 1 = 0 ⇔   2x+3 − 1 = 0 x = 1 − ⇔  2x+3=1 ⇔ x = 1
− thỏa mãn điều kiện Giải hệ phương trình 2 2x+y=x (1) b,  2 (2.5) 2y+x=y (2)
Trừ từng vế 2 phương trình ta có: 2 2 x − y = x − y
⇔ (x − y)(x + y − 1) = 0 x = y x = y ⇔ ⇔   x + y − 1 = 0 x = 1 −   y Ta có: x = y x = y *) ⇔   x(x − 3) = 0 x =   0 hoặc x = 3 Vậy (x; y) = (0;0); (3;3) x = 1 − y x = 1 − y x = 1 − y *) ⇔ ⇔    (*) 2 2 2 2x+y = x 2 − 2y + y = (1 − y) y − y + 1 =    0 Vì phương trình 2
y − y + 1 = 0 vô nghiệm nên hệ (*) vô nghiệm
Vậy hệ đã cho có 2 nghiệm (x; y) = (0; 0); (3; 3) 3. 4x+3
Tìmgiá trị nhỏ nhất của A = 2 x + 1 2 4x+3 x + 4x+4 Ta có: A = = −1 + 2 2 x + 1 x + 1 2 (x + 2) A = −1 + ≥ −1 2 x + 1
Dấu "=" xảy ra ⇔ x + 2 = 0 ⇔ x = 2 − Vậy A = −1 khi x = -2 min 4. a, A (2,5) E O F H B I C K
Gọi I là giao điểm của AH và BC ⇒ AI ⊥ BC Ta có: ∆BHI ∆B S CE (g, g) BH BI ⇒ = ⇒ BH.BE = BC.BI (1) BC BE Ta có: ∆CHI S ∆CBF (g, g) CH CI ⇒ = ⇒ CH.CF = BC.CI (2) CB CF
Từ (1) và (2) suy ra BH.HE + CH.CF = BC(BI + CI) = BC2 b,
Gọi K là điểm đối xứng của H qua BC suy ra HCB = KCB
(2,0) Mà FAI = HCI (do tứ giác AFIC nội tiếp) ⇒ FAI = BCK hay BAK = BCK
⇒ tứ giác BACK nội tiếp đường tròn (O) ⇒ K ∈ (O) 5. + Khi 0 BAC = 90 ⇒ 0 BIC = 90 .
⇒ F trùng với B, E trùng với C lúc đó EF là đường kính.
⇒ EF đi qua điểm O cố định. B F O K I A E C ⇒ + Khi BAC < 900 BIC > 900.
Gọi K là điểm đối xứng của I qua EF.
⇒ EIF = EAF (cùng bù BIC )
EKF = EIF (Do I và K đối xứng qua EF) ⇒ EKF = EAF ⇒ AKFE nội tiếp
⇒ KAB = KEF (cung chắn KF ) (1)
IEF = KEF (Do K và I đối xứng qua EF) (2)
IEF = BIK (cùng phụ KIE ) (3)
Từ (1), (2), (3) ⇒ KAB = BIK
⇒ AKBI là tứ giác nội tiếp ⇒ K ∈(O)
Mà EF là đường trung trực của KI ⇒ E, O, F thẳng hàng.
+ Khi BAC > 900 ⇒ BIC < 900 chứng minh tương tự.
Vậy đường thẳng EF luôn đi qua điểm O cố định.
- - - Hết - - -
S GIÁO DC & ĐÀO TO
ĐỀ THI HC SINH GII VÒNG TNH LP 9 THCS CÀ MAU
NĂM HC 2008-2009 Môn thi: Toán
ĐỀ CHÍNH THC
Ngày thi: 01 – 03 – 2009
Thời gian: 150 phút (Không k thi gian giao đề) Baøi 1 (3,0 ñieåm):
a) Tính giaù trò cuûa bieåu thöùc: S = 2 + 3 2 - 3 + 2 - 3 2 + 3
b) Ruùt goïn bieåu thöùc: y = 2 2 x - 2 x + 1 + x - 4 x + 4 Baøi 2 (3,0 ñieåm):
a) Chöùng minh raèng soá a = 2 ( 3 + ) 1 2 - 3 laø soá höõu tæ.
b) Cho ña thöùc f(x) = mx3 + (m – 2)x2 – (3n – 5)x – 4n. Xaùc ñònh m, n
sao cho ña thöùc f(x) chia heát cho x + 1 vaø x – 3. Baøi 3 (3,0 ñieåm):
Tìm moät soá töï nhieân goàm ba chöõ soá sao cho khi ta laáy chöõ soá ôû haøng ñôn
vò ñaët veà beân traùi cuûa soá goàm hai chöõ soá coøn laïi, ta ñöôïc moät soá coù ba chöõ soá
lôùn hôn chöõ soá ban ñaàu 765 ñôn vò.
Baøi 4 (3,0 ñieåm): Cho ña thöùc f(x – 1) = x2 – (m + 1)x – m2 + 2m – 2 . a) Tìm f(x).
b) Tìm giaù trò nhoû nhaát cuûa f(x) khi m = – 2. Baøi 5 (3,5 ñieåm):
Cho hình bình haønh ABCD. Goïi I laø trung ñieåm cuûa caïnh CD, E laø giao
ñieåm cuûa AC vaø BI, F laø giao ñieåm cuûa hai tia AB vaø DE. Chöùng minh raèng :
a) B laø trung ñieåm cuûa ñoaïn thaúng AF.
b) Neáu BC = BD thì AC = FD.
c) Neáu AC = FD thì BC = BD.
Baøi 6 (4,5 ñieåm): Cho töù giaùc ABCD noäi tieáp ñöôøng troøn (O) trong ñoù hai
ñöôøng cheùo AC vaø BD caét nhau taïi M. Cho bieát ADB laø tam giaùc caân coù goùc A > 900.
a) Chöùng minh raèng: AD2 = AM.AC .
b) Goïi I laø taâm ñöôøng troøn ngoaïi tieáp tam giaùc DCM vaø J laø taâm ñöôøng
troøn ngoaïi tieáp tam giaùc BCM. Chöùng minh raèng: · · IDB = JBD .
c) Chöùng minh raèng: Toång caùc ñoä daøi cuûa hai ñoaïn thaúng ID vaø JB
khoâng tuyø thuoäc vaøo vò trí cuûa ñieåm C treân cung lôùn BD cuûa ñöôøng troøn (O). ---------- HEÁT ----------
SỞ GIÁO DỤC VÀ ĐÀO TẠO KỲ THI CHỌN HỌC SINH GIỎI LỚP 9 CẤP TỈNH LONG AN MÔN THI :TOÁN NGÀY THI: 07/4/2011 ĐỀ C H Í N H T H
C THỜI GIAN :150 phút (không kể thời gian phát đề)
Bài 1:(4 đim)
1/ Không sử dụng máy tính , thực hiện phép tính : 3 + 5 3 − 5 A = + 2 2 + 3 + 5 2 2 − 3 − 5 2/ Cho biểu thức:
B = x − 2 + 4 − x (với 2 ≤ x ≤ 4 )
Tìm giá trị lớn nhất của biểu thức B và giá trị x tương ứng
Bài 2:( 5 đim)
1/ Cho hàm số y = ax2 (a ≠ 0) có đồ thị là (P) đi qua M(-1;2) . Trên (P) lấy A và B có
hoành độ tương ứng là 1 và 2 . Xác định m để đường thẳng y = mx +5 song song với đường thẳng AB 1 1 1 2/ Tìm x thỏa mãn : 2 2 3 2 x − + x + x + =
(2x + x + 2x +1) 4 4 2
Bài 3: (5 đim)
Cho tam giác ABC có ba góc đều nhọn có AB < AC nội tiếp đường tròn O bán kính R.
Ba đường cao AD,BE,CF cắt nhau tại H
a/ Chứng minh H là tâm đường tròn nội tiếp tam giác DEF
b/ Kẻ đường kính AK của đường tròn O.Gọi S là diện tích tam giác ABC A . B AC. Ch BC ứng minh : S = 4R
c/ Gọi M là trung điểm BC . Chứng minh: tứ giác DFEM là nội tiếp
Bài 4 : (3 đim)
Cho điểm M nằm trong tam giác ABC có BC = a, AC = b, AB = c. Gọi các khoảng cách
từ M đến ba cạnh BC, AC, AB tương ứng là x,y,z . Hãy xác định vị trí M trong tam giác sao cho biểu thức : a b c P = + +
đạt giá trị nhỏ nhất x y z
Bài 5 : (3 đim)
Tìm một số chính phương có bốn chữ số , mỗi chữ số nhỏ hơn 9. Biết rằng khi tăng mỗi
chữ số thêm một đơn vị thì số mới được tạo thành cũng là số chính phương. _______________
SỞ GIÁO DỤC VÀ ĐÀO TẠO KỲ THI CHỌN HỌC SINH GIỎI LỚP 9 CẤP TỈNH LONG AN MÔN THI : TOÁN NGÀY THI : 11/4/2012 Đ Ề C H Í N H T H
C THỜI GIAN : 150 phút (không kể thời gian phát đề)
Bài 1: ( 4
đim)
1/ Không sử dụng máy tính, hãy thực hiện phép tính: 2- 3 + 4- 15 + 10 A = 23- 3 5 3x + 6 x x + 1 x + 2 2/ Cho biểu thức B = - + x + x - 2 x + 2 1- x
a/ Tìm điều kiện xác định và rút gọn B.
b/ Tìm giá trị lớn nhất của B và giá trị x tương ứng.
Bài 2: (5 đim)
1/ Tìm hệ số a > 0 sao cho các đường thẳng y = ax – 1 ; y = 1 ; y = 5 và trục tung tạo thành hình
thang có diện tích bằng 8 (đơn vị diện tích). 1 1 1 2 1
2/ Cho các số x, y, z khác 0 thỏa mãn đồng thời + + = 2 và − = 4 . Tính giá trị của x y z 2 xy z
biểu thức P = (x + 2y + z)2012.
Bài 3: (5 đim)
Cho tam giác nhọn ABC nội tiếp đường tròn (O), các đường cao AD, BE, CF (DÎ BC, EÎ AC,
FÎ AB) cắt nhau tại H và cắt đường tròn (O) theo thứ tự ở M, N, K. Chứng minh rằng: a/ BH.BE + CH.CF = BC2. 2 2 2
b/ AH.AD + BH.BE + CH.CF = AB + BC + CA . 2 c/ AM BN CK + + = 4 . AD BE CF
Bài 4: (3 đim)
Cho đoạn thẳng CD = 6 cm, I là một điểm nằm giữa C và D ( IC > ID). Trên tia Ix vuông góc
với CD lấy hai điểm M và N sao cho IC = IM, ID = IN, CN cắt MD tại K ( K MD) , DN cắt MC tại L
(L MC) . Tìm vị trí của điểm I trên CD sao cho CN.NK có giá trị lớn nhất.
Bài 5: (3 đim)
Tìm các cặp số (x; y) nguyên dương thỏa mãn: xy + 2x = 27 – 3y.
----------------------------------------------------- Hết --------------------------------------------
H
và tên thí sinh :………………………………………………….
S
báo danh :………………
SỞ GIÁO DỤC VÀ ĐÀO TẠO KỲ THI CHỌN HỌC SINH GIỎI LỚP 9 CẤP TỈNH LONG AN MÔN THI : TOÁN NGÀY THI : 11/4/2012 Đ Ề C H Í N H T H
C THỜI GIAN : 150 phút (không kể thời gian phát đề)
HƯỚNG DN CHM Bài Câu Ni dung Đim 1 1 2 - 3 + 4 - 15 + 10 (4đ) A = 23- 3 5 2( 2- 3 + 4- 15 + 10) = 0,5 2( 23- 3 5) 4 - 2 3 + 8- 2 15 + 2 5 = 0,25 46 - 6 5 ( 3- )2 1 + ( 5 - 3)2 + 2 5 0,75 = (3 5 - )2 1 3 - 1+ 5 - 3 + 2 5 = 3 5 - 1 0,25 3 5 - 1 = 3 5 - 1 0,25 = 1 2 a/ ĐKXĐ x ³ 0, x ¹ 1 0,25 3x + 6 x x + 1 x + 2 B = - + x + x - 2 x + 2 1- x ( + )( - ) ( + + )2 x 1 x 1 x 2 3x 6 x = ( - - 0,5 x - ) 1 ( x + ) 2 ( x - ) 1 ( x + ) 2 ( x - ) 1 ( x + ) 2 3x + 6 x - x + 1- x - 4 x - 4 = ( x - ) 1 ( x + ) 2 x + 2 x - 3 = ( x - ) 1 ( x + ) 2 0,25 ( x - ) 1 ( x + ) 3 x + 3 = 0,25 ( = x - ) 1 ( x + ) 2 x + 2 x + 3 b) B = Với x ³ 0, x ¹ 1 x + 2 Mà x + 2 ³ 2 1 1 0,25 Û £ x + 2 2 1 3 0,25 Û 1+ £ x + 2 2
Dấu “ = “ xãy ra khi x = 0 Û x = 0 (tmđk) 3 0,25
Vậy giá trị lớn nhất của B là khi x = 0. 2 2 1 (5đ) 6 B C 5 y=5 4 3 0,5 2 A D 1 y=1 -10 -8 -6 -4 -2 2 4 6 8 10 O -1 -2 -3 -4
+) Kí hiệu hình thang ABCD cần tìm như hình vẽ. 6 2
+) Tính được C( ;5) ; D( ;1) a a 0,5 6 2 BC = ; AD = 0,5 a a  6 2  +) S =  + .4 : 2 = 8 0,25 ABCDa a
⇒ a = 2 ( Thỏa ĐK a > 0)
+) Vậy phương trình đường thẳng là y = 2x – 1. 0,25 2 2 1 1 1  1 1 1  0,25 +) Ta có + + = 2 ⇒  + +  = 4 x y zx y z  2  1 1 1  2 1 +) Do đó  + +  = − 2 0.25  x y z xy z 1 1 1 2 2 2 2 1 ⇔ + + + + + − + = 0 0,25 2 2 2 2 x y z xy yz zx xy z  1 2 1   1 2 1  0,5 ⇔  + +  +  + +  = 0 2 2 2 2  x xz z   y yz z  2 2  1 1   1 1  0,5 ⇔  +  +  +  = 0  x z   y z  2  1 1   1 1  +  = 0 − =  x z     x z 0,5 ⇔  ⇔ 
x = y = −z 2 1 1  1 1 −       0 = + =  y z   y z  1 1 1 1 1 Thay vào − + + = 2 ta được x = y = ; z = 0,5 x y z 2 2 2012  1 1 1 Khi − đó P = 2012  + 2. +  = 1 = 1  2 2 2  0,25 3 (5đ) A N K E F o H B C D M a +) Tứ giác DCEH có 0 0 0
HDC + HEC = 90 + 90 = 180
⇒ Tứ giác DCEH nội tiếp ⇒ HED = HCD ( cùng chắn cung HD) 0,5
* ∆ BDE và ∆ BHC có HED = HCD EBC chung.
⇒ ∆ BDE đồng dạng ∆ BHC (g.g) 0,25 0,5 ⇒ BD BE =
BH.BE = BC.BD (*) BH BC
*Chứng minh tương tự đẳng thức (*)ta được : CH.CF = CD.CB (**) 0,25
Cộng (*) và (**) theo vế ta được: BH.BE + CH.CF = BC.BD + CD.CB = (BD + CD).BC = BC.BC = BC2 (1) 0,5 b
+) Chứng minh tương tự đẳng thức (1) ta được:
BH.BE + AH.AD = AB2 (2) và AH.AD + CH.CF = AC2 (3) 0,5
+) Cộng (1), (2), (3) theo vế ta được:
2(AH.AD + BH.BE + CH.CF) = AB2 + AC2 + BC2 0.75 2 2 2
AB + BC + CA 0.25 ⇔ AH.AD + BH.BE + CH.CF = . 2 c +) Ta có:
MBC = MAC ( cùng chắn cung MC)
MAC = CBE ( cùng phụ BCA )
Nên MBC = CBE ⇒ BC là phân giác MBE 0,25
* ∆ MBH có BC là đường cao đồng thời là đường phân giác nên là tam giác cân tại B
⇒ BC đồng thời là đường trung tuyến ứng với cạnh MH.
⇒ D là trung điểm của MH. ⇒ DM = DH. 0,25 *Ta có AM AD + DM = = 1 DM + (*) AD AD AD S DH DM
∆ BHC và ∆ ABC có chung đáy BC nên ta có BHC = = (**) 0,25 S AD AD ABC T AM S ừ (*) và (**) suy ra : = 1 BHC + (1) AD S 0,25 ABC
Chứng minh tương tự đẳng thức (1) ta được: BN CK S = 1 SAHC + (2) và = 1 AHB + (3) 0,25 BE S CF S ABC ABC
Công (1) (2) và (3) theo vế ta được : AM BN CK + + = 1 S S S S BHC + + 1 AHC + + 1 AHB + = 3 ABC + = 3 + 1 = 4 0,25 AD BE CF S S S S ABC ABC ABC ABC 4 (3đ) x M L K N C I D
+) D IND vuông tại I có IN = ID (gt) Þ D IND vuông cân t ại I 0
IND = IDN = 45
* Chứng minh tương tự ta được D IMC vuông cân tại I 0
ICM = IMC = 45 D LCD có · · 0 LCD = LDC = 45 Þ D LCD vuông cân t ại L Þ DL^ MC 0.5 Mà MI ^ CD (gt)
Þ DL và MI là hai đường cao của D CDM cắt nhau tại N Þ N là trực tâm D CDM Þ CN^ MD hay CK^ MD 0,5 D CNI và D MNK có: · · 0 CIN = MKN = 90 · · INC = KNM (đđ) CN NI
Þ D CNI đồng dạngD MNK (g-g)Þ = MN NK Þ 0,5 CN.NK = MN.NI
Ta có: MN.NI = (MI – NI).NI = ( CI – ID).ID = (CD – ID – ID).ID 0,5
Đặt ID = x; x > 0 ta được: 2 æ 3ö 9 9 0,5
MN.NI = (6 – 2x).x = 6.x – 2x2 - ç ÷ = 2 x - ç ÷ + £ ç è 2÷ ø 2 2 3 D
ấu “ = “ xảy ra khi x = (TMĐK x > 0) 2 9 3
Vậy CN. NK có giá trị lớn nhất là khi ID = cm. 0,5 2 2 5 Ta có: xy + 2x = 27 – 3y (3đ) Û xy + 2x + 3y = 27 0,5
x ( y + 2) + 3( y + 2) = 33 Û (x + 3)(y + 2) = 33 0,25 ìï x + 3= 1 ìï x + 3= 33 ìï x + 3= 3 ìï x + 3= 11 Û ïí ho ï ï ï ặcí hoặcí hoặcí ï y + 2 = 33 ïî ï y + 2 = 1 ïî ï y + 2 = 11 ïî ï y + 2 = 3 ïî 1,0 do x > 0, y > 0. ìï x = - 2 ìï x = 30 ìï x = 0 ìï x = 8 Û ïí (lo ï ï ï ại)hoặcí (loại)hoặcí (loại)hoặcí (tđk) 1,0 ï y = 31 ïî ï y = - 1 ïî ï y = 9 ïî ï y = 1 ïî
Vậy cặp số nguyên dương cần tìm là (x; y) = (8;1) 0,25
(Nếu HS trình bày bài gii bng cách khác đúng thì chm theo thang đim tương đương)
S GIÁO DC & ĐÀO TO
K THI CHN HC SINH GII TNH LP 9 THCS NGH AN
NĂM HC 2010 - 2011
ĐỀ CHÍNH THC
Môn thi: TOÁN - BNG A
Thời gian: 150 phút (không kể thời gian giao đề) Câu 1 (4,0 điểm).
a) Cho các số nguyên a1, a2, a3, ... , an. Đặt S = 3 3 3 a + a + ... + a 1 2 n và P = a + a + ... + a . 1 2 n
Chứng minh rằng: S chia hết cho 6 khi và chỉ khi P chia hết cho 6. b) Cho A = 6 4 3 2
n − n + 2n + 2n (với n ∈ N, n > 1). Chứng minh A không phải là số chính phương. Câu 2 (4,5 điểm). a) Giải phương trình: 3 2 10 x + 1 = 3x + 6  1 x + = 3  y   1
b) Giải hệ phương trình: y + =  3 z   1 z + = 3  x  Câu 3 (4,5 điểm). 1 1 1
a) Cho x > 0, y > 0, z > 0 và + + = 4 . x y z 1 1 1 Chứng minh rằng: + + ≤ 1 2x+y+z x + 2y + z x + y + 2z
b) Cho x > 0, y > 0, z > 0 thỏa mãn 2011 2011 2011 x + y + z = 3 .
Tìm giá trị lớn nhất của biểu thức: 2 2 2 M = x + y + z Câu 4 (4,5 điểm).
Cho tam giác ABC có ba góc nhọn nội tiếp đường tròn (O), H là trực tâm của tam giác.
Gọi M là một điểm trên cung BC không chứa điểm A. (M không trùng với B và C). Gọi N và P
lần lượt là điểm đối xứng của M qua các đường thẳng AB và AC.
a) Chứng minh ba điểm N, H, P thẳng hàng. 1 1 b) Khi 0
BOC = 120 , xác định vị trí của điểm M để +
đạt giá trị nhỏ nhất. MB MC Câu 5 (2,5 điểm).
Cho tam giác ABC nội tiếp đường tròn tâm O, một điểm I chuyển động trên cung BC
không chứa điểm A (I không trùng với B và C). Đường thẳng vuông góc với IB tại I cắt đường
thẳng AC tại E, đường thẳng vuông góc với IC tại I cắt đường thẳng AB tại F. Chứng minh
rằng đường thẳng EF luôn đi qua một điểm cố định.
- - - Hết - - -
H và tên thí sinh:................................................................................ S báo danh: .....................................
S GD&ĐT NGH AN
K THI CHN HC SINH GII TNH LP 9 THCS
NĂM HC 2010 - 2011
ĐÁP ÁN ĐỀ CHÍNH THC
Môn: TOÁN - Bng A
-------------------------------------------- Câu: Ni dung 1. 3 V − = − + ới a ∈ Z thì a a (a 1)a(a
1) là tích 3 số tự nhiên liên tiếp nên chia hết cho 2 và 3. Mà (2.3)=1 3 ⇒ a − aM6 3 3 3 ⇒
S − P = (a − a ) + (a − a ) + ... + (a − a )M6 1 1 2 2 n n Vậy SM6 ⇔ PM6 6 4 3 2 2 2 2
n − n + 2n + 2n = n (n + 1) .(n − 2n + 2) 2 2 2 v − + = − + − ới n ∈ N , n > 1 thì n 2n 2 (n 1) 1 > (n 1) 2 2
và n − 2n + 2 = n − 2(n − 1) < 2 n 2 V − ậy (n 1) < 2 n − 2n + 2 < 2 n 2
⇒ n − 2n + 2 không là số chính phương ⇒ đpcm 2. 3 2 10 x + 1 = 3(x + 2) 2 2
⇔ 10 (x + 1)(x − x + 1) = 3(x + 2) điều kiện x ≥ −1 ≥ Đặt x + 1 = a (a 0) 2 x − x + 1 = b (b>0) Ta có: 2 2 10ab = 3a + 3b a = 3b
⇔ (a − 3b)(3a-b) = 0 ⇔  = b 3a Trường hợp1: a = 3b 2
Ta có: x + 1 = 3 x − x + 1 (1) 2 ⇔ 9x − 9x+9=x+1 2 ⇔ 9x − 10x+8 = 0 '
∆ = 25 − 9.8< 0 ⇒ phương trình (1) vô nghiệm Trường hợp 2: b = 3a 2 Ta có: 3 x + 1 = x − x + 1 2 ⇔ 9(x + 1) = x − x + 1 x = 5 + 33 (TM) 1 ⇔  = − 2 ⇔ x − 10x-8 = 0 x 5 33 (TM) 2
Vậy phương trình có 2 nghiệm x = 5 ± 33  1 x + = 3  y   1 y + =  3 z   1 z + = 3  x  3x-1 ⇒ z = ⇒ Từ (3) x thay vào (2) 3xy+3 = 8x+y (4) ⇒ T + = ⇔ ừ (1) xy 1 3y 3xy+3 = 9y (5) ⇒ ⇒ T = ừ (4) và (5) 8x+y = 9y x y
Chứng minh tương tự : y = z ⇒ T = = ừ đó x y z 1 2 ⇒ x + = 3 ⇒ x − 3x+1 = 0 Thay vào (1) x 3 ± 5 ⇒ x = 2 3 ± 5 x = y = z = ⇒ hệ có 2 nghiệm 2 3. 1 1 4 + ≥ Áp d + ụng bất đẳng thức x y x y (với x,y > 0) 1 1 1 1 1 1 1 ≤ ( + ) ≤ + Ta có: 2x+y+z 4 2x y + z ; y + z 4y 4z 1 1 1 1 1 ≤ ( + + ) Suy ra: 2x+y+z 4 2x 4y 4z (1) 1 1 1 1 1 ≤ ( + + ) Tương tự: x+2y+z 4 4x 2y 4z (2) 1 1 1 1 1 ≤ ( + + ) x+y+2z 4 4x 4y 2z (3) 1 1 1 1 1 1 1 ⇒ + + ≤ ( + + ) Từ (1),(2),(3) 2x+y+z x+2y+z x+y+2z 4 x y z 1 1 1 ⇒ + + ≤ 1 2x+y+z x+2y+z x+y+2z 3 ⇔ x = y = z = Dấu "=" xảy ra 4 2011 2011
Áp dụng bất đẳng thức CôSy cho x ,x và 2009 số 1 ta có: 2011 2011 2011 2 2011 x + x + 1 + 1 + ... + 1 ≥ 2011 (x ) 2009 2011 2 ⇒ 2x + 2009 ≥ 2011x (1) 2011 2 T 2y + 2009 ≥ 2011y ương tự: (2) 2011 2 2z + 2009 ≥ 2011z (3) 2011 2011 2011 2(x + y + z ) + 3.2009 2 2 2 ⇒ x + y + z ≤ Từ (1), (2), (3) 2011 2 2 2 ⇒ x + y + z ≤ 3
Giá trị lớn nhất của M là 3 khi và chỉ khi x = y = z = 1 4. A I E P O H N B F C M
Gọi giao điểm của BH với AC là E
AH với BC là F, CH với AB là I
⇒ HECF là tứ giác nội tiếp. ⇒ AHE = ACB (1)
Mà ACB = AMB ( góc nội tiếp cùng chắn một cung)
Ta có: AMB = ANB (Do M, N đối xứng AB) (2)
Từ (1), (2) ⇒ AHBN là tứ giác nội tiếp ⇒ NAB = NHB (*)
Mà NAB = MAB (Do M, N đối xứng qua AB (**) Từ (*), (**) ⇒ NHB = BAM
Chứng minh tương tự: PHC = MAC
⇒ NHB + PHC = BAM + MAC = BAC Mà 0 BAC + IHE = 180 0
⇒ NHB + PHC + BHC = 180 ( vì IHE = BHC ) ⇒ N, H, P thẳng hàng
Gọi J là điểm chính giữa của cung lớn BC 0 BOC = 120 ⇒ B ∆ JC đều
Trên đoạn JM lấy K sao cho MK = MB ⇒ J ∆ KB = C ∆ MB J O K C B M ⇒ BM + MC = JM 1 1 4 + ≥ BM MC BM + MC 1 1 4 ⇒ + ≥ BM MC JM
JM lớn nhất ⇔ JM là đường kính (O) lúc đó M là điểm chính giữa của cung nhỏ BC. 1 1 + Vậy BM
MC nhỏ nhất ⇔ M là điểm chính giữa cung nhỏ BC 5. + Khi 0 BAC = 90 ⇒ 0 BIC = 90 .
⇒ F trùng với B, E trùng với C lúc đó EF là đường kính.
⇒ EF đi qua điểm O cố định. B F O K I A E C
+ Khi BAC < 900 ⇒ BIC > 900.
Gọi K là điểm đối xứng của I qua EF.
⇒ EIF = EAF (cùng bù BIC )
EKF = EIF (Do I và K đối xứng qua EF) ⇒ EKF = EAF ⇒ AKFE nội tiếp
⇒ KAB = KEF (cùng chắn KF ) (1)
IEF = KEF (Do K và I đối xứng qua EF) (2)
IEF = BIK ( cùng phụ KIE ) (3)
Từ (1), (2), (3) ⇒ KAB = BIK
⇒ AKBI là tứ giác nội tiếp ⇒ K ∈(O)
Mà EF là đường trung trực của KI ⇒ E, O, F thẳng hàng.
+ Khi BAC > 900 ⇒ BIC < 900 chứng minh tương tự.
Vậy đường thẳng EF luôn đi qua điểm O cố định.
- - - Hết - - -
S GIÁO DC VÀ ĐÀO TO
K THI CHN HC SINH GII LP 9
THÀNH PH ĐÀ NNG
NĂM HC 2010-2011
ĐỀ CHÍNH THC Môn thi: TOÁN
Thời gian: 150 phút (không tính thời gian giao đề)
Bài 1. (2,0 đim) 2
a +1 a a −1 a − a a + a −1 Cho biểu thức: M = + + với a > 0, a ≠ 1. a a − a a − a a
a) Chứng minh rằng M > 4. 6
b) Với những giá trị nào của a thì biểu thức N = nhận giá trị nguyên? M
Bài 2. (2,0 đim)
a) Cho các hàm số bậc nhất: y = 0,5x + 3, y = 6 − x và y = mx có đồ thị lần
lượt là các đường thẳng (d1), (d2) và (∆m). Với những giá trị nào của tham số m thì
đường thẳng (∆m) cắt hai đường thẳng (d1) và (d2) lần lượt tại hai điểm A và B sao
cho điểm A có hoành độ âm còn điểm B có hoành độ dương?
b) Trên mặt phẳng tọa độ Oxy, cho M và N là hai điểm phân biệt, di động lần
lượt trên trục hoành và trên trục tung sao cho đường thẳng MN luôn đi qua điểm cố
định I(1 ; 2) . Tìm hệ thức liên hệ giữa hoành độ của M và tung độ của N; từ đó, suy 1 1
ra giá trị nhỏ nhất của biểu thức Q = + . 2 2 OM ON
Bài 3. (2,0 đim) 1  7 + x 2 = y 2011 xy
a) Giải hệ phương trình:  − x 2 =  y 3x . y 1
b) Tìm tất cả các giá trị của x, y, z sao cho: x + y − z + z − x = (y + 3). 2
Bài 4. (3,0 đim)
Cho đường tròn (C ) với tâm O và đường kính AB cố định. Gọi M là điểm di động
trên (C ) sao cho M không trùng với các điểm A và B. Lấy C là điểm đối xứng của O
qua A. Đường thẳng vuông góc với AB tại C cắt đường thẳng AM tại N. Đường
thẳng BN cắt đường tròn (C ) tại điểm thứ hai là E. Các đường thẳng BM và CN cắt nhau tại F.
a) Chứng minh rằng các điểm A, E, F thẳng hàng.
b) Chứng minh rằng tích AM⋅AN không đổi.
c) Chứng minh rằng A là trọng tâm của tam giác BNF khi và chỉ khi NF ngắn nhất.
Bài 5. (1,0 đim)
Tìm ba chữ số tận cùng của tích của mười hai số nguyên dương đầu tiên. ---HẾT---
Họ và tên thí sinh: ................................................. Số báo danh: ........................
Chữ ký của giám thị 1: ............................. Chữ ký của giám thị 2: ...........................
S GIÁO DC VÀ ĐÀO TO
KÌ THI CHN SINH HC SINH GII LP 9
THÀNH PH ĐÀ NNG
NĂM HC 2010-2011 Môn thi: TOÁN
HƯỚNG DN CHM MÔN TOÁN LP 9
Dưới đây là sơ lược biu đim ca đề thi Hc sinh gii lp 9. Các Giám kho tho lun
thng nht thêm chi tiết li gii cũng như thang đim ca biu đim đã trình bày. T chm có th
phân chia nh
thang đim đến 0,25 đim cho tng ý ca đề thi. Tuy nhiên, đim tng bài, tng câu
không
được thay đổi. Ni dung tho lun và đã thng nht khi chm được ghi vào biên bn c th để
vi
c chm phúc kho sau này được thng nht và chính xác.
Hc sinh có li gii khác đúng, chính xác nhưng phi nm trong chương trình được hc thì
bài làm đúng đến ý nào giám kho cho đim ý đó.
Vic làm tròn s đim bài kim tra được thc hin theo quy định ca B Giáo dc và Đào
to ti Quyết định s 40/2006/BGD-ĐT. BÀI-Ý
ĐỀ -ĐÁP ÁN ĐIM 2
a +1 a a −1 a − a a + a −1 Cho biểu thức: M = + + với a > 0, a ≠ 1. a a − a a − a a
Bài 1 a) Chứng minh rằng M > 4. 6
b) Với những giá trị nào của a thì biểu thức N = nhận giá trị nguyên. M 2,00 a a −1 ( a −1)(a + a +1) a + a +1 Do a > 0, a ≠ 1 nên: = = và a − a a ( a −1) a 0,25 2
a − a a + a −1 (a +1)(a −1) − a (a −1) (a −1)(a − a +1) −a + a −1 = = = a − a a a (1− a) a (1− a) a 0,25 1.a a +1 (1,25đ) ⇒ M = + 2 a 0,25 Do a > 0; a ≠ 1 nên: 2
( a −1) > 0 ⇔ a +1 > 2 a 0,25 2 a ⇒ M > + 2 = 4 a 0,25 6 3 Ta có 0 < N = <
do đó N chỉ có thể nhận được một giá trị nguyên là 1 M 2 0,25 6 a 1.b Mà N = 1 ⇔ = 1 ⇔ a − 4 a +1 = 0 ⇔ 2 ( a − 2) = 3 (0,75đ) a +1+ 2 a
⇔ a = 2 + 3 hay a = 2 − 3 (phù hợp) 0,25 Vậy, N nguyên ⇔ 2 a = (2 ± 3) 0,25
a) Cho các hàm số bậc nhất: y = 0,5x + 3 , y = 6 − x và y = mx có đồ thị lần lượt
là các đường thẳng (d1), (d2) và (∆m). Với những giá trị nào của tham số m thì đường
thẳng (∆m) cắt hai đường thẳng (d1) và (d2) lần lượt tại hai điểm A và B sao cho điểm
Bài 2 A có hoành độ âm còn điểm B có hoành độ dương?
b) Trên mặt phẳng tọa độ Oxy, cho M và N là hai điểm phân biệt, di động lần lượt
trên trục hoành và trên trục tung sao cho đường thẳng MN luôn đi qua điểm cố định
I(1 ; 2) . Tìm hệ thức liên hệ giữa hoành độ của M và tung độ của N; từ đó, suy ra giá 2,00 1 1
trị nhỏ nhất của biểu thức Q = + . 2 2 OM ON
Điều kiện để (∆m) là đồ thị hàm số bậc nhất là m ≠ 0 0,25
Phương trình hoành độ giao điểm của (d1) và (∆m) là:
0,5x + 3 = mx ⇔ (m − 0,5)x = 3 2.a
Điều kiên để phương trình này có nghiệm âm là m − 0, 5 < 0 hay m < 0,5 0,25
(0,75đ) Phương trình hoành độ giao điểm của (d2) và (∆m) là: 6 − x = mx ⇔ (m +1)x = 6
Điều kiên để phương trình này có nghiệm dương là m +1 > 0 hay m > −1
Vậy điều kiện cần tìm là: −1 < m < 0,5; m ≠ 0 0,25
Đặt m = xM và n = yN ⇒ m⋅n ≠ 0 và m ≠ 1 (*)
Nên đường thẳng qua ba điểm M, I, N có dạng: y = ax+b 0,25 0 = am + b  ⇒ 2 = a + 
b ⇒ hệ thức liên hệ giữa m và n là 2m + n = mn n =  b 0,25 1 2 2.b
Chia hai vế cho m⋅n ≠ 0 ta được: + = 1 (**) m n (1,25đ) 2 2  1 2  1 4 4  1 1   2 1  ⇒ 1 = + = + + = 5 + − −   2 2  2 2     m n  m n mn  m n   m n  0,25 1 1 1 2 1 ⇒ Q = + ≥ ; dấu “=” xảy ra khi =
; kết hợp (**): m = 5, n = 2,5 (thỏa (*)) 2 2 m n 5 m n 0,25 1
Vậy giá trị nhỏ nhất của Q là 5 0,25 1  7 + x 2 =  y 2011 xy
a) Giải hệ phương trình:  (1) Bài 3  − x 2 = y 3x . y
b) Tìm tất cả các giá trị của x, y, z sao cho: 1
x + y − z + z − x = (y + 3) (2) 2 2,0 đ 1  7 2  1 1007  9 + = 2011 = x =   y x    y 9  N 490 ếu xy > 0 thì (1) ⇔ ⇔ ⇔    (phù hợp) 1 2   1 490 9 3  − = = y = y x x 9  1007 0,50 1  7 2 1 −1004 + = − 3.a 2011 =    y x  y 9 (1,25đ) Nếu < xy 0 thì (1) ⇔ ⇔ ⇒ xy >   0 (loại) 1 2  1 1031 − = 3 = − y x x 18 0,25
Nếu xy = 0 thì (1) ⇔ x = y = 0 (nhận). 0,25  9 9 
KL: Hệ có đúng 2 nghiệm là (0;0) và  ;   490 1007  0,25 3.b
Điều kiện x ≥ 0; y − z ≥ 0; z − x ≥ 0 ⇔ y ≥ z ≥ x ≥ 0 0,25
(0,75đ) (2) ⇔ 2 x + 2 y − z + 2 z − x = x + y − z + z − x +3 ⇔ 2 2 2
( x −1) + ( y − z −1) + ( z − x −1) = 0 0,25  x =1 x = 1   ⇔ y − z =  1 ⇔ y = 
3 (thỏa điều kiện)   z − x =  1 z =  2  0,25
Cho đường tròn (C ) với tâm O và đường kính F
AB cố định. Gọi M là điểm di động trên (C )
sao cho M không trùng với các điểm A và B. M
Lấy C là điểm đối xứng của O qua A. Đường
thẳng vuông góc với AB tại C cắt đường thẳng
AM tại N. Đường thẳng BN cắt đường tròn (C ) C A O B
Bài 4 tại điểm thứ hai là E. Các đường thẳng BM và CN cắt nhau tại F.
a) Chứng minh rằng các điểm A, E, F thẳng hàng. (C ) E
b) Chứng minh rằng tích AM⋅AN không đổi.
c) Chứng minh rằng A là trọng tâm của tam
giác BNF khi và chỉ khi NF ngắn nhất. N 3,0 đ MN ⊥ BF và BC ⊥ NF 0,25
⇒ A là trực tâm của tam giác BNF 0,25 4.a ⇒ FA ⊥ NB
(1,00đ) Lại có AE ⊥ NB 0,25 Nên A, E, F thẳng hàng 0,25
CAN = MAB , nên hai tam giác ACN và AMB đồng dạng. 0,25 4.b AN AC Suy ra: = (0,75đ) AB AM 0,25 Hay 2
AM ⋅ AN = AB⋅ AC = 2R không đổi (với R là bán kính đường tròn (C )) 0,25 2
Ta có BA = BC nên A là trong tâm tam giác BNF ⇔ C là trung điểm NF (3) 3 0,25
Mặt khác: CAN = CFM , nên hai tam giác CNA và CBF đồng dạng CN AC 4.c ⇒ 2 = ⇒ CN ⋅CF = BC⋅ AC = 3R BC CF 0,25
(1,25đ) Áp dụng bất đẳng thức Cô-si, ta có: NF=CN+CF≥ 2 CN⋅CF = 2R 3 không đổi 0,25
Nên: NF ngắn nhất ⇔ CN =CF ⇔ C là trung điểm NF (4) 0,25
(3) và (4) cho ta: A là trong tâm tam giác BNF ⇔ NF ngắn nhất 0,25
Bài 5 Tìm ba chữ số tận cùng của tích của mười hai số nguyên dương đầu tiên. 0,75
Đặt: S = 1⋅2⋅3⋅4⋅5⋅6⋅7⋅8⋅9⋅10⋅11⋅12
⇒ S = 3⋅4⋅6⋅7⋅8⋅9⋅11⋅12 (1) là một số nguyên 100
⇒ hai chữ số tận cùng của S là 00 0,50
Mặt khác, trong suốt quá trình nhân liên tiếp các thừa số ở vế phải của (1), nếu chỉ để ý (1,00đ) S
đến chữ số tận cùng, ta thấy
có chữ số tận cùng là 6 (vì 3⋅4=12; 2⋅6=12; 2⋅7=14; 100
4⋅8=32; 2⋅9=18; 8⋅11=88; 8⋅12=96) 0,25
Vậy ba chữ số tận cùng của S là 600 0,25 --- Hết ---
Điều kiện x ≥ 0; y − z ≥ 0; z − x ≥ 0 ⇒ y ≥ z ≥ x ≥ 0 0,25 x +1 y − z +1 z − x +1 Theo BĐT Cauchy: x ≤ ; y − z ≤ ; z − x ≤ 2 2 2 1
⇒ VP = x + y − z + z − x ≤ (y + 3) = VT 3.b 2 0,25 (0,75đ)  x =1 x = 1   Do đó y − z =  1 ⇔ y = 
3 thỏa điều kiện   z − x =  1 z =  2  0,25
S GIÁO DC VÀ ĐÀO TO
ĐỀ THI CHN HC SINH GII LP 9
TP. ĐÀ NNG
NĂM HC 2012 – 2013
MÔN THI: TOÁN – LP 9 THCS (Th
Đề thi chính thc
ời gian làm bài 150 phút không kể thời gian giao đề) Bài 1. (2,5 điểm) Cho biểu thức với a/ Rút gọn biểu thức với
b/ Tìm tất cả các giá trị
sao cho P là một số nguyên tố. Bài 2. (2,0 điểm) a/ Tìm x, biết:
b/ Giải hệ phương trình: Bài 3. (2,0 điểm)
a/ Cho hàm số bậc nhất y = ax + b có đồ thị đi qua điểm M(1;4). Biết rằng đồ thị của hàm số đã cho cắt trục Ox tại điểm
P có hoành độ dương và cắt trục Oy tại điểm Q có tung độ dương. Tìm a và b sao cho OP + OQ nhỏ nhất (với O là gốc tọa độ)
b/ Tìm số tự nhiên có 2 chữ số. Biết rằng nếu lấy tổng của 2 chữ số ấy cộng với 3 lần tích của 2 chữ số ấy thì bằng 17. Bài 4. (2,0 điểm)
Cho tam giác ABC. Gọi I là tâm đường tròn nội tiếp tam giác ABC, qua I vẽ đường thẳng vuông góc với đường thẳng
CI, đường thẳng này cắt các cạnh AC, BC lần lượt tại M và N.
a/ Chứng minh rằng hai tam giác IAM và BAI đồng dạng. b/ Chứng minh rằng Bài 5. (1,5 điểm) Cho tam giác ABC có
là góc tù. Vẽ các đường cao CD và BE của tam giác ABC (D nằm trên đường thẳng AB, E
nằm trên đường thẳng AC). Gọi M,N lần lượt là chân đường vuông góc của các điểm B và C trên đường thẳng DE. Biết
rằng là diện tích tam giác ADE,
là diện tích tam giác BEM và
là diện tích tam giác CDN. Tính diện tích tam giác ABC theo .
S GIÁO DC VÀ ĐÀO TO
K THI CHN HC SINH GII CP TNH LP 9 THCS
TNH ĐĂK NÔNG
NĂM HC 2010-2011 Khóa thi ngày: 10/3/2011 Môn thi: TOÁN
ĐỀ CHÍNH THC
Thời gian: 150 phút ( không k thi gian giao đề)
Bài 1:
(4,0 đim)  2 x 2 x 4x  + − x − 3
1) Cho biểu thức A =  − −  : .  
Tìm điều kiện của x để A > 0.  2 − x 2 + x
x − 4  2 x x 2 2) Cho x = 1 1 − 2 +1 −1 2 +1 +1
Tính giá trị của biểu thức: 4 3 2 2011
B = (x x x + 2x −1)
Bài 2:
(4,0 đim) 1) Giải phương trình: 2
x − 3x + 2 + x + 3 = x − 2 2
+ x + 2x − 3 . 2 x + 2y +1= 0 
2) Cho x, y z là nghiệm của hệ phương trình: 2
y + 2z +1 = 0  2 z  + 2x + 1 = 0.
Tính giá trị của biểu thức: 10 3 2011
C = x + y + z .
Bài 3: (4,0 đim)
1) Tìm các cặp số ( a, b) thỏa mãn hệ thức: a + b − 2011 = a + b − 2011 .
2) Tìm tất cả các số tự nhiên n sao cho: n2 – 14n + 38 là một số chính phương.
Bài 4: (5,0 đim)
Cho đường tròn tâm O, hai đường kính ABCD vuông góc với nhau. E là một điểm
nằm trên cung nhỏ AD . Nối CE cắt OA tại M và nối BE cắt OD tại N.
1) Chứng minh: AM .ED = 2OM .EA OM ON 2) Chứng minh tích ⋅
là một hằng số. Từ đó, suy ra giá trị nhỏ nhất của tổng AM DN OM ON +
, khi đó cho biết vị trí của điểm E? AM DN
Bài 5: (3,0 đim) Cho a, ,
b c là ba số thực dương. Chứng minh bất đẳng thức: 3 3 3 2 2 2 2 2 2 a + b + c a + b b + c c + a 9 + + + ≥ . 2 2 2 2abc ab + c bc + a ca + b 2
--------HT--------
S GD&ĐT VĨNH PHÚC
K THI CHN HSG LP 9 NĂM HC 2011-2012 —————— ĐỀ THI MÔN: TOÁN
ĐỀ CHÍNH THC
Thi gian làm bài: 150 phút, không k thi gian giao đề
————————————
Câu 1 (3,0 đim). 3 x 1. Cho f ( x) =
. Hãy tính giá trị của biểu thức sau: 2 1− 3x + 3x  1   2   2010   2011  A = f   + f   + ... + f   + f    2012   2012   2012   2012  x − 2 x x +1 1+ 2x − 2 x
2. Cho biểu thức P = + + 2 x x −1 x x + x + x x x
Tìm tất cả các giá trị của x sao cho giá trị của P là một số nguyên.
Câu 2 (1,5 đim). 3 2
Tìm tất cả các cặp số nguyên dương ( x; y) thỏa mãn ( x + y) = ( x y − 6) .
Câu 3 (1,5 đim).
Cho a, b, c, d là các số thực thỏa mãn điều kiện:
abc + bcd + cda + dab = a + b + c + d + 2012 Chứng minh rằng: ( 2 a + )( 2 b + )( 2 c + )( 2 1 1 1 d + ) 1 ≥ 2012 .
Câu 4 (3,0 đim).
Cho ba đường tròn (O , O và (O) (kí hiệu ( X ) chỉ đường tròn có tâm là điểm X). Giả sử 1 ) ( 2 )
(O , O tiếp xúc ngoài với nhau tại điểm I và (O , O lần lượt tiếp xúc trong với (O) tại 1 ) ( 2 ) 1 ) ( 2 )
M , M . Tiếp tuyến của đường tròn (O tại điểm I cắt đường tròn (O) lần lượt tại các điểm 1 ) 1 2 ,
A A ' . Đường thẳng AM cắt lại đường tròn (O tại điểm N , đường thẳng AM cắt lại đường 1 ) 1 1 2
tròn (O tại điểm N . 2 ) 2
1. Chứng minh rằng tứ giác M N N M nội tiếp và đường thẳng OA vuông góc với đường 1 1 2 2 thẳng N N . 1 2
2. Kẻ đường kính PQ của đường tròn (O) sao cho PQ vuông góc với AI (điểm P nằm trên
cung AM không chứa điểm M ). Chứng minh rằng nếu PM , QM không song song thì các 1 2 1 2
đường thẳng AI , PM QM đồng quy. 1 2
Câu 5 (1,0 đim)
Tất cả các điểm trên mặt phẳng đều được tô màu, mỗi điểm được tô bởi một trong 3 màu xanh,
đỏ, tím. Chứng minh rằng khi đó luôn tồn tại ít nhất một tam giác cân, có 3 đỉnh thuộc các điểm
của mặt phẳng trên mà 3 đỉnh của tam giác đó cùng màu hoặc đôi một khác màu. —Hết—
Cán b coi thi không gii thích gì thêm.
Họ và tên thí sinh:……….………..…….…….….….; Số báo danh……………….
S GD&ĐT VĨNH PHÚC
K THI CHN HSG LP 9 THCS NĂM HC 2011-2012 ———————
HƯỚNG DN CHM MÔN: TOÁN
——————————— I. LƯU Ý CHUNG:
- Hướng dẫn chấm chỉ trình bày một cách giải với những ý cơ bản phải có. Khi chấm bài học sinh
làm theo cách khác nếu đúng và đủ ý thì vẫn cho điểm tối đa.
- Điểm toàn bài tính đến 0,5 và không làm tròn.
- Với bài hình học nếu thí sinh không vẽ hình phần nào thì không cho điểm tương ứng với phần đó. II. ĐÁP ÁN: Câu Ý
Ni dung trình bày Đim 1 1
1,5 đim
Nhận xét. Nếu x + y = 1 thì f ( x) + f ( y) = 1. x (1− x)3 3 0,5
Thật vậy, ta có f ( x) =
f ( y) = f (1− x) = x + (1− x)3 x + (1− x)3 3 3 x (1− x)3 3
suy ra f ( x) + f ( y) = f ( x) + f (1− x) = + = 1. x + (1− x)3 x + (1− x)3 3 3 0,5  1  1
Vậy, nhận xét được chứng minh. Ta có f   = .  2  2
Theo nhận xét trên ta có:   1   2011    2   2010  A =  f   + f    +  f   + f    + ... +   2012 
 2012     2012   2012   0,5   1005   1007   1006   1   f   + f    + f
 = 1005 + f   = 1005,5   2012   2012    2012   2  2
1,5 đim
Điều kiện: x > 0, x ≠ 1 . Khi đó ta có x + 2 0,5
Rút gọn biểu thức ta được P = x + x +1
Ta có Px + ( P − ) 1
x + P − 2 = 0 , ta coi đây là phương trình bậc hai của x .
Nếu P = 0 ⇒ − x − 2 = 0 vô lí, suy ra P ≠ 0 nên để tồn tại x thì phương trình 2 0,5
trên có ∆ = ( P − )
1 − 4P ( P − 2) ≥ 0 4 4
⇔ −3P + 6P +1 ≥ 0 ⇔ P − 2P +1 ≤ ⇔ ( P − )2 2 2 1 ≤ 3 3
Do P nguyên nên ( P − )2 1 bằng 0 hoặc 1 +) Nếu ( P − )2 1
= 0 ⇔ P = 1 ⇔ x = 1 không thỏa mãn.  0,5 P = +) Nếu ( P − )2 2 1 = 1 ⇔
P = 2 ⇔ 2x + x = 0 ⇔ x =  0 không thỏa mãn P = 0
Vậy không có giá trị nào của x thỏa mãn. 2
1,5 đim
Nếu x y + 6 ⇒ x + y > x − ( y + 6) ≥ 1 ⇒ phương trình vô nghiệm. Do đó 0,5
x < y + 6 ⇒ 2 ≤ x + y < y + 6 − x x < 3 ⇒ x ∈{1; 2}
Với x = 1 thay vào phương trình ban đầu ta được: ( y + )3 2 = y + ⇔ ( y − )( 2 1 ( 5) 3
y + 5y + 8) = 0 ⇔ y = 3 suy ra phương trình có 0,5
nghiệm ( x ; y) = (1; 3) .
Với x = 2 thay vào phương trình ban đầu ta được: ( y + )3 2 3 2 2
= ( y + 4) ⇔ y + 5 y + 4 y − 8 = 0 phương trình này vô nghiệm do 0,5 y ≥ 1.
Vậy phương trình đã cho có nghiệm ( x ; y) = (1; 3) . 3
1,5 đim Ta có: = ( + + + − − − − )2 2012 abc bcd cda dab a b c d 0,5
= ((ab − )(c + d ) + (cd − )(a + b))2 1 1
(ab )2 (a b)2 (cd )2 (c d )2 1 1  ≤ − + + − + +     0,5 = ( 2 2 2 2
a b + a + b + )( 2 2 2 2
c d + c + d + ) = ( 2 a + )( 2 b + )( 2 c + )( 2 1 1 1 1 1 d + ) 1 0,5 Suy ra ( 2 a + )( 2 b + )( 2 c + )( 2 1 1 1 d + ) 1 ≥ 2012 4 P A N1 O O M 1 1 N2 I O2 Q M A' 2 S 1
2,0 đim +) Ta có 2
AM .AN = AM .AN = AI A
N N đồng dạng với AM M 1 1 2 2 1 2 2 1 0,5 suy ra 0
AN N = AM M M N N + AM M = 180 hay tứ giác M N N M 1 2 2 1 1 1 2 2 1 1 1 2 2 0,5 nội tiếp. 1
+) Ta có AN N = AM M =
AOM và tam giác AOM cân tại O nên 1 2 2 1 1 2 1 0 0,5 180 − AOM1 M AO = 1 2 Do đó ta được 0
AN N + M AO = 90 ⇒ OA N N . 0,5 1 2 1 1 2 2
1,0 đim
Gọi S là giao điểm của PM QM . 1 2
Ta có O, O , M thẳng hàng và O I song song với OP IO M = POM 2 2 2 2 2 2
(1). Mặt khác tam giác O IM cân tại O , tam giác OPM cân tại O và kết 2 2 2 2 0,5
hợp với (1) ta được O IM = OPM suy ra P, I , M thẳng hàng. Tương tự ta 2 2 2 2
Q, I , M thẳng hàng. 1
Do PQ là đường kính của đường tròn (O) suy ra 0
PM Q = PM Q = 90 1 2 0,5
I là trực tâm của tam giác SPQ suy ra AI đi qua S hay ba đường thẳng
AI , PM , QM đồng quy. 1 2 5
1,0 đim B A C E D
Xét ngũ giác đều ABCDE, ta nhận thấy ba đỉnh bất kì của ngũ giác luôn tạo thành một tam giác cân.
Do đó khi tô 5 đỉnh A, B, C, D, E bằng 3 màu xanh, đỏ và tím sẽ xảy ra hai khả 0,5 năng sau:
+) Nếu tô 5 đỉnh A, B, C, D, E bởi đủ ba loại màu đã cho thì tồn tại 3 đỉnh có
màu khác nhau và tạo thành một tam giác cân.
+) Nếu tô 5 đỉnh A, B, C, D, E bởi nhiều nhất 2 màu thì có ít nhất 3 đỉnh cùng
màu và tạo thành một tam giác cân. 0,5
Vậy, trong mọi trường hợp luôn tồn tại ít nhất một tam giác cân, có 3 đỉnh
được tô bởi cùng một màu hoặc đôi một khác màu.
UBND tỉnh Thái Nguyên Cộng hoà xã hội chủ nghĩa việt nam
S Giáo dc & Đào to Độc lp - T do - Hnh phúc
K THI CHN HC SINH GII CP TNH LP 9 THCS Tháng 3 / 2012 MôN: Toán
(Thi gian làm bài: 150 phút không k thi gian giao đề) Đề chính thức
Bài 1. Chứng minh rằng tổng bình phương của 5 số nguyên liên tiếp không là số chính phương.
Bài 2. Giải phương trình và hệ phương trình sau:
a, 3 2 − x + x −1 = 1 xy + 2 z = 2  2 b,  yz + x = 2  2 xz + y = 2
Bài 3. Cho ∆ ABC có 3 góc đều nhọn. Gọi O là tâm đường tròn ngoại tiếp ∆ ABC; R, r
theo thứ tự là độ dài bán kính đường tròn ngoại tiếp và đường tròn nội tiếp
∆ ABC; M, N, P lần lượt là hình chiếu vuông góc của O trên AB, BC và AC.
a, Chứng minh: BN . OM + BM . ON = BO . MN
b, Đặt ON = d1 ; OM = d2 ; OP = d3 .
Tính R + r theo d1 , d2 , d3 ?
Bài 4. Lấy một số tự nhiên có 2 chữ số chia cho số có 2 chữ số viết theo thứ tự ngược
lại thì được thương là 4 và dư 15. Nếu lấy số đó trừ đi 9 thì được một số bằng
tổng bình phương của 2 chữ số tạo thành số đó. Tìm số tự nhiên ấy?
-------------- Hết ---------------
Họ tờn thớ sinh:..........................................................Số bỏo danh:......................... áp án Đ1
UBND tỉnh Thái Nguyên Cộng hoà xã hội chủ nghĩa việt nam
S Giáo dc & Đào to Độc lp - T do - Hnh phúc
K THI CHN HC SINH GII CP TNH LP 9 THCS Tháng 3 / 2012
hng dn chm toán 9 Bài 1: 3,5 điểm
C1: Gọi 5 số nguyờn liờn tiếp là n-2, n-1, n, n+1, n+2 với n nguyờn, dễ thấy tổng cỏc
bỡnh phương của 5 số đó là 5(n2 + 2) chia hết cho 5 nhưng không chia hết cho 25 nên
không thể là số chính phương.
C2: Xét tính chẵn lẻ của 5 số nguyên liên tiếp đó. Bài 2: a. 3,5 điểm Đặt a = 3 2 − x b = x −1 ≥ 0  3 2 a + b = 1 Ta có :  (I) a  + b = 1 ⇒ a3 + a2 - 2a = 0 ⇔ a ( a2 + a -2) = 0 a = 0 ⇔   2 a + a − 2 = 0
Hệ ( I ) có ba nghiệm : ( 0 ; 1) ; ( 1 ; 0) ; ( -2 ; 3)
nên phương trình đã cho có nghiệm : 2 ; 1 ; 10 b, 3,5 điểm  + 2 xy z = 2 (1)    + 2 yz x = 2 (2)   + 2 xz y = 2 (3) 
Từ (1) ; (2) ta có : (x – z)(x – y + z) = 0 (4)
Từ (2) và (3) ta có: ( y - x)(x + y –z) = 0 (5)
Từ (3) ; (4) ; (5) ta có hệ : (  x − z)(x − y + z) = 0
(y − x)(x + y − z) = 0   2 xz + y = 2
Để giải hệ trên ta giải 4 hệ   x − z = 0 x − z = 0   y − x = 0 (A) x + y − z = 0 (B)   2 2 xz + y = 2 xz + y = 2  y − x = 0 x − y+ z = 0   x − y + z = 0 (C) x + y − z = 0 (D)   2 2 xz + y = 2 xz + y = 2
Giải 4 hệ trên ta được 8 bộ nghiệm của hệ phương trình :
(1; 1; 1) ; ( -1;-1; -1 ) ; ( 2; 0 ; 2 ) ; (− 2; 0 ;− 2 ) ( 2; 2 ;0) ; (− 2; − 2 0
; ) ; (0 ; 2; 2 ) ; ( 0 ;− 2 ;− 2) Bài 3: A 6 điểm a, Ta có BMO = BNO = 900
=> OMBN là tứ giác nội tiếp M P d2
Trên BO lấy E sao cho BME = OMN 0 d3 O => ∆ BME ∆ NMO E d1 BM NM B C => = N BE NO => BM . NO = BE . NM
Chứng minh tương tự BN. OM = OE .MN
Cộng theo từng vế BM .ON +BN . ON = MN . BO
b. Đặt a , b , c là độ dài các cạnh BC , AC , AB của ∆ ABC a c b theo câu a ta có d1. + d = R . 2 2 2 2
áp dụng câu a đối với các tứ giác OMAP , ONCD ta có b c a d1. + d 2 3. 2 = R. 2 a b c d3 . + d = R. 2 2 . 2 2 Cộng theo từng vế : R 1 . ( a+b+c) = . ( d 2 2
1b + d2b + d3c + d3a + d1a + d2c) r 1
mặt khác SABC = . ( a+b +c ) = .( d 2 2 1c + d3b + d2a )
Do đó ( R + r )( a+b+c) = ( a+b+c)( d1+d2+d3) hay R + r = d1 + d2 + d3 Bài 4: 3,5 điểm Gọi số phải tỡm là (a , b N; 1 a, b 9)
ab = 4.ba +15(1) Ta cú hệ  2 2
ab − 9 = a + b (2)
C1 : Từ (1) ta thấy nếu
=> a = b = 9 khụng thỏamón (1) và (2)
Vậy b = 1 thay b = 1 vào (2) ta được: – 9 = a2 + 1 10a + 1 – 9 = a2 + 1 a2 – 10a + 9 = 0 a1 = 1; a2 = 9 (*) a = 1 => a = b loại (*) a = 9 => = 91 thỏa món (1) 91 = 4 * 19 + 15
Vậy: Số phải tỡm là 91
C2: Từ hệ trờn cú thể dựng PP thế để giải. Rút 1 ẩn từ PT (1) thế vào PT (2) ta sẽ
được một PT bậc 2. Giải PT bậc 2 đó sẽ tỡm được nghiệm.
Chỳ ý: - Học sinh làm theo cách khác mà đúng vẫn cho điểm tối đa.
- GK có thể bàn để thống nhất điểm cho từng phần nhỏ của mỗi bài.
------------------------------------------------------------------------------ SỞ GD&ĐT PHÚ THỌ
ĐỀ THI CHN HC SINH GII CP TNH NĂM HỌC 2012 - 2013
ĐỀ CHÍNH THC
MÔN: TOÁN - LP 9
Thi gian làm bài 150 phút không k thi gian giao đề
Câu1( 3,0 đim)
1) Giải phương trình nghiệm nguyên 2
8x − 3xy − 5y = 25
2)Tìm tất cả số nguyên dương n sao cho A= .4n 3n n + M7
Câu 2( 4,0 đim) 1) Rút g + − − ọn biểu thức: A= 2 10 30 2 2 6 2 : 2 10 − 2 2 3 −1 2 2 2 x yz y zx 2) Cho các s z xy
ố thực dương a,b,c,x,y,z khác 0 thoả mãn . = = a b c 2 2 2 Ch a bc b ca c ab ứng minh rằng = = x y z
Câu 3( 4,0 đim) 1) Cho phương trình: 2
x − 6x − m = 0 (Với m là tham số). Tìm m để phương trình đã
cho có hai nghiệm x1 và x2 thoả mãn 2 2 x x = 12 1 2 3 3 3 8  x y + 27 = 18 2) Gi y ải hệ phương trình:  2 2
4x y + 6x = y
Câu 4( 7,0 đim)
1) Cho đường tròn (O) đường kính BD=2R, dây cung AC của đường tròn (O) thay đổi
nhưng luôn vuông góc và cắt BD tại H. Gọi P,Q,R,S lần lượt là chân các đường vuông
góc hạ từ H xuống AB,AD,CD,CB. a) CMR: 2 2 2 2
HA + HB + HC + HD không đổi. b) CMR : P R
Q S là tứ giác nội tiếp.
2) Cho hình vuông ABCD và MNPQ có bốn đỉnh M,N,P,Q lần lượt thuộc các cạnh AB,BC,CD,DA c
MN + NP + PQ + QM
ủa hình vuông. CMR: SAC ABCD 4
Câu 5( 2,0 đim)
Cho a,b,c là các số thực dương. CMR: ab bc ca a + b + c + + ≤
a + 3b + 2c b + 3c + 2a
c + 3a + 2b 6 ---Hêt—
Hướng dn Câu1.1) 2
8x − 3xy − 5y = 25 2 8x2 − 25 25 ⇔ y 3 ( x + )
5 = 8x − 25 ⇔ y =
⇔ 9 y = 24x − 40 − ∈ Z 3x + 5 3x + 5
Khi 3x+5 là ước 25 từ đó tìm được ( ; x y) ∈{( 1 − ; 0 3 − ) 1 ;(− ; 2 7 − );( ; 0 − } ) 5
( cách khac nhân 2 vế với 9 đưavề tích)
1.2) Với n chẵn n=2k thì 2 2 2 7 −1 k k k k k t A = 2k 4 . + 3 = (2k + ) 1 4 . + 1
( 6 − 9 )M7 ⇒ 2k + 1M7 ⇒ k = ⇒ n = 1 t
4 −1 = 14m + 6(m N ) 2 Với n lẻ n=2k+1 A = (2k 2k +1 2k + ) 1 4 . + + 3 1 = 2k 2k+1 2k +1 2k 4 . + (4 + +
3 1)M7 ⇒ 2k M7 ⇒ k = t
7 ⇒ n = 14m + ( 1 m N )
Vậy n = 14m + 6 hoặc n = 14m +1 ( với mọi n∈ N ) thì A chia hết cho 7 2 10 + 30 − 2 2 − 6 2 Câu2.1) : = 2 10 − 2 2 3 −1 2 2( 5 − ) 1 + 6( 5 − ) 1 3 −1 2 + 3 3 −1 4 + 2 3 3 −1 3 +1 3 −1 1 . = . = . = . = 2 2( 5 − ) 1 2 2 2 4 2 2 2 2 2 2 2 x yz y zx 2.2) z xy = = a b c 2 2 a b c a bc a bc ⇔ = = ⇔ = = ) 1 ( 2 2 2 4 x yz y xz z xy x − 2 2 2 2 2 2 3 3 2 x yz + y z
y z xy xz + x yz x( 3 3 3
x + y + z − 3xyz) 2 2 − Tuongtu : b ac b ac = = (2) 4 y − 2 2 2 2 2 2 3 3 2 y xz + x z
x z x y yz + xy z y( 3 3 3
x + y + z − 3xyz) 2 2 − Tuongtu : c ab c ab = = ) 3 ( 4 Z − 2 2 2 2 2 2 3 3 2 xyz + x y
x y x z y z + xyz z( 3 3 3
x + y + z − 3xyz)
Từ (1) (2) (3) ta co ĐPCM
Câu 3.1)
Để phương trình có nghiệm /
∆ ≥ 0 ⇔ m ≥ −9 (*) x + x = 6 x + x = 6 x = 4 1 2 1 2 1   
Mặt khác ta phải có x .x = −m ⇔ x .x = −m ⇔ x .x = −m m = 8 TM ĐK (*) 1 2 1 2 1 2 −  2 2   x x = 12 x x = 2 x = 2 1 2 1 2 2  3 3 8x y + 27 = 3 18y
3.2)Giải hệ phương trình   2 4x y + 6x = 2 y
HD y =0 không là nghiệm của hệ chia 2 vế PT(1) cho y3 PT(2) cho y2 Ta có  3 27 8x + = 18 2x = a  3 y   3 a + 3 b = 18 a + b = 3 hệ  Đặt  3 ta có hệ  ⇔  2  = b  2 a b + 2 ab = 3 ab = 1 4 x 6 x 1     + =  y  2 y y     3 − 5 6 3 + 5 6 
Hệ có 2 nghiệm (x, y) ∈     ; ; ;       4 3 + 5  4 3 − 5     Câu 4.1) A Q P D B O H S R C a) theo Pitago 2 2 2
HA + HB = AB ; 2 2 2
HC + HB = BC ; 2 2 2
HC + HD = CD ; 2 2 2
HA + HD = AD ; suy ra đpcm
b)Tứ giác HPBS nội tiếp ⇒ H
PS = ∠HBS = DBC
Tứ giác HPAQ là hình chữ nhật ⇒ ∠HPQ = ∠HAQ = CAD = CBD
Do đó ∠SPQ = ∠HPS + HPQ = ∠ 2 CBC
Tương tự ∠SQR = ∠ 2 BDC Do đó 0 0
DBC + ∠BDC = 180 ⇔ ∠SPQ + ∠SRQ = 180 nên tứ giác PQRS nội tiếp ( đ/lí đảo) 4.2) M A B I N K Q L C D P
Cách 1 Gọi T, K, L là trung điểm MQ, MP, NP theo t/c đường trung bình và trung tuyến tam
giác vuông ta có MN + NP + PQ + QM = 2(KL + CL + IK + AI) ≥ 2AC từ đó suy ra đpcm
Cách 2 Ta có theo Pitago 2 2 (BM + BN )2 2 BM + BN
MN = BN + BM ≥ ⇔ MN
( áp dng BĐT Bunhiacoopsky 2 2 T CN + NP DP + DQ AQ + AM ương Tự NP ≥ ; PQ ≥ ; MQ 2 2 2 Nên
BM + NB + NC + CP + PD + DQ + QA + AM 4a
MN + NP + PQ + QM ≥ = = 2a 2 2 2
a 2 (MN + NP + PQ + QM ) = a2 ⇔ dpcm 4
Dấu “=” xảy ra khi MNPQ là hình chữ nhật Câu 5
Cho a,b c>0 .Chứng minh rằng: ab bc ca a + b + c + + ≤
a + 3b + 2c
2a + b + 3c 3a + 2b + c 6
Dự đoán a=b=c tách mẫu để a+c=b+c=2b  1 1 1  1 1  1 1 1 
Tacó áp dụng BĐT (x + y + z) + + ≥ 9 ⇔ ≤    + +   x y z x + y + z 9  x y z ab ab ab  1 1 1  1  ab ab a  = ≤  + +  =  + +  (1)
a + 3b + 2c
(a + c) + (b + c) + 2b
9  a + c b + c 2b  9  a + c b + c 2  Tương tự bc bc bc  1 1 1  1  bc bc b  = ≤  + +  =  + +  (2)
2a + b + 3c
(a + b) + (a + c) + 2c
9  a + c b + c 2b  9  a + b b + c 2  ac ac ac  1 1 1  1  ac ac c  = ≤  + +  =  + +  (2)
3a + 2b + c
(a + b) + (b + c) + 2a
9  a + b b + c 2a  9  a + b b + c 2  Từ (1) (2) (3) 1  ac + bc ab + ac bc + ab
a + b + c a + b + c P ≤  + + +  = 9  a + b b + c a + c 2  6
Dấu “=” xảy ra khi a=b=c
GV Nguyn Minh Sang THCS Lâm Thao-Phú Th
S
GD&ĐT K THI CHN HC SINH GII CP TNH LP 9 THCS
QU
NG BÌNH NĂM HC 2010 - 2011 Môn thi: Toán
ĐỀ THI CHÍNH THC
(Khóa ngày 30 tháng 3 năm 2011)
SỐ BÁO DANH:…………….. Thi gian làm bài: 150 phút (không k thi gian giao đề)
x + 4 x − 4 + x − 4 x − 4
Câu 1:(2.5 đim) Cho biểu thức A = với 4 < x ≤ 8 8 16 1− + 2 x x
a) Rút gọn biểu thức A.
b) Tìm x nguyên để A có giá trị nguyên.
Câu 2:(2.5 đim) Số đo hai cạnh góc vuông của một tam giác là nghiệm của phương trình bậc hai 2
(m − 2)x − 2(m −1)x + m = 0. Xác định m để số đo đường cao ứng 2
với cạnh huyền của tam giác đã cho là 5
Câu 3:(3.0 đim) Cho hai đường tròn (O) và (O’) cắt nhau tại hai điểm A và B. Tiếp
tuyến chung gần B của hai đường tròn lần lượt tiếp xúc (O) và (O’) tại C và D.
Qua A kẻ đường thẳng song song CD cắt (O) và (O’) lần lượt tại M và N. Các
đường thẳng BC, BD lần lượt cắt MN tại P và Q. Các đường thẳng CM, DN cắt
nhau tại E. Chứng minh rằng:
a) Các đường thẳng AE và CD vuông góc nhau. b) Tam giác EPQ cân.
Câu 4:(1.0 đim) Cho x, y, z > 0 thỏa mãn: 2 2 2
x + y + z = 3 . Chứng minh: xy z y x z + + ≥ 3 z x y
Câu 5:
(1.0 đim) Cho a, b, c, d là các số nguyên thỏa mãn : 5 5 5 5
a + b = 4(c + d )
Chứng minh rằng : a + b + c + d chia hết cho 5.
--------------------HẾT----------------------
S
GD&ĐT K THI CHN HC SINH GII CP TNH LP 9 THPT
QU
NG BÌNH NĂM HC 2012- 2013 Môn thi: Toán
ĐỀ THI CHÍNH THC
(Khóa ngày 27 tháng 3 năm 2013)
SỐ BÁO DANH:…………….. Thi gian làm bài: 150 phút (không k thi gian giao đề)
Câu 1:
(2.0 đim) Cho bi x x + x x x − ểu thức: 26 19 2 3 P = − + x + 2 x − 3 x −1 x + 3 a) Rút gọn P.
b) Tìm x để P đạt giá trị nhỏ nhất.
Câu 2:(2.0 đim) Cho phương trình 2
x − 2mx + m − 4 = 0
a) Tìm m để phương trình có hai nghiệm phân biệt x , x + x = 26 1 2 x thỏa mãn 3 3 1 2 m
b) Tìm m nguyên để phương trình có hai nghiệm nguyên.
Câu 3:(3,5 đim)
Cho tam giác ABC đều cố định nội tiếp trong đường tròn (O). Đường thẳng d
thay đổi nhưng luôn đi qua A và cắt cung nhỏ AB tại điểm thứ hai là E (E ≠ A).
Đường thẳng d cắt hai tiếp tại B và C của đường tròn (O) lần lượt tại M và N. MC
cắt BN tại F. Chứng minh rằng:
a) Tam giác CAN đồng dạng với tam giác BMA, tam giác MBC đồng dạng với tam giác BCN.
b) Tứ giác BMEF là tứ giác nội tiếp.
c) Chứng minh đường thẳng EF luôn đi qua một điểm có định khi d thay đổi nhưng luôn đi qua A.
Câu 4:(1,5 đim)
Cho c¸c sè thùc d−¬ng a, b, c tho¶ mn a + b + c =6. Chứng minh rằng: b + c + 5 c + a + 4 a + b + 3 + +
≥ 6 . DÊu ®¼ng thøc x¶y ra khi nµo? 1+ a 2 + b 3 + c
Câu 5:(1,0 đim) Cho n là s 4
ố tự nhiên lớn hơn 1. Chứng minh rằng n n + 4 là hợp số.
--------------------HẾT----------------------
S
GD&ĐT K THI CHN HC SINH GII CP TNH LP 9 THPT
QU
NG BÌNH NĂM HC 2012 - 2013 Môn thi: Toán
(Khóa ngày 27 tháng 3 năm 2013)
HƯỚNG DN CHM
(Đáp án, hướng dn này có 4 trang) yªu cÇu chung
* Đáp án ch trình bày mt li gii cho mi bài. Trong bài làm ca hc sinh yêu cu phi lp
lu
n lô gic cht ch, đầy đủ, chi tiết và rõ ràng.
* Trong mi bài, nếu hc sinh gii sai bước gii trước thì cho đim 0 đối vi nhng bước
gi
i sau có liên quan. câu 3 nếu hc sinh không v hình hoc v hình sai thì cho đim 0.
* Đim thành phn ca mi bài nói chung phân chia đến 0,25 đim. Đối vi đim thành phn
là 0,5
đim thì tu t giám kho thng nht để chiết thành tng 0,25 đim.
* Hc sinh có li gii khác đáp án (nếu đúng) vn cho đim ti đa tu theo mc đim ca tng bài.
* Đim ca toàn bài là tng (không làm tròn s) ca đim tt c các bài. Câu Ni dung Đim 1
1,0 đim
a) ĐK: 0 ≤ x ≠ 1.Ta có: 0,25 x x + 26 x −19 2 x x − 3 P = − +
( x −1)( x + 3) x −1 x + 3
x x + 26 x −19 − 2 x ( x + 3) + ( x − 3)( x −1) = 0,25
( x −1)( x + 3)
x x + 26 x −19 − 2x − 6 x + x − 4 x + 3 0,25 =
( x −1)( x + 3)
x x x +16 x −16 ( x −1)(x +16) x +16 = = = ( 0,25 x −1)( x + 3)
( x −1)( x + 3) x + 3 b) 1,0 đim x +16 25 25 P = = x − 3 + = x + 3 + − 6 0,5 x + 3 x + 3 x + 3 25 ≥ 2 ( x + 3) − 6 = 10 − 6 = 4 0,25 x + 3 Vậy GTNN của P = 4 khi 25 x + 3 = ⇔ x = 4 0,25 x + 3 Trang: 1 - Đáp án Toán 11 2 a) 2
x − 2mx + m − 4 = 0
1,0 đim 2  1  15 2
Ta có: ∆ ' = m m + 4 = m −  + > 0 m ∀  2  4 0,25
Vậy phương trình luôn có 2 nghiệm phân biệt với mọi m. Theo
định lý Viet: x + x = 2 ;
m x x = m − 4 1 2 1 2 0,25
x + x = 26m ⇔ ( x + x )3 3 3
− 3x x (x + x ) = 26 1 2 1 2 1 2 1 2 m 3 2 0,25
⇔ 8m − 6m(m − 4) = 26m m(8m − 6m − 2) = 0 1
m = 0;m = 1;m = − 0,25 4
1,0 đim
b) Gọi x , x (x < x ) là hai nghiệm nguyên của phương trình. 1 2 1 2
Ta có: x + x = 2 ;
m x x = m − 4 . 1 2 1 2 Suy ra
x + x − 2x x = 8 ⇔ 2(x + x ) − 4x x −1 = 15 ⇔ (2x −1)(2x −1) = 1 − 5 . 1 2 1 2 1 2 1 2 1 2 0,25 2x −1 = 1 − x = 0 TH1: 1 1  ⇔  ⇒ m = 4 2x −1 = 15 x = 8 2 2 2x −1 = 5 − x = 2 TH2: 1 1 −  ⇔  ⇒ m = 0 2x −1 = 3 x = 2 2 2 2 0,5 x −1 = 1 − 5 x = 7 TH3: 1 1 −  ⇔  ⇒ m = 3 − 2x −1 = 1 x = 1 2 2 2x −1 = 3 − x = 1 TH4: 1 1 −  ⇔  ⇒ m = 1 2x −1 = 5 x = 3 2 2
Thử lại m=0, m=1, m=-3,m=4 thỏa mãn điều kiện bài toán. 0,25 3
3,5 đim N A E M F O 0,5 I B C Trang: 2 - Đáp án Toán 11
a) Ta có: AC//BM suy ra ∠BMA = CAN
AB//CN suy ra ∠BAM = ∠CNA 0,5
Do đó tam giác CAN đồng dạng với tam giác BMA Suy ra: MB AB MB BC 0,25 = ⇒ = AC NC BC CN 0,25 Mặt khác 0 ∠MBC = BCN = 120 0,25
Suy ra tam giác MBC đồng dạng với tam giác BCN. b) 0 0 ∠BFM = B
CM + ∠NBC = B
CM + ∠BMC = 180 − ∠MBC = 60 0,5 Mặt khác 0
BEM = ∠BCA = 60 (do t/c góc ngoài của tứ giác nội tiếp) 0,25 Suy ra 0 ∠BFM = B
EM = 60 . Do đó tứ giác BMEF nội tiếp. 0,25
c) Gọi I là giao điểm EF với BC.
Ta có ∠IBF = B
MF (câu a), suy ra IB là tiếp tuyến của đường tròn ngoại tứ giác BMEF.
Tương tự chứng minh được IC là tiếp tuyến của đường tròn ngoại tứ giác 0,25 CNEF. Từ đó: 2 2
IB = IE.IF; IC = IE.IF IB = IC hay I là trung điểm BC. 0,25
Vậy d luôn đi qua điểm cố định là I. 0,25 4
1,5 đim
Đặt x = a +1; y = b + 2; z = c + 3 . (x, y, z >0) 0,5 y + z z + x x + y y x x z y z VT = + + = + + + + + x y z x y z x z y 0,5
≥ 2 y . x + 2 z . x + 2 y . z = 6 0,25 x y x z z y
Dấu bằng xảy ra khi x=y=z, suy ra a=3, b=2, c=1 0,25 5
1,0 đim
n là số tự nhiên lớn hơn 1 nên n có dạng n = 2k hoặc n = 2k + 1, với k là
số tự nhiên lớn hơn 0. 0,25 - Với n = 2k, ta có 4 n 4 2k
n + 4 = (2k) + 4 lớn hơn 2 và chia hết cho 2. Do 4 0,25 đó n n + 4 là hợp số. -Với n = 2k+1, tacó 4 n 4 2k 4 k 2 2 k 2 k 2
n + 4 = n + 4 .4 = n + (2.4 ) = (n + 2.4 ) − (2. . n 2 ) 2 k k 2 k k 0,25 = (n + 2.4 − 2. .
n 2 )(n + 2.4 + 2. . n 2 ) = ( k 2
(n − 2 ) + 4k )( k 2 (n + 2 ) + 4k ) M 0,25
ỗi thừa số đều lớn hơn hoặc bằng 2. Vậy n4 + 4n là hợp số Trang: 3 - Đáp án Toán 11
S GIÁO DC VÀ ĐÀO TO K THI CHN HC SINH GII LP 9 CP TNH NĂM HC 2013-2014 QUNG NGÃI Ngày thi : 22/3/2014 Môn : TOÁN
ĐỀ CHÍNH THC
Thi gian làm bài: 150 phút
Bài 1:(4
đim)
a) Cho a;b là hai số nguyên dương khác nhau, thoả mãn 2a2+a = 3b2+b. Ch −
ứng minh a b là phân số tối giản. 2a+2b+1
b) Tìm các cặp số nguyên dương (x; y) thoả mãn: 15x2 − 7y2 = 9
Bài 2: (4 đim) a) Cho 3 3 − ≤ x ≤
; x≠0 và 3+ 2x − 3− 2x = a . 2 2 2 Tính giá tr 6 + 2 9 − 4x ị biểu thức P = theo a. x
b) Cho a,b,c là 3 số dương thoả mãn 1 1 1 + + = 2 . 1+ a 1+ b 1+ c
Tìm giá trị lớn nhất của Q=abc
Bài 3: (4 đim) a) Gi x + 2 ải phương trình: (x − ) 1 (x + 2) + 4(x − ) 1 = 12 . x −1    
b) Giải hệ phương trình: 1 2 x 1+  = 3 và 1 2 y 1−  = 1 .  x + y   x + y 
Bài 4: (6 đim)
Cho nửa đường tròn tâm O đường kính AB cố định. EF là dây cung di động trên nửa đường tròn
đó, sao cho E thuộc cung AF và AB EF=
= R . Gọi H là giao điểm của AF và BE; C là giao điểm của AE 2
và BF; I là giao điểm của CH và AB. a) Tính số đo ·CIF
b) Chứng minh rằng biểu thức AE.AC+BF.BC có giá trị không đổi khi EF di động trên nửa đường tròn.
c) Xác định vị trí của EF trên nửa đường tròn để tứ giác ABFE có diện tích lớn nhất. Tính diện tích lớn nhất đó theo R.
Bài 5: (2 đim)
Tìm cạnh của hình vuông nhỏ nhất, biết rằng: hình vuông đó chứa 5 đường tròn có bán kính bằng 1
và 5 đường tròn này đôi một không có quá 1 điểm chung.
--------------Hết-------------
NGUYN VĂN HÂN TRƯỜNG THCS NGUYN CÁT – TNH HÒA – SƠN
T
NH – QUNG NGÃI.
BÀI GII ĐỀ THI CHN HC SINH GII LP 9 TNH QUNG NGÃI NĂM HC 2013-2014
Môn : TOÁN Ngày thi : 22/3/2014 Câu 1:
1) 2a2+a = 3b2+b ⇔ 2a2+a −2b2−b = b2 ⇔ (a−b)(2a+2b+1) = b2 Gọi (a−b,2a+2b+1) = d
Ta có: a – b ⇔ d, 2a+2b+1⇔d⇔ (a−b) (2a+2b+1) ⇔ d2 ⇔ b2 ⇔ d2 ⇔ b⇔d Mà a – b ⇔ d ⇔ a⇔d
a⇔d; b⇔d mà 2a+2b+1⇔ d nên 1⇔d ⇔ d=1
Vậy phân số đã cho tối giản.
2) Giả sử cặp số nguyên dương (x; y) là nghiệm của phương trình:
15x2 − 7y2 = 9 (1) =>15x2 − 9 =7y2=>7y2 3 => y2 3 => y 3
Đặt y = 3z và thay vào (1) ta có 15x2 − 63z2 = 9 =>5x2 − 21z2 = 3(2) => x 3
Đặt x = 3t và thay vào (2) ta có 45t2 − 21z2 = 3=>15x2 − 7z2 = 1(3)
Nếu z 0(mod3) => VP 0(mod3). VT 1(mod3). Vô lí
Nếu z 1(mod3) => z2 1(mod3) => − 7z2 2(mod3) .
VP 2(mod3). VT 1(mod3). Vô lí
Nếu z 2(mod3) => z2 1(mod3) =>− 7z2 2(mod3)
VP 2(mod3). VT 1(mod3). Vô lí
Vậy không tìm được cặp số nguyên dương (x; y) nào là nghiệm của phương trình đã cho. Câu 2: 2 a) 6 + 2 9 − 4x Cho 3 3 − ≤ x ≤
; x≠0 và 3 + 2x − 3 − 2x = a .Tính giá trị biểu thức P = theo a. 2 2 x 2
3 + 2x+2 (3 + 2x )(3 − 2x ) + 3 − 2x ( 3 + 2x + 3 − 2x ) 3 + 2x + 3 − 2x P = = = x x x 4x 4 = = . x ( 3 + 2x − 3 − 2x ) a
b) Cho ba số dương a , b , c và thỏa mãn điều kiện : 1 1 1 + + = 2 .Tìm giá trị lớn
1+ a 1+ b 1+ c nhất của Q = a.b.c 1 1 1 Gi b c bc ải :Ta có : = 1− + 1− = + ≥ 2 1+ a 1+ b
1+ c 1+ b 1+ c (1+ b)(1+ c) T ca ab ương tự : 1 1 ≥ 2 , ≥ 2 1+ b (1+ c)(1+ a) 1+ c (1+ a)(1+ b) Nhân các b abc
ất đẳng thức vừa nhận được ta có : 1 1 1 . . ≥ 8
1+ a 1+ b 1+ c
(1+ a)(1+ b)(1+ c) Hay : abc 1 ≤
. Dấu = xãy ra khi a = b = c = 1 . Vậy maxQ = 1 8 2 8
Bài 3: (4 đim)
NGUYN VĂN HÂN TRƯỜNG THCS NGUYN CÁT – TNH HÒA – SƠN
T
NH – QUNG NGÃI. a) Gi x + 2 ải phương trình (x − ) 1 (x + 2) + 4(x − ) 1
= 12 . ĐK : x≤ - 2 ; x > 1. x −1 => ( x − ) 1 (x + 2) + 4 (x + 2)(x − ) 1 −12 = 0 . Đặt t = (x + ) 2 (x - )
1 ta có phương trình t2 + 4t – 12 = 0 => t =2 hoặc t = - 6 (loại)
(x+2)(x-1) = 2 => x2 + x – 6 = 0 => x = 2(nhận) hoặc x = - 3 (nhận)   1  2 x 1+  = 3   x + y b)Gi  ải hệ phương trình:  .  1  2 y 1−  = 1    x + y    1   1 3  3 1 2 x 1+  = 3 1+ = 2 = + (Công vê)   x y  +   x + y  2 x  2 x 2 y  =>  =>    1 1  1 2 3 1 1 2 y 1  1   − = = − − = ( tru vê)   x + y   x y 2 y  x + +   y  2 x 2 y 4 9 1 2 2 => = −
( Nhân vê) =>x + 8xy-9y = 0 => (x − y)(x + 9y) = 0 x + y 4x 4y => x = y; x = 9 − y(loai) 3 1 2 => 2 = + => = 2 => x = 1. 2 x 2 x x
Vậy nghiệm của hệ là x = y = 1.
Bài 4: (6 đim) a) Tính số đo ·CIF T 1
ứ giác BFHI nội tiếp => · · º 0 HIF = HBF = sd EF = 30 (tam giác OEF 2 đều)
b) Chứng minh rằng biểu thức AE.AC+BF.BC có giá trị không đổi khi EF di động trên nửa đường tròn.
Ta có AE.AC = AC(AC –CE) = AC2 – AC.AE C
BF.BC = BC(BC –CF) = BC2 – BC.CF
AE.AC+BF.BC = AC2 + BC2 – AC.AE – BC .CF F MÀ AC.AE = BC.CF =CO2 – R2 2 2 2 2AC + 2BC - AB 2 AB 2 CO = => AC2 + BC2 =2CO2 + E 4 4 2 H
Suy ra : AE.AC+BF.BC = 2CO2 + AB – CO2 + R2 – CO2 + R2 = 3R2 4 A I O B
AE.AC+BF.BC= 3R2 Cố định. C
c) Xác định vị trí của EF trên nửa đường tròn để tứ giác ABFE
có diện tích lớn nhất. Tính diện tích lớn nhất đó theo R. E Ta có S P ABEF = SAOF + SFOE + SEOB F
NGUYN VĂN HÂN TRƯỜNG THCS NGUYN CÁT – TNH HÒA – SƠN
T
NH – QUNG NGÃI. A M Q O N B 2 S R 3 FOE =
(Vì tam giác FOE là tam giác đều cạnh R) 4 S +
AOF + SEOB = 1 OA.FM+ 1 OB.EN = R. FM
EN = R.PQ (PQ là đường trung bình của hình thang 2 2 2 EFMN) 2 S R 3 ABEF = + R.PQ mà PQ ≤ OP = R 3 . 4 2 2 2 2 Do R 3 R 3 3R 3 đó SABEF = + =
khi Q trùng với O hay EF // AB. 4 2 4
Bài 5: (2 đim)
Gọi cạnh hình vuông ABCD nhỏ nhất chứa bên trong 5 đường tròn có bán kính bằng 1cm và
đôi một không có quá 1 điểm trong chung là x (cm).
Từ đây suy ra các tâm của 5 đường tròn này nằm trong hình vuông MNPQ có cạnh bằng x – 2
cm. (vì tâm của các đường tròn các đường tròn cách cạnh hình vuông ít nhất 1cm).
Chia hình vuông MNPQ thành 4 hình vuông nh -
ỏ có độ dài mỗi cạnh là x 2 (cm) . (hình vẽ) 2
Theo nguyên lí Dirichlet có ít nhất hai tâm đường tròn cùng thuộc một hình vuông. Giả sử hai tâm đó là O1.O2.
Vì hai đường tròn này có không quá 1 điểm chung nên O A 1O2 B
không nhỏ hơn hai lần bán kính và không lớn hơn độ dài đường chéo M N O3 O1 c -
ủa hình vuông cạnh x 2 (cm) . 2 1cm O2 1cm Hay 2 - ≤ O1.O2 ≤ (x 2) 2 2 O4 O5
=> (x - 2) 2 ³ 2 Þ x - 2 ³ 2 2 Þ x ³ 2 + 2 2 Q 2 P
Vậy cạnh hình vuông nhỏ nhất chứa 5 đường tròn có bán kính D C X cm
bằng 1 và 5 đường tròn này đôi một không có quá 1 điểm chung là 2 + 2 2
NGUYN VĂN HÂN TRƯỜNG THCS NGUYN CÁT – TNH HÒA – SƠN
T
NH – QUNG NGÃI.
SỞ GIÁO DỤC VÀ ĐÀO TẠO
K THI CHN HC SINH GII CP TNH THANH HOÁ
Năm hc 2013 - 2014
ĐỀ THI CHÍNH THC
Môn thi: TOÁN - Lp 9 THCS
Thời gian: 150 phút (không k thi gian giao đề) S Ngày thi: 21/03/2014 ố báo danh
(Đề thi có 01 trang, gồm 05 câu) ........................  x +1 xy x   xy x  + + Câu I ( x +1
4,0 đim): Cho biểu thức A =  + +1 : 1− −   . xy 1 1 xy   xy 1 xy 1 + − − +    
1. Rút gọn biểu thức A. 2. Cho 1 1 +
= 6 . Tìm giá trị lớn nhất của A. x y
Câu II (5,0 đim). 1.Cho phương trình 2 x + ( 2 m − 2) 2
x + m − 2m + 4 = 0 . Tìm m để phương trình có hai nghi 2 1 1 ệm thực phân biệt . 1 x , x th 2 ỏa mãn − = x2 + x2 x x 15m 1 2 1 2
x + y + z = 1
2. Giải hệ phương trình  . 4 4 4
x + y + z = xyz
Câu III (4,0 đim).
1. Tìm tất cả các cặp số nguyên dương (a; b) sao cho (a + b2) chia hết cho (a2b – 1).
2. Tìm x, y, z N thỏa mãn x + 2 3 = y + z .
Câu IV (6,0 đim) : Cho nửa đường tròn tâm O đường kính AB. Một điểm C cố định thuộc đoạn thẳng
AO (C khác A và C khác O). Đường thẳng đi qua C và vuông góc với AO cắt nửa đường tròn đã cho
tại D. Trên cung BD lấy điểm M (M khác B và M khác D). Tiếp tuyến của nửa đường tròn đã cho tại
M cắt đường thẳng CD tại E. Gọi F là giao điểm của AM và CD.
1. Chứng minh tam giác EMF là tam giác cân.
2. Gọi I là tâm đường tròn ngoại tiếp tam giác FDM. Chứng minh ba điểm D, I, B thẳng hàng.
3. Chứng minh góc ABI có số đo không đổi khi M di chuyển trên cung BD.
Câu V (1,0 đim) : Cho x, y là các số thực dương thoả mãn x + y = 1. Tìm giá tr 1 1
ị nhỏ nhất của biểu thức B = + . 3 3 x + y xy ----- HẾT -----
Thí sinh không được s dng tài liu. Cán b coi thi không gii thích gì thêm S K
Ở GIÁO DỤC VÀ ĐÀO TẠO
THI CHN HC SINH GII CP TNH THANH HOÁ
Năm hc 2013 - 2014
HƯỚNG DN CHM
Môn thi: TOÁN - Lp 9 THCS
ĐỀ THI CHÍNH THC
Thời gian: 150 phút (không k thi gian giao đề) Ngày thi: 21/03/2014
(Hướng dẫn chấm gồm 04 trang) Câu Ý
Li gii (vn tt) Đim I 1 Điều kiện: xy ≠ 1. 0,25 (4,0đ) (2,5đ) ( x + )
1 (1− xy ) + ( xy + x )( xy + ) 1 + ( xy + ) 1 (1− xy ) A = : ( xy + ) 1 (1− xy ) ( xy + )
1 (1− xy ) + ( xy + x )( xy + ) 1 − ( x + ) 1 (1− xy ) = ( xy + ) 1 (1− xy ) 0,50 ( x + )
1 (1− xy ) + ( xy + x )( xy + ) 1 + ( xy + ) 1 (1− xy ) = = ( xy + )
1 (1− xy ) + ( xy + x )( xy + ) 1 − ( x + ) 1 (1− xy ) 0,50 1+ x 1 = = . x y + xy xy 1,25 2 Theo Côsi, ta có: 1 1 1 1 6 = + ≥ 2 ⇒ ≤ 9 . (1,5đ) x y xy xy 0,50 1 Dấu bằng xảy ra ⇔ 1 1 = ⇔ x = y = . x y 9 0,50 1
Vậy: maxA = 9, đạt được khi : x = y = . 9 0,50 1
PT đã cho có hai nghiệm phân biệt có điều kiện: (2,5đ) 2
∆'> 0 ⇔ (m − 2) − ( 2
m − 2m + 4) > 0 ⇔ m < 0 (*) 0,50
x + x = 4 − 2m V 1 2
ới m < 0 theo Vi-et ta có:  . x .x = 2 m − 2m + 4 1 2 0,25 2 1 1 2 1 1 Ta có − = ⇔ − = (1) x2 + x2 x x 1 m 5 1 2 1 2 ( 2
x + x ) − 2x x x x 1 m 5 1 2 1 2 1 2 0,50 1 1 1 ⇔ − = m2 − 6m + 4 m2 − 2m + 4 15m 0,50 1 1 1 4 ⇔ − = . Đặt m +
= t do m < 0 ⇒ t < 0 4 4 15 m m + − 6 m + − 2 0,50 m m 1 1 1 t = 4 − Ta cos (1) trở thành − = ⇔ ⇒ t = −  4 ( do t < 0 ) t − 6 t − 2 15 t = 12 0,50 4
Với t = −4 ta có m +
= −4 ⇔ m = −2 thỏa mãn (*) m 0,25 2 Ta có: (2,5đ) 4 4 4 4 4 4 0,50 4 4 4 x + y y + z z + x
x + y + z = + + ≥ 2 2 2 2 2 2
x y + y z + z x = 2 2 2 2 2 2 2 2 2 2 2 2 2 2 2 = x y + y z y z + z x z x + x y 0,50 + +
xyyz + yzzx + zxxy = 2 2 2
= xyz (x + y + z) = xyz ( vì x + y + z = 1). 0,50
x = y = z 1 Dấu bằng xảy ra ⇔ 
x = y = z =
x + y + z = 1 3  1 1 1  V 0,50
ậy nghiệm của hệ phương trình là:  x = ; y = ; z =   3 3 3  III 1
Giả sử (a + b2) M (a2b – 1), tức là: a + b2 = k(a2b – 1), với k ∈ N* ⇔ (4,0đ) (2,0đ)
⇔ a + k = b(ka2 – b) ⇔ a + k = mb (1)
Ở đó m ∈ Z mà: m = ka2 – b ⇔ m + b = ka2 (2) 0,50
Từ (1) và (2) suy ra: (m – 1)(b – 1) = mb – b – m + 1 ⇔
⇔ (m – 1)(b – 1) = (a + 1)(k + 1 – ka) (3)
Do m > 0 (điều này suy ra từ (1) do a, k, b > 0) nên m ≥ 1 (vì m ∈ Z).
Do b > 0 nên b – 1 ≥ 0 (do b ∈ Z) ⇒ (m – 1)(b – 1) ≥ 0.
Vì thế từ (3) suy ra: (a + 1)(k + 1 – ka) ≥ 0. 0,50
Lại do a > 0 nên suy ra: k + 1 – ka ≥ 0 ⇒ k + 1 ≥ ka ⇒ 1 ≥ k(a – 1) (4)
Vì a – 1 ≥ 0 (do a ∈ Z, a > 0) và k ∈ Z, k > 0 nên từ (4) có: a = 1 k(a −1) = 0  ⇔   a = 2 k(a 1) 1  − =  k =1 0,25
- Với a = 1. Thay vào (3) ta được: (m – 1)(b – 1) = 2 ⇔ m −1 = 2  b −1 = 1 b = 2 ⇔   m −1 = 1 b = 3  b −1 = 2
Vậy, trường hợp này ta có: a = 1, b = 2 hoặc a = 1, b = 3. 0,25 b = 1
- Với a = 2 (vì k = 1). Thay vào (3) ta có: (m – 1)(b – 1) = 0 ⇔  . m = 1
Khi b = 1, ta được: a = 2, b = 1.
Khi m = 1: Từ (1) suy ra a + k = b ⇒ b = 3. Lúc này được: a = 2, b = 3. 0,25
Tóm lại, có 4 cặp số (a; b) thỏa mãn bài toán là: (1; 2), (1; 3), (2; 3), (2; 1). 0,25 2 Ta có 2 ⇔ + 2 3 = + + 2 (2,0 x + 3 = y + z x y z yz đ) 0,50 ⇔ ( 2
x y z) + 2 3 = 2 yz ⇒ (x y z) + 4 3(x y z) +12 = 4 yz (1) 4 2
yz − (x y z) 12 TH1. N −
ếu x y z ≠ 0 Ta có 3 = (2) vô lý (
4 x y z) 0,50
( do x, y, z N nên vế phải của (2) là số hữu tỷ ).
x y z = 0
TH2. x y z = 0 khi đó ( ) 1 ⇔  (3) 0.50 yz = 3 x = 4 x = 4  
Giải (3) ra ta được y =1 hoặc y = 3 thử lại thỏa mãn 0,50   z = 3 z = 1 IV (6,0đ) 1 E (2.5đ) D M I H F A C O B
Ta có M thuộc đường tròn tâm O đường kính AB (giả thiết) nên 0 0,50
AMB = 90 (góc nội tiếp chắn nửa đường tròn) 0,50 hay 0 FMB = 90 . Mặt khác 0
FCB = 90 (giả thiết).Do đó 0 FMB + FCB = 180 . 0,50
Suy ra BCFM là tứ giác nội tiếp ⇒ CBM = EFM ( ) 1 (vì cùng bù với CFM ). 0,50
Mặt khác CBM = EMF (2) (góc nội tiếp; góc tạo bởi tiếp tuyến và dây
cung cùng chắn AM ). Từ (1) và (2) ⇒ EFM = EMF .
Suy ra tam giác EMF là tam giác cân tại E. 0,50
(thnhn ra ngay EMF = MBA = MFE nên suy ra EMF cân) DIF
Gọị H là trung điểm của DF. Suy ra IH ⊥ DF và DIH = (3) . 2 2 0,50
(2.5đ) Trong đường tròn (I) ta có: DMF và DIF lần lượt là góc nội tiếp và góc 1
ở tâm cùng chắn cung DF. Suy ra DMF = DIF(4). 2 0,50 T
ừ (3) và (4) suy ra DMF = DIH hay DMA = DIH . Trong 0,50
đường tròn (O) ta có: DMA = DBA (góc nội tiếp cùng chắn DA ) Suy ra DBA = DIH . 0,50
Vì IH và BC cùng vuông góc với EC nên suy ra IH // BC. Do đó 0,50 o DBA + HIB = 180 o
⇒ DIH + HIB = 180 ⇒ Ba điểm D, I, B thẳng hàng. 3(1đ) 1
Vì ba điểm D, I, B thẳng hàng ⇒ ABI = ABD = sđ AD . 2 0,50 1
Mà C cố định nên D cố định ⇒ sđ AD không đổi. 2
Do đó góc ABI có số đo không đổi khi M thay đổi trên cung BD. 0,50 1 1 1 1 1− 2xy Ta có: B = + = + = . 3
(x + y) − 3xy(x + y) xy 1− 3xy xy xy(1− 3xy) 0.25 2 (x + y) Theo Côsi: 1 xy ≤ = . 4 4 1− 2xy
Gọi Bo là một giá trị của B, khi đó, ∃x, y để: B o = ⇔ xy(1− 3xy)
⇔ 3Bo(xy)2 – (2 + Bo)xy + 1 = 0 (1)
Để tồn tại x, y thì (1) phải có nghiệm xy ⇔ ∆ = B 2 o – 8Bo + 4 ≥ 0 ⇔ B ≥ 4 + 2 3 o  0.25 B ≤ 4 − 2 3 o
Để ý rằng với giả thiết bài toán thì B > 0. Do đó ta có: B ≥ 4 + 2 3 . o V(1đ) 2 + B V 3 + 3 3 + 3 ới o B = 4 + 2 3 ⇒ xy = = ⇒ x(1− x) o = ⇔ 6Bo 6(2 + 3) 6(2 + 3) 2 3 2 3 1+ −1 1− −1 3 + 3 3 3 0.25 ⇔ 2 x − x + = 0 ⇔ x = , x = . 6(2 + 3) 2 2 2 3 2 3 1+ −1 1− −1 V 3 3 ậy, B = 4 + 2 3 , x = , y min đạt được khi = 2 2 2 3 2 3 1− −1 1+ −1 ho 3 3 ặc x = , y = . 0.25 2 2 Chú ý:
1) N
ếu hc sinh làm bài không theo cách nêu trong đáp án nhưng đúng thì cho đủ s đim tng
ph
n như hướng dn quy định.
2) Vi
c chi tiết hóa (nếu có) thang đim trong hướng dn chm phi bo đảm không làm sai lch
h
ướng dn chm và phi được thng nht thc hin trong t chm.
3)
Đim bài thi là tng đim không làm tròn.
SỞ GIÁO DỤC VÀ ĐÀO TẠO
ĐỀ THI CHN HC SINH GII LP 9 THCS TỈNH NINH BÌNH
Năm hc 2012 – 2013 MÔN: TOÁN
ĐỀ THI CHÍNH THC
(Thời gian làm bài 150 phút, không kể thời gian giao đề)
Đề thi gm 05 câu, trong 01 trang
Câu 1 (4 điểm). Cho phương trình 2
x + (4m + 1)x + 2(m - 4) = 0 (1)
(x là ẩn số, m là tham số).
1. Chứng minh rằng phương trình (1) luôn có hai nghiệm phân biệt với mọi m.
2. Gọi x1, x2 là hai nghiệm của (1). Tìm m để x − x = 17 . 1 2 2 x - x 2x + x 2(x - 1)
Câu 2 (4 điểm). Cho biểu thức: P = - + (x > 0, x ≠ 1). x + x + 1 x x - 1 1. Rút gọn P.
2. Tìm giá trị của x để P = 3. Câu 3 (4 điểm). x + y + xy = 7
1. Giải hệ phương trình:  3 3 2 2
x + y + 3(x + y ) + 3(x + y) = 70 2. Giải phương trình: 2
( x + 5 - x + 2)(1 + x + 7x + 10) = 3 .
Câu 4 (5 điểm). Cho đường tròn tâm O đường kính AB cố định. Ax và Ay là hai tia thay đổi
luôn tạo với nhau góc 600, nằm về hai phía của AB, cắt đường tròn (O) lần lượt tại M và N.
Đường thẳng BN cắt Ax tại E, đường thẳng BM cắt Ay tại F. Gọi K là trung điểm của đoạn thẳng EF. EF 1. Chứng minh rằng = 3 . AB
2. Chứng minh OMKN là tứ giác nội tiếp.
3. Khi tam giác AMN đều, gọi C là điểm di động trên cung nhỏ AN (C ≠ A, C ≠ N).
Đường thẳng qua M và vuông góc với AC cắt NC tại D. Xác định vị trí của điểm C để diện
tích tam giác MCD là lớn nhất. Câu 5 (3 điểm). 2 3m
1. Cho các số thực m, n, p thoả mãn: n2 + np + p2 = 1 -
. Tìm giá trị lớn nhất và 2
nhỏ nhất của biểu thức S = m + n + p.
2. Cho các số thực dương a, b, c thoả mãn abc = 1. Chứng minh rằng: a b c 1 + + ≥ . 2 2 2 (ab + a +1) (bc + b +1) (ca + c +1) a + b + c
Đẳng thức xảy ra khi nào? -----HẾT-----
Họ và tên thí sinh :....................................................... Số báo danh .............................................
Họ và tên, chữ ký: Giám thị 1:.......................................................................................................
Họ và tên, chữ ký: Giám thị 2:.......................................................................................................
S GIÁO DC & ĐÀO TO K THI CHN HC SINH GII CP TNH
LÂM ĐỒNG
NĂM HC 2010-2011 Môn : TOÁN – THCS ĐỀ CHÍNH THỨC
Thi gian : 150 phút ( không k thi gian giao đề)
(Đề thi gm có 1 trang) Ngày thi : 18/02/2011
Câu 1: (2,0 đim ) Rút gọn A = 127 − 48 7 − 127 + 48 7 .
Câu 2:(2,0 đim) Cho hàm số y = f(x) = (3m2 – 7m +5) x – 2011 (*) . Chứng minh hàm số (*)
luôn đồng biến trên R với mọi m.
Câu 3:( 2,0 đim) Cho hai đường tròn (O) và (O’) cắt nhau tại hai điểm A và B . Trên đường
thẳng AB lấy điểm M sao cho A nằm giữa M và B . Từ M kẻ cát tuyến MCD
với đường tròn (O) và tiếp tuyến MT với đường tròn (O’) (T là tiếp điểm) Chứng minh MC.MD = MT2 .
Câu 4: (2,0 đim ) Cho hai số dương x, y thỏa mãn điều kiện 3x + y – 1 = 0 .
Tìm giá trị nhỏ nhất của biểu thức B = 3x2 + y2 .
Câu 5: (1,5 đim) Chứng minh tổng C = 1 + 2 + 22 + … + 22011 chia hết cho 15 .
Câu 6: (1,5 đim ) Phân tích đa thức x3 – x2 – 14x + 24 thành nhân tử . x + y + z = 2
Câu 7: (1,5 đim) Giải hệ phương trình  2 2xy − z =  4
Câu 8: (1,5 đim ) Chứng minh D = n(n + 1)(n + 2)(n + 3) không phải là số chính phương với mọi n ∈ N *. 1 1 4
Câu 9: (1,5 đim ) Cho hai số dương a và b . Chứng minh + ≥ . a b a + b
Câu 10:(1,5 đim ) Tìm nghiệm tự nhiên của phương trình : 2x2 – xy – y2 – 8 = 0
Câu 11: (1,5 đim ) Cho hình thang vuông ABCD ( 0
A = D = 90 ) , có DC = 2AB . Kẻ DH vuông
góc với AC (H∈ AC) , gọi N là trung điểm của CH .
Chứng minh BN vuông góc với DN .
Câu 12: (1,5 đim). Cho tam giác MNP cân tại M ( 0
M < 90 ) . Gọi D là giao điểm các đường
phân giác trong của tam giác MNP . Biết DM = 2 5 cm , DN = 3 cm .
Tính độ dài đoạn MN . ---------- HẾT---------
Họ và tên thí sinh :……………………………………………...Số báo danh : ………………………
Giám thị 1 :……………………………………………………..Ký tên : …………………………….
Giám thị 2 :……………………………………………………..Ký tên : …………………………….
(Thí sinh không được s dng máy tính )
SỞ GIÁO DỤC &ĐÀO TẠO
KỲ THI CHỌN HỌC SINH GIỎI CẤP TỈNH LÂM ĐỒNG NĂM HỌC 2010-2011
HƯỚNG DN CHM ĐỀ THI CHÍNH THC Môn : TOÁN – THCS Ngày thi 18/02/2011 Câu Hướng dẫn chấm Điểm Câu 1 A = 127 − 48 7 − 127 + 48 7 (2 điểm ) 0,5 điểm = 2 2 (8 − 3 7) − (8 + 3 7) 0,5 = | 8 điểm − 3 7 | − | 8 + 3 7 |
= 8 − 3 7 − 8 − 3 7 (8>3 7 ) 0,5 điểm 0,5 điểm = 6 − 7 Câu 2   (2 − + điểm ) 3m2 – 7m + 5 = 3 2 7 5  m m   3 3  0,5 điểm 2  7 49 60    0,5 điểm = 3  m − − +    6  36 36   2  7 11    0,5 điểm = 3  m − +   > 0 m ∀  6  36   0,5 điểm
Vây f(x) đồng biến trên R với mọi m Câu 3 (2 điểm) Chứng minh MC. MD = MA. MB 0,75 điểm Chứng minh MT2 = MA. MB 0,75 điểm Suy ra MC.MD = MT2 0,5 điểm Câu 4
3x + y – 1 = 0 ⇔ y = 1 – 3x (2 điểm ) 2 2 B = 3x + (1− 3x) 2 = 12x − 6x +1 0,5 điểm 2  1 1    = 12  x − +   0,5 điểm  4  48   2  1  1 1 = 12 x − + ≥   0,5  4  4 4 điểm 1 1 1
Vây GTNN của B là khi x = và y = 0,5 điểm 4 4 4 Câu 5 C = 1 + 2 + 22 + … + 22011 (1,5 điểm )
= (1 + 2 + 22 + 23 ) + (24 + 25 + 26 + 27 ) + …+ ( 22008 + 22009 +22010 + 22011) 0,5 điểm
= (1 + 2 + 22 + 23 )+ 24 (1 + 2 + 22 + 23 )+ …+22008(1 + 2 + 22 + 23 ) 0,5 điểm
= 15 ( 1 + 24 + …+ 22008 ) chia hết cho 15 0,5 điểm Câu 6 x3 – x2 – 14x +24 (1,5 điểm )
= x3 + 4x2 – 5x2 – 20x + 6x + 24 0,5 điểm = (x + 4) (x2 – 5x + 6 ) 0,5 điểm
= (x + 4) (x – 2) (x – 3) 0,5 điểm Câu 7 2 x + y + z = 2 z = 2 − x − y (2 − x − y) = 2xy − 4 (1,5 điểm ) ⇔ ⇔    0,5 2 2 điểm 2xy − z = 4 z = 2xy − 4 z = 2 − x −    y 2 2 (x − 2) + (y − 2) = 0 ⇔  z 0,5 điểm = 2 − x −  y x = y = 2 0,5 điểm ⇔ z = 2 −  Câu 8 D = n(n + 1) (n + 2) (n + 3) (1,5 điểm ) = (n2 + 3n) (n2 + 3n + 2 ) 0,5 điểm = (n2 + 3n)2 +2 (n2 + 3n)
⇒ (n2 + 3n)2 < D < (n2 + 3n)2 +2 (n2 + 3n) +1
⇒ (n2 + 3n)2 < D < (n2 + 3n +1)2 0,5 điểm
Nên D không phải là số chính phương vì (n2 + 3n)2 và (n2 + 3n +1)2 là 2 số chính phương liên tiếp 0,5 điểm Câu 9 Ta có (a – b)2 ≥ 0 (1,5 điểm ) 2 2 ⇔ a + b ≥ 2ab 0,5 điểm 2 ⇔ (a + b) ≥ 4ab 0,5 điểm a + b 4 ⇔ ≥ ( vì (a+b)ab >0 ) ab a + b 1 1 4 ⇔ + ≥ a b a 0,5 điểm + b D
ấu “ = ” xảy ra khi a = b ( thiếu câu này không trừ điểm) Câu 10 2x2 – xy – y2 – 8 = 0 (1,5 điểm) ⇔ (2x + y) (x – y) = 8 0,5 điểm 2x + y = 8 2x + y = 4 ⇔  hoặc  x − y =  1 x − y =  2 0,5 điểm x = 3 x = 2 ⇔  hoặc  y =  2 y =  0 0,5 điểm Câu 11 (1,5 điểm )
Gọi M là trung điểm của DH 0,25 điểm
Chứng minh tứ giác ABNM là hình bình hành ⇒ AM // BN (1) 0,5 điểm Chứng minh MN ⊥ AD 0,25 điểm
Suy ra M là trực tâm của A ∆ DN ⇒ AM ⊥ DN (2) 0,25 điểm
Từ (1) và (2) ⇒ BN ⊥ DN 0,25 điểm Câu 12 (1,5 điểm )
Qua M kẻ tia Mx vuông góc với MN cắt ND tại E , kẻ MF ⊥ ND
Chứng minh D = E ⇒ MD = ME = 2 5 cm và EF =DF 0,5 điểm 1
ME2 = EF .EN = EF .(2EF + DN ) 2 (2 5) = EF(2EF + 3) 2 2EF + 3EF − 20 = 0 (EF + 4)(2EF − 5) = 0 ⇒ EF = 2,5 (vì EF >0) 0,5 điểm ⇒ MN = 2 11 cm 0,5 điểm
(Nếu học sinh giải bằng cách khác đúng , giám khảo dựa theo biểu điểm để cho điểm tương ứng )
S GIÁO DC VÀ ĐÀO TO
KÌ THI CHN HC SINH GII TNH
HI DƯƠNG
LP 9 NĂM HC 2013-2014 MÔN THI: TOÁN
Thi gian làm bài: 150 phút
Ngày thi 20 tháng 03 năm 2014
ĐỀ THI CHÍNH THC
(đề thi gm 01 trang)
Câu 1 (2 đim). 2 1− 1− x .( 3 3
(1+ x) + (1− x) )
a) Rút gọn biểu thức A = với 1 − ≤ x ≤1. 2 2 − 1− x
b) Cho ab là các số thỏa mãn a > b > 0 và 3 2 2 3
a a b + ab − 6b = 0 . 4 4 a − 4b
Tính giá trị của biểu thức B = . 4 4 b − 4a
Câu 2 (2 đim).
a) Giải phương trình 2 2 2
x (x + 2) = 4 − x 2x + 4. 3
x = 2x + y
b) Giải hệ phương trình  . 3
 y = 2 y + x
Câu 3 (2 đim).
a) Tìm các số nguyên dương x, y thỏa mãn phương trình 2
xy + 2xy + x = 32 y .
b) Cho hai số tự nhiên a, b thỏa mãn 2 2
2a + a = 3b + b .
Chứng minh rằng 2a + 2b +1 là số chính phương.
Câu 4 (3
đim).
Cho tam giác đều ABC nội tiếp đường tròn (O, R). H là một điểm di động trên
đoạn OA (H khác A). Đường thẳng đi qua H và vuông góc với OA cắt cung nhỏ AB
tại M. Gọi K là hình chiếu của M trên OB. a) Chứng minh HKM = 2AMH.
b) Các tiếp tuyến của (O, R) tại A và B cắt tiếp tuyến tại M của (O, R) lần lượt
tại D và E. OD, OE cắt AB lần lượt tại F và G. Chứng minh OD.GF = OG.DE.
c) Tìm giá trị lớn nhất của chu vi tam giác MAB theo R.
Câu 5 (1 đim).
Cho a, b, c là các số thực dương thỏa mãn 2ab + 6bc + 2ac = 7abc . Tìm giá trị 4ab 9ac 4bc
nhỏ nhất của biểu thức C = + + . a + 2b a + 4c b + c
----------------------Hết------------------------
Họ và tên thi sinh…………………………………………..số báo danh…………...
Chữ ký của giám thị 1………………………..chữ ký của giám thị 2………………
S GIÁO DC VÀ ĐÀO TO
HƯỚNG DN CHM ĐỀ THI CHN
HI DƯƠNG
HC SINH GII TNH ---------------------------
LP 9 NĂM HC 2013-2014 MÔN THI: TOÁN
Ngày thi 20 tháng 03 năm 2014
(Hướng dn chm gm có 03 trang)
Lưu ý: Nếu hc sinh làm theo cách khác mà kết qu đúng thì giám kho vn
cho đim ti đa. Câu Ni dung Đim 2
1− 1− x .( 1+ x + 1− x )( 2 2 − 1− x ) A = 0.25 2 2 − 1− x Câu 2 = 1− 1− x . 1a:
( 1+ x + 1− x ) 0.25
(1,0 đ) = ( − −x )( +x + −x)2 2 = ( 2 − − x )( 2 1 1 1 1 1 1 2 + 2 1− x ) 0.25 2 = 2x = x 2 0.25 3 2 2 3 2 2
a a b + ab − 6b = 0 ⇔ (a − 2b)(a + ab + 3b ) = 0 (*) 0.25 Vì a > b > 0 2 2
a + ab + 3b > 0 nên từ (*) ta có a = 2 b 0.25 Câu 4 4 4 4 a − 4b 16b − 4b 1b: Vậy biểu thức B = = 4 4 4 4 0.25 b − 4a b − 64b (1,0 đ) 4 12b 4 − B = = 4 0.25 6 − 3b 21 2 t 0.25 Đặt 2 2 t = x x + ⇒ t = ( 4 2 2 4 2 x + 2x ) ⇒ 2 x ( 2 x + 2) = 2 2 tt = 4 − ta được phương trình 2
= 4 − t t + 2t − 8 = 0 ⇔  0.25 2 t = 2 x < 0  x < 0 Với t = -4 ta có 2 x 2x + 4 = 4 − ⇔  ⇔  Câu 2  ( 4 2 x + 2x ) 4 2 = 16
x + 2x − 8 = 0 0.25 2a: x < 0 (1,0 đ) ⇔  ⇔ x = − 2 2 x = 2 x > 0  x > 0 Với t =2 ta có 2
x 2x + 4 = 2 ⇔  ⇔  2  ( 4 2 x + 2x ) 4 2 = 4
x + 2x − 2 = 0 0.25 x > 0 ⇔  ⇔ x =
3 −1 . Kết luận nghiệm của phương trình. 2 x = 3 −1 Câu Từ hệ ta có 3 3 2 2 x y + x = y
x + y x y ( 2 2 (2 ) (2 ) (
) 2xy + x + y ) = 0 0.25 2b: x = y 3 0.25 (1,0 đ)
⇔ (x + y) (x y) = 0 ⇔  x = − y
* Với x = y ta tìm được (x ; y) = (0; 0); ( 3; 3 );( − 3; − 3 ) 0.25
* Với x = - y ta tìm được (x ; y) = (0; 0); (1; −1);( −1;1)
Vậy hệ phương trình có nghiệm 0.25
(x ; y) = (0; 0); 3; 3 );( − 3; − 3 );( −1;1);(1; −1) 2
xy + 2xy + x = 32 y 2
x( y +1) = 32 y 32 y 0.25
Do y nguyên dương ⇒ y +1 ≠ 0 ⇒ x = 2 ( y +1) Vì 2
( y, y +1) = 1 ⇒ ( y +1) ∈U (32) 0.25 Câu mà 5 32 = 2 2 2 ⇒ ( y +1) = 2 và 2 4 ( y +1) = 2 (Do 2 ( y +1) > 1) 0.25 3a: (1,0 ⇒ đ) *Nếu 2 2 ( y +1) = 2 y = 1; x = 8 *Nếu 2 4
( y +1) = 2 ⇒ y = 3; x = 6
Vậy nghiệm nguyên dương của phương trình là: 0.25 x = 8 x = 6  và   y = 1  y = 3 2 2
2a + a = 3b + b 2
⇔ (a b)(2a + 2b +1) = b (*) 0.25
Gọi d là ước chung của (a - b, 2a + 2b + 1) ( * d ∈ ). Thì
(a b)Md
⇒ (a b)(2a + 2b + ) 2 1 Md 0.25
(2a + 2b +1)Md Câu 2 2 ⇒ 3b:
b Md bMd
(1,0 đ) Mà (a b)Md aMd ⇒ (2a + 2b)Md mà (2a + 2b +1)Md ⇒1Md d =1 0.25
Do đó (a - b, 2a + 2b + 1) = 1. Từ (*) ta được a b và 2a + 2b +1 là số chính 0.25
phương => 2a + 2b +1 là số chính phương. x A
Qua A kẻ tia tiếp tuyến Ax của (O). Ta có 1 1 1 A = O = sđ AM (1) 1 1 2 2 1 H M 1 O 0.25 Câu 1 K 4a: B C (1,0 đ)
Có Ax // MH (cùng vuông góc với OA) ⇒ A = M (2) 0.25 1 1
Tứ giác MHOK nội tiếp ⇒ O = K (cùng chắn MH ) (3) 0.25 1 1 1 Từ (1), (2), (3) ta có M = K hay HKM = 2AMH. 0.25 1 1 2 A
Có tứ giác AOMD nội tiếp (4) D 2 1 1 Câu F M 4b: H 1 0.25 1 (1,0 đ) 2 E G O B C 1 1 A = sđ BM ; O = O = sđ BM 1 2 1 2 2 0.25
⇒ A = O ⇒ tứ giác AMGO nội tiếp (5) 1 1
Từ (4), (5) ta có 5 điểm A, D, M, G, O cùng nằm trên một đường tròn ⇒ 0.25 G = D = D 1 2 1
⇒ ∆OGF và ∆ODE đồng dạng ⇒ OG GF 0.25 = hay OD.GF = OG.DE. OD DE
Trên đoạn MC lấy điểm A’ sao cho A
MA’ = MA ⇒ ∆AMA ' đều 1 2 ⇒ A = A 60 BAA' 1 2 ( = 0 − ) H
⇒ ∆MAB = ∆A'AC ⇒ MB = A'C M 0.25 O A' Câu B I C 4c:
(1,0 đ) ⇒ MA + MB = MC 0.25
Chu vi tam giác MAB là MA + MB + AB = MC + AB ≤ 2R + AB
Đẳng thức xảy ra khi MC là đường kính của (O) => M là điểm chính giữa
cung AM => H là trung điểm đoạn AO 0.25
Vậy giá trị lớn nhất của chu vi tam giác MAB là 2R + AB 3 AB 3
Gọi I là giao điểm của AO và BC ⇒ AI = R = ⇒ AB = R 3 2 2 0.25
Giá trị lớn nhất của chu vi tam giác MAB là 2R + AB = (2 + 3)R
Từ gt : 2ab + 6bc + 2ac = 7abc và a,b,c > 0 2 6 2
Chia cả hai vế cho abc > 0 ⇒ + + = 7 c a b 1 1 1
x, y, z > 0
đặt x = , y = , z = ⇒  0.25 a b c
2z + 6x + 2 y = 7 4ab 9ac 4bc 4 9 4 Khi đó C = + + = + + a + 2b a + 4c b + c 2x + y 4x + z y + z Câu 5: 4 9 4
(1,0 đ) C = + 2x + y + + 4x + z +
+ y + z − (2x + y + 4x + z + y + z) 0.25 2x + y 4x + z y + z 2 2 2  2  3  2    =  − x + 2 y  + −  4x + z +  − 
y + z  +17 ≥ 17     0.25 x + 2 y  4x + z     y + z  1 Khi x = ,y = z = 1 thì C = 17 2 0.25
Vậy GTNN của C là 17 khi a =2; b =1; c = 1
S GD&ĐT HI DƯƠNG
KÌ THI CHN HC SINH GII TNH
LỚP 9 THCS NĂM HỌC 2012 – 2013 MÔN THI: TOÁN ĐỀ THI CHÍNH THỨC
Thi gian làm bài: 150 phút (không k thi gian giao đề) Ngày thi: 27/03/2013
( Đề thi gồm có 01 trang )
Câu 1 (2,0 đim):
a) Rút gọn biểu thức: ( − − ) 2 A = x 50 x + 50 x + x − 50 với x ≥ 50
b) Cho x + 3 = 2. Tính giá trị của biểu thức: B = x5 – 3x4 – 3x3 + 6x2 – 20x + 2018
Câu 2 (2,0 đim): 4x 3x a) Giải phương trình + = 6 2 2 x − 5x + 6 x − 7x + 6  x + y + 4 xy = 16
b) Gi¶i hÖ ph−¬ng tr×nh sau:  x + y = 10
Câu 3 (2,0 đim):
a) Với a, b là các số nguyên. Chứng minh rằng nếu 2 2
4a + 3ab −11b chia hết cho 5 thì 4 − 4 a b chia hết cho 5. 5− 3 b) Cho phương trình 2
ax +bx+1= 0 với a, b là các số hữu tỉ. Tìm a, b biết x = 5+ 3
là nghiệm của phương trình.
Câu 4 (3,0 đim):
Cho 3 điểm A, B, C cố định nằm trên một đường thẳng d (B nằm giữa A và C). Vẽ
đường tròn tâm O thay đổi nhưng luôn đi qua B và C (O không nằm trên đường thẳng
d). Kẻ AM và AN là các tiếp tuyến với đường tròn tâm O tại M và N. Gọi I là trung
điểm của BC, AO cắt MN tại H và cắt đường tròn tại các điểm P và Q (P nằm giữa A và O), BC cắt MN tại K.
a) Chứng minh 4 điểm O, M, N, I cùng nằm trên một đường tròn.
b) Chứng minh điểm K cố định khi đường tròn tâm O thay đổi.
c) Gọi D là trung điểm HQ, từ H kẻ đường thẳng vuông góc với MD cắt đường thẳng
MP tại E. Chứng minh P là trung điểm ME.
Câu 5 (1,0 đim): Cho 1 A = v n ới n * ∈ . (2n +1) 2n −1
Chứng minh rằng: A + A + A + ... + A < 1 . 1 2 3 n
------------- HẾT ------------
Họ và tên thí sinh: ……………………………… ….. Số báo danh …………….
Chữ kí giám thị 1 ………………….. Chữ kí giám thị 2 …………………..
S GD&ĐT HI DƯƠNG
ĐÁP ÁN VÀ HƯỚNG DN CHM
ĐỀ THI HC SINH GII TNH
MÔN TOÁNLỚP 9 THCS NĂM HỌC 2012 – 2013
Lưu ý: Thí sinh làm theo các khác đúng vẫn cho điểm tối đa. Điểm bài thi làm tròn đến 0,25 điểm CÂU PHẦN NỘI DUNG ĐIỂM Ta có : 0,25 A = ( x - 50 - x + 50 )2 2 ( 2 x + x - 50 ) 2 A = ( 2 x - 50 + x + 50 - 2 x - 50 )( 2 x + x - 50 ) a) 2 2 2 A = 2x - 2 x - 50 x + x - 50 0,25 1,0 ( )( ) điểm 2 A = 2( 2 2 x - x + 50) 2 Câu 1 A = 100 0,25 2,0 Nh
ưng do theo giả thiết ta thấy đim ( ) 2 A = x - 50 - x + 50 x + x - 50 <0 ⇒ A= -10 0,25đ x + 3 = 2=> 2
x − 2 = − 3 ⇒ (x − 2) = 3 b) 2
x − 4x +1 = 0 0,25 1,0
B = x5 – 3x4 – 3x3 + 6x2 – 20x + 2018
B = (x5 – 4x4 + x3 ) + ( x4 – 4x3 + x2 ) + 5( x2 – 4x + 1) + 2013 0,25 điểm
B = x3( x2 – 4x + 1) +x2( x2 – 4x + 1) +5(x2 – 4x + 1) + 2013 0,25 B = 2013 0,25
Nhận xét x = 0 không là nghiệm của phương trình
Với x ≠ 0 , phương trình đã cho tương đương với: 4 3 + = 6 6 6 x − 5 + x − 7 + x x 6 Đặt t = x − 7 + phương trình trở thành x 4 3 + =6 ( ) 1 (t ≠ 0;t ≠ 2 − ) 0,25 Câu 2 t+2 t 2,0 a) ( ) 2 2
1 ⇔ 4t + 3t + 6 = 6t +12t ⇔ 6t + 5t − 6 = 0 đim 1.0 3 − 2 Gi t = ; t = ( th điểm
ải phương trình ta được 1 2 ỏa mãn ) 2 3 0,25 −3 6 3 V − ới t x − 7 + =
⇔ 2x −11x + 12 = 0 1 = ta có 2 2 x 2 3
Giải phương trình ta được x = ; x = 4 ( th 1 2 ỏa mãn ) 0,25 2 2 6 2 Với t = ta có 2 x − 7 + =
⇔ 3x − 23x + 18 = 0 2 3 x 3 23 + 313 23 − 313
Giải phương trình ta được x = ; x 3 4 = (thỏa mãn) 6 6
Vậy phương trình đã cho có bốn nghiệm là : 3 23 + 313 23 − 313 x = ; x = 4 ; x = ; x 0,25 1 2 = 2 3 4 6 6  x + y + 4 xy = 16  (I) ( x; y ≥ 0 ) x + y = 10 Đặt S= x + y ; P = xy ( S
≥ 0;P ≥ 0 ) hệ (I) có dạng S  + 4P = 16  0,25 2 ( II) b ) S  - 2P = 10 1,0 S  = 4 0,25 ®iÓm
Giải hệ ( II) và đối chiếu điều kiện ta được  P = 3
Khi đó x; y là 2 nghiệm của phương trình t2 – 4t + 3 =0 Gi 0,25
ải phương trình ta được t1 = 3; t2 = 1 x = 9 x = 1 T
ừ đó suy ra hệ phương trình đã cho có hai nghiệm  ; y = 1 y = 9 0,25 2 4a + 3ab − 2 11b M5 ⇒ ( 2 5a + 5ab − 2 10b ) − ( 2 4a + 3ab − 2 11b )M5 0.25 2 2 a) ⇒ a + 2ab + b M5 1.0 ⇒ ( 2 a + b) M5 0,25 điểm ⇒ a 0,25
+ bM5 ( Vì 5 là số nguyên tố) 4 4 ⇒ − = ( 2 2 a b a + b )(a + b)(a − b)M5 0,25 2 5 − 3 ( 5 − 3) x = = = 4 − 15 5 + 3 ( 5 + 3)( 5 − 3) 0,25 Câu 3 2,0 5 − 3 x =
là nghiệm của phương trình nên ta có đim 5 + 3 a ( − )2 4 15 + b(4 − 15) +1= 0 b) 1,0
a (31− 8 15) + b(4 − 15) +1= 0 ®iÓm
⇔ − 15(8a + b) + 31a + 4b + 1 = 0 0,25
a,bQ nên (8a + b), (31a + 4b +1)∈Q 0,25 Do 31a + 4b +1 đ
đó nếu 8a + b ≠ 0 thì 15 = ∈ Q (Vô lí) 8a + b 8  a + b = 0 a = 1 Suy ra  ⇔  3 0,25
 1a + 4b +1 = 0 b  = −8 M Q P D A H O B K I d C N E
I là trung điểm của BC ( dây BC không đi qua O ) a) 0
OI BC OIA = 90 0,25 0,25 1,0 Ta có 0
AMO = 90 ( do AM là hai tiếp tuyến (O) ) ®iÓm 0,25 0
ANO = 90 ( do AN là hai tiếp tuyến (O) ) Suy ra 4 0.25
điểm O, M, N, I cùng thuộc đường tròn đường kính OA
AM, AN là hai tiếp tuyến (O) cắt nhau tại A nên OA là tia phân giác MON mà
∆OMN cân tại O nên OA MN 1 Câu 4
∆ABN đồng dạng với ∆ANC ( vì ANB=ACN= sđ NB và 3,0 2 đim AB AN CAN chung ) suy ra 2 = ⇒ AB.AC=AN 0,25 AN AC
∆ANO vuông tại N đường cao NH nên ta có AH.AO = AN2 b) Suy ra AB.AC = AH.AO 0,25 1,0
∆AHK đồng dạng với ∆AIO ( vì 0 AHK=AIO=90 và OAI chung ) ®iÓm AH AK ⇒ = ⇒ AI.AK=AH.AO AI AO ⇒ AI.AK=AB.AC AB.AC ⇒ AK= 0,25 AI
Ta có A,B,C cố định nên I cố định suy ra AK cố định mà A cố định,
K là giao điểm của dây BC và dây MN nên K thuộc tia AB suy ra K 0,25 cố định Ta có 0
PMQ=90 ( góc nội tiếp chắn nửa đường tròn ).
Xét ∆MHE và ∆QDM có MEH=DMQ ( cùng phụ với DMP ), c) EMH=MQD ( cùng ph ⇒ 0,25 1,0 ụ với MPO ) ME MH = MQ DQ ®iÓm
∆PMH đồng dạng với ∆MQH MP MH MH 0,25 ⇒ = = MQ HQ 2DQ MP 1 ME ⇒ = 0,25 MQ 2 MQ
⇒ ME = 2 MP ⇒ P là trung điểm ME. 0,25 1 2n −1 A = = n (2 0,25 n +1) 2n −1 (2n +1)(2n − ) 1 2n −1  1 1  2n −1  1 1   1 1  A =  −  =  +   −  0,25 n 2
 2n −1 2n +1 2  2n −1
2n +1  2n −1 2n +1  Câu 5 1 1 1 1 2 Vì − > 0 và + < nên 1,0 2n 1 − 2n 1 + 2n 1 − 2n+1 2n 1 − đim 1 1 A < − ( n ∀ ∈ *) 0,25 n 2n −1 2n +1 Do 1 1 1 1 1
đó: A + A + A + ... + A < 1 1 2 3 − + − + ⋅⋅⋅ + − n 3 3 5 2n −1 2n +1 1 0,25
A + A + A + ... + A < 1− < 1 1 2 3 n 2n +1 Hết
S GD&ĐT HÒA BÌNH
ĐỀ THI CHN HC SINH GII CP TNH
NĂM HC 2012 - 2013.
Đề chính thc
MÔN: TOÁN LP 9
(Thi gian 150 phút không k thi gian giao đề). 2   Bài 1: 4x x − 2 x + 2 x − 3
(4,5 điểm): Cho biểu thức A= + + :  2   x − 4 x + 2
2 − x x − 2 1. Rút gọn A.
2. Tìm giá trị của A khi x − 3 = 1 Bài 2 (4 điểm):
1. Giải phương trình x − 2 + 3x −1 = 9
2. Cho các số a,b,c thỏa mãn điềukiện ab + bc + ca = 1. Tính giá trị nhỏ nhất của biểu thức 2 2 2
P = (a + 2bc −1)(b + 2ca −1)(c + 2ab −1) Bài 3 (4 điểm):
a, Một ca nô xuôi một khúc sông dài 52km rồi ngược dòng trở lại 40km mất tổng cộng 4
giờ Biết vận tốc của dòng nước là 3km/h. Tìm vận tốc của ca nô lúc dòng nước yên lặng.
b, Trong mặt phẳng tọa độ Oxy, cho đường thẳng (d) : y= ax +b. Tìm a,b biết (d) đi qua
M(-1; 3) và (d) cắt Ox tại điểm N thỏa mãn ON = 2
Bài 4 (6 điểm): Cho đường tròn tâm O và hai điểm B,C thuộc đường tròn, các tiếp tuyến với
đường tròn tại B và C cắt nhau ở A. Một M là một điểm thuộc cung nhỏ BC. Tiếp tuyến với
đường tròn tại M cắt AB, AC theo thứ tự ở D, E . Gọi giao điểm của OD, OE với BC theo thứ tự
là I và K . Chứng Minh rằng a) BD.OE = OD.BI
b) Tứ giác DIKE nội tiếp c) OM, DK, EI đồng quy
1 1 2 1 2 3 1 2 3 4 1 2 3 4 5 1 2 3
Bài 5 (2 điểm): Cho dãy số sau: ; ; ; ; ; ; ; ; ; ; ; ; ; ; ; ; ; ;....
1 2 1 3 2 1 4 3 2 1 5 4 3 2 1 6 5 4
Tìm số hạng thứ 2013 trong dãy số trên
(Ghi chú: Cán b coi thi không gii thích gì thêm) Së GD & §T Hoµ B×nh
kú thi chän häc sinh giái cÊp tØnh
Líp 9 tHCS n¨m häc 2010 - 2011
§Ò chÝnh thøc §Ò thi m«n : To¸n
Ngµy thi: 22 th¸ng 3 n¨m 2011
Thêi gian lµm bµi: 150 phót (kh«ng kÓ thêi gian giao ®Ò) (§Ò thi gåm cã 01 trang) Bµi 1: (4 ®iÓm)
1. Ph©n tÝch thµnh nh©n tö c¸c biÓu thøc sau: a/ 3 2 2 3
A = x + 3x y − 4xy −12 y b/ 3 2 2 3
B = x + 4 y − 2xy + x + 8 y
2. Cho a = 11+ 6 2 + 11− 6 2 . Chøng minh r»ng a lµ mét sè nguyªn. Bµi 2: (6 ®iÓm) 12 3 1. Gi¶i ph−¬ng tr×nh: − = 1 2 2 x + x + 4 x + x + 2 2. Cho hµm sè 2
y = (m −1)x + m −1 (m: tham sè). T×m m ®Ó ®å thÞ hµm sè lµ ®−êng
th¼ng c¾t hai trôc to¹ ®é t¹i hai ®iÓm A, B sao cho tam gi¸c OAB c©n. x −1
3. T×m x ®Ó biÓu thøc A = ®¹t gi¸ trÞ nhá nhÊt. x +1 Bµi 3: (4 ®iÓm)
1. Cho tam gi¸c ABC nhän néi tiÕp trong ®−êng trßn t©m O, cã b¸n kÝnh b»ng 2. BiÕt 0
BAC = 60 , ®−êng cao AH = 3. TÝnh diÖn tÝch tam gi¸c ABC.
2. Đội cờ vua của trường A thi đấu với đội cờ vua của trường B, mỗi đấu thủ của
trường này thi đấu với mọi đấu thủ của trường kia một trận. Biết rằng tổng số trận đấu bằng
bốn lần tổng số cầu thủ của cả hai đội và số cầu thủ của trường B là số lẻ. Tìm số cầu thủ của mỗi đội.
Bµi 4: (5 ®iÓm) Cho nửa đường tròn tâm O bán kính R, đường kính AB. Hai điểm E, F thay
đổi trên nửa đường tròn sao cho số đo cung AE khác không và nhỏ hơn số đo cung AF, biết
EF = R . Giả sử AF cắt BE tại H, AE cắt BF tại I.
1. Chứng minh rằng tứ giác IEHF nội tiếp được trong một đường tròn.
2. Gọi EG và FQ là các đường cao của tam giác IEF, chứng minh rằng độ dài QG không đổi.
3. Chứng minh rằng QG song song với AB.
Bµi 5: (1 ®iÓm) Gi¶i ph−¬ng tr×nh: 2
x + 2 7 − x = 2 x −1 + −x + 8x − 7 +1
--------------------HÕt-----------------
Hä vµ tªn thÝ sinh:................................ .................. SBD: ..........
Gi¸m thÞ 1 (hä vµ tªn, ch÷ ký): ...................................................
Gi¸m thÞ 2 (hä vµ tªn, ch÷ ký): ....................................................
Së GD&§T Hoµ B×nh H−íng dÉn chÊm m«n to¸n
Kú thi chän häc sinh giái cÊp tØnh cÊp THCS N¨m häc 2010-2011 Bµi ý Néi dung §iÓm 1
a/ A = ( x + 3y ).( x - 2y ).( x + 2y ). 1,0 1.
b/ B = ( x + 2y + 1 ).( x2 - 2xy + 4y2 ). 1,0 2 (4®) 2 2
a = 11+ 6 2 + 11− 6 2 = (3 + 2) + (3 − 2) = 6 1,5
Tõ ®ã a lµ sè nguyªn. 0,5 2 1.
+ HS lËp luËn ®−îc x2 + x + 4 vµ x2 + x + 2 kh¸c 0 råi ®−a PT vÒ d¹ng
9( x2 + x ) + 12 = ( x2 + x + 4 ) ( x2 + x + 2 ) 1,0 (6 ®)
+HS biÕn ®æi PT vÒ d¹ng ( x2 + x - 4 ) ( x2 + x + 1 ) = 0 0,5 − ± 1 17
+HS gi¶i PT tÝch t×m ®−îc 2 nghiÖm lµ x = 0,5 2 2.
+ HS lËp luËn ®−îc ®Ó ®å thÞ hµm sè lµ ®−êng th¼ng c¾t 2 trôc täa ®é t¹i 2 ®iÓm
A vµ B sao cho tam gi¸c OAB c©n th× ®å thÞ hµm sè ®Z cho song song víi ®−êng 1,0
th¼ng y = x ( hoÆc y = - x ) m −1 = 1 m −1 = −1 + Tõ ®ã dÉn ®Õn  hoÆc 
gi¶i 2 hÖ PT ®ã t×m ®−îc 2 m −1 ≠  0 2 m −1 ≠  0 1,0
m = 2 hoÆc m = 0 vµ tr¶ lêi bµi to¸n. 2 = − 0,5 3. + HS viÕt ®−îc A 1 x +1 1,5
+ HS lËp luËn vµ t×m ®−îc gi¸ trÞ nhá nhÊt cña biÓu thøc A b»ng - 1 khi x = 0. 3 (4 ®) A
Gäi K lµ trung ®iÓm cña BC, dÔ cã 0 KOC = 60 . 1.
XÐt tam gi¸c vu«ng OKC cã OC = 2 1,0 O TÝnh ®−îc 0
KC = OC.sin 60 = 3 , TÝnh ®−îc , suy ra diÖn tÝch B BC = 2 3 K C 1,0
tam gi¸c ABC lµ S = 3 3 (§vdt)
Chó ý: Thùc chÊt tam gi¸c ABC ®Òu 1,0
nh−ng kh«ng yªu cÇu HS vÏ h×nh ®óng.
+ Gọi số cÇu thñ ®éi tr−êng A lµ x; Sè cÇu thñ ®éi tr−êng B lµ y đặt đk vµ lËp 1,0 2.
®−îc PT: xy = 4( x + y ) ⇔ (x − 4)( y − 4) = 16
+ HS lập luận vµ t×m ®−îc x = 20 ; y= 5, KL… 1.
1. Chứng minh được tứ giác IEHF nội tiếp I 4
được trong một đường tròn. 2,0 Q G (5 ∆ ∆ (g.g), đ) 2.
2. Chứng minh được IQG IFE F QG IG 1 1 1 từ đó có = =
; QG = EF = R (đpcm). E 1,0 EF IE 2 2 2 H 1,0 3. 3. Chứng minh được IAB I ∆ FE (g.g), kết hợp B A O IQ IG với (2) ta có IQG I ∆ AB , suy ra = dẫn IA IB đến QG song song với AB. 1,0 5
+ HS t×m ®−îc §K 1 ≤ x ≤ 7 vµ biÕn ®æi PT vÒ d¹ng tÝch 0,5 (1®)
( x −1 − 2 ).( x −1 − 7 − x ) = 0
+ HS gi¶i PT tÝch t×m ®−îc x = 5 hoÆc x = 4 ®Òu tháa mZn vµ tr¶ lêi. 0,5
Chó ý: Mäi lêi gi¶i ®óng kh¸c ®Òu ®−îc cho ®iÓm t−¬ng ®−¬ng
SỞ GIÁO DỤC VÀ ĐÀO TẠO
K THI CHN HC SINH GII LP 9 THCS HÀ NAM
NĂM HC 2012-2013 Môn thi: TOÁN
ĐỀ THI CHÍNH THC
Thi gian làm bài: 150 phút, không k thi gian giao đề
Bài 1.
(4,0 đim) Cho bi x y xy ểu thức: P = − −
( x + y )(1− y ) ( x + y )( x +1) ( x +1)(1− y )
1. Rút gọn biểu thức P.
2. Tìm các giá trị x, y nguyên thỏa mãn P = 2.
Bài 2. (4,0 đim)
1. Cho hai số thực a, b không âm thỏa mãn18a + 4b ≥ 2013 . Chứng minh rằng
phương trình sau luôn có nghiệm: 2
18ax + 4bx + 671− 9a = 0 .
2. Tìm tất cả các nghiệm nguyên x, y của phương trình 3 2 3
x + 2x + 3x + 2 = y .
Bài 3. (4,5 đim)
1. Cho p2p + 1 là hai số nguyên tố lớn hơn 3. Chứng minh rằng 4p + 1 là một hợp số. 2. Giải phương trình: 2 3 2
4x + 3x + 3 = 4 x + 3x + 2 2x −1
Bài 4. (6,0 đim)
Cho góc xOy có số đo bằng 60o. Đường tròn có tâm K nằm trong góc xOy tiếp
xúc với tia Ox tại M và tiếp xúc với tia Oy tại N. Trên tia Ox lấy điểm P thỏa mãn
OP = 3OM. Tiếp tuyến của đường tròn (K) qua P cắt tia Oy tại Q khác O. Đường
thẳng PK cắt đường thẳng MN ở E. Đường thẳng QK cắt đường thẳng MN ở F.
1. Chứng minh tam giác MPE đồng dạng với tam giác KPQ.
2. Chứng minh tứ giác PQEF nội tiếp được trong đường tròn.
3. Gọi D là trung điểm của đoạn PQ. Chứng minh tam giác DEF là một tam giác đều.
Bài 5. (2,0 đim)
Cho a, b, c là ba số thực dương thỏa mãn: a + b + c = 3. Chứng minh rằng: a +1 b +1 c +1 + + ≥ 3 2 2 2 1+ b 1+ c 1+ a
------HT------
Thí sinh không được s dng máy tính cm tay.
Họ và tên thí sinh: ................................................. Số báo danh: ............................
Chữ ký của giám thị 1: ............................. Chữ ký của giám thị 2: ...............................
S GIÁO DC VÀ ĐÀO TO
K THI CHN HC SINH GII LP 9 THCS HÀ NAM
NĂM HC 2012-2013 Môn thi: TOÁN
ĐÁP ÁN CHÍNH THC ĐÁP ÁN-BIỂU ĐIỂM
(Đáp án biu đim này gm 3 trang) Câu Ni dung Đim Câu
§iÒu kiÖn ®Ó P x¸c ®Þnh lµ : x ≥ 0 ; y ≥ 0 ; y ≠ 1 ; x + y ≠ 0 . 0,5 1.1 x(1 + x ) − y(1 −
y ) − xy ( x + y ) (x y) + (2,5
(x x + y y ) − xy( x + y ) đ) P = = 0,5
( x + y )(1 + x )(1 − y )
( x + y )(1 + x)(1 − y )
( x + y )( x y + x xy + y xy) x ( x + ) 1 − y ( x + )
1 + y (1 + x )(1 − x ) = = 0,5
( x + y )(1 + x )(1 − y )
(1 + x )(1 − y ) x
y + y y x
x (1 − y )(1 + y ) − y (1 − y ) = = 0,5 (1 − y ) (1 − y ) = x + xy y 0,5 Câu P = 2 ⇔ x + xy −
y = 2 với x ≥ 0 ; y ≥ 0 ; y ≠ 1 ; x + y ≠ 0 1.2 ⇔ x (1,5 đ)
(1 + y ) − ( y + 1) = 1 ⇔ ( x − 1)(1 + y ) = 1 0,5
Ta cã: 1 + y ≥ 1 ⇒ x −1 ≤ 1 ⇔ 0 ≤ x ≤ 4 ⇒ x = 0; 1; 2; 3 ; 4 0,5
Thay vµo P ta cã c¸c cÆp gi¸ trÞ (4; 0) vµ (2 ; 2) tho¶ m,n 0,5 Câu
Cho hai s thc a, b tha mãn 18a + 4b ≥ 2013 (1) 2.1
(2,0 đ) Chng minh rng phương trình sau có nghim: 2
18ax + 4bx + 671 − 9a = 0 (2)
TH1 : Với a = 0 thì (2) ⇔ 4bx + 671 = 0
Từ (1) ⇒ b ≠ 0 . Vậy (2) luôn có nghiệm 671 x = − 0,5 4b
TH2 : Với a ≠ 0 , ta có : 2 2 2 ∆ ' = 4b −18 ( a 671 − 9a) = 4b − 6 .2013 a + 162a 0,5 2 2 2 2 2 2 ≥ 4b − 6 (
a 18a + 4b) + 162a = 4b − 24ab + 54a = (2b − 6a) + 16a ≥ 0, ∀ , a b 0,5 Vậy pt luôn có nghiệm 0,5 Câu
Tìm các s nguyên x, y tha mãn phương trình: 3 2 3
x + 2x + 3x + 2 = y 2.2 2   (2,0 đ) Ta có 3 7 3 3 2
y x = 2x + 3x + 2 = 2  x +  +
> 0 ⇒ x < y (1) 0,5  4  8 2   3 3 2 9 15
( x + 2) − y = 4 x + 9 x + 6 = (2)  2 x +  +
> 0 ⇒ y < x + 2 0,5  4  16
Từ (1) và (2) ta có x < y < x+2 mà x, y nguyên suy ra y = x + 1 0,5
Thay y = x + 1 vào pt ban đầu và giải phương trình tìm được x = -1; x = 1 từ đó 0,5
tìm được hai cặp số (x, y) thỏa mãn bài toán là (1 ; 2), (-1 ; 0) Câu
Do p là số nguyên tố lớn hơn 3 nên p có dạng p = 3k ± 1 0,5 3.1
*) Nếu p = 3k + 1 thì 2 p + 1 = 6k + 3 = 3(2k + 1) (2,0đ) 0,5
⇒ 2p + 1 là hợp số (Vô lý)
*) Nếu p = 3k −1, k ≥ 2 thì 4 p + 1 = 12k − 3 = 3(4k −1) 0,5
Do 4k −1 ≥ 7 nên 4p + 1 là một hợp số. 0,5 Câu 1 Điều kiện: x ≥ 3.2 2 0,5 (2,5 đ) PT 2
⇔ 4x + 3x + 3 = 4x x + 3 + 2 2x − 1 0,5 ⇔ ( 2
4x − 4x x + 3 + x + 3) + (1− 2 2x −1 + 2x −1) = 0 ⇔ ( x − x + )2 + ( − x − )2 2 3 1 2 1 = 0 0,5 2x = x + 3 ⇔  0,5 1 = 2x −1 2 4x = x + 3 ⇔  ⇔ x = 1 (tmđk) 0,5  1 = 2x −1 Câu 4 Câu Hình vẽ đúng. 0,5 4.1 y +PK là phân giác góc QPO (2,5 đ) 0,5
MPE = KPQ (*) . Q + Tam giác OMN đều 0 ⇒ EMP = 120 .
+ QK cũng là phân giác OQP 0,5 N E D 0 QKP = 180 − K (KQP+ KPQ) Mà 0 0 0
2KQP + 2KPQ = 180 − 60 = 120 x 0 O ⇒ 0,5
QKP = 120 . Do đó: EMP = QKP M P (* ) * . F
Từ (*) và (**), ta có ∆MPE ∆KPQ 0,5 Câu
Do hai tam giác MPE và KPQ đồng dạng nên: MEP = KQP 0,5 4.2
(1,0 đ) hay: FEP = FQP Suy ra, tứ giác PQEF nội tiếp được trong đường tròn. 0,5 Câu
Gi D là trung đim ca đon PQ. Chng minh tam giác DEF là mt tam giác đều. 4.3 PM PE PM PK
Do hai tam giác MPE và KPQ đồng dạng nên: = . Suy ra: = . (2,5 đ) PK PQ PE PQ 0,5
Ngoài ra: MPK = EPQ . Do đó, hai tam giác MPK và EPQ đồng dạng. Từ đó: 0 PEQ = PMK = 90 . 0,5
Suy ra, D là tâm của đường tròn ngoại tiếp tứ giác PQEF. 0,5
Vì vậy, tam giác DEF cân tại D.
Ta có: FDP = 2FQD = OQP ; EDQ = 2EPD = OPQ . 0,5 0 = − ( + ) 0 FDE 180 FDP EDQ = POQ = 60 0,5
Từ đó, tam giác DEF là tam giác đều.
Câu 5 Cho a, b, c là ba s thc dương tha mãn: a + b + c = 3 . Chng minh rng: (2,0 đ) a + 1 b + 1 c + 1 + + ≥ 3 2 2 2 1 + b 1 + c 1 + a
Theo bất đẳng thức Cauchy ta có: 2 1 + b ≥ 2b nên: 2 2 a + 1 b (a + 1) b (a + 1) ab + b = (a + 1) − ≥ (a + 1) − = a + 1 − 2 2 1 + b b + 1 2b 2 a + 1 ab + b ⇔ ≥ a + 1 − 2 1 + b 2 0,5 Tương tự ta có: b + 1 bc + c ≥ b + 1 − (2) 2 1 + c 2 c + 1 ca + a ≥ c + 1 − (3) 2 1 + a 2
Cộng vế theo vế (1), (2) và (3) ta được: a + 1 b + 1 c + 1 a + b + c − ab − bc − ca 0,5 + + ≥ 3 + (*) 2 2 2 1 + b 1 + c 1 + a 2 M a + b + c − ab − bc − ca ặt khác: ab + bc + ca ≤ (a + b + c)2 3( ) = 9 ⇒ ≥ 0 2 0,5 a + 1 b + 1 c + 1 Nên (*) ⇔ + + ≥ 3 (đpcm) 2 2 2 1 + b 1 + c 1 + a
Dấu "=" xảy ra khi và chỉ khi a = b = c = 1
---------------HT--------------
Lưu ý: - Các cách giải đúng khác cho điểm tương đương với biểu điểm
- Điểm toàn bài không làm tròn
SỞ GIÁO DỤC VÀ ĐÀO TẠO
KỲ THI CHỌN HỌC SINH GIỎI TỈNH HÀ NAM LỚP 9 THCS NĂM 2011 Môn Toán ĐỀ CHÍNH THỨC
Thời gian: 150 phút (không k thi gian giao đề)
Bài 1.(6 đim) 6 + 2 5 − 13 + 48 1. Cho biểu thức A = 3 +1
a) Rút gọn biểu thức A.
b) Tìm nghiệm nguyên của phương trình: 2 2 y − A = x(A + x)(A + x ) 2. Gọi d ,d là các 1 2
đường thẳng lần lượt có phương trình: d :y = 2x + 3m + 2 và 2 d : y = (m + m)x − 4 1 2
a) Tìm m để hai đường thẳng d ,d song song. 1 2
b) Tuỳ theo giá trị của m, tìm giá trị nhỏ nhất của biểu thức: 2 = ( − + + )2 2 B
2x y 3m 2 + (m + m)x − y − 4  
Bài 2.(6 đim) 1. Gi 2 3
ải phương trình: 2(x + 2) = 3( x + 8 + 2x)
2. Tìm m để phương trình sau có 4 nghiệm phân biệt: 4 3 2 + − − − ( + ) 2 x 3x (2m 1)x 3m 1 x + m + m = 0
Bài 3.(1 đim)
x(2 y −1 − x) + y(2 x −1 − y) = 0
Giải hệ phương trình :  3 3 x + y =16
Bài 4.(6 đim)
Cho 3 điểm cố định A, B, C phân biệt và thẳng hàng theo thứ tự đó. Đường tròn (O) đi
qua B và C (O không thuộc BC). Qua A kẻ các tiếp tuyến AE và AF đến đường tròn (O) (E và
F là các tiếp điểm). Gọi I là trung điểm của đoạn thẳng BC, N là trung điểm của đoạn thẳng EF.
1. Chứng minh rằng: E và F nằm trên một đường tròn cố định khi đường tròn (O) thay đổi.
2. Đường thẳng FI cắt đường tròn (O) tại E’(khác F). Chứng minh tứ giác BCE’E là hình thang.
3. Chứng minh rằng: Tâm đường tròn ngoại tiếp tam giác ONI nằm trên một đường
thẳng cố định khi đường tròn (O) thay đổi.
Bài 5.(1 đim)
Cho tam giác ABC. Xác định vị trí của điểm M nằm trong tam giác ABC sao cho
AM.BC + BM.CA + CM.AB đạt giá trị nhỏ nhất.
-----------------------------HẾT---------------------------
(Giám th không gii thích gì thêm )
H
và tên: ................................................. S báo danh:.................................................
Ch
kí ca giám th 1.................................Ch kí ca giám th 2....................................
PHÒNG GIÁO DỤC – ĐÀO TẠO ĐỨC THỌ
ĐỀ THI CHN ĐỘI TUYN HC SINH GII CP HUYN NĂM HC 2013-2014 MÔN TOÁN 9
Thời gian làm bài: 150 phút
Bài 1: Rút gọn các biểu thức sau:
a) A = 4 + 10 + 2 5 + 4 − 10 + 2 5 − 5 x y ( − )2 ( − )2 2 2 2 2 x y x x y y b) B = + − với xy > 0; x ≠ y xy x (x − y) y (x − y)
Bài 2: Tìm các số nguyên x, y thỏa mãn 2 y + 2xy − 7x −12 = 0
Bài 3: Giải các phương trình    a) 5 − x 5 − x 10 14 x   x +  = 6
b) (x − 2013) + (x − 2014) = 1  x +1  x +1 
Bài 4: Cho ∆ABC vuông tại A (AC > AB), đường cao AH (H ∈ BC). Trên tia HC lấy điểm D sao cho HD =
HA. Đường vuông góc với BC tại D cắt AC tại E.
a) Chứng minh rằng ∆BEC ∼ ∆ADC. Tính BE theo m = AB
b) Gọi M là trung điểm của BE. Chứng minh rằng ∆BHM ∼ ∆BEC. Tính AHM GB HD
c) Tia AM cắt BC tại G. Chứng minh rằng = BC AH + HC Bài 5: a) Cho 3 3 + + ( 2 2 x y
3 x + y ) + 4(x + y) + 4 = 0 và xy > 0 1 1 Tìm GTLN của M = + x y
b) Với a, b, c là các số thực dương. Chứng minh rằng 5 5 5 3 3 3 a b c a + b + c + + ≥ 2 2 2 2 2 2 a + ab + b b + bc + c c + ca + a 3
Bài gii ca Nguyn Ngc Hùng – THCS Hoàng Xuân Hãn Bài 1: a) Đặt = + + + − + ⇒ 2 x 4 10 2 5 4 10 2 5
x = 8 + 2 6 − 2 5 = 8 + 2 ( 5 − ) 1 = 6 + 2 5 ⇒ x = 5 +1. Do đó A = 1 (x − y)x (x − y) y b) B = 1+ − x (x − y) y (x − y)
Xét các trường hợp x < y < 0; y < x < 0; x > y > 0 và y > x > 0 ta đều được B = 1 Bài 2: 2 Cách 1: 2
y + 2xy − 7x −12 = 0 ⇔ (x + y) = (x + 3)(x + 4)
(x + 3)(x + 4) là tích của 2 số nguyên liên tiếp nên không thể là 1 số chính phương x + 3 = 0 x = 3 − Dó đó ⇔ 
Từ đó ta tìm được (x; y) ∈ {(-3; 3); (-4; 4)} x 4 0  + = x = 4 − Cách 2: 2 + − − = ⇔ 2 + − − = ⇔ 2 y 2xy 7x 12 0 4y 8xy 28x 48 0
4y − 49 + 4x (2y − 7) = −1 2y − 7 = 1 x = 4 − 2y − 7 = 1 − x = 3 −
⇔ (2y − 7)(2y + 7 + 4x ) = 1 − ta có  ⇔   ⇔  2y + 7 + 4x = −1 y = 4 2y + 7 + 4x = 1 y = 3  5 x  Bài 3: a) 5 x
Cách 1: ĐKXĐ: x ≠ -1. Đặt − x   = a và − x + = b .  x +1  x + 1  5 x   5 x  2 2 Ta có − − 5x − x + x + x + 5 − x a + b = x   +  x +  = = 5  x +1   x + 1  x +1 a = 2   5 − x   x   = 2 ab = 6 2 b = 3 a = 2   x +1   x − 3x + 2 = 0 Do đó  ⇔ . Với 2  ⇒  ⇔  ⇒ x − 3x + 2 = 0 a b 5  + = a = 3 2 b = 3  5 − x x − 3x + 2 = 0  x + = 3   b = 2  x +1 x = 1 ⇔ ( x − ) 1 (x − 2) = 0 ⇔  x = 2   5 − x  x   = 3 2 a = 3   x +1   x − 2x + 3 = 0 V 2 ới 2  ⇒  ⇔ 
⇒ x − 2x + 3 = 0 ⇔ (x − ) 1 + 2 = 0 , vô nghiệm 2 b = 2  5 − x x − 2x + 3 = 0 x + = 2  x +1
Vậy phương trình có tập nghiệm S = {1; 2}  5 x  5 x  − − 2 Cách 2: x   x +  = 6 ⇔ (5x − 2 x )( 2 x + 5) = 6(x + ) 1 ⇔ 4 x − 3 5x + 2 11x −13x + 6 = 0  x +1  x + 1  ⇔ 4 − 3 + 2 − + = ⇔ ( 2 − + )( 2 x 5x 11x 13x 6 0 x 3x 2 x − 2x + 3) = 0
Từ đó ta tìm được tập nghiệm S = {1; 2} b) ( 10 14 5 7
x − 2013) + (x − 2014) = 1 ⇔ x − 2013 + x − 2014 = 1
Ta có x = 2013, x = 2014 là 2 nghiệm của phương trình. Ta chứng minh 2 nghiệm này là duy nhất Xét x < 2013 ⇒ 7 5 7
x − 2014 < −1 ⇒ x − 2014 > 1 ⇒ x − 2014 > 1 ⇒ x − 2013 + x − 2014 > 1 5 0 < x − 2013 < 1 0 x 2013 1   < − <  x − 2013 < x − 2013
Xét 2013 < x < 2014 ⇒  ⇔  ⇔  7  1 − < x − 2014 < 0 0 < x − 2014 < 1   x − 2014 < x − 2014  5 7
⇔ x − 2013 + x − 2014 < x − 2013 + x − 2014 = x − 2013 + 2014 − x = 1 Xét x > 2014 ⇒ 5 5 7
x − 2014 < −1 ⇒ x − 2013 > 1 ⇒ x − 2013 > 1 ⇒ x − 2013 + x − 2014 > 1
Vậy phương trình có nghiệm x = 2013, x = 2014 0  A EDC = BAC = 90 (gt)
Bài 4: a) Xét ∆EDC và ∆BAC có  C chung E EC BC m
⇒ ∆EDC ∼ ∆BAC (g – g) ⇒ = M DC AC Xét ∆BEC và ∆ADC có C B H G D  EC BC  =  DC
AC ⇒ ∆BEC ∼ ∆ADC (c – g - c)  C chung
⇒ BEC = ADC . Mặt khác AH = HD (gt) nên 0 0 0 0
⇒ ADH = 45 ⇒ ADC = 135 ⇒ BEC = 135 ⇒ AEB = 45 ⇒ ∆AEB vuông cân tại A. Do đó BE = m 2 0 AHB = CAB = 90 (gt)
b) Xét ∆AHB và ∆CAB có 
⇒ ∆AHB ∼ ∆CAB (g – g) B chung AB BH 2 2 2 BE BH BM BH ⇒ =
⇒ AB = BH.BC ⇒ 2AB = 2BH.BC ⇒ BE = 2BH.BC ⇒ = ⇒ = BC AB 2BC BE BC BE BM BH  =
(Vì BE = 2BM). Xét ∆BHM và ∆BEC có  BC BE
⇒ ∆BHM ∼ ∆BEC (c – g - c)  MBH chung 0 0
⇒ BHM = BEC = 135 ⇒ AHM = 45 0 AHC = BAC = 90 (gt) AH AB
c) Xét ∆AHC và ∆BAC có 
⇒ ∆AHC ∼ ∆BAC (g – g) ⇒ = (1) C chung HC AC
Mặt khác ∆AEB vuông cân tại A có AM là trung tuyến thì AM cũng là phân giác hay AG là đường GB AB
phân giác của ∆ABC. Suy ra =
(2). Từ (1) và (2) ta có: GC AC GB AH =
⇒ GB.HC = AH.GC ⇒ GB.HC = AH.(BC − GB) ⇒ GB.HC = AH.BC − AH.GB GC HC GB HD
⇒ AH.GB + GB.HC = HD.BC (Vì HD = AH) ⇒ GB.(AH + HC) = HD.BC ⇒ = BC AH + HC Bài 5: a) 3 3 + + ( 2 2 x y 3 x + y ) + 4(x + y) + 4 = 0 ⇔ ( + )( 2 2 − + ) + ( 2 2 − + ) + ( 2 2 x y x xy y 2 x xy y
x + 2xy + y ) + 4(x + y) + 4 = 0 ⇔ ( − + ) 1 x
xy y (x + y + 2) + (x + y + 2)2 2 2 = 0 ⇔ (x + y + 2)( 2 2
2x − 2xy + 2y + 2x + 2y + 4) = 0 2 1 (x y 2)(x y)2 (x )2 1 ( y )2 1 2 ⇔ + + − + + + + +
= 0 ⇔ x + y + 2 = 0 ⇔ x + y = 2 − 2  
Mà xy > 0 do đó x, y < 0 (−x) + (−y) 2 Áp d −
ụng BĐT CauChy ta có (−x)(−y) ≤ = 1 nên xy ≤ 1, do đó ≤ 2 − 2 xy 1 1 x + y Vậy M = + = ≤ 2
− , GTLN của M là -2. Đạt được khi x = y = -1 x y xy 3 a 2a − b b) Cách 1: Ta có: 3 ≥
⇔ 3a ≥ (2a − b) a + ab + b ⇔ a + b ≥ ab a + b 2 2 ( 2 2 ) 3 3 ( ) a + ab + b 3 ⇔ − + ≥ ⇔ ( − )2 2 2 a ab b ab a b ≥ 0 luôn đúng. 3 5 3 2 a 2a − b a 2a − a b Do đó ≥ ⇔ ≥
. Chứng minh tương tự ta được 2 2 2 2 a + ab + b 3 a + ab + b 3 5 5 5 3 3 3 3 3 3 2 2 2 a b c a + b + c
a + b + c − a b − b c − c a + + ≥ + 2 2 2 2 2 2 a + ab + b b + bc + c c + ca + a 3 3
Mặt khác: Vai trò a, b, c như nhau nên giả sử a ≥ b ≥ c > 0 3 3 3 2 2 2 2 + + − − − = ( − ) 2 + ( − ) 2 a b c a b b c c a a a b b b c + c (c − a) = ( − ) + ( − + − ) + ( − ) = ( − )2 2 2 2 a a b b b a a c c c a
a b (a + b) + (a − c)(b − c)(b + c) ≥ 0 5 5 5 3 3 3 a b c a + b + c Từ đó suy ra + + ≥
. Dấu “=” xảy ra khi a = b = c 2 2 2 2 2 2 a + ab + b b + bc + c c + ca + a 3
Cách 2: Áp dụng BĐT Bunhia mở rộng ta có 5 5 5 6 6 6 a b c a b c + + = + + 2 2 2 2 2 2 3 2 2 3 2 2 3 2 2 a + ab + b b + bc + c c + ca + a a + a b + ab b + b c + bc c + c a + ca (a + b + c )2 3 3 3 ≥ 3 3 3 2 2 2 2 2 2
a + b + c + a b + ab + b c + bc + c a + ca M 2 ặt khác ( − ) 2 2 3 3
a b ≥ 0 ⇔ a − ab + b ≥ ab ⇔ a + b ≥ ab(a + b) tương tự 3 3 b + c ≥ bc(b + c) 3 3
c + a ≥ ca (c + a) . Suy ra ( 3 3 3
2 a + b + c ) ≥ ab(a + b) + bc(b + c) + ca (c + a) ⇔ ( 3 3 3 + + ) 3 3 3 3 a b
c ≥ a + b + c + ab(a + b) + bc(b + c) + ca (c + a) (a + b + c )2 3 3 3 3 3 3 a + b + c ⇒ ≥ 3 3 3 2 2 2 2 2 2
a + b + c + a b + ab + b c + bc + c a + ca 3
D đoán: Mỗi câu 1 đ theo thang điểm 10 và mỗi câu 2 đ theo thang điểm 20
PHÒNG GD&ĐT BÌNH GIANG
ĐỀ THI CHN HC SINH GII VÒNG I
NĂM HC 2012-2013 ĐỀ CHÍNH THỨC
MÔN: TOÁN - LP 9
(Thi gian làm bài: 150 phút)
Câu I (2,0 đim). x + 2 x +1 1 Cho biểu thức: A = + + với x ≥ 0, x ≠ 1 x x −1 x + x +1 1− x 1) Rút gọn A 1
2) Chứng tỏ rằng: A < 3
Câu II (2,0 đim).
1) Giải phương trình: x − x −15 = 17 2) Tìm x, y sao cho: − ( + ) 2 5x 2 x 2 y + y +1 = 0
Câu III (2,0 đim).
1) Tìm số nguyên x, sao cho : 2
x + x − p = 0 với p là số nguyên tố. 2 2) Tìm m m 2013m 2012 để hàm số bậc nhất − + y = x − 2011 là hàm số 2 m − 2 2m + 3 nghịch biến.
Câu IV (3,0 đim).
1) Cho tam giác ABC có ba góc nhọn nội tiếp đường tròn (O ; R), hai đường
cao BE và CF của tam giác cắt nhau tại H. Kẻ đường kính AK của đường tròn (O ;
R), gọi I là trung điểm của BC. a) Chứng minh AH = 2.IO. b) Biết 0
BAC = 60 , tính độ dài dây BC theo R. 2) Cho 0 A
∆ BC(A = 90 ) , BC = a. Gọi bán kính của đường tròn nội tiếp r 2 −1 A
∆ BC là r. Chứng minh rằng: ≤ . a 2
Câu V (1,0 đim).
Cho x + 3y ≥ 1. Tìm giá trị nhỏ nhất của biểu thức: 2 2 C = x + y
–––––––– Hết ––––––––
HƯỚNG DN CHM ĐỀ THI CHN HSG VÒNG I NĂM HC 2012-2013
MÔN: TOÁN - LP 9 Câu Phn Ni dung Đim x + 2 x +1 1 A = + − ( 0.25 x − ) 1 (x + x + ) 1 x + x +1 x −1 x + 2 + x −1− x − x −1 A = 0.25 ( x − )1(x + x + )1 1 (1,0 đ) x − x A = ( 0.25 x − ) 1 (x + x + ) 1 x ( x − ) 1 Câu I x A = = , với x ≥ 0, x ≠ 1 0.25 (2,0 điểm) ( x − )1(x + x + )1 x + x +1 ( − )2 x 1 1 1 x Xét − A = − = 0.50 3 3 x + x +1 3(x + x +1) 2 Do x ≥ 0, x ≠ 1 (1,0 2 đ) ⇒ ( − )2  1  3
x 1 > 0 và x + x +1 =  x +  + > 0 0.25  2  4 1 1 ⇒ − A > 0 ⇔ A < 0.25 3 3 ĐKXĐ: x ≥ 15
x − x −15 = 17 ⇔ x −15 − x −15 − 2 = 0 0.25 ⇒ 1 Đặt 2 t = x −15 (t ≥ 0) t − t − 2 = 0 0.25 (1,0 đ) t = 2 (TM§K)
⇔ (t − 2)(t + 1) = 0 ⇔  0.25 t = −  1 (lo¹i)
Với t = 2 ⇒ x −15 = 2 ⇔ x −15 = 4 ⇔ x = 19 (TMĐK) 0.25 ĐKXĐ: x ≥ 0 Câu II − ( + ) 2 5x 2 x 2 y + y +1 = 0 ⇔ 2
4x − 4 x +1+ x − 2y x + y = 0 0.25 (2,0 điểm) ⇔ ( − )2 + ( − )2 2 x 1 x y = 0 (1) 0.25 2 2 Vì (2 x − ) 1 ≥ 0, ( x − y) ≥ 0 x ∀ ≥ 0, y 2 (1,0 đ) ⇒ ( − )2 + ( − )2 2 x 1 x y ≥ 0 . 0.25  1 x 2 x 1 0 =   − =  4 Để (1) xẩy ra thì  ⇔  (TM) 0.25  x − y = 0 1 y =  2 Theo bài ra: 2 p = x + x = x (x + )
1 mà x, x + 1 là số nguyên liên tiếp Câu III 1 nên x (x + )
1 là số chẵn ⇒ p là số chẵn. 0.25 (2,0 điểm) (1,0 đ)
Mặt khác p là số nguyên tố nên p = 2 0.25 ⇒ 2
x + x − 2 = 0 ⇔ (x + 2)(x − ) 1 = 0 ⇔ x = 1 ho ặc x = - 2 (TM) 0.50 2 m − 2013m + 2012 Để hàm số y =
x − 2011 nghịch biến thì 2 m − 2 2m + 3 2 m − 2013m + 2012 < 0 (1). − + = ( − )2 2 m 2 2m 3 m 2 +1 > 0 m ∀ 0.25 2 m − 2 2m + 3 (1) 2
⇔ m − 2013m + 2012 < 0 ⇔ (m − ) 1 (m − 2012) < 0 2 0.25 (1,0 đ) m −1 > 0 m >1   m − 2012 < 0 m < 2012 ⇔ ⇔  m 1 0  − < m < 1 0.25     m − 2012 > 0 m > 2012 ⇒ 1 < m < 2012 0.25 A
Vì B, C thuộc đường tròn đường kính AK 0 ⇒ ABK = ACK = 90 E ⇒ KB ⊥ AB, KC ⊥ AC 0.25 CH ⊥ AB, BH ⊥ AC (gt) F O ⇒ BK // CH,CK // BH H 1a
⇒ BHCK là hình bình hành 0.25 (1,0 đ) B C I
I là trung điểm của BC (gt)
⇒ I là trung điểm của HK K O là trung điểm của AK (gt)
⇒ OI là đường trung bình của K ∆ AH 0.25 1 ⇒ OI = AH ⇒ AH = 2.IO 0.25 2 OA = OC ⇒ O
∆ AC cân tại O ⇒ OAC = OCA
KOC = OAC + OCA (T/c góc ngoài của tam giác) ⇒ KOC = 2.OAC 0.25
Chứng minh tương tự: KOB = 2.OAB Câu IV 1b (3,0 điểm) ⇒ + = ( + ) 0
KOC KOB 2 OAC OAB ⇒ BOC = 2.BAC = 120 0.25 (1,0 đ) OB = OC ⇒ O ∆ BC cân tại O ⇒ = ( 0 0 − ) 0 OCI 180 120 : 2 = 30 0.25 Vì I là trung
điểm của BC (gt) ⇒ OI ⊥ BC 3 Trong ∆ ( 0 OIC I$ = 90 ): 0 IC = OC.cos30 = R. ⇒ BC = R 3 0.25 2 B r 2 −1 ≤
⇔ 2r ≤ a 2 − a ⇔ 2r + a ≤ a 2 0.25 D a 2 C/m r được AB + AC = 2r + a 0.25 E O ⇒ AB + AC ≤ BC 2 2 A F C 2 2 2 ⇔ AB + 2AB.AC + AC ≤ 2BC (1,0 đ) 2 2 2 2
⇔ AB + 2AB.AC + AC ≤ 2AB + 2AC 0.25 ⇔ (AB − AC)2 ≥ 0( ) 1 r 2 −1 BĐT (1) đúng ⇒ ≤ , dấu “=” xảy ra khi A ∆ BC v/cân tại A. 0.25 a 2 Câu V
(1,0 điểm) (1,0 đ) Do x + 3y ≥ 1, đặt x + 3y = 1+ a với a ≥ 0 ⇒ x = 1 + a – 3y, thay vào biểu thức C: 2 2
⇒ C = 10y − 6ay − 6y + a + 2a +1 0.25 2  3  1 1 1 C = 10 y − (a + ) 1 +  ( 2 a + 2a ) + ≥  . 0.50  10  10 10 10 1 ⇒ min C = khi: 10  3  3  3  3 y y = (a ) 1 0 y y  − + = = =  10  10 ⇔  10 ⇔  10 ⇔  1 0.25 a 0 a 0 x 3y 1  = = + = x =  10
* Hc sinh làm bng cách khác đúng vn cho đim ti đa.